Sie sind auf Seite 1von 479

" Item: 1 of 46 111 f> Mark .

<:J [::> il ~ ~
w
2
Q. ld: 1149 [ Previous Next Lab Values Notes Calculator
4
5
6 A 54-year-old alcoholic w ho has been sober for 6 months "falls off the w agon" and imbibes a large amount of
7 isopropyl alcohol. After an unknow n period of time she is found by her neighbor and brought to the emergency
8 room . The patient eventually arouses but is troubled by a persistent right foot drop. Prolonged compression
9 of w hat nerve most likely accounts for her symptoms?
10
11
12 O A. Common peroneal
13
0 B. Femoral
14
15 o c. Tibial
16
O D. Radial
17
18 O E Pudendal
19
0 F. Lateral sural cutaneous
20
21
22
23
24
25
26
27
28
29
30
31
32
33
34
35
36
37
38
39
40

~
41
42 F!!ck suW.nd
---------------------------------------------------------------------------------------------------------------------------------------------
EnQck
" Item: 1 of 46 111 f> Mark .<:J [::> il ~ ~
w
2
Q. ld: 1149 [ Previous Next Lab Values Notes Calculator
4
"' @ A. Common peroneal [82%]
5
6 0 B. Femoral [2%]
7 0 C. Tibial [12%]
8
9
0 D. Radial [1 %]
10 0 E. Pudendal [2%]
11 0 F. Lateral sural cutaneous [1 %]
12
13
14 Explanation: User ld:
15
16 The common peroneal nerve is the lateral branch of the sciatic nerve and originates at the superior aspect of
17 the popliteal fossa . It then traces the lateral aspect of the fossa, crosses the head of the fibula, and comes
18 into direct contact w ith the lateral neck of the fibula as it enters the leg. At this point it divides into the deep
19 peroneal nerve and superficial peroneal nerve. The deep peroneal innervates the extensor and the great
20 dorsiflexors. The superficial peroneal branch supplies the peroneal muscles and the skin of most of the toes.
21 The common peroneal nerve is particularly susceptible to damage as it traces the lateral neck of the fibula .
22 Damage typically occurs via compression or leg fracture. The result is inability to dorsiflex the foot.
23
24 (Choice B) The femoral nerve (L2-L4} is the largest nerve in the lumbar plexus and innervates the muscles of
the anterior and medial thigh. Damage to the femoral nerve w ill result in loss of knee jerk.
25
26 (Choice C) The tibial nerve is the larger of the two terminal branches of the sciatic nerve and it provides
27 motor innervation for the popliteus and the flexors of the foot. Damage of this nerve would yield difficulty w ith
28 plantar flexion .
29
30 (Choice D) The radial nerve innervates the upper extremity and damage results in w rist drop.
31
32 (Choice E) The pudendal nerve provides the majority of the motor and sensory input to the pelvic floor.
33
34 (Choice F) The lateral sural cutaneous is a cutaneous branch of the tibial nerve and supplies sensation to the
35 calf.
36
37 Educational Objective:
38 Common peroneal nerve injury as it traces the lateral aspect of the fibular neck is common . Bony fracture
39 and compression are the most common causes. Clinically this manifests as foot drop.
40

~
41
42 F!!ck
---------------------------------------------------------------------------------------------------------------------------------------------
suW.nd EnQck
1 "
Item: 2 of 46
Q. ld: 1535 [
111 f> Mark .<:J
Previous
[::>
Next
il
Lab Values
~
Notes
~
Calculator

6 A 26-year-old male presents to the ER w ith a sharp pain in his neck and shoulder. He has some breathing
7 difficulty secondary to pain w ith inspiration. His past medical history is not significant except for a mild
8 respiratory disease a few days ago. His blood pressure is 120/70 mmHg and his heart rate is 100/min. The
9 pain experienced by this patient is most likely carried by w hich of the follow ing nerves?
10
11
12 0 A. Phrenic
13 0 B. Accessory
14
15 0 C. Vagus
16 0 D. Superior epigastric
17
18 0 E. Long thoracic
19
20
21
22
23
24
25
26
27
28
29
30
31
32
33
34
35
36
37
38
39
40
41
42
~imeRemaini~ ~ 0
I Tuto' Feedback Suspend End Block
---------------------------------------------------------------------------------------------------------------------------------------------
1 "
Item: 2 of 46
Q. ld: 1535 [
111 f> Mark .<:J
Previous
[::>
Next
il
Lab Values
~
Notes
~
Calculator

~ @ A. Phrenic [72%]
6 0 B. Accessory [13%]
7 0 C. Vagus [9%]
8
9 0 D. Superior epigastric [ 1%]
10 0 E. Long thoracic [5%]
11
12
13 Explanation: User ld:
14
Pleuritic chest pain can result from any condition that causes inflammation of the pleura . The pleura is divided
15
16 into segments, as follow s:
17 1. Visceral pleura: The visceral pleura, or pulmonary pleura, covers all surfaces of the lungs, including
18 the surfaces w ithin the pulmonary fissures.
19 2. Parietal pleura: This represents the remainder of the pleura that is not in contact w ith the lungs and
20 can be subdivided as follow s:
21
22 Costal pleura: Covers the thoracic w all including the ribs, sternum, intercostal spaces, costal
23 cartilages, and the sides of the thoracic vertebrae.
24 Mediastinal pleura: Covers the mediastinum
25 Diaphragmatic pleura: Covers the surface of the diaphragm located w ithin the thoracic cavity
26 Cervical pleura: Extends w ith the apices of the lung into the neck.
27
28
29 The parietal pleura is innervated by somatic sensory (sensory afferent) nerves, w hich allow the sensation of
30 sharp and localized pain . The phrenic nerve, w hich is derived from the C3-C5 nerve roots, delivers motor
31 innervation to the diaphragm and additionally carries pain fibers from the diaphragmatic and mediastinal
32 pleura . Irritation of the pleura in either of these areas w ill cause a sharp pain w orsened by inspiration that w ill
33 be "referred" to the C3-C5 distribution, w hich lies at the base of the neck and over the shoulder. Sensory
34 innervation of the remainder of the parietal pleura is accomplished by intercostal nerves.
35
36 (Choice B) The spinal accessory nerve is the 11 '" cranial nerve and provides motor innervation to the
sternocleidomastoid and trapezius muscles.
37
38
(Choice C) The vagus nerve is the 101" cranial nerve and is the major source of parasympathetic innervation
39
to the viscera of the chest w all and the foregut.
40
41
42
~imeRemaini~ ~ 0
I Tuto' Feedback Suspend End Block
---------------------------------------------------------------------------------------------------------------------------------------------
1 "
Item: 2 of 46
Q. ld: 1535 [
111 f> Mark .<:J
Previous
[::>
Next
il
Lab Values
~
Notes
~
Calculator
into segments, as follow s:
1. Visceral pleura: The visceral pleura, or pulmonary pleura, covers all surfaces of the lungs, including
6
the surfaces w ithin the pulmonary fissures.
7
2. Parietal pleura: This represents the remainder of the pleura that is not in contact w ith the lungs and
8
can be subdivided as follow s:
9
10 Costal pleura: Covers the thoracic w all including the ribs, sternum, intercostal spaces, costal
11 cartilages, and the sides of the thoracic vertebrae.
12 Mediastinal pleura: Covers the mediastinum
13 Diaphragmatic pleura: Covers the surface of the diaphragm located w ithin the thoracic cavity
14 Cervical pleura: Extends w ith the apices of the lung into the neck.
15
16
17 The parietal pleura is innervated by somatic sensory (sensory afferent) nerves, w hich allow the sensation of
18 sharp and localized pain. The phrenic nerve, w hich is derived from the C3-C5 nerve roots, delivers motor
19 innervation to the diaphragm and additionally carries pain fibers from the diaphragmatic and mediastinal
20 pleura . Irritation of the pleura in either of these areas w ill cause a sharp pain w orsened by inspiration that w ill
21 be "referred" to the C3-C5 distribution, w hich lies at the base of the neck and over the shoulder. Sensory
22 innervation of the remainder of the parietal pleura is accomplished by intercostal nerves.
23
24 (Choice B) The spinal accessory nerve is the 11 '" cranial nerve and provides motor innervation to the
25 sternocleidomastoid and trapezius muscles.
26
27 (Choice C) The vagus nerve is the 101" cranial nerve and is the major source of parasympathetic innervation
28 to the viscera of the chest w all and the foregut.
29
30 (Choice D) There is no superior epigastric nerve. The superior epigastric artery and vein course over the
31 superior half of the abdominal w all and anastomose w ith the internal thoracics and the inferior epigastrics.
32
(Choice E) The long thoracic nerve innervates the serratus anterior. Damage to this nerve causes w inged
33
34 scapula .
35 Educational Objective:
36 Irritation of the mediastinal or diaphragmatic parietal pleura w ill cause sharp pain, w orse on inspiration, in the
37 C3-C5 distribution. Pain sensation from these areas is carried by the phrenic nerve.
38
39
Time Spent: 1 seconds Copyright USMLEW orld,LLC. Last updated: [7/7/201 0]
40
41
42
~imeRemaini~ ~ 0
I Tuto' Feedback Suspend End Block
---------------------------------------------------------------------------------------------------------------------------------------------
Item: 3 of 46
Q. ld: 8332 [
111 f> Mark .<:J
Previous
[::>
Next
il
Lab Values
~
Notes
~
Calculator

5
6 The following vignette applies to the next 2 items.
7
8 A 52-year-old man is brought to the emergency department for evaluation of fever, chills, and malaise.
9 Cardiac examination reveals a new holosystolic heart murmur that radiates tow ard the axilla . Blood cultures
10 are obtained and he undergoes transesophageal echocardiography. The ultrasound probe is placed in the
11 mid-esophagus facing anteriorly and cardiac chambers are interrogated.
12
13 Item 1 of 2
14
Anatomically, w hich of the follow ing chambers is closest to the probe?
15
16
17 0 A. Left atrium
18
0 B. Left ventricle
19
20 0 C. Right atrium
21 0 D. Right ventricle
22
23
24
25
26
27
28
29
30
31
32
33
34
35
36
37
38
39
40
41
42
~imeRemaini~ ~ 0
I Tuto' Feedback Suspend End Block
---------------------------------------------------------------------------------------------------------------------------------------------
Item: 3 of 46
Q. ld: 8332 [
111 f> Mark .<:J
Previous
[::>
Next
il
Lab Values
~
Notes
~
Calculator

"' @ A. Left atrium [75%]


5
6 0 B. Left ventricle [6%]
7 0 C. Right atrium [13%]
8
0 D. Right ventricle [6%]
9
10
11 Explanation: User ld:
12
13 Transesophageal echocardiography (TEE)
14
15
16
17
18
Trach~ TEE probe
19
20
21
22
23
24 Aortic arch
25
26
27
28
29 Pulmonary trunk
30
31
32
_ _ Descending
33
34 aorta
35
36
37
38
39
40
41
42
Item: 3 of 46
Q. ld: 8332 [
111 f> Mark .<:J
Previous
[::>
Next
il
Lab Values
~
Notes
~
Calculator

Transesophageal echocardiography (TEE)


5
6
7
8
9 Trach~ TEE probe
10
11
12
13
14
15
Aortic arch
16
17
18
19
20
Pulmonary trunk
21
22
23
24 _ _ Descending
25 aorta
26
27
28
29
30
31
32
33
34
35 Ventricular
36 septum
37
38
39
40 Mitral valve
41
42
~imeRemaini~ ~ 0
I Tuto' Feedback Suspend End Block
---------------------------------------------------------------------------------------------------------------------------------------------
Item: 3 of 46
Q. ld: 8332 [
111 f> Mark .<:J
Previous
[::>
Next
il
Lab Values
~
Notes
~
Calculator

This patient's presentation suggests acute endocarditis, w ith confirmation pending the results of the blood
5 cultures and cardiac imaging. Transesophageal echocardiography (TEE) uses ultrasound w aves generated
6 from w ithin the esophagus to produce clear images of the neighboring cardiac structures. The left atrium
7 makes up the majority of the heart's posterior surface, w ith the esophagus passing immediately posterior to
8 the heart. Therefore, the esophagus lies w ithin closest proximity to the left atrium . This allow s the left
9 atrium, atrial septum, and mitral valve to be particularly w ell visualized on TEE. Due to its adjacent proximity,
10 conditions that result in left atrial enlargement (eg, atrial fibrillation, mitral stenosis) can cause dysphagia
11 through external compression of the esophagus.
12
13
14
15
16
17
18
19
20
21
22
23
24
25
26
27
28
29
30
31
32
33
34
35
36
37
38
39
40
41
42
5
6
7
8
9
10
11
12
13
14
15
16
17
18
19
20
21
22
23
24
25
26
27
28 (Choice B) The left ventricle forms the apex and left border of the heart on frontal chest x-ray.
29
30 (Choice C) The right atrium, along w ith the superior vena cava, forms the right lateral cardiac border on
31 frontal chest x-ray.
32
(Choice D) The right ventricle forms the anterior (sternal} surface of the heart and the majority of its inferior
33
34 border.
35 Educational objective:
36 The left atrium forms the majority of the posterior surface of the heart and resides adjacent to the esophagus.
37 Enlargement of the left atrium can compress the esophagus and cause dysphagia .
38
39
Time Spent: 2 seconds Copyright USMLEW orld,LLC. Last updated: [1 / 14/2014]
40
41
42
~imeRemaini~ ~ 0
I Tuto' Feedback Suspend End Block
--------------------------------------------------------------------------------------------------------------------------------------
1
2
"
Item: 4 of 46
Q. ld: 8333 [
111 f> Mark .<:J
Previous
[::>
Next
il
Lab Values
~
Notes
~
Calculator

5
6 Item 2 of 2
7
8 In the same mid-esophageal position, the probe is rotated so that it now faces posteriorly. Anatomically,
9 w hich of the follow ing w ill be best interrogated in this position?
10
11
12
0 A. Descending aorta
13 0 B. Pulmonary artery
14
15 0 C. Pulmonary veins
16 0 D. Superior vena cava
17
18 0 E. Tricuspid valve
19
20
21
22
23
24
25
26
27
28
29
30
31
32
33
34
35
36
37
38
39
40
41
42
~imeRemaini~ ~ 0
I Tuto' Feedback Suspend End Block
---------------------------------------------------------------------------------------------------------------------------------------------
1
2
"
Item: 4 of 46
Q. ld: 8333 [
111 f> Mark .<:J
Previous
[::>
Next
il
Lab Values
~
Notes
~
Calculator

ol @ A. Descending aorta [69%]


5
6 0 B. Pulmonary artery [8%]
7 0 C. Pulmonary veins [1 0%]
8 0 D. Superior vena cava [11 %]
9
10 0 E. Tricuspid valve [2%]
11
12
Explanation: User ld:
13
14 Transesophageal echocardiography (TEE)
15
16
17
18
Trachea
19 TEE probe
20
21
~
22 Esophagus
23
24
25 Aortic arch
26
27
28
29
30 Pulmonary trunk
31
32
33 Pulmonary valve
34 _ _ Descending
35 aorta
36
37
38
39
40
41
42
1
2
"
Item: 4 of 46
Q. ld: 8333 [
111 f> Mark .<:J
Previous
[::>
Next
il
Lab Values
~
Notes
~
Calculator

Transesophageal echocardiography (TEE)


5
6
7
8
Trachea
9 TEE probe
10
11 ~
12 Esophagus
13
14
15 Aortic arch
16
17
18
19
20
21
22
23
Pulmonary valve _ _ Descending
24
25 aorta
26
27
28
29 atrium
30
31
32
33
34
Ventricular
35
septum
36
37
38
Left ven1tricle
39 Mitral valve
40
41
42
~imeRemaini~ ~ 0
I Tuto' Feedback Suspend End Block
---------------------------------------------------------------------------------------------------------------------------------------------
1
2
"
Item: 4 of 46
Q. ld: 8333 [
111 f> Mark .<:J
Previous
[::>
Next
il
Lab Values
~
Notes
~
Calculator

The aorta has four major divisions: the ascending aorta, the aortic arch, the descending thoracic aorta, and
5 the abdominal aorta . The ascending aorta lies posterior and to the right of the main pulmonary artery. The
6 aortic arch travels above the right pulmonary artery and the left bronchus. The brachiocephalic, left common
7 carotid, and left subclavian arteries originate (in that order) from its superior aspect. The descending thoracic
8 aorta abuts the left anterior surface of the vertebral column and lies posterior to the esophagus and the left
9 atrium . This permits clear visualization of the descending aorta during transesophageal echocardiography
10 (TEE), allow ing for the detection of abnormalities such as dissection or aneurysm . The descending thoracic
11 aorta continues to travel dow n the left anterior surface of the vertebral column, becoming the abdominal aorta
12 as it crosses the diaphragm and eventually branching into the common iliac arteries.
13
14
15
16
17
18
19
20
21
22
23
24
25
26
27
28
29
30
31
32
33
34
35
36
37
38
39
40
41
42
1
2
"
Item: 4 of 46
Q. ld: 8333 [
111 f> Mark .<:J
Previous
[::>
Next
il
Lab Values
~
Notes
~
Calculator

5
6
7
8
9
10
11
12
13
14
15
16
17
18
19
20
21
22
23
24
25
26
27
28
29
30
31
32
33
34
35
36
37
38
39
40 (Choice B) The main pulmonary artery ascends anteriorly and to the left of the ascending aorta and is
41
42
~imeRemaini~ ~ 0
I Tuto' Feedback Suspend End Block
---------------------------------------------------------------------------------------------------------------------------------------------
1
2

5
6
7
8
9
10
11
12
13
14
15
16
17
18 (Choice B) The main pulmonary artery ascends anteriorly and to the left of the ascending aorta and is
19 directed tow ard the left shoulder. After the pulmonary artery bifurcates, the right pulmonary artery travels
20 horizontally under the aortic arch posterior to the superior vena cava and the left pulmonary artery courses
21 superiorly over the left main bronchus.
22
23 (Choice C) The superior and inferior pulmonary veins arise bilaterally from each lung and enter the left
24 atrium. The proximal 2-3 em of the pulmonary veins contain cardiac muscle w ithin the media and function like
25 sphincters during atrial systole.
26
27 (Choice D) The superior vena cava (SVC) is formed behind the right 1., costal cartilage by the merger of the
28 right and left brachiocephalic veins. It returns blood from the head, neck, and upper extremities to the right
29 atrium of the heart. Mediastinal neoplasms can compress the SVC and result in SVC syndrome.
30
(Choice E) The tricuspid valve is located between the right atrium and right ventricle and is composed of 3
31
valve leaflets, the annulus, supporting chordae tendineae, and the papillary muscles. It is commonly infected
32
(endocarditis) in IV drug users.
33
34 Educational objective:
35 The descending thoracic aorta lies posterior to the esophagus and the left atrium . This position permits clear
36 visualization of the descending aorta by transesophageal echocardiography, allow ing for the detection of
37 abnormalities such as dissection or aneurysm .
38
39
Time Spent: 1 seconds Copyright USMLEW orld,LLC. Last updated: [1 / 14/2014]
40

~
41
42 F!!ck suW.nd EnQck
---------------------------------------------------------------------------------------------------------------------------------------------
1
2
" Item: 5 of 46
Q. ld: 1538 [
111 f> Mark .<:J
Previous
[::>
Next
il
Lab Values
~
Notes
~
Calculator
'""3
4

!!..J!L=l
6
. 7 A 63-year-old male w ith a history of gastrointestinal bleed has developed a sudden onset shortness of breath.

. Evaluation show s pulmonary embolism . A filter placement is planned to prevent further embolization. The
. 89 filter w ill most likely be placed in w hich of the follow ing structures?
. 10
. 11
.
. 1213
. 14
. 15
.
. 1617
. 18
. 19
.
. 2021
. 22
. 23
. 24
. 25
. 26
. 27
.
. 2829
. 30
. 31
.
. 32
33 0 A. A
. 34 0 B. B
. 35
. 0 C. C
. 3637 0 D. D
. 38 0 E. E
. 39 ~

. 40
. 41
. 42 ~ F!!ck
---------------------------------------------------------------------------------------------------------------------------------------------
suW.nd EnQck
1
2
" Item: 5 of 46
Q. ld: 1538 [
111 f> Mark .<:J
Previous
[::>
Next
il
Lab Values
~
Notes
~
Calculator
'""3
4
0 A.A [18%]
!!..J!L=l
6 .; @ B. B [61%]
. 7 0 C. C [13%]
.
. 89 0 D. D [5%]
. 10 0 E. E [2%]
. 11
.
. 1213 Explanation: User ld:
. 14
. 15 The IVC filter is designed to prevent the travel of deep vein thrombosis from the legs to the lung vasculature.
. Inferior vena cava filter is used to prevent pulmonary embolism in patients who have contraindications to
. 1617 anticoagulation. This abdominal computed tomography (CT) scan is taken at the level of L2. The bilateral
. 18 kidneys are viewed in section; the inferior pole of the right lobe of the liver is noted on the left side of the above
. 19 image anterior to the right kidney and posterolateral to the large bowel.
.
. 2021 Choice A represents the lumen of the second part of the duodenum, which classically lies at the level of L2.

. 22 Choice B is medial to the descending part of the duodenum and lies anterior to the right side of the vertebral
. 23 body. This structure is the inferior vena cava . The right renal vein drains into the IVC near this plane of
. 24 section.
. 25
. 26 Choice C represents the abdominal aorta. The bifurcation of the abdominal aorta occurs at the level of L4.
. 27
. Choice D, if observed carefully, looks like a figure eight. This structure represents two segments of the ileum
. 2829 lying next to one another.
. 30
. 31 Choice E represents the left renal vein. The left renal vein receives drainage from the left gonadal vein and
. courses anterior to the aorta, but posterior to the superior mesenteric artery, before draining into the inferior
. 32
33
vena cava .
. 34
. 35 Educational Objective:
. The inferior vena cava courses through the abdomen and inferior thorax in a location anterior to the right half
. 3637 of the vertebral bodies. The renal veins join the IVC at the level of L1/L2, and the common iliac veins merge to
. 38 become the IVC at the level of L4.
. 39 ~

. 40 Time Spent: 6 seconds Copyright USMLEWorld,LLC. Last updated: [7/7/201 OJ


. 41 ~imeRemaini~ ~ 0
. 42 I Tuto' Feedback Suspend End Block
---------------------------------------------------------------------------------------------------------------------------------------------
1
2
" Item: 6 of 46
Q. ld: 1737 [
111 f> Mark .<:J
Previous
[::>
Next
il
Lab Values
~
Notes
~
Calculator

CJ
5
A 63-year-old male complains of having to "strain" on urination. The patient feels much better several w eeks
7 after starting doxazosin. Involvement of w hich of the follow ing structures is most likely responsible for this
8 patient's symptoms?
9
10
11
12
13
14
15
16
17
18
19
20
21
22
23
24
25
26
27
28
29 0 A. A
30 0 B. B
31
32 0 C. C
33 0 D. D
34 0 E. E
35
36 0 F. F
37
38
39
40
41
42
~imeRemaini~ ~ 0
I Tuto' Feedback Suspend End Block
---------------------------------------------------------------------------------------------------------------------------------------------
1
2
" Item: 6 of 46
Q. ld: 1737 [
111 f> Mark .<:J
Previous
[::>
Next
il
Lab Values
~
Notes
~
Calculator

CJ
5
.; @ A. A [67%]
0 B. B [8%]
7 0 C. C [18%]
8 0 D. D [5%]
9
10
0 E. E [2%]
11 0 F. F[1%]
12
13
Explanation: User ld:
14
15 The patient described in the question stem is suffering from symptoms of benign prostatic hyperplasia
16 (BPH). On autopsy, this condition is present in >90% of men aged eighty and above. Urinary complaints are
17 the primary symptoms in BPH, including urinary hesitancy, urgency, frequency, incomplete voiding, post-void
18 leakage of urine, and nocturia. Diagnosis is made from the history and physical, including the digital rectal
19
exam.
20
21 Medical treatment of BPH is w ith a-adrenergic blockers, w hich relax the smooth muscle in the bladder neck,
22 or w ith 5-a reductase inhibitors, w hich reduce hormonal influence on the prostate by preventing the
23 conversion of testosterone to dihydrotestosterone. In the pelvic CT "slice" given in the question stem, the
24 prostate is labeled "(A)." At this level of sectioning, the prostate is located posterior to the pubic symphysis
25 and anterior to the anal canal "(B)."
26
27 (Choice C) C is the distal end of bladder.
28
29 (Choice D) D labels the ischioanal fossa .
30
31 (Choice E) E is the rectus abdominus muscle.
32
(Choice F) F is the gluteus maximus muscle.
33
34 Educational Objective:
35 BPH is a common, age-related condition that causes urinary symptoms and can be medically treated w ith a-
36 adrenergic blockers or 5-a reductase inhibitors. The prostate is located between the pubic symphysis and the
37 anal canal in inferior sections of the pelvis on CT scan.
38
39
Time Spent: 5 seconds Copyright USMLEW orld,LLC. Last updated: [8/ 10/201 0]
40
41
42
~imeRemaini~ ~ 0
I Tuto' Feedback Suspend End Block
--------------------------------------------------------------------------------------------------------------------------------------
1
2
" Item: 7 of 46
Q. ld: 8671 [
111 f> Mark .<:J
Previous
[::>
Next
il
Lab Values
~
Notes
~
Calculator

CJ
5
6 A 65-year-old man comes to the physician because of right low er abdominal and groin pain and difficulty
w alking for the past w eek. Climbing stairs has been particularly difficult for him, and he has fallen several
8 times due to "knee buckling." His past medical history is significant for diabetes mellitus, hypertension, and
9 atrial fibrillation . He takes w arfarin for chronic anticoagulation. On physical examination, the right patellar
10 reflex is decreased compared to the left. His abdominal CT is show n below.
11
12
13
14
15
16
17
18
19
20
21
22
23
24
25
26
27
28
29
30
31
32
33 W hich of the follow ing additional findings w ould you most expect on physical examination?
34
35
36
0 A. Impaired adduction of the right thigh
37 0 B. Loss of sensation on the anterior aspect of the right thigh
38 0 C. Loss of sensation on the dorsum of the right foot
39
40
0 D. Loss of sensation on the sole of the right foot
41
42
~imeRemaini~ ~ 0
I Tuto' Feedback Suspend End Block
---------------------------------------------------------------------------------------------------------------------------------------------
1
2
" Item: 7 of 46
Q. ld: 8671 [
111 f> Mark .<:J
Previous
[::>
Next
il
Lab Values
~
Notes
~
Calculator

CJ
5
.... ......... "'' ~...... '""'"""'"*"'~ ........... u .................... ~ .... ............. , ....... '''""''-'~'"' "I"'"'"'~ "'''1"''1'-''" ... ' '"'"._.,,,,, ... ~....... ,

6 0 A. Impaired adduction of the right thigh [ 15%1


"' @ B. l oss of sensation on the anterior aspect of the right thigh [45%1
8
9
0 C. loss of sensation on the dorsum of the right foot [14%1
10 0 D. loss of sensation on the sole of the right foot [9%1
11 0 E. Weakened abduction of the right thigh [17%1
12
13
14 Explanation: User ld:
15
16
17 Lower extremity Nerve
Cause of injury Sensory deficit Motor deficit
18 nerve roots
19
20
21
22
23 Anterior hip dislocation,
24 ) Thigh adduction
Obturator L2L4 iatrogenic
25 (Choice A)
26 (eg, pelvic surgery)
27
28
29
30
31
32
33
34 Pelvic fracture or mass
35 involving iliopsoas/ iliacus Flexion of thigh,
Femoral L2L4
36 muscle (eg, hematoma or extension of leg
37 abscess)
38
39
40
41
42
1
2
" Item: 7 of 46
Q. ld: 8671 [
111 f> Mark .<:J
Previous
[::>
Next
il
Lab Values
~
Notes
~
Calculator

CJ
5 Lower extremity Nerve
6 Cause of injury Sensory deficit Motor deficit
nerve roots

8
9
10 t
11
12 Anterior hip dislocation, ) Thigh adduction
13 Obturator L2L4 iatrogenic
14 (Choice A)
(eg, pelvic surgery)
15
16
17
18
19
20 r;:;, ~
I
21
22 Pelvic fracture or mass I
23
24 Femoral L2L4
involving iliopsoas/iliacus .~ Flexion of thigh,

25
26
muscle (eg, hematoma or
abscess) [ extension of leg

27
28
-
29
\:...
30
\
31 I
32
33 1\
34 Fibula neck fract ure or \ Foot eversion,
35 Common
L4S2 nerve compression at l dorsiflexion,
36 peroneal
37 fibular neck toe extension
38
39
40
~~
41
42
~imeRemaini~ ~ 0
I Tuto' Feedback Suspend End Block
---------------------------------------------------------------------------------------------------------------------------------------------
1
2

CJ
5 Tibial L4-S3 Trauma to the knee plantar flexion
6 & toe flexion

8
9
10 (Choice D)
11 Thigh abduction
12 Superior gluteal L4-S1 Iatrogenic (eg, posterior None
(Choice E)
13 hip dislocation or buttocks
14
Inferior gluteal LS-52 injection) None Thigh extension
15
16
()USMLWottd, LLC
17
18
19 The CT image reveals a large fluid collection in the right retroperitoneum lying anterior to the psoas muscle.
20 The fluid is isodense w ith muscle and displaces the right kidney anteriorly. These findings are consistent w ith
21 a spontaneous retroperitoneal hematoma, most likely secondary to w arfarin use. The risk of bleeding w hile on
22 w arfarin therapy is greatest in patients w ith risk factors such as increased age, diabetes mellitus,
23 hypertension, and alcoholism.
24
The femoral nerve descends through the fibers of the psoas major muscle, emerges laterally between the
25
psoas and iliacus muscle, and then runs beneath the inguinal ligament into the thigh. Femoral nerve
26
mononeuropathy can occur due to trauma (eg, pelvic fracture), compression from a hematoma or abscess,
27
stretch injury, or ischemia. Patients w ith femoral neuropathy develop w eakness involving the quadriceps
28
muscle group and may have w eakening of the iliopsoas w ith more proximal nerve injuries. They often
29
complain of difficulty w ith stairs and frequent falling secondary to "knee buckling." On examination, the patellar
30
reflex is generally diminished. In addition, sensory loss over the anterior and medial thigh and medial leg is
31
typical. Acute, severe pain in the groin, lower abdomen, or back may also occur if the neuropathy is caused
32
by a retroperitoneal hematoma.
33
34 Educational objective:
35 Lesions of the femoral nerve can occur due to trauma, nerve compression, stretch injury, or ischemia.
36 Patients develop w eakness of the quadriceps muscle, loss of the patellar reflex, and loss of sensation over
37 the anterior and medial thigh and medial leg.
38
39
Time Spent: 10 seconds Copyright USMLEWorld,LLC. Last updated: [2112/2014]
40
41
42
~imeRemaini~ ~ 0
I Tuto' Feedback Suspend End Block
---------------------------------------------------------------------------------------------------------------------------------------------
1
2
" Item: 8 of 46
Q. ld: 1904 [
111 f> Mark .<:J
Previous
[::>
Next
il
Lab Values
~
Notes
~
Calculator

CJ
5
6 A 16-year-old male presents to your office w ith difficulty w alking. On physical examination, he has thoracic
7 kyphoscoliosis and multiple small scars on his hands. W hen asked about these, he reports that he
repeatedly accidentally burns his hands because he "cannot feel the heat." Neurologic examination reveals
9 1+ biceps reflexes and 3+ patellar reflexes bilaterally, as w ell as decreased muscle strength in the bilateral
10 upper and low er extremities. W hich of the follow ing is the most likely cause of this patient's problems?
11
12
13 0 A. Amyotrophic lateral sclerosis
14
0 B. Guillain-Barre syndrome
15
16 0 C. Syringomyelia
17 0 D. Normal pressure hydrocephalus
18
19 0 E. Multiple sclerosis
20 0 F. Medulloblastoma
21
22
23
24
25
26
27
28
29
30
31
32
33
34
35
36
37
38
39
40
41
42
~imeRemaini~ ~ 0
I Tuto' Feedback Suspend End Block
---------------------------------------------------------------------------------------------------------------------------------------------
1
2
" Item: 8 of 46
Q. ld: 1904 [
111 f> Mark .<:J
Previous
[::>
Next
il
Lab Values
~
Notes
~
Calculator

CJ
5
" @ C. Syringomyelia [63%]

6 0 D. Normal pressure hydrocephalus [1%]


7 0 E. Multiple sclerosis [5%]
0 F. Medulloblastoma [3%]
9
10
11 Explanation: User ld:
12
13 This patient has chronic loss of upper extremity pain and temperature sensation, upper extremity w eakness
14 and hyporeflexia, low er extremity w eakness and hyperreflexia, and kyphoscoliosis. These findings are
15 consistent w ith a diagnosis of syringomyelia.
16
17 In syringomyelia, a central cystic dilation in the cervical spinal cord (a syrinx) slow ly enlarges,
18 characteristically causing damage to the ventral w hite commissure and anterior horns. The ventral w hite
19 commissure is the site of decussation of second-order lateral spinothalamic tract neurons, and the ventral
20 horns are the site of low er motor neuron cell bodies. The syrinx is most commonly situated at the C8-T1 cord
21 levels and may extend rostrally, caudally, and centrifugally. Further expansion w ithin the cervical cord in later
22 stages of the disease can produce low er extremity w eakness and hyperreflexia (upper motor neuron defect)
23 by affecting the lateral corticospinal tract.
24
25 (Choice A) ALS is a progressive degenerative disease of upper and low er motor neurons only; it does not
26 cause sensory loss .
27
(Choice B) Guillain-Barre syndrome is a peripheral nervous system autoimmune demyelinating
28
polyneuropathy that typically causes low er motor neuron manifestations (e.g. areflexia, w eakness) in a
29
symmetric ascending pattern.
30
31 (Choice D) Normal pressure hydrocephalus is characterized by gait apraxia and urinary incontinence. It is
32 rare in patients younger than 60 years of age. It arises due to defective CSF resorption by the arachnoid
33 granulations.
34
35 (Choice E) Multiple sclerosis is an autoimmune CNS demyelinating disorder that presents w ith episodic
36 neurologic symptoms reflecting defects in random w hite matter tracts.
37
38 (Choice F) Medulloblastoma is the most common malignant brain tumor of children. It is a posterior fossa
39 tumor that causes symptoms related to obstructive hydrocephalus.
40
41
42
~imeRemaini~ ~ 0
I Tuto' Feedback Suspend End Block
---------------------------------------------------------------------------------------------------------------------------------------------
1
2
" Item: 8 of 46
Q. ld: 1904 [
111 f> Mark .<:J
Previous
[::>
Next
il
Lab Values
~
Notes
~
Calculator

CJ
5 Explanation: User ld:
6
This patient has chronic loss of upper extremity pain and temperature sensation, upper extremity w eakness
7
and hyporeflexia, low er extremity w eakness and hyperreflexia, and kyphoscoliosis. These findings are
consistent w ith a diagnosis of syringomyelia.
9
10 In syringomyelia, a central cystic dilation in the cervical spinal cord (a syrinx) slow ly enlarges,
11 characteristically causing damage to the ventral w hite commissure and anterior horns. The ventral w hite
12 commissure is the site of decussation of second-order lateral spinothalamic tract neurons, and the ventral
13 horns are the site of low er motor neuron cell bodies. The syrinx is most commonly situated at the C8-T1 cord
14 levels and may extend rostrally, caudally, and centrifugally. Further expansion w ithin the cervical cord in later
15 stages of the disease can produce low er extremity w eakness and hyperreflexia (upper motor neuron defect)
16 by affecting the lateral corticospinal tract.
17
18 (Choice A) ALS is a progressive degenerative disease of upper and low er motor neurons only; it does not
19 cause sensory loss.
20
21 (Choice B) Guillain-Barre syndrome is a peripheral nervous system autoimmune demyelinating
22 polyneuropathy that typically causes low er motor neuron manifestations (e.g. areflexia, w eakness) in a
23 symmetric ascending pattern.
24
25 (Choice D) Normal pressure hydrocephalus is characterized by gait apraxia and urinary incontinence. It is
26 rare in patients younger than 60 years of age. It arises due to defective CSF resorption by the arachnoid
27 granulations.
28
29 (Choice E) Multiple sclerosis is an autoimmune CNS demyelinating disorder that presents w ith episodic
30 neurologic symptoms reflecting defects in random w hite matter tracts.
31
(Choice F) Medulloblastoma is the most common malignant brain tumor of children. It is a posterior fossa
32
tumor that causes symptoms related to obstructive hydrocephalus.
33
34 Educational Objective:
35 The combination of fixed segmental loss of upper extremity pain and temperature sensation, upper extremity
36 low er motor neuron signs, and/or low er extremity upper motor neuron signs in the setting of scoliosis
37 suggests a diagnosis of syringomyelia .
38
39
Time Spent: 2 seconds Copyright USMLEW orld,LLC. Last updated: [7/7/201 0]
40
41
42
~imeRemaini~ ~ 0
I Tuto' Feedback Suspend End Block
---------------------------------------------------------------------------------------------------------------------------------------------
1
2
" Item: 9 of 46
Q. ld: 1738 [
111 f> Mark .<:J
Previous
[::>
Next
il
Lab Values
~
Notes
~
Calculator

CJ
5
6 A 48-year-old male presents to your office w ith recent w eight loss and vague abdominal pain . He admits to
7 drinking alcohol regularly, but denies any past medical problems. A 72-hour stool collection reveals excessive
8 excretion of neutral fat. Involvement of w hich of the follow ing structures is most likely responsible for this
patient's symptoms?
10
11
12
13
14
15
16
17
18
19
20
21
22
23
24
25
26
27
28
29
30
31
32
33
34 0 A. A
35 0 B. B
36
0 C. C
37
38 0 D. D
39 0 E. E
40

~
41
42 F!!ck
---------------------------------------------------------------------------------------------------------------------------------------------
suW.nd EnQck
1
2
" Item: 9 of 46
Q. ld: 1738 [
111 f> Mark .<:J
Previous
[::>
Next
il
Lab Values
~
Notes
~
Calculator

CJ
5 II @ B. B [87%]
6 0 C. C [1 %]
7
0 0 . 0 [1 %]
8
0 E. E [8%]
10
11
Explanation: User ld:
12
13 The patient described in the question stem is suffering from exocrine pancreatic insufficiency, w hich is
14 evidenced by his body's inability to normally metabolize fat. People w ho suffer from chronic pancreatitis, often
15 caused by alcoholism, have severe abdominal pain, stool that floats in the toilet bow l (due to excess fat
16 content), and w eight loss.
17
18 The head, neck, and body of the pancreas are retroperitoneal, w hile the tail is peritoneal. The head of the
19 pancreas lies in the curve of the duodenum and overlies the L2 vertebra . The body overlies the L1 and L2
20 vertebrae and makes contact posteriorly w ith the aorta, the superior mesenteric vessels, and the left kidney.
21 The tail of the pancreas courses w ithin the splenorenalligament alongside the splenic vessels.
22
23 (Choice A) This structure is the second (descending) part of the duodenum.
24
25 (Choice C) This structure is the inferior vena cava (IVC). The IVC lies on the right side of the vertebrae in a
26 retroperitoneal position .
27
28 (Choice D) This structure labels the abdominal aorta . The aorta in this region lies on the left side of the
29 vertebrae in a retroperitoneal position .
30
(Choice E) These structures are loops of the jejunum. At this level in the abdominal cavity, loops of small
31
bow el represent jejunum. If this CT slice w ere of the inferior (low er) abdomen, the small bow el w ould more
32
likely be the ileum.
33
34 Educational Objective:
35 On abdominal CT scans, the pancreas can be identified by its head in close association w ith the second part
36 of the duodenum; by its body overlying the left kidney, aorta, IVC and superior mesenteric vessels; and also by
37 the tail lying in the splenorenal ligament.
38
39
Time Spent: 7 seconds Copyright USMLEW orld,LLC. Last updated: [7/7/201 0]
40

~
41
42 F!!ck
---------------------------------------------------------------------------------------------------------------------------------------------
suW.nd EnQck
1
2
" Item: 10 of 46
Q.ld: 8564 [
lil f> Mark .<:J
Previous
[::>
Next
il
Lab Values
~
Notes
~
Calculator

CJ
5
6 A 4-day-old premature infant in the neonatal intensive care unit develops a decreased level of consciousness
7 and hypotonia . She w as delivered vaginally at 30 w eeks of gestation and her birth w eight w as 1200 g (2 lb 10
8 oz). Physical examination reveals a lethargic infant w ith a w eak and high-pitched cry, prominent scalp veins,
9 and tense fontanels. Cranial ultrasound reveals blood in the lateral ventricles. W hich of the follow ing
structures is the most likely source of the bleeding?
11
12
13 0 A. Bridging cortical veins
14
0 B. Germinal matrix
15
16 0 C. Meningeal arteries
17 0 D. Sagittal sinus
18
19 0 E. Vessels of the circle of W illis
20
21
22
23
24
25
26
27
28
29
30
31
32
33
34
35
36
37
38
39
40

~
41
42 F!!ck
---------------------------------------------------------------------------------------------------------------------------------------------
suW.nd EnQck
1
2
" Item: 10 of 46
Q.ld: 8564 [
lil f> Mark .<:J
Previous
[::>
Next
il
Lab Values
~
Notes
~
Calculator

CJ
5
.; @ B. Germinal matrix [40%]

6 0 C. Meningeal arteries [8%]


7 0 D. Sagittal sinus [1 8%]
8
0 E. Vessels of the circle of Willis [1 6%]
9

11 Explanation: User ld:


12
13 Germinal matrix hemorrhage
14
15
16
17
Hemorrhage
18
extending into
19
20 lateral ventricle
21 Lateral
22 ventricle
23
24
25 Germinal
26 matrix -~--------./
27
28
29
30
31
32
33
34
35
36
37
38
39
40
41
42
~imeRemaini~ ~ 0
I Tuto' Feedback Suspend End Block
--------------------------------------------------------------------------------------------------------------------------------------
1
2
" Item: 10 of 46
Q.ld: 8564 [
lil f> Mark .<:J
Previous
[::>
Next
il
Lab Values
~
Notes
~
Calculator

CJ
5
<!>USMLEWO<ki.LLC

6 Intraventricular hemorrhage (IVH) is a common complication of prematurity that can lead to long-term
7 neurodevelopmental impairment. It occurs most frequently in infants born before 32 w eeks gestation and/or
8 w ith birth w eight < 1500 g, and almost always occurs w ithin the first 5 postnatal days. IVH in the new born can
9 be clinically silent or present w ith an altered level of consciousness, hypotonia, and decreased spontaneous
movements. Symptoms of catastrophic bleeding include a bulging anterior fontanelle, hypotension,
11 decerebrate posturing, tonic-clonic seizures, irregular respirations, and coma .
12
13 IVH in preterm infants usually originates from the germinal matrix, a highly cellular and vascularized layer in
14 the subventricular zone from w hich neurons and glial cells migrate out during brain development. The matrix
15 contains numerous thin-w alled vessels lacking the glial fibers that support other blood vessels throughout the
16 brain, w hich contributes to the risk of hemorrhage. It is especially vulnerable to hemodynamic instability as
17 premature infants can have impaired autoregulation of cerebral blood flow . Between 24-32 w eeks of
18 gestation, the germinal matrix becomes less prominent and its cellularity and vascularity decrease, reducing
19 the risk of IVH.
20
21 (Choice A) Subdural hematoma can result from the rupture of cortical bridging veins. In infants, this can
22 occur from violent shaking (shaken baby syndrome).
23
24 (Choice C) Epidural hematoma frequently occurs due to head trauma . It is often associated w ith temporal
25 bone fracture and subsequent tearing of the middle meningeal artery.
26
27 (Choice D) The dural venous sinuses are venous channels found between layers of dura mater in the brain.
28 Certain conditions (eg, malignancy, hypercoagulable states, and trauma) can result in thrombosis w ithin the
29 dural sinuses.
30
(Choice E) Saccular (berry) aneurysms are the most common cause of subarachnoid hemorrhage. They
31
typically occur near the circle of W illis. Berry aneurysms are associated w ith Ehlers-Danlos syndrome and
32
autosomal dominant polycystic kidney disease.
33
34 Educational objective:
35 Neonatal intraventricular hemorrhage usually occurs in the fragile germinal matrix and increases in frequency
36 w ith decreasing age and birth w eight. It is a common complication of prematurity that can lead to long-term
37 neurodevelopmental impairment.
38
39
Time Spent: 2 seconds Copyright USMLEW orld,LLC. Last updated: [1 2116/2013]
40
41
42
~imeRemaini~ ~ 0
I Tuto' Feedback Suspend End Block
---------------------------------------------------------------------------------------------------------------------------------------------
1
2
" Item: 11 of 46
Q.ld: 1491 [
111 f> Mark .<:J
Previous
[::>
Next
il
Lab Values
~
Notes
~
Calculator

CJ
5
6 A 46-year-old gardener presents to your office w ith knee pain . He reports that he spends hours on his knees
7 several times per w eek w hile gardening. W hich of the follow ing bursae do you most expect to be affected in
8 this patient?
9
10
0 A. Suprapatellar bursa
12 0 B. Prepatellar bursa
13
14 0 C. Anserine bursa
15 0 D. Semimembranous (popliteal) bursa
16
17 0 E. Gastrocnemius bursa
18
19
20
21
22
23
24
25
26
27
28
29
30
31
32
33
34
35
36
37
38
39
40
41
42
~imeRemaini~ ~ 0
I Tuto' Feedback Suspend End Block
---------------------------------------------------------------------------------------------------------------------------------------------
1
2
" Item: 11 of 46
Q.ld: 1491 [
111 f> Mark .<:J
Previous
[::>
Next
il
Lab Values
~
Notes
~
Calculator

CJ
5
A 'lo-year-ota garaener presents to your omce w nn Knee pa1n . He reports mat ne spenas nours on ms Knees
several times per w eek w hile gardening. W hich of the follow ing bursae do you most expect to be affected in
6 this patient?
7
8 0 A. Suprapatellar bursa [25%]
9
~ @ B. Prepatellar bursa [56%]
10
0 C. Anserine bursa [6%]
12 0 D. Semimembranous (popliteal) bursa [11 %]
13
14
0 E. Gastrocnemius bursa [2%]
15
16 Explanation: User ld:
17
18 The prepatellar bursa is located between the patella and the overlying skin and prepatellar tendon . It is lined
19 by synovium and contains very little fluid. Its function is to diminish friction and ensure maximal range of
20 motion at the knee. Chronic trauma from repeated kneeling is one cause of prepatellar bursitis, also called
21 "housemaid's knee." It is common in roofers, plumbers and carpet layers.
22
23 Symptoms of prepatellar bursitis include knee pain, sw elling, redness and inability to flex the knee on the
24 affected side. The symptoms are usually relieved by rest. Physical examination reveals tenderness to
25 palpation, erythema, crepitance, and fluctuant edema over the low er pole of the patella .
26
27 (Choices A, D and E) Prolonged kneeling is not associated w ith inflammation of the suprapatellar, popliteal or
28 gastrocnemius bursae. Popliteal and gastrocnemius bursitis can be associated w ith the formation of Baker's
29 cysts.
30
(Choice C) Anserine bursitis (pes anserinus bursitis) presents w ith pain along the medial aspect of the
31
knee. It results from overuse in athletes or from chronic trauma in patients w ith heavy body habitus. There is
32
typically tenderness to palpation approximately 4 em distal to the anteromedial joint margin of the knee.
33
34 Educational Objective:
35 Repeated and prolonged kneeling can cause prepatellar bursitis. Dubbed "housemaid's knee," today it is
36 most commonly seen in roofers, carpenters and plumbers. Signs and symptoms of prepatellar bursitis
37 include knee pain, erythema, sw elling and inability to kneel on the affected side.
38
39
Time Spent: 2 seconds Copyright USMLEW orld,LLC. Last updated: [1 /10/2013]
40
41
42
~imeRemaini~ ~ 0
I Tuto' Feedback Suspend End Block
---------------------------------------------------------------------------------------------------------------------------------------------
1
2
" Item: 12 of 46
Q.ld: 654 [
lil f> Mark .<:J
Previous
[::>
Next
il
Lab Values
~
Notes
~
Calculator

CJ
5
6 A 23-year-old male w ho has been previously healthy is brought to the ER by his friends after a street fight.
7 They tell you that they w ere "walking dow n the street minding their ow n business w hen a guy jumped out from
8 behind a dumpster and stabbed him in the chest". They state that they disarmed the man after his single
9 attack and that he ran off into an alley. The patient is conscious but in distress. Physical examination reveals
10 a stab w ound at the fifth intercostal space along the left midclavicular line. W hich of the follow ing is most
11 likely punctured in this patient?

13
14 0 A. Azygous vein
15 0 B. Inferior vena cava
16
17 0 C. Left atrium
18 0 D. Left lung
19
20 0 E. Right ventricle
21
22
23
24
25
26
27
28
29
30
31
32
33
34
35
36
37
38
39
40
41
42
~imeRemaini~ ~ 0
I Tuto' Feedback Suspend End Block
---------------------------------------------------------------------------------------------------------------------------------------------
1
2
" Item: 12 of 46
Q.ld: 654 [
lil f> Mark .<:J
Previous
[::>
Next
il
Lab Values
~
Notes
~
Calculator

CJ
5
.; ~

0
D. Left lung [50%]
E. Right ventricle [39%]
6
7
8 Explanation: User ld:
9
10
11

13
14
15
16
17
18
19
20
21
22
23
24
25
26
27
28
29
30
31
32
33
34
35
36
USMLEW01Id, LLC ([) 20l 1
37
38
39 Most of the volume of the thoracic cavity is occupied by the lungs. The apex of each lung extends into the
40 neck approximately three to four centimeters above the first rib. The lunq bases are in direct contact w ith the
41
42
~imeRemaini~ ~ 0
I Tuto' Feedback Suspend End Block
---------------------------------------------------------------------------------------------------------------------------------------------
1
2
" Item: 12 of 46
Q.ld: 654 [
lil f> Mark .<:J
Previous
[::>
Next
il
Lab Values
~
Notes
~
Calculator

CJ
5 USMLEWO<Id. LLC 11> 2011
6
7
Most of the volume of the thoracic cavity is occupied by the lungs. The apex of each lung extends into the
8
neck approximately three to four centimeters above the first rib. The lung bases are in direct contact w ith the
9
diaphragm, w hich separates the right lung from the right lobe of the liver, and the left lung from the left lobe of
10
the liver, the stomach, and the spleen . The mediastinal surface of each lung has a cardiac impression that
11
accommodates the heart.
13 The heart is located behind the sternum and its anterior surface is partially covered by the lungs. The apex is
14 formed by the left ventricle and is covered by the left lung. It lies behind the fifth left intercostal space at the left
15 midclavicular line. The sternocostal (anterior) surface of the heart is formed chiefly by the right ventricle. The
16 diaphragmatic (inferior) surface is formed by left and right ventricles, and is in contact w ith the central tendon
17 of the diaphragm. The posterior surface of the heart is formed mainly by the left atrium .
18
19 Penetrating injury involving the fifth intercostal space at the left midclavicular line would most likely injure the
20 left lung. Penetration of the left lung at this location could lead to injury of the apex of the heart (left ventricle)
21 as w ell, if the v.ound ~-~ere deep enough. All other heart chambers lie medial to the left midclavicular line and
22 would not be affected.
23
24 (Choice A) The azygos vein lies in the posterior mediastinum immediately to the right of the midline. It drains
25 blood from the posterior intercostal veins into the superior vena cava .
26
27 (Choice B) The inferior vena cava is located in the mediastinum, lies to the right of midline, and drains into
28 the right atrium .
29
30 (Choice C) The left atrium is located posterior to the left ventricle and forms the base of the heart.
31
(Choice E) The right ventricle forms the anterior (sternal) surface of the heart and the majority of its inferior
32
border. It lies medially to the left midclavicular line.
33
34 Educational Objective:
35 The left ventricle forms the apex of the heart and reaches as far as the fifth intercostal space at the left
36 midclavicular line. All other chambers of the heart lie medial to the left midclavicular line. The lungs overlap
37 much of the anterior surface of the heart.
38
39 Time Spent: 1 seconds Copyright USMLEWorld,LLC. Last updated: [11 /212013]
40
41
42
~imeRemaini~ ~ 0
I Tuto' Feedback Suspend End Block
---------------------------------------------------------------------------------------------------------------------------------------------
1
2
" Item: 13 of 46
Q.ld: 1525 [
lil f> Mark .<:J
Previous
[::>
Next
il
Lab Values
~
Notes
~
Calculator

CJ
5
6 A 56-year-old female w ith a long history of gallstones presents to the emergency room complaining of
7 cramping mid-abdominal pain, abdominal distention and vomiting for the last 12 hours. Abdominal X-ray
8 reveals air in the gallbladder and biliary tree. In w hich of the follow ing sites is a gallstone most likely lodged?
9
10
11 0 A. Cystic duct
12 0 B. Common bile duct
14 0 C. Duodenum
15 0 D. Jejunum
16
17 0 E. Ileum
18
19
20
21
22
23
24
25
26
27
28
29
30
31
32
33
34
35
36
37
38
39
40
41
42
~imeRemaini~ ~ 0
I Tuto' Feedback Suspend End Block
---------------------------------------------------------------------------------------------------------------------------------------------
1
2
" Item: 13 of 46
Q.ld: 1525 [
lil f> Mark .<:J
Previous
[::>
Next
il
Lab Values
~
Notes
~
Calculator

CJ
5
I " @ E. Ileum [33%]
6
7 Explanation: User ld:
8
9 This patient is most likely suffering from gallstone ileus, a condition that occurs in patients w ith longstanding
10 cholelithiasis (often middle-aged to elderly females). A large (typically 2.5 em or greater) gallstone causes the
11 formation of a cholecystenteric fistula between the gallbladder and adjoining gut tissue due to persistent
12 pressure on these tissues. The fistula ultimately allow s passage of the gallstone into the small bowel, and the
resulting communication between the gallbladder and small bowel allow s intestinal gas to enter the gallbladder
14 and biliary tree. Often this gas can be observed on abdominal X-rays.
15
16 After entering the small bowel, the gallstone typically passes freely through the duodenum, jejunum and ileum
17 until it encounters the ileocecal valve. The ileocecal valve typically handles material in a liquid state, and the
18 large gallstones associated w ith gallstone ileus are generally unable to pass through this valve. Obstruction of
19 the small bow el at the ileocecal valve leads to the classic signs and symptoms of small bow el obstruction
20 including abdominal pain and distention, vomiting, tenderness to palpation and tinkling bow el sounds.
21
22 The diagnosis can be made w ith an upper Gl series to demonstrate the cholecystenteric fistula and small
23 bowel obstruction . Treatment is by surgical removal of the offending stone. Cholecystectomy and repair of
24 the fistula are rarely necessary.
25
(Choices A and B) Obstruction of the cystic duct or common bile duct by a gallstone is referred to as
26
choledocolithiasis. This can lead to biliary colic, jaundice and cholangitis. Signs and symptoms of cholangitis
27
include fever, right upper quadrant abdominal pain, and an inspiratory pause during right upper quadrant
28
palpation (Murphy's sign). Free air is typically not seen in the biliary tree. Right upper quadrant ultrasound w ill
29
reveal stones in the biliary ducts.
30
31 (Choices C and D) The caliber of the duodenum and jejunum is typically sufficient to allow the passage of the
32 larger gallstones that cause gallstone ileus.
33
34 Educational Objective:
35 Gallstone ileus results from the passage of a large gallstone (typically greater than 2.5 em) through a
36 cholecystenteric fistula into the small bowel w here it ultimately causes obstruction at the ileocecal valve. Gas
37 is seen w ithin the gallbladder and biliary tree on abdominal X-ray due to the presence of the fistula, and
38 patients present w ith signs and symptoms of small bowel obstruction .
39
40 f"'.,... .,., . ..: ... l..+ ~ I I C'fdll 1\AL., ..I..J I I f"' 1 ........+, __ ........+-. ...1 r7 t7 t'"~f\" n1

~
41
42
---------------------------------------------------------------------------------------------------------------------------------------------
F!!ck suW.nd EnQck
1
2
" Item: 14 of 46
Q.ld: 1696 [
lil f> Mark .<:J
Previous
[::>
Next
il
Lab Values
~
Notes
~
Calculator

CJ
5
6 Show n below are the cerebral arteriography findings from a 45-year-old patient.
7
8
9
10
11
12
13

15
16
17
18
19
20
21
22
23
24
25
26
27 Occlusion of the artery labeled 'X' w ould most likely result in an inability to:
28
29
30 0 A. Climb stairs
31 0 B. Grip
32 0 C. W histle
33
34 0 D. Sw allow food
35
36
37
38
39
40
41
42
~imeRemaini~ ~ 0
I Tuto' Feedback Suspend End Block
---------------------------------------------------------------------------------------------------------------------------------------------
1
2
" Item: 14 of 46
Q.ld: 1696 [
lil f> Mark .<:J
Previous
[::>
Next
il
Lab Values
~
Notes
~
Calculator

CJ
5
., @ A. Climb stairs [70%]

6 0 B. Grip [9%]
7 0 C. W histle [9%]
8
0 D. Swallow food [11 %]
9
10
11 Explanation: User ld:
12
Anterior communicating artery
13

15 Medial striate artery


16 (recu rrent artery of Heubner)
17 Anterior cerebral artery
18
19 Anteromedial central
20 rr-t-- - - . / (perforating) arteries
21
Ophthalmic artery
22
23
24
Anterolateral central
Internal carotid artery (lenticulostriate) arteries
25
26
27 Middle cerebral
28 artery
29
30
31
32
33 Posterior communicating --~ Anterior choroidal artery
34 artery
35
36 Posterior cerebral artery
37
38
39
40
41
42
1
2
" Item: 14 of 46
Q.ld: 1696 [
lil f> Mark .<:J
Previous
[::>
Next
il
Lab Values
~
Notes
~
Calculator

CJ
5
Anterior communicating artery
6
7 Medial striate artery
8 (recurrent artery of Heubner)
9 Anterior cerebral artery
10
11
12
13
Ophthalmic artery
15
16
Anterolateral central
Internal carotid artery (lenticulostriate) arteries
17
18
19 Middle cerebral
20 artery
21
22
23
24
25 Posterior communicating ----..!ll Anterior choroidal artery
26 artery
27
28 Posterior cerebral artery
29
30
31
32
33
34
Basilar artery
35
36 Labyrinthine
37 - - -- (internal acoustic) artery
38 Anterior inferior
39 cerebellar artery- ........~
40
41
42
1
2

CJ
5
6 Posterior communicating --~ Anterior choroidal artery
7 artery
8
9 Posterior cerebral artery
10
11
12
13

15 Basilar artery
16
17 Labyrinthine
18
Anterior inferior ---~ (internal acoustic) artery
19
20 cerebellar artery
21 Vertebral artery
22 USMLEWorld, LLC 201 1
23
24
25 This cerebral angiogram image is projected axially, w ith the top of the image representing anterior regions and
26 the bottom representing posterior regions. The very large vessels seen bilaterally are the internal carotid
27 arteries. Each gives off a middle cerebral artery to supply more lateral regions and an anterior cerebral artery,
28 w hich extends medially. The above "X'' marks an anterior cerebral artery. Each anterior cerebral artery
29 supplies the medial region of the ipsilateral hemisphere, from the frontal pole to the parietooccipital sulcus.
30 Occlusion of the anterior cerebral artery would affect motor function of the contralateral leg and foot.
31
(Choices B, C and D) Motor control of the hand, head and neck would be affected by occlusion of the middle
32
cerebral artery.
33
34 Educational Objective:
35 The anterior cerebral arteries supply the medial portions of the two hemispheres. Occlusion causes
36 w eakness of the contralateral legs and sensory deficits of the contralateral legs, trunk and genitals. There
37 may also be behavior and mood changes due to injury of frontal lobe structures.
38
39
Time Spent: 31 seconds Copyright USMLEWorld,LLC. Last updated: [1 / 1/2014]
40
41
42
~imeRemaini~ ~ 0
I Tutor Feedback Suspend End Block
--------------------------------------------------------------------------------------------------------------------------------------
1
2
" Item: 15 of 46
Q.ld: 8587 [
lil f> Mark .<:J
Previous
[::>
Next
il
Lab Values
~
Notes
~
Calculator

CJ
5
6 A 47-year-old morbidly obese woman comes to the physician seeking advice regarding w eight loss. She has
7 tried diet and exercise a number of times w ithout success. Her other medical problems include type 2
8 diabetes mellitus and obstructive sleep apnea . Her body mass index is 43 kg/m 2. After a discussion about
9 available surgical options, she expresses interest in the adjustable gastric band, an inflatable silicone device
10 that is placed around the cardiac part of the stomach. In order to encircle the stomach, the band must pass
11 through w hich of the follow ing structures?
12
13
14 0 A. Falciform ligament
0 B. Gastrocolic ligament
16
17 0 C. Greater omentum
18 0 D. Lesser omentum
19
20 0 E. Splenorenal ligament
21
22
23
24
25
26
27
28
29
30
31
32
33
34
35
36
37
38
39
40
41
42
~imeRemaini~ ~ 0
I Tuto' Feedback Suspend End Block
---------------------------------------------------------------------------------------------------------------------------------------------
1
2
" Item: 15 of 46
Q.ld: 8587 [
lil f> Mark .<:J
Previous
[::>
Next
il
Lab Values
~
Notes
~
Calculator

CJ
5
.; ~ D. Lesser omentum [56%]
0 E. Splenorenal ligament [3%]
6
7
8 Explanation: User ld:
9
10
11 Greater & lesser omentum
12
13
14 Falciform ligament Lesser omentum
Rou nd ligament of liver
16
17 Hepatoduodenal ligament
18 Quadrate lobe of liver
19 Hepatogastric ligament
20
21 Gallbladder
Left lobe of liver
22
23
24
25
26
27
28
29
30
31
32
33
34
35
36 Left colic
37 Right kidney
~--(splenic)
38 flexure
39
40
41
42
1
2
" Item: 15 of 46
Q.ld: 8587 [
lil f> Mark .<:J
Previous
[::>
Next
il
Lab Values
~
Notes
~
Calculator

CJ
5
Great er & lesser omentum
6
7
Falciform ligament Lesser omentum
8
9 Rou nd ligament of liver
10
11 Hepatoduodenal ligament
Quadrate lobe of liver
12
Hepatogastric ligament
13
14
Gallbladder
Left lobe of liver
16
17
18
19
20
21
22 Right lobe
23 of liver
24
Spleen
25 Omental
26 foramen
27
28
29
30 Left colic
Right kidney
31 ~-- (splenic)
32 flexure
33
34
35 Right colic
36 (hepatic) ----;.:._~_Greater
37 flexure omentum
38
39
40
41
42
~imeRemaini~ ~ 0
I Tuto' Feedback Suspend End Block
--------------------------------------------------------------------------------------------------------------------------------------
1
2
" Item: 15 of 46
Q.ld: 8587 [
lil f> Mark .<:J
Previous
[::>
Next
il
Lab Values
~
Notes
~
Calculator

CJ
5 USMlEWOtld, tlC
6
7 Adjustable gastric banding is a type of restrictive bariatric surgery designed for obese patients. The
8 adjustable gastric band is an inflatable silicone device placed around the gastric cardia. It is intended to slow
9 the passage of food, increasing satiety and limiting the amount of food consumed. To encircle the upper
10 stomach, the gastric band must pass through the lesser omentum.
11
12 The lesser omentum is a double layer of peritoneum that extends from the liver to the lesser curvature of the
13 stomach and the beginning of the duodenum. Anatomically, the lesser omentum is divided into 2 ligaments:
14
1. Hepatogastric ligament: the portion connecting to the lesser curvature of the stomach
2. Hepatoduodenal ligament: the portion connecting to the duodenum
16
17
18 Between the 2 layers of the lesser omentum, close to the right-sided free margin, lie the hepatic artery,
19 common bile duct, portal vein, lymphatics, and hepatic plexus. The right and left gastric arteries and gastric
20 veins also lie between the 2 layers, near w here the lesser omentum attaches to the stomach.
21
22 (Choice A) The falciform ligament attaches the liver to the anterior body w all. It is a derivative of the
23 embryonic ventral mesentery and contains the round ligament, the remnant of the fetal umbilical vein.
24
25 (Choices B and C) The greater omentum is a large fold of visceral peritoneum that extends from the greater
26 curvature of the stomach, travels inferiorly over the small intestine, and then reflects on itself and ascends to
27 encompass the transverse colon before reaching the posterior abdominal w all. The gastrocolic ligament is
28 the section that stretches from the greater curvature of the stomach to the transverse colon . It forms part of
29 the anterior w all of the lesser sac and is often divided during surgery to provide access to the anterior
30 pancreas and posterior w all of the stomach.
31
(Choice E) The splenorenal ligament is derived from the peritoneum and lies between the left kidney and the
32
spleen . It contains the splenic vessels and the tail of the pancreas.
33
34 Educational objective:
35 The lesser omentum is a double layer of peritoneum that extends from the liver to the lesser curvature of the
36 stomach and the beginning of the duodenum. It is divided into the hepatogastric and hepatoduodenal
37 ligaments.
38
39 Time Spent: 1 seconds Copyright USMLEW orld,LLC. Last updated: [1 / 14/201 4]
40
41
42
~imeRemaini~ ~ 0
I Tuto' Feedback Suspend End Block
---------------------------------------------------------------------------------------------------------------------------------------------
1
2
" Item: 16 of 46
Q.ld: 1884 [
lil f> Mark .<:J
Previous
[::>
Next
il
Lab Values
~
Notes
~
Calculator

CJ
5
A 52-year-old man is evaluated for nonspecific, chronic abdominal pain . An axial computed tomography
image of his abdomen is show n below .
6
7
8
9
10
11
12
13
14
15

17
18
19
20
21
22
23
24
25
26
27
28
29
30 W hich of the follow ing statements best describes the structure indicated by the arrow ?
31
32
33 A. It drains into the left subclavian vein in the thorax
0
34
35 B. It drains through the cystic duct into the common bile duct
36 0
37
38 C. It gives rise to the right and left renal arteries
0
39
40

~
41
42 F!!ck suW.nd
---------------------------------------------------------------------------------------------------------------------------------------------
EnQck
1
2

CJ
5
6
7
8
9
10
11
12
13
14
15

17
18
19
20
21
22
23
24
25
W hich of the follow ing statements best describes the structure indicated by the arrow ?
26
27
28 A. It drains into the left subclavian vein in the thorax
29 0
30
B. It drains through the cystic duct into the common bile duct
31 0
32
33 C. It gives rise to the right and left renal arteries
34 0
35
D. It is formed by the union of the common iliac veins
36 0
37
38 E. It joins the splenic vein to form the portal vein
39 0
40
41
42
~imeRemaini~ ~ 0
I Tuto' Feedback Suspend End Block
---------------------------------------------------------------------------------------------------------------------------------------------
1
2
" Item: 16 of 46
Q.ld: 1884 [
lil f> Mark .<:J
Previous
[::>
Next
il
Lab Values
~
Notes
~
Calculator

CJ
5
0 B. It drains through the cystic duct into the common bile duct [1 0%]
0 C. It gives rise to the right and left renal arteries [7%]
6
.; @ D. It is formed by the union of the common iliac veins [57%]
7
8 0 E. It joins the splenic vein to form the portal vein [19%]
9
10
11 Explanation: User ld:
12
13
14
15

17
18
19
20
21
22
23
24
25
26
27
28
29
30
31
32
33
34
35
36
37
38
39 This axial CT image show s the abdomen near the L1 vertebral level. At this level, the renal arteries originate
40

~
41
42 F!!ck
---------------------------------------------------------------------------------------------------------------------------------------------
suW.nd EnQck
1
2
" Item: 16 of 46
Q.ld: 1884 [
lil f> Mark .<:J
Previous
[::>
Next
il
Lab Values
~
Notes
~
Calculator

CJ
5
0 B. It drains through the cystic duct into the common bile duct [1 0%]
0 C. It gives rise to the right and left renal arteries [7%]
6
.; @ D. It is formed by the union of the common iliac veins [57%]
7
8 0 E. It joins the splenic vein to form the portal vein [19%]
9
10
11 Explanation: User ld:
12
13
14
15

17
18
19
20
21
22
23
24
25
26
27
28
29
30
31
32
33
34
35
36
37
38
39 This axial CT image show s the abdomen near the L1 vertebral level. At this level, the renal arteries originate
40
41
42
~imeRemaini~ ~ 0
I Tuto' Feedback Suspend End Block
---------------------------------------------------------------------------------------------------------------------------------------------
1
2

CJ
5
6
7
8
9
10
11
12
13
14
15 This axial CT image show s the abdomen near the L1 vertebral level. At this level, the renal arteries originate
from the aorta . The inferior vena cava (IVC) lies just anterior to the right renal artery and to the right of the
17 aorta . The IVC is formed by the union of the right and left common iliac veins at the level of L4-L5 and drains
18 into the right atrium just above the level of the diaphragm at T8. The IVC returns venous blood to the right
19 atrium from the low er extremities, portal system, and abdominal and pelvic viscera .
20
21 (Choice A) The thoracic duct drains lymph from the entire left side of the body and all regions inferior to the
22 umbilicus (including the gut structures). It enters the thorax through the aortic hiatus and empties into the left
23 subclavian vein near its junction w ith the internal jugular vein .
24
25 (Choice B) The cystic duct drains bile from the gallbladder into the common bile duct. The plane of this
26 cross-sectional image is too low to see the gallbladder, but in higher sections it can be seen just below the
27 liver in the anterior right abdomen .
28
29 (Choice C) The image show s the abdominal aorta giving rise to the right and left renal arteries. The
30 abdominal aorta lies posteromedial to the IVC w ithin the abdomen .
31
(Choice E) The superior mesenteric vein (SMV) joins the splenic vein to form the portal vein. The SMV can
32
be seen vaguely near the head of the pancreas in this cross-sectional image.
33
34 Educational objective:
35 The inferior vena cava (IVC) is formed by the union of the right and left common iliac veins at the level of
36 L4-L5. The renal arteries and veins lie at the level of L1. The IVC returns venous blood to the heart from the
37 low er extremities, portal system, and abdominal and pelvic viscera .
38
39 Time Spent: 10 seconds Last updated: [3/ 11/201 4]
Copyright USMLEW orld,LLC.
40
41
42
~imeRemaini~ ~ 0
I Tuto' Feedback Suspend End Block
--------------------------------------------------------------------------------------------------------------------------------------
1
2
" Item: 17 of 46
Q.ld: 1638 [
111 f> Mark .<:J
Previous
[::>
Next
il
Lab Values
~
Notes
~
Calculator

CJ
5
6 A 34-year-old male has difficulty w alking after sustaining a traumatic injury to his right leg. Physical
7 examination reveals a right foot that is dorsiflexed and everted. The patient is unable to stand on his tiptoes.
8 W hat is the most likely area of sensory loss in this patient?
9
10
11 0 A. Anterior thigh
12
0 B. Medial leg
13
14 0 C. Medial foot
15
16 0 D. Dorsal foot
0 E. Plantar foot
18
19
20
21
22
23
24
25
26
27
28
29
30
31
32
33
34
35
36
37
38
39
40
41
42
~imeRemaini~ ~ 0
I Tuto' Feedback Suspend End Block
---------------------------------------------------------------------------------------------------------------------------------------------
1
2
" Item: 17 of 46
Q.ld: 1638 [
111 f> Mark .<:J
Previous
[::>
Next
il
Lab Values
~
Notes
~
Calculator

CJ
5
I ~ @ E. Plantar foot [55%]
6
7 Explanation: User ld:
8
9 Sciatic nerve
10
11
12
13 Common
- - peroneal
14 nerve
15
16 Saphenous nerve
(b<anch of femoml nerve)
18
19 Tibial
20 nerve
21
22
23
24
25
26
27
28 Deep peroneal nerve
29
30 The tibial nerve is the large medial branch of the sciatic nerve that descends through the popliteal fossa
31 together w ith the popliteal vein and artery. Injury to this nerve can occur after deep penetrating trauma to the
32 popliteal fossa and can cause a number of different deficits due to its numerous functions. The tibial nerve
33 innervates the gastrocnemius, soleus and plantaris muscles, w hich are responsible for plantar flexion of the
34 foot. It also supplies the flexor digitorum longus and flexor hallucis longus, w hich are responsible for toe
35 flexion . The tibial nerve also innervates the tibialis posterior muscle, w hich is responsible for inversion of the
36 foot. (Patients w ith injury to the tibial nerve often hold their lower extremity in a calcaneovalgocavus position-
37 the foot is dorsiflexed and everted.) After delivering its motor innervation, the tibial nerve terminates by dividing
38 into the medial and lateral plantar nerves to provide sensory innervation to the skin of the distal plantar surface
39 of the foot.
40
41
42
~imeRemaini~ ~ 0
I Tuto' Feedback Suspend End Block
---------------------------------------------------------------------------------------------------------------------------------------------
1
2

CJ
5
6
7
8
9
10
11 Deep peroneal nerve
12
13 The tibial nerve is the large medial branch of the sciatic nerve that descends through the popliteal fossa
14 together w ith the popliteal vein and artery. Injury to this nerve can occur after deep penetrating trauma to the
15 popliteal fossa and can cause a number of different deficits due to its numerous functions. The tibial nerve
16 innervates the gastrocnemius, soleus and plantaris muscles, w hich are responsible for plantar flexion of the
foot. It also supplies the flexor digitorum longus and flexor hallucis longus, w hich are responsible for toe
18 flexion . The tibial nerve also innervates the tibialis posterior muscle, w hich is responsible for inversion of the
19 foot. (Patients w ith injury to the tibial nerve often hold their low er extremity in a calcaneovalgocavus position-
20 the foot is dorsiflexed and everted.) After delivering its motor innervation, the tibial nerve terminates by dividing
21 into the medial and lateral plantar nerves to provide sensory innervation to the skin of the distal plantar surface
22 of the foot.
23
24 (Choice A) The cutaneous branch of the femoral nerve innervates the majority of the skin of the anterior thigh .
25
26 (Choice B) Cutaneous branches of the saphenous nerve innervate the skin of the medial leg. The
27 saphenous nerve is a branch of the femoral nerve and courses to the medial leg together w ith the great
28 saphenous vein .
29
30 (Choice C) The medial aspect of the foot is innervated by the superficial peroneal nerve, a branch of the
31 common peroneal nerve, as w ell as by the saphenous nerve.
32
(Choice D) The superficial peroneal nerve innervates the dorsum of the foot, except for the skin between the
33
great toe and the second toe, w hich is innervated by the deep peroneal nerve.
34
35 Educational Objective:
36 The tibial nerve innervates the flexors of the low er leg, the extrinsic digital flexors of the toes, and the skin of
37 the sole of the foot.
38
39
Time Spent: 2 seconds Copyright USMLEW orld,LLC. Last updated: [1 2119/2012]
40
41
42
~imeRemaini~ ~ 0
I Tuto' Feedback Suspend End Block
---------------------------------------------------------------------------------------------------------------------------------------------
1
2
" Item: 18 of 46
Q.ld: 1969 [
lil f> Mark .<:J
Previous
[::>
Next
il
Lab Values
~
Notes
~
Calculator

CJ
5
6 A 23-year-old athlete presents to your office w ith right knee pain after playing basketball. A coronal MRI image
7 of his knee is show n below.
8
9
10
11
12
13
14
15
16
17

19
20
21
22
23
24
25
26
27 The asterisk marks the attachment site of w hich of the follow ing ligaments?
28
29 0 A. Anterior cruciate
30
31 0 B. Tibial collateral
32 0 C. Transverse genicular
33 0 D. Posterior cruciate
34
35
0 E. Patellar
36
37
38
39
40
41
42
~imeRemaini~ ~ 0
I Tuto' Feedback Suspend End Block
---------------------------------------------------------------------------------------------------------------------------------------------
1
2
" Item: 18 of 46
Q.ld: 1969 [
lil f> Mark .<:J
Previous
[::>
Next
il
Lab Values
~
Notes
~
Calculator

CJ
5
~ @ D. Posterior cruciate [45%]
0 E. Patellar [2%]
6
7
8 Explanation: User ld:
9
10 In this coronal MRI image of the right knee, the opaque (white) regions, from top to bottom, correspond to: the
11 lateral and medial condyles of the femur (the medial condyle is adjacent to the asterisk), the lateral and medial
12 condyles of the tibia, and the head of the fibula. The anterior and posterior cruciate ligaments are ligaments
13 w ithin the articular capsule of the knee joint that cross one another as each spans from the tibia to the femur.
14 The posterior cruciate ligament attaches to the posterior part of the intercondylar area of the tibia and the
15 anterior lateral surface of the medial epicondyle of the femur (asterisk).
16 Anterior view Posterior view
17

19
20
21 Posterior cruciate ligament
22 Lateral collateral ligament
23 (fibular collateral ligament )
24
25
Medial collateral ligament Popliteus tendon
26
(tibial collateral ligament)
27
28
29
30 Anterior cruciate ligament
31
32
33
34
35 USMI..EW<>tld.LL< () 2011

36
37
(Choice A} The anterior cruciate ligament is more commonly injured than the posterior cruciate ligament. It
38
spans from the anterior portion of the intercondylar tibia to the posterior medial side of the lateral femoral
39
condyle. It prevents posterior displacement of the femur relative to the tibia w hen the knee is fully extended.
40
41
42
~imeRemaini~ ~ 0
I Tuto' Feedback Suspend End Block
---------------------------------------------------------------------------------------------------------------------------------------------
1
2

CJ
5
6 Posterior cruciate ligament
7 Lateral collateral ligament
8 (fibular collateral ligament)
9
10 Medial collateral ligament_ Popliteus tendon
11 (tibial collateral ligament)
12
13
14
15 Anterior cruciate ligament
16
17

19
USMUWOrld.lLC () 201 I
20
21
22 (Choice A) The anterior cruciate ligament is more commonly injured than the posterior cruciate ligament. It
23 spans from the anterior portion of the intercondylar tibia to the posterior media! side of the latera! femora!
24 condyle. It prevents posterior displacement of the femur relative to the tibia w hen the knee is fully extended.
25
26 (Choice B) The tibial collateral ligament is a very strong band of tissue that spans from the medial femoral
27 epicondyle to the medial condyle of the tibia . It also attaches to the medial meniscus.
28
29 (Choice C) The transverse genicular ligament attaches the medial and lateral menisci anteriorly on the
30 proximal articular surface of the tibia .
31
(Choice E) The patellar ligament spans from the patella to the tibial tuberosity. It is the continuation of the
32
quadriceps femoris tendon.
33
34 Educational Objective:
35 The posterior cruciate ligament prevents anterior displacement of the femur relative to the tibia w hen the knee
36 is flexed. It attaches to the posterior part of the intercondylar area of the tibia and the anterior part of the lateral
37 surface of the medial epicondyle of the femur.
38
39
Time Spent: 8 seconds Copyright USMLEW orld,LLC. Last updated: [1 /8/2014]
40
41
42
~imeRemaini~ ~ 0
I Tuto' Feedback Suspend End Block
---------------------------------------------------------------------------------------------------------------------------------------------
1
2
" Item: 19 of 46
Q.ld : 303 [
lil f> Mark .<:J
Previous
[::>
Next
il
Lab Values
~
Notes
~
Calculator

CJ
5
6 A 32-year-old female presents to your office w ith severe nausea and recurrent bilious vomiting. Her
7 symptoms initially began as postprandial epigastric pain and early satiety, but have progressed over the last
8 two w eeks. She w orks as an actress, and tells you that her symptoms only started w hen she "landed a role
9 in a soap opera" and w as inspired to lose 25 lbs. on a "crash diet". On physical exam, her abdomen is tender
10 and slightly distended w ith high-pitched bow el sounds. Concerned about a small bow el obstruction, you admit
11 the patient to the hospital. A laparotomy is performed, and it is observed that the angle between her superior
12 mesenteric artery and her aorta is significantly decreased. W hich of the follow ing structures is most likely to
13 be obstructed by the artery?
14
15
16 0 A. Ascending portion of the duodenum
17 0 B. Descending portion of the duodenum
18
0 C. Duodenal bulb
20 0 D. Duodenojejunal flexure
21
22 0 E. Gastric antrum
23 0 F. Transverse portion of the duodenum
24
25
26
27
28
29
30
31
32
33
34
35
36
37
38
39
40
41
42
~imeRemaini~ ~ 0
I Tuto' Feedback Suspend End Block
---------------------------------------------------------------------------------------------------------------------------------------------
1
2
" Item: 19 of 46
Q.ld: 303 [
lil f> Mark .<:J
Previous
[::>
Next
il
Lab Values
~
Notes
~
Calculator

CJ
5
I .; @ F. Transverse portion of the duodenum [54%]
6
7 Explanation: User ld:
8
9 The superior mesenteric artery (SMA) leaves the aorta at the level of L1 and supplies the intestine from the
10 duodenum and pancreas to the left colic flexure. The transverse portion of the duodenum lies horizontally at
11 the level of L3, between the aorta and superior mesenteric artery. Normally, the SMA and aorta form an
12 approximately 45 degree angle. If this angle diminishes to less than 20 degrees, the transverse portion of the
13 duodenum can get entrapped between the SMA and aorta, leading to symptoms of partial small bow el
14 obstruction. This condition is called superior mesenteric artery syndrome.
15
16 Superior mesenteric artery syndrome
17 Anterior view
18

20 Right crus
21 of diaphragm
22
23
24
25
26
Stomach
27
Inferior
28
29
vena cava ---.L /
30
31
32
33 Aort a - - --1--
34
35
36 Celiac trunk - ----1-
37
38
39
40
41
42
1
2
" Item: 19 of 46
Q.ld: 303 [
lil f> Mark .<:J
Previous
[::>
Next
il
Lab Values
~
Notes
~
Calculator

CJ
5
Superior mesenteric artery syndrome
6 Anterior view
7
8 Right crus
9 of diaphragm
10
11
12
13
14
15 Stomach
Inferior
16
17
18
vena cava ~--4~ /
20
21
22
Aorta - - ---1--
23
24 Celiac trunk - ----1-
25
26
27
28
29
30
31
32 Liga ment ofTreitz
Duodenum - --;.-
33
34
35
36
37
Jejunum
38
39
40
41
42
1
2

CJ
5
6
7
8
9
10
11
12 Ligam ent ofTreitz
13
14
15
16
17 Jejunum
18
Left renal vein
20
21
22
Superior m esenteric
23
24 art ery
25
26 USMLEWorld, LLC 2011
27
28
Narrowing of the aortomesenteric angle can occur with any condition that causes diminished mesenteric
29
fat, including low body w eight, recent w eight loss, severe burns or other inducers of catabolism, and
30
prolonged bed rest. It can also occur with pronounced lordosis or after surgical correction of scoliosis, as this
31
procedure lengthens the spine resulting in decreased mobility of the SMA.
32
33 Educational Objective:
34 Superior mesenteric artery syndrome occurs when the transverse portion of the duodenum is entrapped
35 between the SMA and aorta, causing symptoms of partial intestinal obstruction . This syndrome occurs when
36 the aortomesenteric angle critically decreases, secondary to diminished mesenteric fat, pronounced lordosis,
37 or surgical correction of scoliosis.
38
39
Time Spent: 1 seconds Copyright USMLEW orld,LLC. Last updated: [1 / 13/2014]
40
41
42
~imeRemaini~ ~ 0
I Tuto' Feedback Suspend End Block
---------------------------------------------------------------------------------------------------------------------------------------------
1
2
" Item: 20 of 46
Q.ld: 1744 [
111 f> Mark .<:J
Previous
[::>
Next
il
Lab Values
~
Notes
~
Calculator

CJ
5
6 A 56-year-old male is undergoing evaluation for fatigue, vague abdominal discomfort, and bloody stools.
7 Colonoscopy reveals a mass in the descending colon. Surgical removal of the mass requires proper ligation
8 of the:
9
10
11 0 A. Celiac trunk
12 0 B. Superior mesenteric artery
13
14 0 C. Inferior mesenteric artery
15 0 D. Internal iliac artery
16
17 0 E. External iliac artery
18
19

21
22
23
24
25
26
27
28
29
30
31
32
33
34
35
36
37
38
39
40
41
42
~imeRemaini~ ~ 0
I Tuto' Feedback Suspend End Block
---------------------------------------------------------------------------------------------------------------------------------------------
1
2
" Item: 20 of 46
Q.ld: 1744 [
111 f> Mark .<:J
Previous
[::>
Next
il
Lab Values
~
Notes
~
Calculator

CJ
5
A 56-year-old male is undergoing evaluation for fatigue, vague abdominal discomfort, and bloody stools.
Colonoscopy reveals a mass in the descending colon . Surgical removal of the mass requires proper ligation
6 of the:
7
8 0 A. Celiac trunk [1 %]
9
10 0 B. Superior mesenteric artery [4%]
11 " @ C. Inferior mesenteric artery [91 %]
12 0 D. Internal iliac artery [3%]
13
14 0 E. External iliac artery [1 %]
15
16 Explanation: User ld:
17
18 This patient has a tumor in the descending colon . The distal 1/3 of the transverse colon, descending colon,
19 sigmoid colon, and rectum are all hindgut derivatives. Hindgut structures receive arterial blood from the
inferior mesenteric artery (IMA), a branch of the aorta . These structures are drained by the inferior mesenteric
21 vein (IMV), w hich does no/course w ith the IMA. The IMV drains blood from the hindgut into the splenic vein,
22 w hich drains into the portal vein .
23
24 (Choice A) The celiac trunk supplies blood to most foregut structures w ith the exception of the mouth,
25 pharynx and proximal esophagus. The foregut encompasses all structures from the mouth to the second part
26 of the duodenum. Foregut derivatives include the liver, pancreas, and gallbladder.
27
28 (Choice B) The superior mesenteric artery supplies blood to the midgut, w hich spans from the third part of
29 the duodenum to the proximal 213 of the transverse colon.
30
31 (Choice D) The internal iliac artery supplies portions of the gut through branches like the middle rectal artery
32 and the inferior rectal artery (a branch of the pudendal artery, w hich arises from the internal iliac artery).
33
(Choice E) Branches of the external iliac artery supply the lower extremity and anterior abdominal w all.
34
35 Educational Objective:
36 The hindgut encompasses the distal 1/3 of the transverse colon, the descending colon, the sigmoid colon,
37 and the rectum. These structures receive their main arterial blood supply from the inferior mesenteric artery.
38
39
Time Spent: 5 seconds Copyright USMLEWorld,LLC. Last updated: [3/ 12/2014]
40
41
42
~imeRemaini~ ~ 0
I Tuto' Feedback Suspend End Block
---------------------------------------------------------------------------------------------------------------------------------------------
1
2
" Item: 21 of 46
Q.ld: 1812 [
111 f> Mark .<:J
Previous
[::>
Next
il
Lab Values
~
Notes
~
Calculator

CJ
5
6 A 34-year-old Caucasian female has difficulty abducting her right arm past the horizontal position . An
7 abnormal prominence of the right inferior scapular angle is also observed. W hich of the follow ing is the most
8 likely cause of this patient's condition?
9
10
11 0 A. Thyroidectomy
12 0 B. Anterior dislocation of the shoulder joint
13
14 0 C. Violent stretch between the head and the shoulder
15 0 D. Clavicular fracture
16
17 0 E. Mastectomy
18
19
20

22
23
24
25
26
27
28
29
30
31
32
33
34
35
36
37
38
39
40
41
42
~imeRemaini~ ~ 0
I Tuto' Feedback Suspend End Block
---------------------------------------------------------------------------------------------------------------------------------------------
1
2
" Item: 21 of 46
Q.ld: 1812 [
111 f> Mark .<:J
Previous
[::>
Next
il
Lab Values
~
Notes
~
Calculator

CJ
5
I .; E. Mastectomy [52%]
6
7 Explanation: User ld:
8
9 The patient described in the question stem is exhibiting symptoms consistent w ith paralysis of the serratus
10 anterior muscle due to long thoracic nerve injury. Paralysis of the serratus anterior causes two classic signs.
11 First, w hen a patient is asked to press anteriorly against a w all scapular "winging" can be observed. A w inged
12 scapula results from an inability of the serratus anterior to hold the medial border and inferior angle of the
13 scapula against the posterior chest w all. Second, patients w ith serratus anterior paralysis are unable to
14 abduct the arm higher than the horizontal position . The deltoid and supraspinatus muscles abduct the arm up
15 to the horizontal position, but at that point the action of the serratus anterior is required to rotate the glenoid
16 cavity superiorly, thereby allow ing complete abduction of the arm over the head. The long thoracic nerve
17 injury can occur during penetrating trauma or iatrogenically during axillary lymph node dissections as may
18 occur during a radical mastectomy.
19
(Choice A) Thyroidectomy carries many risks due to the complicated anatomy of the midline neck. Nerve
20
injuries that can occur w ith this procedure include damage to the recurrent laryngeal nerve during ligature of
the inferior thyroid artery and damage to the external branch of the superior laryngeal nerve during ligature of
22
the superior thyroid artery.
23
24 (Choice B) Anterior dislocation of the shoulder joint or a fracture of the neck of the humerus can injure the
25 axillary nerve, resulting in paralysis of the deltoid and teres minor and some sensory loss to the upper lateral
26 arm.
27
28 (Choice C) Violent stretch between the head and the shoulder can occur during delivery or by trauma in the
29 adult. This results in damage to the upper trunk of the brachial plexus and the classic Erb-Duchenne palsy.
30 This palsy results from damage to the musculocutaneous and suprascapular nerves and causes a w aiter's
31 tip posturing of the arm (shoulder adducted, arm pronated, and elbow extended}.
32
33 (Choice D) Clavicular fracture does not cause any characteristic nerve palsies, but it is important to note that
34 these fractures typically occur in the middle third of the clavicle due to the strength of the ligamentous
35 structures at either end of this bone.
36
37 Educational Objective:
38 Mastectomy is a commonly tested cause of long thoracic nerve injury, but any trauma or surgery in the axillary
39 region is at risk of damaging this nerve. Injury to this nerve causes w inging of the scapula and inability to
40 abduct the shoulder oast 90 dearees.
41
42
~imeRemaini~ ~ 0
I Tuto' Feedback Suspend End Block
---------------------------------------------------------------------------------------------------------------------------------------------
1
2
" Item: 22 of 46
Q.ld: 1631 [
111 f> Mark .<:J
Previous
[::>
Next
il
Lab Values
~
Notes
~
Calculator

CJ
5
6 A 52-year-old male presents to your office w ith a painless mass in his groin. On physical examination, you
7 palpate several hard lymph nodes in the right inguinal area . Lymph node biopsy reveals malignant cells that
8 most likely originated from the:
9
10
11 0 A. Testes
12 0 B. Prostate
13
14 0 C. Bladder
15 0 D. Anal canal
16
17 0 E. Sigmoid colon
18
19
20
21

23
24
25
26
27
28
29
30
31
32
33
34
35
36
37
38
39
40
41
42
~imeRemaini~ ~ 0
I Tuto' Feedback Suspend End Block
---------------------------------------------------------------------------------------------------------------------------------------------
1
2
" Item: 22 of 46
Q.ld: 1631 [
111 f> Mark .<:J
Previous
[::>
Next
il
Lab Values
~
Notes
~
Calculator

CJ
5
palpate several hard lymph nodes in the right inguinal area . Lymph node biopsy reveals malignant cells that
most likely originated from the:
6
7 0 A. Testes [26%]
8
9 0 B. Prostate [23%]
10 0 C. Bladder [7%]
11 ~ @ D. Anal canal [38%]
12
13 0 E. Sigmoid colon [5%]
14
15 Explanation: User ld:
16
17 This patient has palpable firm superficial inguinal lymph nodes. The superficial inguinal lymph nodes drain all
18 cutaneous lymph from the umbilicus to the feet, including the external genitalia and anus (up to the beginning
19 of the rectum, the dentate line), but excluding the posterior calf, w hich drains to the popliteal lymph nodes.
20 The superficial inguinal lymph nodes lie in a region bounded by the inguinal ligament, sartorius muscle, and
21 adductor longus muscle, and overlie the femoral nerve, artery and vein .

23 (Choice A) In general, lymph vessels follow the paths of the arteries and veins supplying a particular
24 structure. The testes receive their blood supply from the testicular arteries, w hich arise directly from the
25 abdominal aorta just inferior to the renal arteries. The lymph vessels that drain the testes follow a path from
26 the testes to the abdominal aortic lymph nodes.
27
28 (Choice B) Lymph from the prostate drains primarily to the internal iliac nodes, but also to the external iliac
29 nodes and sacral nodes (minor pathw ays).
30
31 (Choice C) Lymph from the superior portion of the bladder drains to the external iliac nodes w hile lymph from
32 the inferior portion of the bladder drains to the internal iliac nodes.
33
(Choice E) Lymph from the sigmoid colon drains to the inferior mesenteric lymph nodes.
34
35 Educational Objective:
36 All skin from the umbilicus dow n, including the anus (up to the dentate/pectinate line) but excluding the
37 posterior calf, drains to the superficial inguinal lymph nodes.
38
39
Time Spent: 2 seconds Copyright USMLEW orld,LLC. Last updated: [7/7/201 0]
40
41
42
~imeRemaini~ ~ 0
I Tuto' Feedback Suspend End Block
--------------------------------------------------------------------------------------------------------------------------------------
1
2
" Item: 23 of 46
Q.ld: 1141 [
111 f> Mark .<:J
Previous
[::>
Next
il
Lab Values
~
Notes
~
Calculator

CJ
5
A 34-year-old male begins treatment w ith combination chemotherapy for Hodgkin's lymphoma. Shortly
thereafter, he experiences severe vomiting and requires intravenous fluid supplementation. W hich of the
6 follow ing brain sites is responsible for his symptoms?
7
8
9
10
11
12
13
14
15
16
17
18
19
20
21
22

24
25
26
27
28
29
30
31
32
33
34
35
36
37
38 0 A. A
39 0 B. B
40
41
42
~imeRemaini~ ~ 0
I Tuto' Feedback Suspend End Block
---------------------------------------------------------------------------------------------------------------------------------------------
1
2

CJ
5
6
7
8
9
10
11
12
13
14
15
16
17
18
19
20
21
22

24
25
26
27
28
29
30
31
32
33
34 0 A. A
35 0 B. B
36
37 0 C. C
38 0 D. D
39 0 E. E
40
41
42
~imeRemaini~ ~ 0
I Tuto' Feedback Suspend End Block
----------------------------
1
2

CJ
5
6
7
8
9
10 0 A.A [19%]
11
12 0 B. B [1 8%]
13 0 C. C [11 %]
14 .; @ D. D [44%]
15
16 0 E. E [6%]
17
18
Explanation: User ld:
19
20 The vomiting that results from administration of systemic chemotherapy is believed to be triggered by the
21 chemoreceptor trigger zone (CTZ), located on the dorsal surface of the medulla at the caudal end of the fourth
22 ventricle (Choice D) in a region known as the area postrema. The area postrema receives blood from
fenestrated vessels (absent blood-brain barrier), which allows it to sample chemicals circulating in the blood.
24
25 (Choice A} This is the thalamus, which helps to coordinate the sensory and motor systems.
26
27 (Choice B) This dorsal midbrain site is the location of the superior and inferior colliculi. The third ventricle is
28 located dorsal to these structures.
29
30 (Choice C) This is the ventral pons. Fibers of cranial nerves V-VIII, the corticospinal tract, the medial
31 lemniscus, and the lateral spinothalamic tract course through this region .
32
(Choice E) The cerebellum is marked as choice E.
33
34 Educational Objective:
35 Acute nausea following administration of systemic chemotherapy results from stimulation of the
36 chemoreceptor trigger zone (CTZ), which lies in the area postrema of the dorsal medulla near the fourth
37 ventricle.
38
39
Time Spent: 11 seconds Copyright USMLEWorld,LLC. Last updated: [4/4/2014]
40
41
42
~imeRemaini~ ~ 0
I Tuto' Feedback Suspend End Block
--------------------------------------------------------------------------------------------------------------------------------------
1
2
" Item: 24 of 46
Q.ld: 8294 [
111 f> Mark .<:J
Previous
[::>
Next
il
Lab Values
~
Notes
~
Calculator

CJ
5
A 33-year-old man comes to the emergency department complaining of fever, chills, and dyspnea on
exertion . He admits to smoking one pack of cigarettes a day, consuming 2-3 alcoholic beverages daily, and
6 using intravenous illicit drugs 2-3 times per w eek. He is febrile and tachycardic. Blood cultures grow
7 Staphylococcus aureus. A diagnosis of infective endocarditis is established. The valve most likely affected in
8 this patient can be best evaluated by auscultation in w hich of the follow ing areas?
9
10
11
12
13
14
15
16
17
18
19
20
21
22
23

25
26
27
28
29
30
31
32
33
34
35
36
37
38 0 A. A
39 0 B. B
40

~
41
42 F!!ck
---------------------------------------------------------------------------------------------------------------------------------------------
suW.nd EnQck
1
2
" Item: 24 of 46
Q.ld: 8294 [
111 f> Mark .<:J
Previous
[::>
Next
il
Lab Values
~
Notes
~
Calculator

CJ
5
us1ng Intravenous 1111c1t drugs :i-3 t1mes per weeK. He IS tebnle and tachycard1c. l::llood cultures grow
Staphylococcus aureus. A diagnosis of infective endocarditis is established. The valve most likely affected in
6 this patient can be best evaluated by auscultation in which of the following areas?
7
8
9
10
11
12
13
14
15
16
17
18
19
20
21
22
23

25
26
27
28
29
30
31
32
~U,C () )01 1
33
34
35 0 A. A
36
37 0 B. B
38 0 C. C
39 0 D. D
40

~
41
42 F!!ck
---------------------------------------------------------------------------------------------------------------------------------------------
suW.nd EnQck
1
2
" Item: 24 of 46
Q.ld: 8294 [
111 f> Mark .<:J
Previous
[::>
Next
il
Lab Values
~
Notes
~
Calculator

CJ
5
.; @ C. C [81%]
0 D. D [13%]
6
7
8 Explanation: User ld:
9
10
11
12 Aortic area
13
14
15 Pulmonic area
16
17
18
19 Tricuspid area
20
21
22
23
Mitral area
25
26
27
28
29
30
31
32
33
34
35
36
USMl.lWotld.UC 0 1011
37
38
39 Right-sided endocarditis involving the tricuspid valve commonly occurs in IV drug users and is most often due
40 to Staphylococcus aureus. Specifically, infective endocarditis can cause tricuspid requrQitation , which is
41
42
~imeRemaini~ ~ 0
I Tuto' Feedback
---------------------------------------------------------------------------------------------------------------------------------------------
Suspend End Block
1
2
" Item: 24 of 46
Q.ld: 8294 [
111 f> Mark .<:J
Previous
[::>
Next
il
Lab Values
~
Notes
~
Calculator

CJ
5
6
7 USKI'Nolld.UC O lOi t
8
9
Right-sided endocarditis involving the tricuspid valve commonly occurs in IV drug users and is most often due
10
to Staphylococcus aureus. Specifically, infective endocarditis can cause tricuspid regurgitation, w hich is
11
identified as an early systolic murmur best heard over the left lower sternal border that is accentuated by
12
inspiration .
13
14 Heart sounds/murmurs result from reverberation of blood secondary to valve closure. Auscultation to
15 determine function of a valve is performed at the position closest to w here the sound w aves reverberate and
16 not at the position of the valve.
17
18
19 Location best heard
Va lve
20
21
22 Aortic valve Second right intercostal space at the right sternal border
23

25 Pulmonic valve Second left intercostal space at the left sternal border
26
27
28 Tricuspid valve Fourth left intercostal space at the lower left sternal border
29
30
31 Mitral valve Fifth left intercostal space, medial to the mid-clavicular line
32
33 @ US.MLEWo>1d. LlC
34
35 Educational objective:
36 An early systolic murmur best heard over the left lower sternal border that is accentuated by inspiration is
37 most likely due to tricuspid regurgitation .
38
39
Time Spent: 10 seconds Copyright USMLEWorld,LLC. Last updated: [1 / 12/2014]
40
41
42
~imeRemaini~ ~ 0
I Tuto' Feedback Suspend End Block
--------------------------------------------------------------------------------------------------------------------------------------
1
2
" Item: 25 of 46
Q.ld : 513 [
111 f> Mark .<:J
Previous
[::>
Next
il
Lab Values
~
Notes
~
Calculator

CJ
5
6 A 26-year-old female presents to your office w ith a three-month history of amenorrhea . She also notes that
7 during this time her breasts have become engorged . She is taking a drug that helps her 'not to hear voices'
8 and acetaminophen for occasional headaches. You proceed w ith giving her a pregnancy test that returns w ith
9 a negative result. W hich of the follow ing dopaminergic pathw ay disturbances are most likely responsible for
10 this patient's symptoms?
11
12
13 0 A. Mesolimbic-mesocortical
14
0 B. Nigrostriatal
15
16 0 C. Tuberoinfundibular
17
0 D. Periventricular
18
19 0 E. lncertohypothalamic
20
21
22
23
24

26
27
28
29
30
31
32
33
34
35
36
37
38
39
40
41
42
~imeRemaini~ ~ 0
I Tuto' Feedback Suspend End Block
---------------------------------------------------------------------------------------------------------------------------------------------
1
2
" Item: 25 of 46
Q.ld : 513
111 f> Mark .<:J
Previous
[::>
Next
il
Lab Values
~
Notes
~
Calculator

CJ
5
.; @ C. Tuberoinfundibular [56%]
0 D. Periventricular [4%]
6
7 0 E. lncertohypothalamic [14%]
8
9
Explanation: User ld:
10
11 The patient appears to be suffering from galactorrhea, a non-puerperal secretion of w atery or milky fluid that
12 contains neither pus nor blood. The tuberoinfundibular dopaminergic pathw ay connects the hypothalamus
13 and the pituitary gland and is responsible for dopamine-dependent prolactin tonic inhibition. The
14 neurotransmitter dopamine is secreted from these neural axons and inhibits prolactin secretion from the
15 anterior pituitary gland. Some antipsychotic drugs block dopamine in the tuberoinfundibular pathway, w hich
16 can cause an increase in blood prolactin levels, leading to hyperprolactinemia and abnormal lactation
17 (galactorrhea), disruptions to the menstrual cycle in w omen, and sexual dysfunction.
18
19 (Choice A) The mesolimbic-mesocortical pathway is primarily involved in regulating behavior. This area is
20 hyperactive in schizophrenia . Antipsychotic action of the neuroleptics is by primarily blocking the dopamine in
21 the mesolimbic-mesocortical (frontal cortex) pathway.
22
23 (Choice B) In the nigrostriatal system the neurons are projected from the substancia nigra to the caudate
24 nucleus and putamen. This pathway primarily regulates coordination of voluntary movements. In this
pathway dopamine inhibits the release of acetylcholine. Degeneration of the substantia nigra thus causes
26 decreased dopamine and subsequent increased acetylcholine; this leads to movement disorders such as
27 Parkinsonism. Administering high potency antipsychotics can cause some dopamine antagonism in this
28 pathway resulting in drug-induced Parkinsonism .
29
30 (Choices D and E) The other 2 pathways are not really w ell defined and not important for purpose of board
31 examination .
32
33 Educational Objective:
34 Remember the 3 important Dopaminergic systems:
35 Pathway Function Diseases associated
36 Mesolimbic-mesocortical Regulates behavior Schizophrenia
37 Coordination of voluntary
Nigrostriatal Parkinsonism
38 movements
39 Tuberoinfundibular Controls prolactin secretion Hyperprolactinemia
40
41
42
~imeRemaini~ ~ 0
I Tuto' Feedback Suspend End Block
---------------------------------------------------------------------------------------------------------------------------------------------
1
2
" Item: 25 of 46
Q.ld : 513 [
111 f> Mark .<:J
Previous
[::>
Next
il
Lab Values
~
Notes
~
Calculator

CJ
5
0
0
D. Periventricular [4%]
E. lncertohypothalamic [14%]
6
7
8 Explanation: User ld:
9
10 The patient appears to be suffering from galactorrhea, a non-puerperal secretion of w atery or milky fluid that
11 contains neither pus nor blood. The tuberoinfundibular dopaminergic pathw ay connects the hypothalamus
12 and the pituitary gland and is responsible for dopamine-dependent prolactin tonic inhibition. The
13 neurotransmitter dopamine is secreted from these neural axons and inhibits prolactin secretion from the
14 anterior pituitary gland. Some antipsychotic drugs block dopamine in the tuberoinfundibular pathw ay, w hich
15 can cause an increase in blood prolactin levels, leading to hyperprolactinemia and abnormal lactation
16 (galactorrhea), disruptions to the menstrual cycle in w omen, and sexual dysfunction.
17
18 (Choice A) The mesolimbic-mesocortical pathw ay is primarily involved in regulating behavior. This area is
19 hyperactive in schizophrenia . Antipsychotic action of the neuroleptics is by primarily blocking the dopamine in
20 the mesolimbic-mesocortical (frontal cortex) pathw ay.
21
22 (Choice B) In the nigrostriatal system the neurons are projected from the substancia nigra to the caudate
23 nucleus and putamen. This pathway primarily regulates coordination of voluntary movements. In this
24 pathway dopamine inhibits the release of acetylcholine. Degeneration of the substantia nigra thus causes
decreased dopamine and subsequent increased acetylcholine; this leads to movement disorders such as
26 Parkinsonism . Administering high potency antipsychotics can cause some dopamine antagonism in this
27 pathway resulting in drug-induced Parkinsonism.
28
(Choices D and E) The other 2 pathways are not really w ell defined and not important for purpose of board
29
examination .
30
31 Educational Objective:
32 Remember the 3 important Dopaminergic systems:
33 Pathway Function Diseases associated
34 Mesolimbic-mesocortical Regulates behavior Schizophrenia
35 Coordination of voluntary
36 Nigrostriatal Parkinsonism
movements
37 Tuberoinfundibular Controls prolactin secretion Hyperprolactinemia
38
39
Time Spent: 2 seconds Copyright USMLEW orld,LLC. Last updated: [2/16/2012]
40
41
42
~imeRemaini~ ~ 0
I Tuto' Feedback Suspend End Block
---------------------------------------------------------------------------------------------------------------------------------------------
1
2
" Item: 26 of 46
Q.ld: 1636 [
111 f> Mark .<:J
Previous
[::>
Next
il
Lab Values
~
Notes
~
Calculator

CJ
5
6 A young athlete presents to your office after sustaining an injury to the right upper extremity. Physical
7 examination reveals w eak right forearm flexion and an absent biceps reflex. Given these findings, sensation
8 loss in w hich of the following areas is most likely to be found in this patient?
9
10
11 0 A. Posterior arm
12 0 B. Posterior forearm
13
14 0 C. Lateral forearm
15 0 D. Medial forearm
16
17 0 E. Thenar eminence
18
19
20
21
22
23
24
25

27
28
29
30
31
32
33
34
35
36
37
38
39
40
41
42
~imeRemaini~ ~ 0
I Tuto' Feedback Suspend End Block
---------------------------------------------------------------------------------------------------------------------------------------------
1
2
" Item: 26 of 46
Q.ld: 1636 [
111 f> Mark .<:J
Previous
[::>
Next
il
Lab Values
~
Notes
~
Calculator

CJ
5
examination reveals w eak right forearm flexion and an absent biceps reflex. Given these findings, sensation
loss in w hich of the follow ing areas is most likely to be found in this patient?
6
7 0 A. Posterior arm [8%1
8
9 0 B. Posterior forearm [7%1
10 ~ @ C. Lateral forearm [51 %1
11 0 D. Medial forearm [24%1
12
13 0 E. Thenar eminence [8%1
14
15 Explanation: User ld:
16
17 This patient has symptoms suggestive of injury to the musculocutaneous nerve. The musculocutaneous
18 nerve is derived from the C5-C7 ventral rami . As its name implies, the musculocutaneous nerve provides
19 both motor innervation and somatic sensory innervation to the arm . It innervates the major upper arm flexors
20 including the biceps brachii, coracobrachialis, and brachialis muscles. After innervating these muscles, the
21 remaining fibers continue as the lateral cutaneous nerve of the forearm, providing sensory innervation to the
22 skin of the lateral forearm .
23
24 (Choice A) The posterior arm is innervated by a branch of the radial nerve (posterior cutaneous nerve of the
25 arm).

27 (Choice B) The posterior forearm is innervated by a branch of the radial nerve (posterior cutaneous nerve of
28 the forearm).
29
30 (Choice D) The medial forearm is innervated by a branch of the ulnar nerve (medial cutaneous nerve of the
31 forearm).
32
(Choice E) The thenar eminence is innervated by the recurrent branch of the median nerve.
33
34 Educational objective:
35 The musculocutaneous nerve innervates the flexor muscles of the upper arm and provides sensory
36 innervation to the lateral forearm. The musculocutaneous nerve is derived from the upper trunk of the brachial
37 plexus and can be injured by forceful injuries that cause separation of the neck and shoulder.
38
39
Time Spent: 1 seconds Copyright USMLEW orld,LLC. Last updated: [1 / 1/20141
40
41
42
~imeRemaini~ ~ 0
I Tuto' Feedback Suspend End Block
---------------------------------------------------------------------------------------------------------------------------------------------
1
2
" Item: 27 of 46
Q.ld: 1742 [
111 f> Mark .<:J
Previous
[::>
Next
il
Lab Values
~
Notes
~
Calculator

CJ
5
6 A patient presents to your office w ith right leg numbness. W hen he w alks, you notice that he lifts his right foot
7 higher than he does his left, and that his right foot slaps to the ground w ith each step. On neurologic exam,
8 you also ascertain that he is unable to evert the right foot. W hich of the follow ing nerves has most likely been
9 injured?
10
11
12 0 A. Tibial
13 0 B. Common peroneal
14
15 0 C. Superficial peroneal
16 0 D. Sural
17
18 0 E. Obturator
19
20
21
22
23
24
25
26

28
29
30
31
32
33
34
35
36
37
38
39
40
41
42
~imeRemaini~ ~ 0
I Tuto' Feedback Suspend End Block
---------------------------------------------------------------------------------------------------------------------------------------------
1
2
" Item: 27 of 46
Q.ld: 1742 [
111 f> Mark .<:J
Previous
[::>
Next
il
Lab Values
~
Notes
~
Calculator

CJ
5
., @ B. Common peroneal [70%]

6 0 C. Superficial peroneal [12%]


7 0 D. Sural [2%]
8 0 E. Obturator [3%]
9
10
11 Explanation: User ld:
12
13 Sciatic nerve
14
15
16
17
18
19 on peroneal nerve
20
21
22
23
24
25
26
Tibial nerve Deep peroneal nerve
28
29
30
31
32
33
34
35
36
37
38
39 The common peroneal nerve is the most commonly injured nerve in the leg. It begins in the proximal popliteal
40 fossa, w here the sciatic nerve divides into the tibial nerve and common peroneal nerve. From there, the
41
42
~imeRemaini~ ~ 0
I Tuto' Feedback Suspend End Block
---------------------------------------------------------------------------------------------------------------------------------------------
1
2

CJ
5
6
7
8
9
10
The common peroneal nerve is the most commonly injured nerve in the leg. It begins in the proximal popliteal
11
fossa, w here the sciatic nerve divides into the tibial nerve and common peroneal nerve. From there, the
12
common peroneal nerve courses laterally into the anterior compartment of the leg, passing superficially
13
around the head and neck of the fibula. Blunt trauma to the lateral leg and fractures of the neck of the fibula
14
can injure this nerve.
15
16 Patients w ith common peroneal nerve damage present w ith an equinovarus (plantarflexed and inverted)
17 posture of the affected foot due to paralysis of the peroneus longus and peroneus brevis muscles (mediate
18 foot eversion), paralysis of the tibialis anterior muscle (mediates dorsiflexion), and paralysis of the extrinsic
19 extensors of the toes. Injury to this nerve also causes loss of sensation to the anterolateral leg.
20
21 The classic finding on gait exam in patients w ith common peroneal nerve injury is "foot drop," w here the
22 affected leg is lifted high off of the ground w hile w alking due to an inability to dorsiflex the foot. The affected
23 foot w ill also classically slap to the ground w ith each step .
24
25 (Choice A) Tibial nerve injury causes dorsiflexion and eversion of the foot w ith sensory loss on the sole of the
26 foot.

28 (Choice C) The superficial peroneal nerve innervates the peroneus longus and peroneus brevis muscles in
29 the lateral compartment of the leg, w hich together evert the foot. It also provides sensory innervation to the
30 skin of the distal anterior leg and dorsal foot.
31
32 (Choice D) The sural nerve is a purely sensory nerve that supplies the posterolateral leg and lateral foot.
33
(Choice E) Obturator nerve injury classically causes spasms of the adductor muscles of the thigh.
34
35 Educational Objective:
36 Common peroneal nerve injury is common and typically results from trauma to the leg near the head of the
37 fibula. Signs include foot drop and a characteristic high-stepping gait.
38
39
Time Spent: 1 seconds Copyright USMLEW orld,LLC. Last updated: [7/7/201 0]
40
41
42
~imeRemaini~ ~ 0
I Tuto' Feedback Suspend End Block
---------------------------------------------------------------------------------------------------------------------------------------------
1
2
" Item: 28 of 46
Q.ld: 1687 [
111 f> Mark .<:J
Previous
[::>
Next
il
Lab Values
~
Notes
~
Calculator

CJ
5
A 45-year-old male presents with left arm clumsiness. Physical examination reveals motor w eakness and
6
increased muscle tone in the left arm. Specifically, with passive extension of the left arm there is initial
7
resistance then sudden release when maximum extension is reached. Pathology within which of the following
8
brain structures is most likely responsible?
9
10
11
12
13
14
15
16
17
18
19
20
21
22
23
24
25
26
27

29
30 Dlspl1yed with permission from Sprinaer Hnlthcue Ltd.
31 CCopyrlcht 1995, 2004 by Current Medicine
32
33
34 0 A. A
35 0 B. B
36
37 0 C. C
38 0 D. D
39 0 E. E
40
41
42
~imeRemaini~ ~ 0
I Tuto' Feedback Suspend End Block
------------------------------------------------------------------------------------------------------------------------------------------------------
1
2

CJ
5
6 Dlspl1yed with permi ssion from Sprlncer Hnlthure Ltd.
7 CCopyrlcht 1995, 2004 by C.urrent Medicine
8
9
10 0 A.A [1 5%]
11
.; @ B. B [42%]
12
13 0 C. C [9%]
14 0 D. D [1 7%]
15
16 0 E. E [1 6%]
17
18
Explanation: User ld:
19
20 This patient has "clasp knife spasticity," characterized by initial jerking resistance to passive extension
21 follow ed by sudden release of resistance . This form of spasticity is typical of an upper motor neuron lesion.
22 Potential sites for upper motor neuron lesions include the corticospinal tracts of the spinal cord; the medulla,
23 pons and midbrain; the internal capsule; or the precentral gyrus (primary motor cortex). Of these, the internal
24 capsule is the only structure labeled above (Choice B). In general, upper motor neuron lesions cause spastic
25 paresis, hyperreflexia, and a positive Babinski sign. The clasp knife phenomenon is a form of spasticity that
26 results from a lack of upper motor neuron inhibitory control on the spinal muscle stretch reflex arc.
27
(Choice A) This is the caudate nucleus, the structure affected by Huntington's disease.
29
30 (Choice C) This is the insular cortex (insula), w hich plays a role in the limbic system (emotion).
31
32 (Choice D) This is the putamen .
33
(Choice E) This is the globus pallidus.
34
35 Educational Objective:
36 Upper motor neuron lesions cause spastic rigidity, hyperreflexia, and paresis. Corticospinal tract, internal
37 capsule (posterior limb), and primary motor cortex lesions can cause these symptoms.
38
39
Time Spent: 9 seconds Copyright USMLEWorld,LLC. Last updated: [11 / 10/2011]
40
41
42
~imeRemaini~ ~ 0
I Tuto' Feedback Suspend End Block
---------------------------------------------------------------------------------------------------------------------------------------------
1
2
" Item: 29 of 46
Q.ld : 2022 [
111 f> Mark .<:J
Previous
[::>
Next
il
Lab Values
~
Notes
~
Calculator

CJ
5
6 A 45-year-old patient presents w ith vague abdominal pain and undergoes CT imaging of the abdomen (see
7 slide below). W hich of the follow ing labeled organs is supplied by an artery of the foregut although it is not a
8 foregut derivative?
9
10
11
12
13
14
15
16
17
18
19
20
21
22
23
24
25
26
27
28

30
31
32
33 0 A. A
34 0 B.B
35 0 C. C
36
37 0 D. D
38 0 E. E
39
40

~
41
42 F!!ck
---------------------------------------------------------------------------------------------------------------------------------------------
suW.nd EnQck
1
2
" Item: 29 of 46
Q.ld : 2022 [
111 f> Mark .<:J
Previous
[::>
Next
il
Lab Values
~
Notes
~
Calculator

CJ
5
0 A.A[11%]

6 0 B. B [8%]
7 0 C. C [10%]
8 0 D. D [18%]
9
10 v @ E. E [53%]
11
12 Explanation: User ld:
13
14 The "E" above labels the spleen, a mesoderm-derived organ that originates from the dorsal mesentery. The
15 spleen receives its blood supply from the splenic artery, w hich is a branch of the celiac trunk, the primary
16 artery of the foregut. The foregut includes the esophagus through the second part of the duodenum, the liver
17 (Choice A), the gallbladder, and a portion of the pancreas. Unlike the spleen, the pancreas and liver develop
18 as outpouchings of the primitive gut tube during embryogenesis.
19
20 (Choice A) The liver is an endoderm-derived foregut structure that forms from a diverticulum of the primitive
21 gut tube w ithin the ventral mesentery. Its main arterial supply is the proper hepatic artery, a branch of the
22 celiac trunk.
23
24 (Choice B) The kidneys are retroperitoneal organs derived from the mesoderm. They are not derived from
25 the gut.
26
27 (Choice C) The first 213 of the transverse colon is considered part of the midgut, and the last 1/3 is part of the
28 hindgut. The superior mesenteric artery supplies the midgut portion of the transverse colon via the middle
colic artery w hile the inferior mesenteric artery supplies the distal portion via the left colic artery. These
30 vessels anastomose via the marginal artery.
31
(Choice D) The pancreas is an endoderm-derived structure of the foregut and midgut. It receives blood from
32
the superior and inferior pancreaticoduodenal arteries as w ell as other arterial branches of the celiac trunk and
33
superior mesenteric artery.
34
35 Educational Objective:
36 The spleen is not a gut derivative; it forms from the mesodermal dorsal mesentery. The splenic artery,
37 how ever, is a branch of the celiac trunk (the primary blood supply of the foregut).
38
39
Time Spent: 10 seconds Copyright USMLEWorld,LLC. Last updated: [7/7/2010]
40
41
42
~imeRemaini~ ~ 0
I Tuto' Feedback Suspend End Block
--------------------------------------------------------------------------------------------------------------------------------------
1
2
" Item: 30 of 46
Q.ld: 1634 [
111 f> Mark .<:J
Previous
[::>
Next
il
Lab Values
~
Notes
~
Calculator

CJ
5
6 A 19-year-old female presents to your office complaining of "right hand clumsiness." Physical examination
7 reveals decreased sensation over the fifth finger and a flattened hypothenar eminence. The affected nerve is
8 commonly injured at w hich of the following locations?
9
10
11 0 A. Carpal tunnel
12
0 B. Hook of the hamate
13
14 0 C. Surgical neck of the humerus
15
16
0 D. Head of the radius
17 0 E. Coracobrachialis muscle
18
19
20
21
22
23
24
25
26
27
28
29

31
32
33
34
35
36
37
38
39
40
41
42
~imeRemaini~ ~ 0
I Tuto' Feedback Suspend End Block
---------------------------------------------------------------------------------------------------------------------------------------------
1
2
" Item: 30 of 46
Q.ld: 1634 [
111 f> Mark .<:J
Previous
[::>
Next
il
Lab Values
~
Notes
~
Calculator

CJ
5
~ @ B. Hook of the hamate [73%]

0 C. Surgical neck of the humerus [5%]


6
7 0 D. Head of the radius [8%]
8 0 E. Coracobrachialis muscle [3%]
9
10
11 Explanation: User ld:
12
13 The ulnar ne!Ve is a branch of the medial cord of the brachial plexus derived from the C8-T1 ventral rami . The
14 ulnar ne!Ve provides sensory inne!Vation to the fifth digit and the medial half of the fourth digit as w ell as to the
15 palmar and dorsal surfaces of the hand. The ulnar neiVe also provides motor inneiVation to the flexor carpi
16 ulnaris and to the medial section of the flexor digitorum profundis in the forearm . In the hand, the ulnar neiVe
17 inne!Vates all of the palmar and dorsal interosseus muscles, the muscles of the hypothenar eminence, the
18 palmaris brevis muscle, the third and fourth lumbrical muscles, and the adductor pollicis muscle.
19
The ulnar ne!Ve enters the forearm after passing behind the medial epicondyle of the humerus w here it is
20
covered by a small amount of overlying soft tissue. This region, sometimes referred to as the "funny bone," is
21
a common site of ulnar neiVe injury. Ulnar ne!Ve injury at this site classically causes a "claw hand" deformity.
22
In the w rist, the ulnar ne!Ve passes between the hook of the hamate and the pisiform bone in a fibroosseous
23
tunnel know n as Guyon's canal. Ulnar ne!Ve injury at Guyon's canal causes dysesthesia of the ulnar side of
24
the hand and w eakness of the intrinsic muscles of the hand.
25
26 (Choice A) In carpal tunnel syndrome, some factor reduces the size of the carpal tunnel causing median
27 ne!Ve compression. Patients typically experience difficulty w ith fine motor control of the thumb.
28
29 (Choice C) Fracture of the surgical neck of the humerus may cause axillary ne!Ve injury leading to paralysis
of the deltoid and teres minor muscles as w ell as loss of sensation of the lateral upper arm .
31
32 (Choice D) The deep branch of the radial ne!Ve can be affected by radial head subluxation leading to
33 w eakness or paralysis of many of the muscles of the ex1ensor compartment of the forearm.
34
35 (Choice E) The coracobrachialis muscle is an arm flexor that lies deep to the biceps brachii and overlies the
36 median ne!Ve and brachial artery. It is inne!Vated by the musculocutaneous ne!Ve.
37
38 Educational Objective:
39 Ulnar ne!Ve injury classically causes a "claw hand" deformity. The ulnar ne!Ve can be injured either near the
40 medial epicondyle of the humerus or in Guyon's canal near the hook of the hamate and pisiform bone in the
41
42
~imeRemaini~ ~ 0
I Tuto' Feedback Suspend End Block
---------------------------------------------------------------------------------------------------------------------------------------------
1
2

CJ
5 0 E. Coracobrachialis muscle [3%]
6
7
Explanation: User ld:
8
9 The ulnar nerve is a branch of the medial cord of the brachial plexus derived from the C8-T1 ventral rami. The
10 ulnar nerve provides sensory innervation to the fifth digit and the medial half of the fourth digit as w ell as to the
11 palmar and dorsal surfaces of the hand. The ulnar nerve also provides motor innervation to the flexor carpi
12 ulnaris and to the medial section of the flexor digitorum profundis in the forearm . In the hand, the ulnar nerve
13 innervates all of the palmar and dorsal interosseus muscles, the muscles of the hypothenar eminence, the
14
palmaris brevis muscle, the third and fourth lumbrical muscles, and the adductor pollicis muscle.
15
16 The ulnar nerve enters the forearm after passing behind the medial epicondyle of the humerus w here it is
17 covered by a small amount of overlying soft tissue. This region, sometimes referred to as the "funny bone," is
18 a common site of ulnar nerve injury. Ulnar nerve injury at this site classically causes a "claw hand" deformity.
19 In the w rist, the ulnar nerve passes between the hook of the hamate and the pisiform bone in a fibroosseous
20 tunnel know n as Guyon's canal. Ulnar nerve injury at Guyon's canal causes dysesthesia of the ulnar side of
21 the hand and w eakness of the intrinsic muscles of the hand.
22
23 (Choice A) In carpal tunnel syndrome, some factor reduces the size of the carpal tunnel causing median
24 nerve compression. Patients typically experience difficulty w ith fine motor control of the thumb.
25
26 (Choice C) Fracture of the surgical neck of the humerus may cause axillary nerve injury leading to paralysis
27 of the deltoid and teres minor muscles as w ell as loss of sensation of the lateral upper arm.
28
29 (Choice D) The deep branch of the radial nerve can be affected by radial head subluxation leading to
w eakness or paralysis of many of the muscles of the ex1ensor compartment of the forearm.
31
(Choice E) The coracobrachialis muscle is an arm flexor that lies deep to the biceps brachii and overlies the
32
median nerve and brachial artery. It is innervated by the musculocutaneous nerve.
33
34 Educational Objective:
35 Ulnar nerve injury classically causes a "claw hand" deformity. The ulnar nerve can be injured either near the
36 medial epicondyle of the humerus or in Guyon's canal near the hook of the hamate and pisiform bone in the
37 w rist.
38
39
Time Spent: 2 seconds Copyright USMLEW orld,LLC. Last updated: [7/ 17/2012]
40
41
42
~imeRemaini~ ~ 0
I Tuto' Feedback Suspend End Block
---------------------------------------------------------------------------------------------------------------------------------------------
1
2
" Item: 31 of 46
Q.ld : 292 [
lil f> Mark .<:J
Previous
[::>
Next
il
Lab Values
~
Notes
~
Calculator

CJ
5
6 A 55-year-old man is brought to the emergency department w ith profuse vomiting. Over the last hour, he has
7 vomited large amounts of bright red blood. His past medical history is significant for long-term use of
8 antacids. The patient's blood pressure is 70/40 mm Hg and pulse is 120/min. He dies 1 hour after admission
9 despite resuscitation efforts. Autopsy show s a deep peptic ulcer localized proximally on the lesser curvature
10 of the stomach . The ulcer most likely penetrated w hich of the follow ing arteries?
11
12
13 0 A. Common hepatic
14 0 B. Right gastroepiploic
15
16 0 C. Left gastric
17 0 D. Gastroduodenal
18
19 0 E. Inferior pancreaticoduodenal
20 0 F. Splenic
21
22 0 G. Superior mesenteric
23
24
25
26
27
28
29
30

32
33
34
35
36
37
38
39
40
41
42
~imeRemaini~ ~ 0
I Tuto' Feedback Suspend End Block
---------------------------------------------------------------------------------------------------------------------------------------------
1
2
" Item: 31 of 46
Q.ld: 292 [
lil f> Mark .<:J
Previous
[::>
Next
il
Lab Values
~
Notes
~
Calculator

CJ
5
~ @ C. l eft gastric [67%]
0 D. Gastroduodenal [11 %]
6
7 0 E. Inferior pancreaticoduodenal [1 %]
8 0 F. Splenic [6%]
9
0 G. Superior mesenteric [1 %]
10
11
12 Explanation: User ld:
13
14
Stomach vasculature
15
16
17 Esophageal branch
18
19 Celiac trunk
20
21 Common hepatic artery Short gastric arteries
22
23 left & right hepatic arteries
24
25
26 Cystic artery
27
28
29
30

32
33
34
35
36 Supraduodenal ;ut<>nr-__,.- J l eft gastroepiploic artery
37
38
39 Duodenal bulb /
40
41
42
1
2
" Item: 31 of 46
Q.ld : 292 [
lil f> Mark .<:J
Previous
[::>
Next
il
Lab Values
~
Notes
~
Calculator

CJ
5
Stomach vasculature
6
7 Esophageal branch
8
9 Celiac trunk
10
11 Common hepatic artery Short gastric arteries
12
13 left & right hepatic arteries
14
15
16 Cystic artery
17
18
19
20 Right gastric artery
21
22
23
24
25
26 Supraduodenal l eft gastroepiploic artery
27
28
29 Duodena l bulb /
30
Right gastroepiploic artery
32
Superior
33
34 Pancreaticoduodenal artery
USMLEWortd, lLC
35
36
37 Most gastric ulcers arise along the lesser curvature of the stomach. They are usually located at the
38 transitional zone between the acid-secreting epithelium of the gastric corpus and gastrin-producing epithelium
39 of the antrum. Glands in the corpus contain parietal cells that produce HCI and intrinsic factor; mucosal
40 glands in the antrum contain G-cells that secrete gastrin. The predilection of ulcers for the transitional zone
41
42
~imeRemaini~ ~ 0
I Tuto' Feedback Suspend End Block
--------------------------------------------------------------------------------------------------------------------------------------
1
2
" Item: 31 of 46
Q.ld : 292 [
lil f> Mark .<:J
Previous
[::>
Next
il
Lab Values
~
Notes
~
Calculator

CJ
5
Most gastric ulcers arise along the lesser curvature of the stomach. They are usually located at the
transitional zone between the acid-secreting epithelium of the gastric corpus and gastrin-producing epithelium
6
of the antrum . Glands in the corpus contain parietal cells that produce HCI and intrinsic factor; mucosal
7
glands in the antrum contain G-cells that secrete gastrin. The predilection of ulcers for the transitional zone
8
may be due to the microenvironment having an optimal pH for H e/icobacter py/otiproliferation, or perhaps non-
9
optimal local conditions induce the expression of inflammatory bacterial stress proteins.
10
11 A number of arteries supply the stomach:
12
13 Anatomic area Supplying artery Source artery I
14 Proximal lesser curvature Celiac
Left gastric I
15
16 Proper hepatic +-
17 Distal lesser curvature Right gastric common hepatic +-
18 celiac
19 Splenic+-
20 Proximal greater curvature Left gastroepiploic
celiac
21
22 Gastroduodenal +-
23 Distal greater curvature Right gastroepiploic common hepatic +-
24 celiac
25 Proximal greater curvature Splenic+-
26 Short gastric arteries
above splenic artery celiac I
27
28
The left and right gastric arteries run along and perfuse the lesser curvature of the stomach. They are likely to
29
be penetrated by ulcers, leading to hemorrhage.
30
(Choice A) The common hepatic artery perfuses the liver, gallbladder, pylorus, duodenum, and pancreas. It
32 arises from the celiac trunk and ends w hen it bifurcates into the proper hepatic and gastroduodenal arteries.
33
34 (Choice B) The right gastroepiploic artery arises from gastroduodenal artery and perfuses the distal greater
35 curvature of the stomach. Gastric ulcers do not commonly occur in the greater curvature.
36
37 (Choice D) The gastroduodenal artery supplies blood to the pylorus and the proximal part of the
38 duodenum. Ulcers in the posterior duodenal bulb can erode into the gastroduodenal artery and cause
39 bleeding.
40
41
42
~imeRemaini~ ~ 0
I Tuto' Feedback Suspend End Block
---------------------------------------------------------------------------------------------------------------------------------------------
1
2
" Item: 31 of 46
Q.ld : 292 [
lil f> Mark .<:J
Previous
[::>
Next
il
Lab Values
~
Notes
~
Calculator

CJ
5
Distal lesser curvature Right gastric
I I VtJCI IICtJOU\.. -

common hepatic ._
6 celiac
7 Splenic._
Proximal greater curvature l eft gastroepiploic
8 celiac
9
Gastroduodenal +-
10
Distal greater curvature Right gastroepiploic common hepatic +-
11
celiac
12
13 Proximal greater curvature Splenic+-
Short gastric arteries
14 above splenic artery celiac I
15
16 The left and right gastric arteries run along and perfuse the lesser curvature of the stomach. They are likely to
17 be penetrated by ulcers, leading to hemorrhage.
18
19 (Choice A) The common hepatic artery perfuses the liver, gallbladder, pylorus, duodenum, and pancreas. It
20 arises from the celiac trunk and ends w hen it bifurcates into the proper hepatic and gastroduodenal arteries.
21
22 (Choice B) The right gastroepiploic artery arises from gastroduodenal artery and perfuses the distal greater
23 curvature of the stomach . Gastric ulcers do not commonly occur in the greater curvature.
24
25 (Choice D) The gastroduodenal artery supplies blood to the pylorus and the proximal part of the
26 duodenum. Ulcers in the posterior duodenal bulb can erode into the gastroduodenal artery and cause
27 bleeding.
28
29 (Choices E and G) The superior mesenteric and inferior pancreaticoduodenal arteries do not perfuse the
30 stomach.

(Choice F) The splenic artery arises from the celiac trunk and supplies blood to the spleen. It is separated
32
from the posterior w all of the stomach by the pancreas and is only rarely penetrated by gastric ulcers.
33
34 Educational objective:
35 The great majority of gastric ulcers occur over the lesser curvature of the stomach, at the border between
36 acid-secreting and gastrin-secreting mucosa. The left and right gastric arteries run along the lesser curvature
37 and are likely to be penetrated by ulcers, causing gastric bleeding.
38
39
Time Spent: 1 seconds Copyright USMl EW orld,l lC. l ast updated: [12115/2013]
40
41
42
~imeRemaini~ ~ 0
I Tuto' Feedback Suspend End Block
---------------------------------------------------------------------------------------------------------------------------------------------
1
2
" Item: 32 of 46
Q.ld: 1967 [
111 f> Mark .<:J
Previous
[::>
Next
il
Lab Values
~
Notes
~
Calculator

CJ
5
6 A 54-year-old man comes to the physician for evaluation of exertional chest pain. He undergoes coronary
7 angiography and is found to have severe stenosis of the right coronary artery, left anterior descending, and
8 circumflex coronary arteries. He is referred to a surgeon for coronary artery bypass grafting. During the
9 procedure, a portion of his great saphenous vein is removed and grafted to 1 of his diseased coronary arteries
10 to bypass its atherosclerotic narrow ing. The saphenous vein can be best harvested from w hich of the
11 follow ing sites?
12
13
14 0 A. Just superior to the inguinal ligament
15 0 B. Just inferior to the anterior superior iliac spine
16
17 0 C. Just inferolateral to the pubic tubercle
18 0 D. At the midline of the popliteal fossa
19
20 0 E. Over the lateral aspect of the foot
21
22
23
24
25
26
27
28
29
30
31

33
34
35
36
37
38
39
40
41
42
~imeRemaini~ ~ 0
I Tuto' Feedback Suspend End Block
---------------------------------------------------------------------------------------------------------------------------------------------
1
2
" Item: 32 of 46
Q.ld: 1967 [
111 f> Mark .<:J
Previous
[::>
Next
il
Lab Values
~
Notes
~
Calculator

CJ
5
~ @ C. Just inferolateral to the pubic tubercle [36%1

6 0 D. At the midline of the popliteal fossa [33%1


7 0 E. Over the lateral aspect of the foot [20%1
8
9
10 Explanation: User ld:
11
12 Superficial & deep veins of leg
13
14
15
16
17
18
19
20
21 If-- -- - Saphenofemoral junction
22
23 Saphenous opening
24 (fossa ovale)
25
26 Medial circumflex
27 femoral vein
28
29
30
I-- - Great saphenous vein
31 17'-ii-- - Femoral vein

33 Adductor can aI
34
35 Popliteal vein
36
37
Superior /
38 . -------:,.--1(
. I
L. /_
39
40
gemcu ar vetn
I
41
42
~imeRemaini~ ~ 0
I Tuto' Feedback Suspend End Block
--------------------------------------------------------------------------------------------------------------------------------------
1
2
" Item: 32 of 46
Q.ld: 1967 [
111 f> Mark .<:J
Previous
[::>
Next
il
Lab Values
~
Notes
~
Calculator

CJ
5
Superficial & deep vein s of leg

6
7
8
9
10
11
12
13 fl_- -- - - Saphenofemoral junction
14
15 Saphenous opening
16 (fossa ovale)
17
18 Medial circumflex
19 femoral vein
20
21
22 1-- - Great saphenous vein
23 t-;f-1\-- - Femoral vein
24
25
26
27 Popliteal vein
28
29
Superior _ _ _____,__~/
30
genicular vein
31

33
34
35
36
37
38
39
40
41
42
1
2
" Item: 32 of 46
Q.ld: 1967 [
111 f> Mark .<:J
Previous
[::>
Next
il
Lab Values
~
Notes
~
Calculator

CJ
5 Superior -------'-- --{/
6 genicular vein
7
8
9
10
11
12
13
14 Anterior tibial veins- - - - --t<:n
15
16
17
18
19
20
21
22
23
24
25
26
27
28
29
30
31 Dorsal venous arch ----..,q

33
34 @ USMLEWorld, LLC
35
36 When the left anterior descending artery (LAD) alone is occluded by an atherosclerotic plaque, the left internal
37 rnarnrnary artery (left internal thoracic artery) is the preferred vessel for bypass grafting because it has
38 superior short- and long-terrn patency rates compared to saphenous vein grafts. For cases in which there
39 are multiple coronary arteries or vessels other than the LAD requiring revascularization, saphenous vein grafts
40 ..
......,.. ..,.., ; .... ,...., ...... ,.. ....

~
41
42 F!!ck
---------------------------------------------------------------------------------------------------------------------------------------------
suW.nd EnQck
1
2
" Item: 32 of 46
Q.ld: 1967 [
111 f> Mark .<:J
Previous
[::>
Next
il
Lab Values
~
Notes
~
Calculator

CJ
5
@ USMLEWorld, LlC

6 W hen the left anterior descending artery (LAD) alone is occluded by an atherosclerotic plaque, the left internal
7 mammary artery (left internal thoracic artery) is the preferred vessel for bypass grafting because it has
8 superior short- and long-term patency rates compared to saphenous vein grafts. For cases in w hich there
9 are multiple coronary arteries or vessels other than the LAD requiring revascularization, saphenous vein grafts
10 are routinely used.
11
12 The great saphenous vein is located superficially in the leg and is the longest vein in the body. It courses
13 superiorly from the medial foot, anterior to the medial malleolus, and up the medial aspect of the leg and
14 thigh . In the proximal anterior thigh 3-4 centimeters inferolateral to the pubic tubercle, the great saphenous
15 vein dives deep through the cribriform fascia of the saphenous opening to join the femoral vein . Surgeons
16 commonly access the great saphenous vein in the medial leg or at its point of origin in the upper thigh near the
17 femoral triangle. The femoral triangle is bordered by the inguinal ligament superiorly, sartorius muscle
18 laterally, and adductor longus muscle medially.
19
20 (Choice A) The deep circumflex iliac vessels course parallel to and just superior to the inguinal ligament.
21 The superficial and inferior epigastric veins course above the midportion of the inguinal ligament.
22
23 (Choice B) No major vessels are located immediately inferior to the anterior superior iliac spine (ASIS}. The
24 ASIS serves as the superior attachment of the inguinal ligament, and a penetrating injury to the region below
25 the ASIS could damage the lateral cutaneous nerve of the thigh .
26
27 (Choice D) The popliteal artery and vein course centrally through the popliteal fossa together w ith the tibial
28 nerve. Common medical problems that occur in the popliteal fossa include popliteal artery aneurysms, w hich
29 account for the majority of peripheral artery aneurysms, and synovial (Baker) cysts, w hich are commonly
30 associated w ith arthritis.
31
(Choice E) The small saphenous vein can be found at the lateral aspect of the foot. This vein courses
posteriorly to drain into the popliteal vein.
33
34 Educational objective:
35 The great saphenous vein is a superficial vein of the leg that originates on the medial side of the foot, courses
36 anterior to the medial malleolus, and then travels up the medial aspect of the leg and thigh . It drains into the
37 femoral vein w ithin the region of the femoral triangle, a few centimeters inferolateral to the pubic tubercle.
38
39
Time Spent: 1 seconds Copyright USMLEW orld,LLC. Last updated: [3/ 17/2014]
40

~
41
42 F!!ck
---------------------------------------------------------------------------------------------------------------------------------------------
suW.nd EnQck
1
2
" Item: 33 of 46
Q.ld: 1623 [
111 f> Mark .<:J
Previous
[::>
Next
il
Lab Values
~
Notes
~
Calculator

CJ
5
6 A 65-year-old female is evaluated for progressive dyspnea and cardiomegaly. She also complains of
7 occasional dysphagia . Her dysphagia is potentially explained by enlargement of the:
8
9
10 0 A. Right ventricle
11 0 B. Right atrium
12
13 0 C. Left ventricle
14 0 D. Left atrium
15
16 0 E. Pulmonary artery
17
18
19
20
21
22
23
24
25
26
27
28
29
30
31
32

34
35
36
37
38
39
40

~
41
42 F!!ck suW.nd
---------------------------------------------------------------------------------------------------------------------------------------------
EnQck
1
2
" Item: 33 of 46
Q.ld: 1623 [
111 f> Mark .<:J
Previous
[::>
Next
il
Lab Values
~
Notes
~
Calculator

CJ
5
ol @ D. l eft atrium [67%1

6 0 E. Pulmonary artery [3%1


7
8 Explanation: User ld:
9
10
11
12
13
14
15
16
17
18
19
20
21
22
23
24
25
26
27
28 Given her history of dyspnea and cardiomegaly, this patient's dysphagia may be secondary to a cardiac
29 cause. Cardiovascular dysphagia is uncommon, but w hen it does occur, it is usually the result of left atrial
30 enlargement. (The left atrium is located posteriorly, directly overlying the esophagus.) l eft atrial enlargement
31 most commonly occurs in the setting of mitral stenosis and left ventricular failure. Increasing pressures in the
32 left atrium cause distention of this chamber and pressure on the underlying esophagus.

34 (Choice A) The right ventricle is located on the right lateral aspect of the heart and abuts the right lung.
35
36 (Choice B) The right atrium is located on the anterior and superior right aspect of the heart and receives
37 blood from the inferior and superior venae cavae as w ell as the coronary sinus.
38
(Choice C) The left ventricle is located on the left lateral side of the heart and abuts the left lung.
39
40

~
41
42 F!!ck
---------------------------------------------------------------------------------------------------------------------------------------------
suW.nd EnQck
1
2

CJ
5
6
7
8
9
10
11
12
13
14
15
16
17
18
19
20 Given her history of dyspnea and cardiomegaly, this patient's dysphagia may be secondary to a cardiac
21 cause. Cardiovascular dysphagia is uncommon, but w hen it does occur, it is usually the result of left atrial
22 enlargement. (The left atrium is located posteriorly, directly overlying the esophagus.) Left atrial enlargement
23 most commonly occurs in the setting of mitral stenosis and left ventricular failure. Increasing pressures in the
24 left atrium cause distention of this chamber and pressure on the underlying esophagus.
25
26 (Choice A) The right ventricle is located on the right lateral aspect of the heart and abuts the right lung.
27
28 (Choice B) The right atrium is located on the anterior and superior right aspect of the heart and receives
29 blood from the inferior and superior venae cavae as w ell as the coronary sinus.
30
31 (Choice C) The left ventricle is located on the left lateral side of the heart and abuts the left lung.
32
(Choice E) The pulmonary artery emerges from the right ventricle and branches into the right and left
pulmonary arteries below the arch of the aorta .
34
35 Educational Objective:
36 Cardiovascular dysphagia can result from pressure on the esophagus by a dilated left atrium. The left atrium
37 is commonly enlarged in patients w ith mitral stenosis and left ventricular failure.
38
39
Time Spent: 2 seconds Copyright USMLEW orld,LLC. Last updated: [1 /9/2013]
40
41
42
~imeRemaini~ ~ 0
I Tuto' Feedback Suspend End Block
---------------------------------------------------------------------------------------------------------------------------------------------
1
2
" Item: 34 of 46
Q.ld: 8547 [
111 f> Mark .<:J
Previous
[::>
Next
il
Lab Values
~
Notes
~
Calculator

CJ
5
6 A 52-year-old man is brought to the emergency department for hematemesis that began 1 hour earlier. He
7 has regularly consumed large amounts of alcohol for the last 20 years and has been hospitalized numerous
8 times due to ethanol intoxication . Physical examination show s jaundice and scleral icterus. Examination of
9 his abdomen reveals a palpable spleen and moderate ascites. Endoscopy show s bleeding esophageal
10 varices. This patient's bleeding most likely resulted from chronic shunting of portal blood to the systemic
11 circulation through w hich of the follow ing veins?
12
13
14 0 A. Left gastric
15 0 B. Middle colic
16
17 0 C. Paraumbilical
18 0 D. Right gastroepiploic
19
20 0 E. Splenic
21 0 F. Superficial epigastric
22
23
24
25
26
27
28
29
30
31
32
33

35
36
37
38
39
40
41
42
~imeRemaini~ ~ 0
I Tuto' Feedback Suspend End Block
---------------------------------------------------------------------------------------------------------------------------------------------
1
2
" Item: 34 of 46
Q.ld: 8547 [
111 f> Mark .<:J
Previous
[::>
Next
il
Lab Values
~
Notes
~
Calculator

CJ
5 ~ @ A. Left gastric [66% 1
6 0 B. Middle colic [1 %1
7 0 C. Paraumbilical [6%1
8
9 0 D. Right gastroepiploic [8%1
10 0 E. Splenic [9%1
11
0 F. Superficial epigastric [9%1
12
13
14 Explanation: User ld:
15
16
17 Portacaval anastomoses
18
19
20 Clinical manifestation Portal circulation Systemic circulation
21
22
23 Esophageal varices Left gastric vein Esophageal vein
24
25 Superior rectal vein Middle & inferior recta l veins
Hemo rrhoids
26
27
28 Superficial & inferior epigastric
Caput medusae Paraumbilical veins
29 veins
30
@USMLEWorld, LLC
31
32
33 This patient has numerous cirrhotic stigmata including ascites, jaundice, and splenomegaly. An important
cause of cirrhosis-related morbidity and mortality is the development of variceal hemorrhaging as a direct
35 consequence of portal hypertension .
36
Cirrhosis (of any type) represents the endstage of progressive hepatic fibrosis. It is characterized by
37
distortion of the hepatic architecture and formation of regenerative nodules. Cirrhosis is the most common
38
cause of portal hypertension, w hich arises from increased resistance to portal flow at the hepatic sinusoids.
39
Chronic portal hypertension leads to dilation of small, pre-existing vascular channels between the portal and
40
41
42
~imeRemaini~ ~ 0
I Tuto' Feedback Suspend End Block
--------------------------------------------------------------------------------------------------------------------------------------
1
2
" Item: 34 of 46
Q.ld: 8547 [
111 f> Mark .<:J
Previous
[::>
Next
il
Lab Values
~
Notes
~
Calculator

CJ
5 Caput medusae Pa raumbilical veins
Superficial & inferior epigastric

6 veins
7 @ USMLEWorld, LLC
8
9 This patient has numerous cirrhotic stigmata including ascites, jaundice, and splenomegaly. An important
10 cause of cirrhosis-related morbidity and mortality is the development of variceal hemorrhaging as a direct
11 consequence of portal hypertension .
12
13 Cirrhosis (of any type} represents the endstage of progressive hepatic fibrosis. It is characterized by
14 distortion of the hepatic architecture and formation of regenerative nodules. Cirrhosis is the most common
15 cause of portal hypertension, w hich arises from increased resistance to portal flow at the hepatic sinusoids.
16 Chronic portal hypertension leads to dilation of small, pre-existing vascular channels between the portal and
17 systemic circulations. These dilated collateral vessels (portosystemic anastomoses} commonly form in the
18 anterior abdomen (caput medusae}, low er rectum (hemorrhoids}, and inferior end of the esophagus
19 (esophageal varices}.
20
21 Treatment sometimes includes inserting a transjugular intrahepatic portosystemic shunt (TIPS} between the
22 portal vein and hepatic vein percutaneously, relieving portal hypertension by shunting blood to the systemic
23 circulation .
24
25 (Choice B) The middle colic vein drains the transverse colon .
26
27 (Choice D) The right gastroepiploic vein drains venous blood from the low er parts of the antero-superior and
28 postero-inferior surfaces of the stomach. It runs from left to right along the greater curvature of the stomach.
29
(Choice E) The splenic vein drains blood from the spleen. It joins w ith the superior mesenteric vein to form
30
the hepatic portal vein . Patients w ith portal hypertension can develop splenomegaly due to congestion of the
31
splenic red pulp.
32
33 Educational objective:
In cirrhosis, portal hypertension arises from increased resistance to portal flow at the hepatic sinusoids. This
35 causes increased pressure in the portosystemic collateral veins w ithin the low er end of esophagus, anterior
36 abdomen, and low er rectum . Dilation of these collaterals is responsible for the esophageal varices, caput
37 medusae, and hemorrhoids commonly seen in cirrhotic patients.
38
39
Time Spent: 1 seconds Copyright USMLEW orld,LLC. Last updated: [11 /30/2013]
40
41
42
~imeRemaini~ ~ 0
I Tuto' Feedback Suspend End Block
---------------------------------------------------------------------------------------------------------------------------------------------
1
2
" Item: 35 of 46
Q.ld : 417 [
111 f> Mark .<:J
Previous
[::>
Next
il
Lab Values
~
Notes
~
Calculator

CJ
5
6 A 65-year-old w oman comes to the emergency department for nausea, vomiting, and abdominal pain that
7 began about 6 hours ago. She also complains of vague pelvic pain over the last few months, but says her
8 pain has never been this severe. Her temperature is 38.4 C (101oF), blood pressure is 141/90 mm Hg,
9 pulse is 92/min, and respirations are 18/min. Physical examination show s a moderately distended abdomen.
10 There is a tender bulge below the inguinal ligament, just lateral to the pubic tubercle. The overlying skin is
11 erythematous. W hich of the follow ing structures is most likely immediately lateral to the bulge?
12
13
14 0 A. Inferior epigastric vessels
15 0 B. Cooper's ligament
16
17 0 C. Femoral vein
18 0 D. Transversalis fascia
19
20 0 E. Rectus muscle sheath
21 0 F. Round ligament
22
23
24
25
26
27
28
29
30
31
32
33
34

36
37
38
39
40
41
42
~imeRemaini~ ~ 0
I Tuto' Feedback Suspend End Block
---------------------------------------------------------------------------------------------------------------------------------------------
1
2
" Item: 35 of 46
Q.ld: 417 [
111 f> Mark .<:J
Previous
[::>
Next
il
Lab Values
~
Notes
~
Calculator

CJ
5
v @ C. Femoral vein [64%]
0 D. Transversalis fascia [3%]
6
7 0 E. Rectus muscle sheath [3%]
8 0 F. Round ligament [5%]
9
10
11 Explanation: User ld:
12
13 Groin hernias
14
15
Rectus
16
17 Indirect ~i-- abd ominis
18 inguinal muscle
19 hernia
20 (Internal
21 inguinal Inferior
22
ring) epigastric
23
24 vessels
25
26
27
Inguinal
28
29 ligament Direct
30 inguinal
31 h ernia
32 (Hessel bach's
33 triangle)
34 Femoral
artery
36
Femoral Femoral
37
38 vein hernia
39
40
41
42
1
2
" Item: 35 of 46
Q.ld: 417 [
111 f> Mark .<:J
Previous
[::>
Next
il
Lab Values
~
Notes
~
Calculator

CJ
5
Groin hernias
6
7 Rectus
8 Indirect ..rli - - abd ominis
9 inguinal muscle
10
hernia
11
12 (Internal
13 inguinal , ....... Inferior
14 ring) epigastric
15 vessels
16
17
18
19 Inguinal
20 ligament Direct
21 inguinal
22
hernia
23
24 (Hessel bach's
25 Femoral triangle)
26 artery
27
28 Femoral Femoral
29 vein
30
31
32
33
34

36
37 USMlEWorld.llC 1 2010
38
39 There are 3 types of groin hernias: direct inguinal, indirect inguinal, and femoral. Direct and indirect inguinal
40

~
41
42 F!!ck
---------------------------------------------------------------------------------------------------------------------------------------------
suW.nd EnQck
1
2
" Item: 35 of 46
Q.ld : 417
111 f> Mark .<:J
Previous
[::>
Next
il
Lab Values
~
Notes
~
Calculator

CJ
5
6 There are 3 types of groin hernias: direct inguinal, indirect inguinal, and femoral. Direct and indirect inguinal
7 hernias occur above the inguinal ligament; femoral hernias are located below it.
8
Femoral hernias protrude through the femoral ring, w hich can w iden and become lax w ith age. They are
9
lateral to the pubic tubercle and lacunar ligament and medial to the femoral artery and vein. Femoral hernias
10
are more common in w omen and tend to occur on the right side. Femoral hernias may present w ith upper
11
thigh, groin, or pelvic discomfort.
12
13 Because the femoral canal is small, femoral hernias are prone to incarceration. Incarcerated hernias cannot
14 be reduced; if a segment of bow el is present in the hernia sac, incarceration can cause bow el obstruction
15 (nausea/vomiting/abdominal pain and distention). Impaired blood flow to contents trapped in the hernia sac
16 occurs a few hours after incarceration, causing strangulation. Ischemia and necrosis of the contents of the
17 hernia sac occur, w hich may lead to systemic symptoms such as fever.
18
19 (Choice A) Direct inguinal hernias lie medial to the inferior epigastric vessels, and indirect inguinal hernias
20 are lateral to the vessels.
21
22 (Choice B) The pectineal (Cooper's) ligament is a thickened part of the pectineal fascia. It overlies the
23 pectineal ridge of the pubic bone and is located posteriorly to the femoral canal.
24
25 (Choice D) The transversalis fascia is found between the transversalis muscle and the extraperitoneal fat. It
26 forms the posterior w all of the inguinal canal. The deep inguinal ring is an opening in the transversalis fascia
27 and is the site of protrusion of indirect inguinal hernias.
28
29 (Choice E) The rectus muscle sheath forms the medial border of Hesselbach's triangle. Direct inguinal
30 hernias are located just lateral to the rectus abdominis muscle sheath .
31
(Choice F) The round ligament in w omen is homologous to the spermatic cord in men. It leaves the pelvis
32
through the deep inguinal ring and passes through the inguinal canal.
33
34 Educational objective:
Femoral hernias are inferior to the inguinal ligament, lateral to pubic tubercle, and medial to the femoral vein.
36 They can present w ith groin discomfort or manifest w ith a bulge on the upper thigh . Incarceration and
37 strangulation are common complications of femoral hernias.
38
39
Time Spent: 2 seconds Copyright USMLEW orld,LLC. Last updated: [212/2014]
40
41
42
~ime Remaining:o9:1l ~ 0
I Tuto' Feedback Suspend End Block
---------------------------------------------------------------------------------------------------------------------------------------------
1
2
" Item: 36 of 46
Q.ld: 1145 [
111 f> Mark .<:J
Previous
[::>
Next
il
Lab Values
~
Notes
~
Calculator

CJ
5
6 A 63-year-old male w ith a history of stroke and resultant severe oropharyngeal dysphagia develops a right
7 lower lobe pneumonia after an episode of vomiting. This type of aspiration pneumonia commonly affects the
8 right lower lung lobe because:
9
10
11 A. The right main bronchus is longer than the left main bronchus
12
13
14
15 0
16
17
18
19
20 0 B. The right main bronchus is straighter than the left main bronchus
21
0 C. The right main bronchus is narrow er than the left main bronchus
22
23 0 D. The right lower lobe receives a richer blood supply than the left lower lobe
24 0 E. Vascular resistance is higher in the right low er lobe than the right upper lobe
25
26
27
28
29
30
31
32
33
34
35

37
38
39
40
41
42
~imeRemaini~ ~ 0
I Tuto' Feedback Suspend End Block
---------------------------------------------------------------------------------------------------------------------------------------------
1
2
" Item: 36 of 46
Q.ld: 1145 [
111 f> Mark .<:J
Previous
[::>
Next
il
Lab Values
~
Notes
~
Calculator

CJ
5
6 A 63-year-old male w ith a history of stroke and resultant severe oropharyngeal dysphagia develops a right
7 low er lobe pneumonia after an episode of vomiting. This type of aspiration pneumonia commonly affects the
8 right low er lung lobe because:
9
10
11 0 A. The right main bronchus is longer than the left main bronchus [2%]
12 .; @ B. The right main bronchus is straighter than the left main bronchus [94%]
13 0 C. The right main bronchus is narrow er than the left main bronchus [1 %]
14
15 0 D. The right low er lobe receives a richer blood supply than the left low er lobe [1 %]
16 0 E. Vascular resistance is higher in the right low er lobe than the right upper lobe [1 %]
17
18
19 Explanation: User ld:
20
21 The right main bronchus is more prone to foreign body aspiration than the left main bronchus because the
22 right main bronchus has a larger diameter, is shorter, and is oriented more vertically than the left main
23 bronchus. This fact can be remembered using the mnemonic, "Sw allow a bite, goes dow n the right."
24
25 Educational objective:
26 Aspirated or inhaled particles are most likely to become lodged in the right main bronchus or its branches
27 because this bronchus is shorter, w ider and more vertically oriented than the left main bronchus.
28
29 Time Spent: 4 seconds Copyright USMLEW orld,LLC. Last updated: [1 / 1/2014]
30
31
32
33
34
35

37
38
39
40
41
42
~imeRemaini~ ~ 0
I Tuto' Feedback Suspend End Block
---------------------------------------------------------------------------------------------------------------------------------------------
1
2
" Item: 37 of 46
Q.ld: 1682 [
111 f> Mark .<:J
Previous
[::>
Next
il
Lab Values
~
Notes
~
Calculator

CJ
5
A 34-year-old female presents to the emergency room after falling in the bathroom. She complains of w rist
pain and numbness in her right palm . Based on her symptoms, you suspect that she has dislocated the
6 lunate bone in the w rist. Identify the lunate bone on the X-ray below .
7
8
9
10
11
12
13
14
15
16
17
18
19
20
21
22
23
24
25
26
27
28
29
30
31
32
33
34 0 A. A
35
36 0 B. B
0 C. C
38 0 D. D
39
40 0 E. E
41
42
~imeRemaini~ ~ 0
I Tuto' Feedback Suspend End Block
---------------------------------------------------------------------------------------------------------------------------------------------
1
2

CJ
5
6
7
8
9
10
11 0 A.A [7%]
12
0 B. B [1 3%]
13
14 0 C. C [31%]
15 -' @ D. D [41 %]
16
0 E. E [8%]
17
18
19 Explanation: User ld:
20
21 A fall on an outstretched, dorsiflexed hand is the classic injury that causes lunate dislocation. The lunate
22 (Choice D) is one of the proximal carpal bones. It can be identified on an X-ray of the hand as the more
23 medial of the two bones that articulate with the radius. The scaphoid is the other (more lateral) bone that
24 articulates with the radius (Choice C). Injuries that cause lunate dislocation also often cause fracture of the
25 scaphoid. A potential late sequela of such an injury is avascular necrosis of the proximal scaphoid fracture
26 fragment.
27
28 (Choice A) The trapezium is the most lateral of the distal carpal bones. Recall that the "thumb swings on the
29 trapezium ."
30
31 (Choice B) The capitate bone lies in the center of the wrist.
32
(Choice E) The triquetrum bone is located in the proximal medial wrist and together with the lunate forms the
33
small articular surface between the distal ulna and the carpal bones.
34
35 Educational Objective:
36 The lunate bone can be identified on a hand X-ray as the more medial of the two carpal bones that articulate
with the radius. It lies immediately medial to the scaphoid bone.
38
39
Time Spent: 12 seconds Copyright USMLEWorld,LLC. Last updated: [7/7/201 OJ
40
41
42
~imeRemaini~ ~ 0
I Tuto' Feedback Suspend End Block
--------------------------------------------------------------------------------------------------------------------------------------
1
2
" Item: 38 of 46
Q.ld: 833 [
111 f> Mark .<:J
Previous
[::>
Next
il
Lab Values
~
Notes
~
Calculator

CJ
5
6 A 39-year-old female is brought to the emergency room after a motor vehicle collision. Upon arrival she is
7 determined to be stable, but complains of mild abdominal pain. CT scan of the abdomen reveals a
8 retroperitoneal hematoma. Trauma of w hich of the follow ing anatomic structures is most likely responsible for
9 this finding?
10
11
12 0 A. Abdominal aorta
13
14
0 B. Transverse colon
15 0 C. Liver
16
17 0 D. Spleen
18 0 E. Pancreas
19
20
21
22
23
24
25
26
27
28
29
30
31
32
33
34
35
36
37

39
40
41
42
~imeRemaini~ ~ 0
I Tuto' Feedback Suspend End Block
---------------------------------------------------------------------------------------------------------------------------------------------
1
2
" Item: 38 of 46
Q.ld: 833 [
111 f> Mark .<:J
Previous
[::>
Next
il
Lab Values
~
Notes
~
Calculator

CJ
5
V

0
v. LIV"I [.l7 0J

D. Spleen [32%]
6 .; @ E. Pancreas [37%]
7
8
9 Explanation: User ld:
10
11 The retroperitoneum is the space located behind the peritoneal cavity. It includes the follow ing structures:
12
1. Major vessels: abdominal aorta, inferior vena cava and their branches.
13
2. Solid organs: pancreas (except the tail), kidneys, adrenal glands.
14
3. Hollow organs: parts 2 & 3 and a portion of part 4 of the duodenum, the ascending and descending
15
colon (secondarily retroperitoneal), rectum, ureters and bladder.
16
4. Vertebral column and pelvic muscles.
17
18
19 Retroperitoneal hematomas are commonly associated w ith abdominal and pelvic trauma; they develop in up
20 to half of patients w ith blunt abdominal trauma . Pancreatic injury w ith formation of a retroperitoneal hematoma
21 can occur due to severe blunt or penetrating abdominal trauma . It can frequently be caused by direct
22 abdominal blow s from malpositioned seat belts or by steering w heels during automobile collisions.
23 Pancreatic injury can cause mild symptoms or be asymptomatic. An abdominal CT scan should be
24 performed in stable patients w ith blunt abdominal trauma to exclude the possibility of retroperitoneal
25 hematoma.
26
27 (Choice A) Though the abdominal aorta is a retroperitoneal structure, rupture of the abdominal aorta w ould
28 most likely result in massive and rapid blood loss leading to hemorrhagic shock.
29
(Choices B, C and D) The transverse colon, liver, and spleen are intraperitoneal organs. Lacerations or
30
rupture of these organs can occur in blunt abdominal trauma, but these injuries w ould lead to
31
hemoperitoneum (free blood in the peritoneal space) not retroperitoneal hematoma.
32
33 Educational Objective:
34 Retroperitoneal hematoma in a stable patient is likely to occur due to pancreatic injury. It may present w ith
35 mild symptoms, be asymptomatic or be masked by symptoms from other injuries related to trauma .
36 Abdominal CT is the diagnostic modality of choice. Frequently nonexpanding hematomas in this location w ill
37 be treated conservatively (i.e. non-operatively}.

39
Time Spent: 4 seconds Copyright USMLEW orld,LLC. Last updated: [7/7/201 OJ
40
41
42
~imeRemaini~ ~ 0
I Tuto' Feedback Suspend End Block
---------------------------------------------------------------------------------------------------------------------------------------------
1
2
" Item: 39 of 46
Q.ld : 8670 [
111 f> Mark .<:J
Previous
[::>
Next
il
Lab Values
~
Notes
~
Calculator

CJ
5
A 56-year-old man comes to the emergency department because of persistent left w rist pain. W hile w alking
his dog 5 hours ago, he fell forward to the ground and landed on his outstretched left hand. Examination
6
show s mild sw elling over the dorsum of the left hand w ith preserved range of motion. There is point
7
tenderness present over the anatomic snuffbox. An x-ray of his left w rist is show n below .
8
9
10
11
12
13
14
15
16
17
18
19
20
21
22
23
24
25
26
27
28
29
30
31
32
33
34
35
36
37
38
This patient is at greatest risk for developing w hich of the follow ing complications?
40
41
42
~imeRemaini~ ~ 0
I Tuto' Feedback Suspend End Block
---------------------------------------------------------------------------------------------------------------------------------------------
1
2

CJ
5
6
7
8
9
10
11
12
13
14
15
16
17
18
19
20
21
22
23
24
25
26
27
28
29
30
31
This patient is at greatest risk for developing w hich of the follow ing complications?
32
33
34 0 A. Avascular necrosis
35 0 B. Carpal tunnel syndrome
36
37 0 C. Compartment syndrome
38 0 D. Dupuytren's contracture
0 E. Osteomyelitis
40
41
42
~imeRemaini~ ~ 0
I Tuto' Feedback Suspend End Block
---------------------------------------------------------------------------------------------------------------------------------------------
1
2
" Item: 39 of 46
Q.ld : 8670 [
111 f> Mark .<:J
Previous
[::>
Next
il
Lab Values
~
Notes
~
Calculator

CJ
5
" @ A. Avascular necrosis [76%]

6 0 B. Carpal tunnel syndrome [9%]


7 0 C. Compartment syndrome [8%]
8 0 D. Dupuytren's contracture [6%]
9
10 0 E. Osteomyelitis (1 %]
11
12
Explanation: User ld:
13
14
15 Scaph oid avascular necrosis
16
17
18
19
20
21
22
23
24
25
26
27
28
29
30 ,---'--'-------=~--- Palmar scaphoid
31 branch
32 Superficial
,--___:.--:,.:.:._--!:--- - Dorsal carpal branch
33 palmar branch
34
35
36
Dorsal scaphoid _ __ ~,.....,..,.-----~~~=------Scaphoid fracture
37 branch
Necrosis of proximal
38 --~-------
pole of scaphoid
Radial artery
40
41
42
1
2
" Item: 39 of 46
Q.ld : 8670 [
111 f> Mark .<:J
Previous
[::>
Next
il
Lab Values
~
Notes
~
Calculator

CJ
5 Scaphoid avascular necrosis
6
7
8
9
10
11
12
13
14
15
16
17
18
19
20 r---'~-----.,..----=>;.---- Palmar scaphoid
21 branch
22
Superficial
r--__:.-..;;.:._-~-- Dorsal carpal branch
23 palmar branch
24
25 Dorsal scaphoid _ __ ~,....,.-.._,~~~=------Scaphoid fracture
26 branch
27 Necrosis of proximal
28 --~------- pole of scaphoid
29 Radial artery
30
31
32
33
34
35
36
37
38

40 <!:) USMLoW Id. LLC

41
42
~imeRemaini~ ~ 0
I Tuto' Feedback Suspend End Block
--------------------------------------------------------------------------------------------------------------------------------------
1
2
" Item: 39 of 46
Q.ld: 8670 [
111 f> Mark .<:J
Previous
[::>
Next
il
Lab Values
~
Notes
~
Calculator

CJ
5 Scaphoid fractures are the most common of the carpal bone fractures. They frequently result from falls
6 onto an outstretched arm that cause direct axial compression or wrist hyperextension. A scaphoid fracture
7 should be suspected in any patient with persistent wrist pain and tenderness in the anatomical snuff box
8 following a fall. The snuffbox is a triangular deepening on the dorsoradial aspect of the hand at the level of the
9 carpal bones. The scaphoid and trapezium bones form the floor of the snuffbox.
10
The scaphoid is the largest bone of the proximal carpal row and is located on the radial aspect of the hand just
11
distal to the radius itself. The dorsal scaphoid branch of the radial artery supplies the majority of the scaphoid
12
after entering near the bone's distal pole. Blood supply to the proximal pole proceeds in a retrograde manner
13
and can be easily interrupted by a fracture. Thus, scaphoid fractures are at risk for avascular necrosis and
14
nonunion.
15
16 (Choice B) Carpal tunnel syndrome is caused by median nerve compression as it travels through the carpal
17 tunnel. It is associated with repetitive wrist movements, hypothyroidism, diabetes mellitus, and rheumatoid
18 arthritis.
19
20 (Choice C) Acute compartment syndrome occurs when increased pressure within a fascial compartment
21 compromises blood circulation and tissue function within that space. It develops after significant trauma,
22 particularly long-bone fractures of the leg or forearm .
23
24 (Choice D) Dupuytren's contracture is a benign, slowly progressive fibroproliferative disease of the palmar
25 fascia. As the scarring progresses, nodules form on the palmar fascia and the fingers gradually lose their
26 flexibility, eventually resulting in contractures that draw the fingers into flexion.
27
28 (Choice E) Osteomyelitis commonly results from either hematogenous seeding of bone with bacteria or by
29 contiguous spread of organisms from a nearby wound. However, unexposed bone is unlikely to develop
30 osteomyelitis. Staphylococcus aureus is the most common cause of hematogenous osteomyelitis.
31
32 Educational objective:
33 Fall on an outstretched hand may cause fracture of the scaphoid bone. Examination shows tenderness in the
34 anatomical snuff box. The scaphoid bone is vulnerable to avascular necrosis and nonunion due to its tenuous
35 blood supply.
36
37 References:
38 1. Fractures in brief: scaphoid fractures.
40

~
41
42 F!!ck
---------------------------------------------------------------------------------------------------------------------------------------------
suW.nd EnQck
1
2
" Item: 39 of 46
Q.ld: 8670 [
111 f> Mark .<:J
Previous
[::>
Next
il
Lab Values
~
Notes
~
Calculator

CJ
5
V II ~V (,UI VU ~~U'-' ~"'11'-'U U l l l l UIU~ .... UU"''-' Ull'-' .... ~ U./\I(,U I..V IIIiJI '-'"'"''VII VI

should be suspected in any patient w ith persistent w rist pain and tenderness in the anatomical snuff box
follow ing a fall. The snuffbox is a triangular deepening on the dorsoradial aspect of the hand at the level of the
1111 11"'~ IIJtJ'-'I'-'A~'-'II..,IVII . I"\ "''"-UtJIIVIU I I U .... U .U'-'

6
carpal bones. The scaphoid and trapezium bones forrn the floor of the snuffbox.
7
8 The scaphoid is the largest bone of the proximal carpal row and is located on the radial aspect of the hand just
9 distal to the radius itself. The dorsal scaphoid branch of the radial artery supplies the majority of the scaphoid
10 after entering near the bone's distal pole. Blood supply to the proximal pole proceeds in a retrograde rnanner
11 and can be easily interrupted by a fracture. Thus, scaphoid fractures are at risk for avascular necrosis and
12 nonunion.
13
14 (Choice B) Carpal tunnel syndrome is caused by rnedian nerve compression as it travels through the carpal
15 tunnel. It is associated w ith repetitive w rist rnovernents, hypothyroidism, diabetes mellitus, and rheumatoid
16 arthritis.
17
18 (Choice C) Acute cornpartrnent syndrome occurs w hen increased pressure w ithin a fascial cornpartrnent
19 cornprornises blood circulation and tissue function w ithin that space. It develops after significant traurna,
20 particularly long-bone fractures of the leg or forearm .
21
22 (Choice D) Dupuytren's contracture is a benign, slow ly progressive fibroproliferative disease of the palrnar
23 fascia. As the scarring progresses, nodules forrn on the palrnar fascia and the fingers gradually lose their
24 flexibility, eventually resulting in contractures that draw the fingers into flexion.
25
26 (Choice E) Osteomyelitis cornrnonly results frorn either hematogenous seeding of bone w ith bacteria or by
27 contiguous spread of organisms frorn a nearby w ound. How ever, unexposed bone is unlikely to develop
28 osteomyelitis. Staphylococcus aureus is the rnost cornrnon cause of hematogenous osteomyelitis.
29
Educational objective:
30
Fall on an outstretched hand rnay cause fracture of the scaphoid bone. Examination show s tenderness in the
31
anatomical snuff box. The scaphoid bone is vulnerable to avascular necrosis and nonunion due to its tenuous
32
blood supply.
33
34
35 References:
36 1. Fractures in brief: scaphoid fractures.
37
38
Tirne Spent: 13 seconds Copyright USMLEW orld,LLC. Last updated: [1 0/2212013]
40

~
41
42
---------------------------------------------------------------------------------------------------------------------------------------------
F!!ck suW.nd EnQck
1
2
" Item: 40 of 46
Q.ld: 1968 [
111 f> Mark .<:J
Previous
[::>
Next
il
Lab Values
~
Notes
~
Calculator

CJ
5
6 A 20-year-old man comes to the physician w ith right knee pain follow ing a soccer game. W hen asked how he
7 sustained the injury, he says, "I fell to the ground after colliding w ith m y teammate and then an opponent fell on
8 m y shin ." On physical examination, his knee appears slightly sw ollen, and there is excessive posterior
9 displacement of the tibia w hen pressure is applied to his anterior tibia w ith the knee in the flexed position. The
10 physician orders an MRI to confirm the diagnosis. An axial MRI cross section of the right knee is show n
11 below. W hich of the follow ing structures is most likely injured in this patient?
12
13
14
15
16
17
18
19
20
21
22
23
24
25
26
27
28
29
30
31
32 O A.A
33
34
0 B. B
35 0 C. C
36 0 D. D
37
O E.E
38
39

~
41
42 F!!ck
---------------------------------------------------------------------------------------------------------------------------------------------
suW.nd EnQck
1
2
" Item: 40 of 46
Q.ld: 1968 [
111 f> Mark .<:J
Previous
[::>
Next
il
Lab Values
~
Notes
~
Calculator

CJ
5
.; @ D. D [65%]
0 E. E [4%]
6
7
8 Explanation: User ld:
9
10 In this axial MRI of the right knee, the patellar ligament is visible anteriorly at the top of the image and the
11 gastrocnemius muscle is seen posteriorly at the bottom. The anterior and posterior cruciate ligaments are
12 found w ithin the articular capsule of the knee joint and cross one another as each spans from the femur to the
13 tibia. The posterior cruciate ligament originates from the anterolateral surface of the medial femoral condyle
14 and inserts into the posterior intercondylar area of the tibia. It prevents posterior displacement of the tibia
15 relative to the femur. Its integrity can be tested in the clinical setting by using the posterior draw er test (the
16 maneuver described in the question stem). The posterior cruciate ligament is most commonly injured during
17 sporting activities or motor vehicle accidents follow ing a direct blow to the anterior proximal tibia .
18
19 Anterior view Posterior view
20
21
22
23
24 Posterior cruciate ligament
25 Lateral collateral ligament
26 (fibular collateral ligament)
27
28 Medial collateral ligament Popliteus tendon
29 (tibial collateral ligament)
30
31
32
33 Anterior cruciate ligament
34
35
36
37
USMLEWorld.LLC 0 lOll
38
39

~
41
42 F!!ck suW.nd EnQ ck
---------------------------------------------------------------------------------------------------------------------------------------------
1
2

CJ
5
6
7
8 Lateral collateral ligament
(fibular collateral ligament)
9
10
11 Medial collateral ligament Popliteus tendon
12 (tibial collateral ligament)
13
14
15
16 Anterior cruciate ligament
17
18
19
20 USMLEWorld.LLC 0 lOll
21
22
23 (Choice A) The anterior cruciate ligament functions to resist anterior displacement of the tibia relative to the
24 femur.
25
26 (Choices 8 and C) The lateral condyle of the femur articulates w ith the lateral superior articular surface of
27 the tibia . The periphery of this articulation is normally covered by the lateral meniscus, w hich is rounder and
28 covers a larger portion of the articular surface than the medial meniscus.
29
(Choice E) The medial condyle of the femur articulates w ith the medial superior articular surface of the
30
tibia. The periphery of this articulation is normally covered by the medial meniscus. The medial articular
31
surfaces of the knee are larger than the lateral articular surfaces, as this compartment bears more of the body'
32
s w eight.
33
34 Educational objective:
35 The posterior cruciate ligament prevents posterior displacement of the tibia relative to the femur. It originates
36 from the anterolateral surface of the medial femoral condyle and inserts into the posterior intercondylar area of
37 the tibia . Its integrity can be tested in the clinical setting by using the posterior draw er test.
38
39
Time Spent: 9 seconds Copyright USMLEW orld,LLC. Last updated: [11 /6/2013]
41
42
~imeRemaini~ ~ 0
I Tuto' Feedback Suspend End Block
---------------------------------------------------------------------------------------------------------------------------------------------
1
2
" Item: 41 of 46
Q.ld: 1704 [
111 f> Mark .<:J
Previous
[::>
Next
il
Lab Values
~
Notes
~
Calculator

CJ
5
6 A 24-year-old mountain climber presents to the emergency room w ith right shoulder pain follow ing a fall. He
7 w as climbing a cliff at a nearby national park w hen he lost his grip and fell approximately 5 meters onto the
8 boulders below. He says that he "landed on his right arm and heard the bone snap". Physical examination
9 reveals total inability to extend the right w rist. X-ray reveals a fracture of the right humeral shaft. W hich of the
10 follow ing arteries is most likely to be injured in this patient?
11
12
13 0 A. Anterior circumflex humeral artery
14 0 B. Brachial artery
15
16 0 C. Common interosseous artery
17 0 D. Deep brachial artery
18
19 0 E. Radial collateral artery
20
21
22
23
24
25
26
27
28
29
30
31
32
33
34
35
36
37
38
39
40

42
~imeRemaini~ ~ 0
I Tuto' Feedback Suspend End Block
---------------------------------------------------------------------------------------------------------------------------------------------
1
2
" Item: 41 of 46
Q.ld: 1704 [
111 f> Mark .<:J
Previous
[::>
Next
il
Lab Values
~
Notes
~
Calculator

CJ
5
.; @ D. Deep brachial artery [43%]
0 E. Radial collateral artery [14%]
6
7
8 Explanation: User ld:
9 Axillary artery
10
11
12
Anterior & Posterior _.
13
circumftex humeral arteries
14
15 Deep brachial artery
16 (runs with the radial nerve)
17
18
19
20 Brachial artery
21
22
23
24
25
26
27
28
29 Common interosseous artery
30
31
32
33
34
35
36
37
38
39
40 W hen the midshaft of the humerus is fractured, there is significant associated risk of injury to the radial nerve

42
~imeRemaini~ ~ 0
I Tuto' Feedback Suspend End Block
---------------------------------------------------------------------------------------------------------------------------------------------
1
2
" Item: 41 of 46
Q.ld: 1704 [
111 f> Mark .<:J
Previous
[::>
Next
il
Lab Values
~
Notes
~
Calculator

CJ
5
6 W hen the midshaft of the humerus is fractured, there is significant associated risk of injury to the radial nerve
7 and deep brachial artery. The radial nerve innervates most of the forearm extensors at the elbow and most of
8 the hand extensors at the w rist. It also innervates the extrinsic extensors of the digits, and the brachioradialis
9 and supinator muscles. The radial nerve also provides cutaneous sensory innervation to the dorsal hand,
10 forearm and upper arm . The deep brachial artery branches off of the brachial artery high in the arm, passes
11 inferior to the teres major muscle, and courses posteriorly along the humerus in close association w ith the
12 radial nerve.
13
14 The patient described above is exhibiting signs of radial nerve injury. The combination of radial nerve deficits
15 w ith radiographic evidence of a humeral fracture should raise concern for an associated injury to the deep
16 brachial artery.
17
18 (Choice A) The anterior circumflex humeral artery is a branch of the axillary artery that passes anterior to the
19 surgical neck of the humerus and anastomoses w ith the posterior circumflex humeral artery. A fracture to the
20 surgical neck of the humerus may damage this vessel and the axillary nerve.
21
22 (Choice B) The axillary artery gives off the posterior circumflex humeral artery and becomes the brachial
23 artery. The brachial artery courses anteromedially in the arm w ithin the bicipital groove and branches into the
24 radial and ulnar arteries after entering the forearm. Supracondylar fractures of the humerus may injure this
25 artery.
26
27 (Choice C) The common interosseous artery is a short branch of the ulnar artery that gives rise to anterior,
28 posterior and recurrent branches w ithin the proximal forearm.
29
(Choice E) The deep brachial artery divides into the radial collateral artery (the continuation of the deep
30
brachial artery) and middle collateral artery at the midpoint of the humerus. The radial collateral artery also
31
courses w ith the radial nerve. However, injury to the radial collateral artery is less likely because it originates
32
at the low er end of the spiral groove w here the deep brachial artery pierces the lateral intramuscular septum .
33
34 Educational Objective:
35 The deep brachial artery and radial nerve course along the posterior aspect of the humerus. Midshaft
36 fractures of the humerus risk injury to these structures. Supracondylar fractures are associated w ith injury to
37 the brachial artery.
38
39
Time Spent: 1 seconds Copyright USMLEW orld,LLC. Last updated: [219/2014]
40

42
~imeRemaini~ ~ 0
I Tuto' Feedback Suspend End Block
---------------------------------------------------------------------------------------------------------------------------------------------
1
2
" Item: 42 of 46
Q.ld: 8703 [
111 f> Mark .<:J
Previous
[::>
Next
il
Lab Values
~
Notes
~
Calculator

CJ
5
6 A 23-year-old man comes to the emergency department complaining that he has food stuck in his throat. His
7 symptoms started 2 hours ago after eating fish at a local seafood restaurant. He has tried coughing and
8 sw allow ing multiple times in an attempt to clear the food, but has so far been unsuccessful. The patient
9 denies any difficulty w ith breathing. He does not appear to be in any distress on physical examination.
10 Laryngoscopy reveals a fish bone lodged in the left piriform recess. During an attempt to retrieve the fish
11 bone, a nerve is injured deep to the mucosa overlying the recess. W hich of the follow ing is most likely to be
12 impaired in this patient?
13
14
15 0 A. Cough reflex
16 0 B. Gag reflex
17
18 0 C. Mastication
19 0 D. Salivation
20
21 0 E. Taste sensation
22
23
24
25
26
27
28
29
30
31
32
33
34
35
36
37
38
39
40
41 ~imeRemaini~ ~ 0
I Tuto' Feedback Suspend End Block
---------------------------------------------------------------------------------------------------------------------------------------------
1
2
" Item: 42 of 46
Q.ld: 8703 [
111 f> Mark .<:J
Previous
[::>
Next
il
Lab Values
~
Notes
~
Calculator

CJ
5
., @ A. Cough reflex [35%]

6 0 B. Gag reflex [55%]


7 0 C. Mastication [2%]
8
0 D. Salivation [3%]
9
10 0 E. Taste sensation [5%]
11
12
Explanation: User ld:
13
14
15 Posterior view of pharynx
16
17
18
19
20
21 Palatine tonsils- -..:.r.
22
23
24
25
26 Root of tongue Aryepiglottic fold
27
28
29
Epiglottis
30 Internal branch of
31 superior laryngeal
32
33
34 Mucosa overlying
35 internal laryngeal Thyroid cartilage
36
37
nerve
38
39
40
1
2
" Item: 42 of 46
Q.ld: 8703 [
111 f> Mark .<:J
Previous
[::>
Next
il
Lab Values
~
Notes
~
Calculator

CJ
5 Posterior view of pharynx
6
7
8
9
10
11 Palatine tonsils- -.!!
12
13
14
15
Root of tongue Aryepiglottic fold
16
17
18 Epiglottis
19
20 Internal branch of
21 superior laryngeal
22
23
24 Mucosa overlying
25 internal laryngeal Thyroid cartilage
26 nerve
27
28
29 Piriform recess
30
31
32
33
34
35
36
37
38
39
40
1
2

CJ
5
6
7
8 @ USMLEWOrld, LLC
9
10 The piriform recesses are small cavities that lie on either side of the laryngeal orifice. They are bounded
11 medially by the aryepiglottic folds and laterally by the thyroid cartilage and thyrohyoid membrane. During
12 normal sw allow ing, food is diverted by the epiglottis laterally through the piriform recesses into the esophagus
13 w ithout endangering the airway.
14
15 A thin layer of mucosa overlying the piriform recess is all that protects the superficially coursing internal
16 laryngeal nerve, a branch of the superior laryngeal nerve (CN X). Unlike the recurrent and external laryngeal
17 nerves that carry motor fibers to the muscles involved in vocal cord function, the internal laryngeal nerve
18 contains only sensory and autonomic fibers. It mediates the afferent limb of the cough reflex by carrying
19 sensation from the mucosa superior to the vocal cords. Foreign bodies, such as chicken or fish bones, can
20 become lodged in the piriform recess. The internal laryngeal nerve can be damaged by the sharpness of
21 these objects or attempts to retrieve them .
22
23 (Choice B) The afferent limb of the gag reflex is mediated predominantly by the glossopharyngeal nerve
24 (CN IX}, w hile the efferent limb is carried by the vagus nerve (CN X). The internal laryngeal nerve does not
25 carry motor fibers and mediates sensation mainly from larynx and epiglottis (stimulation of these areas
26 induces coughing, not gagging).
27
28 (Choice D) Salivation is mediated in part by parasympathetic fibers originating from the glossopharyngeal
29 nerve. These fibers synapse on the otic ganglion, and postganglionic fibers travel via the auriculotemporal
30 nerve to reach the parotid gland.
31
(Choice E) Taste from the base of the tongue is mediated by the glossopharyngeal nerve (CN IX); taste from
32
the anterior two-thirds of the tongue is mediated by the facial nerve (CN VII).
33
34 Educational objective:
35 The internal laryngeal nerve mediates the afferent limb of the cough reflex above the vocal cords. Foreign
36 bodies (eg, chicken or fish bones) can become lodged in the piriform recess and may cause damage to the
37 nerve, impairing the cough reflex.
38
39 Time Spent: 2 seconds Copyright USMLEW orld,LLC. Last updated: [1 0/ 15/2013]
40
41 ~imeRemaini~ ~ 0
I Tutor Feedback Suspend End Block
---------------------------------------------------------------------------------------------------------------------------------------------
1
2
" Item: 43 of 46
Q.ld : 7640 [
111 f> Mark .<:J
Previous
[::>
Next
il
Lab Values
~
Notes
~
Calculator

CJ
5
A 65-year-old man w ith a history of coronary artery disease comes to the physician complaining of
progressive exertional shortness of breath, fatigue, and low er extremity sw elling. His medical history is
6 significant for a m yocardial infarction 5 years ago and an electronic pacemaker implanted 2 years ago. His
7 chest x-ray is show n below. A segment of one of the leads is highlighted (arrow).
8
9
10
11
12
13
14
15
16
17
18
19
20
21
22
23
24
25
26
27
28
29
30
31
32
33
34
35
36
37
38 The highlighted segment most likely lies w ithin w hich of the follow ing structures?
39
40 ~ A 1\...,4-.....: ..... :,...,4.... .., . ...........; ... , . 1...... ,..,, . 1 .... ,,..,

41
42
~imeRemaini~ ~ 0
I Tuto'
---------------------------------------------------------------------------------------------------------------------------------------------
Feedback Suspend End Block
1
2

CJ
5
6
7
8
9
10
11
12
13
14
15
16
17
18
19
20
21
22
23
24
25
26
27
28
29
30
31
The highlighted segment most likely lies w ithin w hich of the follow ing structures?
32
33
34 0 A. Anterior interventricular sulcus
35 0 B. Atrioventricular groove
36
37 0 C. Pulmonary artery
38 0 D. Right atrium
39 0 E. Right ventricle
40
41
42
~imeRemaini~ ~ 0
I Tuto' Feedback Suspend End Block
---------------------------------------------------------------------------------------------------------------------------------------------
1
2
" Item: 43 of 46
Q.ld : 7640 [
111 f> Mark .<:J
Previous
[::>
Next
il
Lab Values
~
Notes
~
Calculator

CJ
5 The highlighted segment most likely lies within w hich of the following structures?
6
7 0 A. Anterior interventricular sulcus [8%]
8
.; @ B. Atrioventricular groove [47%]
9
10 0 C. Pulmonary artery [9%]
11 0 D. Right atrium [23%]
12
0 E. Right ventricle [1 3%]
13
14
15 Explanation: User ld:
16
17
18
Biventricular pacemaker
19
20
21
22
23
24
25
26
27
28
29
30
31 Pulse generator
32
33
34
35
36 left ventricular
37 coronary sinus lead
38
39
40
41
42
1
2
" Item: 43 of 46
Q.ld : 7640 [
111 f> Mark .<:J
Previous
[::>
Next
il
Lab Values
~
Notes
~
Calculator

CJ
5
Biventricular pacemaker
6
7
8
9
10
11
12
13
14
15
16
17
18 Pulse generator
19
20
21
22
23 Left ventricular
24 coronary sinus lead
25
26
27
28
29
30
31
32
33
34
35
Right atrial lead
36
37
38
39
40
41
42
1
2

CJ
5
6
7
8
Right atrial lead
9
10
11
12
13
14
15
16 Right ventricular lead
17
18 @USMLEWo~d, llC
19
20 This patient has a biventricular pacemaker, a device that requires 2 or 3 leads. If 3 leads are used, the first 2
21 are placed in the right atrium and right ventricle. The third lead is used to pace the left ventricle. Right atrial
22 and ventricular leads are easy to place as they only need to traverse the left subclavian vein and superior vena
23 cava to reach these cardiac chambers. In contrast, the lead that paces the left ventricle is more difficult to
24 position . The preferred transvenous approach involves passing the left ventricular pacing lead from the right
25 atrium into the coronary sinus, w hich resides in the atrioventricular groove on the posterior aspect of the
26 heart. It is then advanced into one of the lateral venous tributaries in order to optimize left ventricular pacing.
27
28 (Choice A) The anterior interventricular sulcus courses toward the apex of the heart on its anterior surface.
29 The anterior descending vessels lie in this sulcus.
30
31 (Choice C) The pulmonary artery lies just to the left of center on the anterior surface of the heart. It courses
32 from the right ventricle toward the aortic arch . Swan-Ganz catheters traverse the pulmonary artery.
33
(Choices D & E) The right atrium and ventricle are visible on the rightmost section of the cardiac silhouette.
34
35 Educational objective:
36 Left ventricular leads in biventricular pacemakers course through the coronary sinus, w hich resides in the
37 atrioventricular groove on the posterior aspect of the heart.
38
39
Time Spent: 9 seconds Copyright USMLEWorld,LLC. Last updated: [2/9/2014]
40

~
41
42 F!!ck suW.nd EnQck
---------------------------------------------------------------------------------------------------------------------------------------------
1
2
" Item: 44 of 46
Q.ld: 1639 [
111 f> Mark .<:J
Previous
[::>
Next
il
Lab Values
~
Notes
~
Calculator

CJ
5
6 A 43-year-old male presents to your office w ith difficulty w alking. On gait examination, you note that he leans
7 to the right side w hile w alking. W hen he is asked to stand on his left foot, his right hip tilts significantly
8 dow nward. W hich of the follow ing ne!Ves is most likely injured in this patient?
9
10
11 0 A. Superior gluteal
12
13
0 B. Inferior gluteal
14 0 C. Obturator
15
16 0 D. Femoral
17 0 E. Sciatic
18
19
20
21
22
23
24
25
26
27
28
29
30
31
32
33
34
35
36
37
38
39
40
41
42
~imeRemaini~ ~ 0
I Tuto' Feedback Suspend End Block
---------------------------------------------------------------------------------------------------------------------------------------------
1
2
" Item: 44 of 46
Q.ld: 1639 [
111 f> Mark .<:J
Previous
[::>
Next
il
Lab Values
~
Notes
~
Calculator

CJ
5
.; @ A. Superior gluteal [63%1
6 0 B. Inferior gluteal [21 %1
7 0 C. Obturator [8%1
8
9 0 D. Femoral [2%1
10 0 E. Sciatic [5%1
11
12
13 Explanation: User ld:
14
This patient demonstrates a positive Trendelenburg sign . The Trendelenburg test involves observing a patient
15
standing facing aw ay from the examiner w hile lifting each foot off of the ground. The sign is positive w hen the
16
hip dips tow ard the unaffected side w hen the patient stands on the affected leg. This can occur due to
17
w eakness of the gluteus medius and gluteus minimus muscles because these muscles function to pull the
18
pelvis dow n and abduct the thigh . These muscles are supplied by the superior gluteal nerve.
19
20 Patients w ith injury of the superior gluteal nerve may exhibit a w addling gait or a characteristic limp know n as
21 the "gluteus medius limp." The Trendelenburg sign and gluteus medius limp can also be seen in patients w ith
22 injuries to the gluteus medius and gluteus minimus muscles themselves as w ell as in patients w ith injuries to
23 the hip joint.
24
25 (Choice B) The inferior gluteal nerve innervates the gluteus maximus muscle. This muscle is responsible for
26 extension of the thigh at the hip and external rotation of the thigh . Injury of the inferior gluteal nerve causes
27 difficulty rising from the seated position and climbing stairs.
28
29 (Choice C) Injury of the obturator nerve causes deficits of thigh adduction as w ell as medial thigh sensory
30 loss.
31
32 (Choice D) The femoral nerve innervates the muscles responsible for flexion of the thigh at the hip (iliacus
33 and sartorius muscles) and extension of the leg at the knee (quadriceps femoris). It also supplies sensory
34 innervation to the skin on the anterior thigh and medial leg. Injury results in loss of the knee reflex and anterior
35 thigh sensory loss.
36
37 (Choice E) The sciatic nerve innervates all of the knee flexors (hamstrings) except the short head of the
38 biceps femoris . It subsequently divides into the tibial and common peroneal nerves, w hich each go on to
39 provide motor and sensory innervation to the leg and foot.
40
41
42
~imeRemaini~ ~ 0
I Tuto' Feedback Suspend End Block
---------------------------------------------------------------------------------------------------------------------------------------------
1
2
" Item: 44 of 46
Q.ld: 1639 [
111 f> Mark .<:J
Previous
[::>
Next
il
Lab Values
~
Notes
~
Calculator

CJ
5
Explanation: User ld:
6
7 This patient demonstrates a positive Trendelenburg sign . The Trendelenburg test involves observing a patient
8 standing facing aw ay from the examiner w hile lifting each foot off of the ground. The sign is positive w hen the
9 hip dips tow ard the unaffected side w hen the patient stands on the affected leg. This can occur due to
10 w eakness of the gluteus medius and gluteus minimus muscles because these muscles function to pull the
11 pelvis dow n and abduct the thigh . These muscles are supplied by the superior gluteal nerve.
12
13 Patients w ith injury of the superior gluteal nerve may exhibit a w addling gait or a characteristic limp know n as
14 the "gluteus medius limp." The Trendelenburg sign and gluteus medius limp can also be seen in patients w ith
15 injuries to the gluteus medius and gluteus minimus muscles themselves as w ell as in patients w ith injuries to
16 the hip joint.
17
18 (Choice B) The inferior gluteal nerve innervates the gluteus maximus muscle. This muscle is responsible for
19 extension of the thigh at the hip and external rotation of the thigh . Injury of the inferior gluteal nerve causes
20 difficulty rising from the seated position and climbing stairs.
21
22 (Choice C) Injury of the obturator nerve causes deficits of thigh adduction as w ell as medial thigh sensory
23 loss.
24
25 (Choice D) The femoral nerve innervates the muscles responsible for flexion of the thigh at the hip (iliacus
26 and sartorius muscles) and extension of the leg at the knee (quadriceps femoris). It also supplies sensory
27 innervation to the skin on the anterior thigh and medial leg. Injury results in loss of the knee reflex and anterior
28 thigh sensory loss.
29
(Choice E) The sciatic nerve innervates all of the knee flexors (hamstrings) except the short head of the
30
biceps femoris . It subsequently divides into the tibial and common peroneal nerves, w hich each go on to
31
provide motor and sensory innervation to the leg and foot.
32
33 Educational Objective:
34 1) Injury to the superior gluteal nerve causes w eakness of the gluteus medius and gluteus minimus muscles,
35 producing a positive Trendelenburg test.
36 2) The inferior gluteal nerve innervates the gluteus maximus muscle. Injury of the inferior gluteal nerve causes
37 difficulty rising from the seated position and climbing stairs.
38
39
Time Spent: 2 seconds Copyright USMLEW orld,LLC. Last updated: [11 / 10/2011]
40
41
42
~imeRemaini~ ~ 0
I Tuto' Feedback Suspend End Block
---------------------------------------------------------------------------------------------------------------------------------------------
1
2
" Item: 45 of 46
Q.ld: 8326 [
111 f> Mark .<:J
Previous
[::>
Next
il
Lab Values
~
Notes
~
Calculator

CJ
5
6 A 31-year-old man comes to the emergency department complaining of right-sided scrotal pain and sw elling
7 that has gradually w orsened over the last 3 days. His temperature is 38.3 C (101 F). On physical
8 examination, his right hemiscrotum is w arm, tender, and erythematous. The cremasteric reflex is present. A
9 scrotal ultrasound reveals a fluid collection consistent w ith a superficial scrotal abscess. W hich of the
10 follow ing lymph node groups is most likely to be tender and sw ollen?
11
12
13 0 A. Common iliac
14 0 B. Inferior mesenteric
15
16 0 C. Infraclavicular
17 0 D. Para-aortic
18
19 0 E. Superficial inguinal
20
21
22
23
24
25
26
27
28
29
30
31
32
33
34
35
36
37
38
39
40
41
42
~imeRemaini~ ~ 0
I Tuto' Feedback Suspend End Block
---------------------------------------------------------------------------------------------------------------------------------------------
1
2
" Item: 45 of 46
Q.ld: 8326 [
111 f> Mark .<:J
Previous
[::>
Next
il
Lab Values
~
Notes
~
Calculator

CJ
5 0 A. Common iliac [3%]
6
0 B. Inferior mesenteric [2%]
7
8 0 C. Infraclavicular (0%]
9 0 D. Para-aortic [26%]
10
11
v @ E. Superficial inguinal [69%]
12
13 Explanation: User ld:
14
15
Lymph vessels & nodes of male genitalia
16
17
18
19
20
21
22 Para-aortic nodes
23 ---::::---~,;.-;____ _ (receive lymph
24 drainage from testis)
25
26
27
28
29
30
31
32
33
34
35
36
37
38
39
40
41
42
1
2
" Item: 45 of 46
Q.ld: 8326 [
111 f> Mark .<:J
Previous
[::>
Next
il
Lab Values
~
Notes
~
Calculator

CJ
5
Lymph vessels & nodes of male genitalia
6
7
8
9
10
11 Para-aortic nodes
12 -:::>-----'":~--- (receive lymph
13 drainage from testis)
14
15
16
17
18
19
20
21
22
23
24
25
26
27
28 Deep inguinal nodes
29 -:7----~l---- (receive lymph drain age
30 from glans penis &
31
superficia l nodes)
32
33
34 Superficial inguinal nodes
35 \ ~~y>--j~-~- (receive lymph d rainage
36 from scrotum)
37
38
39
40
41
42
1
2

CJ
5
6
7
8 Deep inguinal nodes
9 =----~1----- (receive lymph drain age
10 from glans penis &
11 superficia l nodes)
12
13
14 Super ficial inguinal nodes
15 t/..1\\__:.>---T T _ I _ (receive lymph d rainage
16 from scrotum)
17
18
19
20
21
22
23
24
25
26 lJSMLEWorld.LLC
27
28 Lymphatic drainage of the scrotum occurs via the superficial inguinal lymph nodes. These lymph nodes drain
29 nearly all cutaneous lymph from the umbilicus to the feet, including the external genitalia and anus (up to the
30 dentate line). The exceptions are the testis, glans penis, and the cutaneous portion of the posterior calf.
31 Lymph frorn the testes drains directly into the para-aortic (retroperitoneal) lymph nodes (Choice D). Lymph
32 frorn the glans penis and the cutaneous portion of the posterior calf drains into the deep inguinal lymph
33 nodes. The superficial inguinal lymph nodes also drain into the deep inguinal lymph nodes.
34
35 (Choice A) The common iliac nodes are located alongside the common iliac artery and drain the internal and
36 external iliac nodes. The external iliac nodes receive lymph from the deep inguinal lymph nodes.
37
38 (Choice B) The inferior mesenteric nodes drain the structures supplied by branches of the inferior
39 mesenteric artery (eg, the left colic, sigmoid, and superior rectal arteries}. Thus, these nodes drain the
40 descending and sigmoid colon as w ell as the upper part of the rectum, and their efferents drain into the

~
41
42 F!!ck suW.nd EnQck
---------------------------------------------------------------------------------------------------------------------------------------------
1
2

CJ
5 _::;:,----:--:-- ..,..-- (receive lymph d rain ag e
6 from scrotum)
7
8
9
10
11
12
13
14
15
16 @ lJSMLEWorld, UC
17
18 Lymphatic drainage of the scrotum occurs via the superficial inguinal lymph nodes. These lymph nodes drain
19 nearly all cutaneous lymph from the umbilicus to the feet, including the external genitalia and anus (up to the
20 dentate line). The exceptions are the testis, glans penis, and the cutaneous portion of the posterior calf.
21 Lymph from the testes drains directly into the para-aortic (retroperitoneal) lymph nodes (Choice D). Lymph
22 from the glans penis and the cutaneous portion of the posterior calf drains into the deep inguinal lymph
23 nodes. The superficial inguinal lymph nodes also drain into the deep inguinal lymph nodes.
24
25 (Choice A) The common iliac nodes are located alongside the common iliac artery and drain the internal and
26 external iliac nodes. The external iliac nodes receive lymph from the deep inguinal lymph nodes.
27
28 (Choice B) The inferior mesenteric nodes drain the structures supplied by branches of the inferior
29 mesenteric artery (eg, the left colic, sigmoid, and superior rectal arteries). Thus, these nodes drain the
30 descending and sigmoid colon as w ell as the upper part of the rectum, and their efferents drain into the
31 pre-aortic nodes.
32
(Choice C) Infraclavicular lymph nodes are found beside the cephalic vein between the pectoralis major and
33
deltoid, immediately below the clavicle. They drain lymph from portions of the upper limb and breast.
34
35 Educational objective:
36 Due to its intra-abdominal origin, lymphatic drainage of the testis is to the para-aortic lymph nodes. In
37 contrast, lymph drainage from the scrotum goes into the superficial inguinal lymph nodes.
38
39 Time Spent: 1 seconds Copyright USMLEW orld,LLC. Last updated: [2118/2014]
40

~
41
42 F!!ck suW.nd EnQck
---------------------------------------------------------------------------------------------------------------------------------------------
1
2
" Item: 46 of 46
Q.ld: 1956 [
111 f> Mark .<:J
Previous
[::>
Next
il
Lab Values
~
Notes
~
Calculator

CJ
5
6 A 65-year-old Caucasian female is brought to the emergency department w ith severe left-sided hip pain after
7 falling in the bathroom . She is agitated and demands quick pain relief. A pelvic x-ray is show n below.
8
9
10
11
12
13
14
15
16
17
18
19
20
21
22
23
24
25
26
27
28
29 W hich of the follow ing arteries is most likely to be damaged in this patient?
30
31 0 A. Femoral
32
33 0 B. Medial circumflex
34 0 C. Lateral circumflex
35 0 D. Obturator
36
37 0 E. Inferior gluteal
38
39
40

~
41
42 F!!ck suW.nd
---------------------------------------------------------------------------------------------------------------------------------------------
EnQck
1
2
" Item: 46 of 46
Q.ld: 1956 [
111 f> Mark .<:J
Previous
[::>
Next
il
Lab Values
~
Notes
~
Calculator

CJ
5
~ @ B. Medial circumflex [49%]
0 C. Lateral circumflex [19%]
6
7 0 D. Obturator [12%]
8 0 E. Inferior gluteal [6%]
9
10
11 Explanation: User ld:
12
13 This patient's x-ray show s a nondisplaced left subcapital femoral neck fracture. Note below how the femoral
14 neck is markedly shortened compared to the right side. The dashed lines demarcate the intertrochanteric
15 line, and the red dots label the fracture plane.
16
17
18
19
20
21
22
23
24
25
26
27
28
29
30
31
32
33
34
35
36 Femoral neck fractures are common in elderly patients w ith osteoporosis w ho have sustained a fall. The
37 femoral head and neck derive their blood supply from the superior and inferior gluteal arteries and the medial
38 and lateral femoral circumflex arteries, vessels that together form the trochanteric anastomosis. The medial
39 femoral circumflex artery makes the largest contribution to the blood supply of this region and is vulnerable to
40 rl':lr-n<:~t"'o f.-rH''n for-n,..,r':ll nor V f.-':!rh uoc- ,..!, 10 t" itC" r ll"'c-o ':lc-c-r.r i-:~t-i "n llltith tho nnC'torin.- <:~C"n.ort nf tho for-n,..,.--:~1

~
41
42
---------------------------------------------------------------------------------------------------------------------------------------------
F!!ck suW.nd EnQck
1
2

CJ
5
6
7
8
9
10
11
12
13 Femoral neck fractures are common in elderly patients w ith osteoporosis w ho have sustained a fall. The
14 femoral head and neck derive their blood supply from the superior and inferior gluteal arteries and the medial
15 and lateral femoral circumflex arteries, vessels that together form the trochanteric anastomosis. The medial
16 femoral circumflex artery makes the largest contribution to the blood supply of this region and is vulnerable to
17 damage from femoral neck fractures due to its close association w ith the posterior aspect of the femoral
18 neck. Injury or thrombosis of this vessel predisposes to avascular necrosis of the femoral head.
19
20 (Choice A) The femoral artery gives rise to the medial and lateral femoral circumflex arteries. Injury to the
21 femoral artery is unlikely here given its anatomical relationship to the fractured femoral head and because
22 lower extremity claudication and other signs/symptoms of seriously compromised limb perfusion (including
23 limb necrosis} would be expected w ith such an injury.
24
25 (Choice C) The lateral femoral circumflex artery courses anterior to the femoral neck and contributes to the
26 blood supply of the femoral head and neck.
27
28 (Choice D) The obturator artery gives rise to an artery that supplies the femoral head. This vessel is
29 especially important in children because it supplies blood to the region of the femoral head proximal to the
30 epiphyseal growth plate. This artery is of minimal clinical significance in adults.
31
(Choice E) The superior and inferior gluteal arteries contribute to the trochanteric anastomosis that supplies
32
the head and neck of the femur.
33
34 Educational Objective:
35 A femoral neck fracture can damage the blood supply to the femoral head and neck. This is most common
36 w ith displaced fractures. The medial femoral circumflex artery provides the majority of the blood supply to the
37 femoral head and neck; injury to this vessel can cause avascular necrosis of the femoral head.
38
39 Time Spent: 7 seconds Copyright USMLEWorld,LLC. Last updated: [7/7/201 OJ
40

~
41
42 F!!ck suW.nd EnQck
---------------------------------------------------------------------------------------------------------------------------------------------
2
" Item: 1 of 46
Q. ld: 1812 [
111 f> Mark .<:J
Previous
[::>
Next
il
Lab Values
~
Notes
~
Calculator
3
4
5
6 A 34-year-old Caucasian female has difficulty abducting her right arm past the horizontal position . An
7 abnormal prominence of the right inferior scapular angle is also observed. W hich of the follow ing is the most
8 likely cause of this patient's condition?
9
10
11 0 A. Thyroidectomy
12
13
0 B. Anterior dislocation of the shoulder joint
14 0 C. Violent stretch between the head and the shoulder
15
16 0 D. Clavicular fracture
17 0 E. Mastectomy
18
19
20
21
22
23
24
25
26
27
28
29
30
31
32
33
34
35
36
37
38
39
40
41
42
~imeRemaini~ ~ 0
I Tuto' Feedback Suspend End Block
---------------------------------------------------------------------------------------------------------------------------------------------
2
" Item: 1 of 46
Q. ld: 1812 [
111 f> Mark .<:J
Previous
[::>
Next
il
Lab Values
~
Notes
~
Calculator
3
4
5
I " @ E. Mastectomy [52%]
6
7 Explanation: User ld:
8
9 The patient described in the question stem is exhibiting symptoms consistent w ith paralysis of the serratus
10 anterior muscle due to long thoracic nerve injury. Paralysis of the serratus anterior causes two classic signs.
11 First, w hen a patient is asked to press anteriorly against a w all scapular "w inging" can be observed. A w inged
12 scapula results from an inability of the serratus anterior to hold the medial border and inferior angle of the
13 scapula against the posterior chest w all. Second, patients w ith serratus anterior paralysis are unable to
14 abduct the arm higher than the horizontal position. The deltoid and supraspinatus muscles abduct the arm up
15 to the horizontal position, but at that point the action of the serratus anterior is required to rotate the glenoid
16 cavity superiorly, thereby allow ing complete abduction of the arm over the head. The long thoracic nerve
17 injury can occur during penetrating trauma or iatrogenically during axillary lymph node dissections as may
18 occur during a radical mastectomy.
19
20 (Choice A) Thyroidectomy carries many risks due to the complicated anatomy of the midline neck. Nerve
21 injuries that can occur w ith this procedure include damage to the recurrent laryngeal nerve during ligature of
22 the inferior thyroid artery and damage to the external branch of the superior laryngeal nerve during ligature of
23 the superior thyroid artery.
24
25 (Choice B) Anterior dislocation of the shoulder joint or a fracture of the neck of the humerus can injure the
26 axillary nerve, resulting in paralysis of the deltoid and teres minor and some sensory loss to the upper lateral
27 arm .
28
(Choice C) Violent stretch between the head and the shoulder can occur during delivery or by trauma in the
29
adult. This results in damage to the upper trunk of the brachial plexus and the classic Erb-Duchenne palsy.
30
This palsy results from damage to the musculocutaneous and suprascapular nerves and causes a w aiter's
31
tip posturing of the arm (shoulder adducted, arm pronated, and elbow extended).
32
33 (Choice D) Clavicular fracture does not cause any characteristic nerve palsies, but it is important to note that
34 these fractures typically occur in the middle third of the clavicle due to the strength of the ligamentous
35 structures at either end of this bone.
36
37 Educational Objective:
38 Mastectomy is a commonly tested cause of long thoracic nerve injury, but any trauma or surgery in the axillary
39 region is at risk of damaging this nerve. Injury to this nerve causes w inging of the scapula and inability to
40 -:ah.-.11 .... t tho C'hnl .lrlor n-:r.c-t on rlo.nr-o.o.C'

~
41
42 F!!ck
---------------------------------------------------------------------------------------------------------------------------------------------
suW.nd EnQck
2
" Item: 1 of 46
Q. ld: 1812 [
111 f> Mark .<:J
Previous
[::>
Next
il
Lab Values
~
Notes
~
Calculator
3
4
Explanation: User ld:
5
6 The patient described in the question stem is exhibiting symptoms consistent w ith paralysis of the serratus
7 anterior muscle due to long thoracic nerve injury. Paralysis of the serratus anterior causes two classic signs.
8 First, w hen a patient is asked to press anteriorly against a w all scapular "w inging" can be observed. A w inged
9 scapula results from an inability of the serratus anterior to hold the medial border and inferior angle of the
10 scapula against the posterior chest w all. Second, patients w ith serratus anterior paralysis are unable to
11 abduct the arm higher than the horizontal position . The deltoid and supraspinatus muscles abduct the arm up
12 to the horizontal position, but at that point the action of the serratus anterior is required to rotate the glenoid
13 cavity superiorly, thereby allow ing complete abduction of the arm over the head. The long thoracic nerve
14 injury can occur during penetrating trauma or iatrogenically during axillary lymph node dissections as may
15 occur during a radical mastectomy.
16
17 (Choice A) Thyroidectomy carries many risks due to the complicated anatomy of the midline neck. Nerve
18 injuries that can occur with this procedure include damage to the recurrent laryngeal nerve during ligature of
19 the inferior thyroid artery and damage to the external branch of the superior laryngeal nerve during ligature of
20 the superior thyroid artery.
21
22 (Choice B) Anterior dislocation of the shoulder joint or a fracture of the neck of the humerus can injure the
23 axillary nerve, resulting in paralysis of the deltoid and teres minor and some sensory loss to the upper lateral
24 arm .
25
26 (Choice C) Violent stretch between the head and the shoulder can occur during delivery or by trauma in the
27 adult. This results in damage to the upper trunk of the brachial plexus and the classic Erb-Duchenne palsy.
28 This palsy results from damage to the musculocutaneous and suprascapular nerves and causes a waiter's
29 tip posturing of the arm (shoulder adducted, arm pronated, and elbow extended).
30
(Choice D) Clavicular fracture does not cause any characteristic nerve palsies, but it is important to note that
31
these fractures typically occur in the middle third of the clavicle due to the strength of the ligamentous
32
structures at either end of this bone.
33
34 Educational Objective:
35 Mastectomy is a commonly tested cause of long thoracic nerve injury, but any trauma or surgery in the axillary
36 region is at risk of damaging this nerve. Injury to this nerve causes w inging of the scapula and inability to
37 abduct the shoulder past 90 degrees.
38
39
Time Spent: 286 seconds Copyright USMLEW orld,LLC. Last updated: [7/7/201 0]
40
41
42
~imeRemaini~ ~ 0
I Tuto' Feedback Suspend End Block
---------------------------------------------------------------------------------------------------------------------------------------------
1
" Item: 2 of 46
Q. ld: 1733 [
111 f> Mark .<:J
Previous
[::>
Next
il
Lab Values
~
Notes
~
Calculator
3
4
5
6 A 52-year-old golfer presents to your office w ith right shoulder pain that has started to interfere w ith his daily
7 activities, such as getting dressed. Imaging studies suggest calcification of the supraspinatus tendon . W hich
8 of the following is most likely to provoke pain in this patient?
9
10
11 0 A. Flexion of the humerus
12 0 B. Extension of the humerus
13
14 0 C. Adduction of the humerus
15 0 D. Abduction of the humerus
16
17 0 E. Medial rotation of the humerus
18
19
20
21
22
23
24
25
26
27
28
29
30
31
32
33
34
35
36
37
38
39
40
41
42
~imeRemaini~ ~ 0
I Tuto' Feedback Suspend End Block
---------------------------------------------------------------------------------------------------------------------------------------------
1
" Item: 2 of 46
Q. ld: 1733 [
111 f> Mark .<:J
Previous
[::>
Next
il
Lab Values
~
Notes
~
Calculator
3
4 v @ D. Abduction of the humerus [75%]
5
6
0 E. Medial rotation of the humerus [ 10%]
7
8 Explanation: User ld:
9
10
11
12
13
14
15
16 Subacromial bursa
17
18
19
20
21
22 Deltoid muscle
23
24
25
26
27
28
29 A
30
31 The rotator cuff is made up of the tendons of the supraspinatus, infraspinatus, subscapularis, and teres minor
32 muscles. These tendons, along w ith the tendon of the long head of the biceps brachii muscle and the
33 ligaments of the glenohumeral joint, contribute to the stability of the joint. In rotator cuff syndrome, the most
34 commonly injured tendon is the supraspinatus because this tendon is prone to repeated impingement trauma
35 between the humeral head and the acromion . The supraspinatus originates on the supraspinous fossa of the
36 scapula and inserts on the superior portion of the head of the humerus and is primarily an abductor of the
37 arm. Therefore, rotator cuff syndrome is most commonly associated w ith pain during abduction of the
38 humerus.
39
(Choice A) Flexion of the humerus at the shoulder is not accomplished by any of the rotator cuff muscles.
40 . . .. . .
41
42
~imeRemaini~ ~ 0
I Tuto' Feedback Suspend End Block
---------------------------------------------------------------------------------------------------------------------------------------------
1

3
4
5
6
Deltoid muscle
7
8
9
10
11
12
13 A
14
15 The rotator cuff is made up of the tendons of the supraspinatus, infraspinatus, subscapularis, and teres minor
16 muscles. These tendons, along w ith the tendon of the long head of the biceps brachii muscle and the
17 ligaments of the glenohumeral joint, contribute to the stability of the joint. In rotator cuff syndrome, the most
18 commonly injured tendon is the supraspinatus because this tendon is prone to repeated impingement trauma
19 between the humeral head and the acromion. The supraspinatus originates on the supraspinous fossa of the
20 scapula and inserts on the superior portion of the head of the humerus and is primarily an abductor of the
21 arm . Therefore, rotator cuff syndrome is most commonly associated w ith pain during abduction of the
22 humerus.
23
24 (Choice A) Flexion of the humerus at the shoulder is not accomplished by any of the rotator cuff muscles.
25 The anterior segment of the deltoid is an important flexor of the humerus.
26
27 (Choice B) Extension of the humerus is not accomplished by any of the rotator cuff muscles.
28
29 (Choice C) Adduction of the humerus is not accomplished by any of the rotator cuff muscles. The main
30 adductor of the humerus is the latissimus dorsi muscle.
31
(Choice E) Medial rotation of the humerus is accomplished by the subscapularis muscle, w hich originates on
32
the subscapular fossa and inserts onto the lesser tubercle of the head of the humerus.
33
34 Educational Objective:
35 The most commonly injured structure in rotator cuff syndrome is the tendon of the supraspinatus muscle.
36 Because the supraspinatus is an abductor of the humerus, injury to its tendon causes pain on abduction of
37 the arm .
38
39
Time Spent: 1 seconds Copyright USMLEW orld,LLC. Last updated: [7/7/201 0]
40
41
42
~imeRemaini~ ~ 0
I Tuto' Feedback Suspend End Block
---------------------------------------------------------------------------------------------------------------------------------------------
1
2
" Item: 3 of 46
Q. ld: 1747 [
111 f> Mark .<:J
Previous
[::>
Next
il
Lab Values
~
Notes
~
Calculator
4
5
6 A 54-year-old female undergoes thyroidectomy for several thyroid nodules that w ere suspicious for
7 malignancy on fine needle aspiration. W hile ligating the superior thyroid artery, the surgeon accidentally
8 severs a nearby nerve. W hich of the follow ing muscles is denervated as a result of this injury?
9
10
11 0 A. Thyroarytenoid
12 0 B. Lateral cricoarytenoid
13
14 0 C. Posterior cricoarytenoid
15 0 D. Cricothyroid
16
17 0 E. Aryepiglotticus
18
19
20
21
22
23
24
25
26
27
28
29
30
31
32
33
34
35
36
37
38
39
40
41
42
~imeRemaini~ ~ 0
I Tuto' Feedback Suspend End Block
---------------------------------------------------------------------------------------------------------------------------------------------
1
2
" Item: 3 of 46
Q. ld: 1747 [
111 f> Mark .<:J
Previous
[::>
Next
il
Lab Values
~
Notes
~
Calculator
4 v @ D. Cricothyroid [69%]
5
6 0 E. Aryepiglotticus [3%]
7
8 Explanation: User ld:
9
10 The superior thyroid artery, a branch of the external carotid artery, and the inferior thyroid artery, a branch of
11 the thyrocervical trunk, provide the blood supply to the thyroid and parathyroid glands. The superior thyroid
12 artery, superior thyroid vein, and external branch of the superior laryngeal nerve course together in a
13 neurovascular triad that originates superior to the thyroid gland and lateral to the thyroid cartilage. Because
14 the external branch of the superior laryngeal nerve courses just deep to the superior thyroid artery, it is at risk
15 of injury during thyroidectomy. The cricothyroid muscle is the only muscle innervated by this nerve. The
16 remaining laryngeal muscles are innervated by the recurrent laryngeal nerves, w hich also provide sensory
17 innervation to the larynx below the vocal folds. The internal branch of the superior laryngeal nerve does not
18 innervate any muscles, but provides sensory innervation to the laryngeal mucosa above the vocal folds.
19
20 Larynx
21
(Vagus nerve)
22
23
24
25
26 Superior laryngeal nerve Recurrent laryngeal nerve
27 !
28 All laryngeal muscles except cricothyroid
Sensory innervation below the vocal cords
29
30 External branch Internal branch
31 ! !
32 Cricothyroid muscle Sensory innervation above the vocal cords
33
34 (Choices A, B, C and E) These muscles are all innervated by the recurrent laryngeal nerve.
35
36 Educational Objective:
37 The external branch of the superior laryngeal nerve is at risk of injury during thyroidectomy due to its proximity
38 to the superior thyroid artery and vein . This nerve innervates the cricothyroid muscle.
39
40 1 ...,,...4. ............4--....1 r ... t"><l l '"lf\,..,1

~
41
42
---------------------------------------------------------------------------------------------------------------------------------------------
F!!ck suW.nd EnQck
1
2
" Item: 4 of 46
Q. ld: 1968 [
111 f> Mark .<:J
Previous
[::>
Next
il
Lab Values
~
Notes
~
Calculator
3

5
6 A 20-year-old man comes to the physician w ith right knee pain follow ing a soccer game. W hen asked how he
7 sustained the injury, he says, "I fell to the ground after colliding w ith m y teammate and then an opponent fell on
8 m y shin ." On physical examination, his knee appears slightly sw ollen, and there is excessive posterior
9 displacement of the tibia w hen pressure is applied to his anterior tibia w ith the knee in the flexed position. The
10 physician orders an MRI to confirm the diagnosis. An axial MRI cross section of the right knee is show n
11 below. W hich of the follow ing structures is most likely injured in this patient?
12
13
14
15
16
17
18
19
20
21
22
23
24
25
26
27
28
29
30
31
32 O A.A
33
34
0 B. B
35 0 C. C
36 0 D. D
37
O E.E
38
39
40
41
42
~imeRemaini~ ~ 0
I Tuto' Feedback Suspend End Block
---------------------------------------------------------------------------------------------------------------------------------------------
1
2
" Item: 4 of 46
Q. ld: 1968 [
111 f> Mark .<:J
Previous
[::>
Next
il
Lab Values
~
Notes
~
Calculator
3

~ @D. D [65%]
5
6 0 E. E [4%]
7
8
9 Explanation: User ld:
10
In this axial MRI of the right knee, the patellar ligament is visible anteriorly at the top of the image and the
11
gastrocnemius muscle is seen posteriorly at the bottom. The anterior and posterior cruciate ligaments are
12
found within the articular capsule of the knee joint and cross one another as each spans from the femur to the
13
tibia . The posterior cruciate ligament originates from the anterolateral surface of the medial femoral condyle
14
and inserts into the posterior intercondylar area of the tibia . It prevents posterior displacement of the tibia
15
relative to the femur. Its integrity can be tested in the clinical setting by using the posterior drawer test (the
16
maneuver described in the question stem}. The posterior cruciate ligament is most commonly injured during
17
sporting activities or motor vehicle accidents following a direct blow to the anterior proximal tibia .
18
19
20 Anterior view Posterior view
21
22
23
24
Posterior cruciate ligament
25
Lateral collateral ligament
26
{fibular collateral ligament )
27
28
29 Medial collateral ligament Popliteus tendon
30 {tibial collateral ligament) - ,;.::;:::::::;:::~
31
32
33
34 Anterior cruciate ligament
35
36
37
38 USMlfWOrld.ll( ) 201 I
39
40
41
42
~imeRemaini~ ~ 0
I Tuto' Feedback Suspend End Block
---------------------------------------------------------------------------------------------------------------------------------------------
1
2

6
7
8 Lateral collateral ligament
(fibular collateral ligament)
9
10
11 Medial collateral ligament Popliteus tendon
12 (tibial collateral ligament)
13
14
15
16 Anterior cruciate ligament
17
18
19
20 USMLEWorld.LLC 0 lOll
21
22
23 (Choice A) The anterior cruciate ligament functions to resist anterior displacement of the tibia relative to the
24 femur.
25
26 (Choices 8 and C) The lateral condyle of the femur articulates w ith the lateral superior articular surface of
27 the tibia . The periphery of this articulation is normally covered by the lateral meniscus, w hich is rounder and
28 covers a larger portion of the articular surface than the medial meniscus.
29
(Choice E) The medial condyle of the femur articulates w ith the medial superior articular surface of the
30
tibia. The periphery of this articulation is normally covered by the medial meniscus. The medial articular
31
surfaces of the knee are larger than the lateral articular surfaces, as this compartment bears more of the body'
32
s w eight.
33
34 Educational objective:
35 The posterior cruciate ligament prevents posterior displacement of the tibia relative to the femur. It originates
36 from the anterolateral surface of the medial femoral condyle and inserts into the posterior intercondylar area of
37 the tibia . Its integrity can be tested in the clinical setting by using the posterior draw er test.
38
39
Time Spent: 2 seconds Copyright USMLEW orld,LLC. Last updated: [11 /6/2013]
40
41
42
~imeRemaini~ ~ 0
I Tuto' Feedback Suspend End Block
---------------------------------------------------------------------------------------------------------------------------------------------
1
2
" Item: 5 of 46
Q. ld: 8557 [
111 f> Mark .<:J
Previous
[::>
Next
il
Lab Values
~
Notes
~
Calculator
3
4

6 A 34-year-old w oman comes to the physician complaining of double vision . She first experienced difficulty
7 focusing her eyes w hile at w ork yesterday. Upon w aking up this morning, her symptoms had progressed to
8 frank diplopia. Her past medical history is insignificant. A complete physical examination is performed,
9 including a full neurologic assessment. Shining light into her right eye causes constriction of her right pupil,
10 but not the left pupil. Shining light into her left eye causes constriction of only her right pupil. W hich of the
11 follow ing additional physical examination findings is likely to be found in this patient?
12
13
14 0 A. Absence of the left corneal reflex
15 0 B. Drooping of the left eyelid
16
17 0 C. Inability to close the left eye
18 0 D. Inward deviation of the left eye
19
20 0 E. Visual loss in the left eye
21
22
23
24
25
26
27
28
29
30
31
32
33
34
35
36
37
38
39
40

~
41
42 F!!ck
---------------------------------------------------------------------------------------------------------------------------------------------
suW.nd EnQck
1
2
" Item: 5 of 46
Q. ld: 8557 [
111 f> Mark .<:J
Previous Next
[::> il
Lab Values
~
Notes
~
Calculator
3
4 ., @ B. Drooping of the left eyelid (55%]

6 0 C. Inability to close the left eye (4%]


7 0 0 . Inw ard deviation of the left eye [1 3%]
8 0 E. Visual loss in the left eye [9%]
9
10
11 Explanation: User ld:
12
13 Pupillary light reflex
14
15
16
17
18
19
20
21
22
23
24
25 nerve
26
27
28
29
30
31
32
33 Oculomotor
34 \ , - - -- - : - -- -
n erve
35
36
37
38
39
40
41
42
1
2
" Item: 5 of 46
Q. ld: 8557 [
111 f> Mark .<:J
Previous
[::>
Next
il
Lab Values
~
Notes
~
Calculator
3
4
Explanation: User ld:
6
7
8
Pupillary light reflex
9
10
11
12
13
14
15
16
17
18
19
20
21
22
23
24
25
26 ..
27 ... Oculomotor
28 \;-- -- . . , -- -
29 nerve
30
31
32
33
34
35
36
37 Edinger-Westpha l nucleus
38
39 Pretectal nucleus
40 ff\ II~U I r;w,.u lrl I tr

~
41
42 F!!ck suW.nd
---------------------------------------------------------------------------------------------------------------------------------------------
EnQck
1
2
" Item: 5 of 46
Q. ld: 8557 [
111 f> Mark .<:J
Previous
[::>
Next
il
Lab Values
~
Notes
~
Calculator
3
4
The pupillary light reflex is assessed by shining light in an eye and observing the response in that eye (direct)
6 as w ell as the opposite eye (consensual). The optic nerve (CN II) is responsible for the afferent limb of this
7 reflex; the oculomotor nerve (CN Ill) is responsible for the efferent limb. In this patient, the right CN II and right
8 CN Ill are functional as there w as a direct response seen w hen light w as shined into the right eye. In addition,
9 the left CN II is also functional as a consensual response occurred w hen light w as shined into the left
10 eye. How ever, constriction of the left pupil w as not seen in either circumstance. Thus, this patient's lesion
11 most likely involves her left CN Ill.
12
13 The symptoms of CN Ill palsy relate to the function of CN Ill:
14
15 1. A somatic component innervates the inferior, superior, and medial rectus; inferior oblique; and levator
palpebrae muscles. Ptosis occurs due to paralysis of the levator palpebrae, and the unopposed
16
action of the lateral rectus (CN VI) and superior oblique (CN IV} muscles leads to a "dow n-and-out"
17
18 gaze.
19 2. Parasympathetic fibers of CN Ill innervate the iris sphincter and ciliary muscle. Paralysis of these
fibers causes a fixed, dilated pupil and loss of accommodation.
20
21
22 CN Ill palsy can result from lesions anywhere along the nerve's path from the oculomotor nucleus in the
23 midbrain to the extraocular muscles w ithin the orbit. The most dreaded cause of isolated acute third nerve
24 palsy is an actively enlarging intracranial aneurysm (at risk of imminent rupture).
25
26 (Choice A) Absence of the corneal reflex can result from lesions involving CN V, (afferent limb} or CN VII
27 (efferent limb).
28
29 (Choice C) A lesion involving the left CN VII w ould result in inability to close the left eye.
30
31 (Choice D) Inward deviation of the left eye w ould result from a CN VI lesion and the resultant unopposed
32 action of CN Ill.
33
(Choice E) The left CN II is functional as it w as able to produce a consensual response in the right eye.
34
35 Educational objective:
36 The pupillary light reflex is assessed by shining light in an eye and observing the response in that eye (direct)
37 and the opposite eye (consensual). The optic nerve (CN II) is responsible for the afferent limb of the pupillary
38 reflex, and the oculomotor nerve (CN Ill) is responsible for the efferent limb.
39
40
41
42
~imeRemaini~ ~ 0
I Tuto' Feedback Suspend End Block
---------------------------------------------------------------------------------------------------------------------------------------------
1
2
" Item: 5 of 46
Q. ld: 8557 [
111 f> Mark .<:J
Previous
[::>
Next
il
Lab Values
~
Notes
~
Calculator
3
4 The pupillary light reflex is assessed by shining light in an eye and observing the response in that eye (direct)
as w ell as the opposite eye (consensual). The optic nerve (CN II) is responsible for the afferent limb of this
6 reflex; the oculomotor nerve (CN Ill) is responsible for the efferent limb. In this patient, the right CN II and right
7 CN Ill are functional as there w as a direct response seen w hen light w as shined into the right eye. In addition,
8 the left CN II is also functional as a consensual response occurred w hen light w as shined into the left
9 eye. How ever, constriction of the left pupil w as not seen in either circumstance. Thus, this patient's lesion
10 most likely involves her left CN Ill.
11
12 The symptoms of CN Ill palsy relate to the function of CN Ill:
13
14 1. A somatic component innervates the inferior, superior, and medial rectus; inferior oblique; and levator
palpebrae muscles. Ptosis occurs due to paralysis of the levator palpebrae, and the unopposed
15
action of the lateral rectus (CN VI) and superior oblique (CN IV) muscles leads to a "dow n-and-out"
16
17 gaze.
18 2. Parasympathetic fibers of CN Ill innervate the iris sphincter and ciliary muscle. Paralysis of these
19 fibers causes a fixed, dilated pupil and loss of accommodation.
20
21 CN Ill palsy can result from lesions anywhere along the nerve's path from the oculomotor nucleus in the
22 midbrain to the extraocular muscles w ithin the orbit. The most dreaded cause of isolated acute third nerve
23 palsy is an actively enlarging intracranial aneurysm (at risk of imminent rupture).
24
25 (Choice A) Absence of the corneal reflex can result from lesions involving CN V, (afferent limb) or CN VII
26 (efferent limb).
27
28 (Choice C) A lesion involving the left CN VII w ould result in inability to close the left eye.
29
30 (Choice D) Inw ard deviation of the left eye w ould result from a CN VI lesion and the resultant unopposed
31 action of CN Ill.
32
(Choice E) The left CN II is functional as it w as able to produce a consensual response in the right eye.
33
34 Educational objective:
35 The pupillary light reflex is assessed by shining light in an eye and observing the response in that eye (direct)
36 and the opposite eye (consensual). The optic nerve (CN II) is responsible for the afferent limb of the pupillary
37 reflex, and the oculomotor nerve (CN Ill) is responsible for the efferent limb.
38
39 Time Spent: 2 seconds Copyright USMLEW orld,LLC. Last updated: [1/4/201 4]
40
41
42
~imeRemaini~ ~ 0
I Tuto' Feedback Suspend End Block
--------------------------------------------------------------------------------------------------------------------------------------
1
2
" Item: 6 of 46
Q. ld: 1749 [
111 f> Mark .<:J
Previous
[::>
Next
il
Lab Values
~
Notes
~
Calculator
3
4
5
A 45-year-old Caucasian female presents to your office w ith numbness of the right palm . After taking a
7 detailed history and doing a physical examination, you suspect a median nerve lesion at the w rist. Before
8 reaching the w rist, the median nerve courses between the:
9
10
11 0 A. Biceps and coracobrachialis muscles
12 0 B. Flexor carpi ulnaris and flexor digitorum profundus muscles
13
14 0 C. Flexor digitorum superficialis and flexor digitorum profundus muscles
15 0 D. Olecranon and the medial epicondyle of the humerus
16
17 0 E. Supinator muscle and the head of the radius
18
19
20
21
22
23
24
25
26
27
28
29
30
31
32
33
34
35
36
37
38
39
40
41
42
~imeRemaini~ ~ 0
I Tuto' Feedback Suspend End Block
---------------------------------------------------------------------------------------------------------------------------------------------
1
2
" Item: 6 of 46
Q. ld: 1749 [
111 f> Mark .<:J
Previous
[::>
Next
il
Lab Values
~
Notes
~
Calculator
3
4 ~ @ C. Flexor digitorum superficialis and flexor digitorum profundus muscles [51 %]
5
0 D. Olecranon and the medial epicondyle of the humerus [1 0%]
7 0 E. Supinator muscle and the head of the radius [1 0%]
8
9
10 Explanation: User ld:
11
The median nerve receives contributions from the C6 through T1 spinal nerves and emerges as a distinct
12 structure from a combination of fibers from the lateral and medial cords of the brachial plexus. From this
13 location, the median nerve courses w ith the brachial artery in the groove betw een the biceps brachii and the
14
brachialis muscles. It gains access to the forearm in the medial aspect of the antecubital fossa and
15 immediately courses between the humeral and ulnar heads of the pronator teres muscle. It then courses
16 between the flexor digitorum superficialis and the flexor digitorum profundus muscles before entering the w rist
17 and hand w ithin the flexor retinaculum. Injury of the median nerve at the w rist can result from suicide
18
attempts, carpal tunnel syndrome, and other traumatic insults to the anterior w rist. Denervation atrophy leads
19
to loss of the thenar eminence and an "ape hand" deformity. Sensation is also lost on the palmar surface of
20 the first three and one-half fingers (including the thumb).
21
22 (Choice A) The musculocutaneous nerve courses directly between the biceps brachii and the
23 coracobrachialis muscles. Injury to this nerve leads to paralysis of the biceps and the brachialis, w hich
24 results in an inability to flex the forearm .
25
26 (Choice B) The ulnar nerve courses between the flexor carpi ulnaris and flexor digitorum profundus in the
27 forearm . Injury to this nerve leads to a "claw hand" deformity due to paralysis of most of the intrinsic muscles
28 of the hand.
29
30 (Choice D) The ulnar nerve also courses between the olecranon and the medial epicondyle of the humerus.
31 This is the location of the "funnybone" and is a common site of injury to the ulnar nerve due to the superficial
32 location of the nerve in this region.
33
34 (Choice E) The radial nerve courses through the supinator muscle near the head of the radius. Injury to this
35 nerve causes "wrist drop" due to the fact that the radial nerve innervates the extensors of the hand at the
36 forearm.
37
Educational Objective:
38
The median nerve courses between the humeral and ulnar heads of the pronator teres muscle and then runs
39
40 between the flexor digitorum superficialis and the flexor digitorum profundus muscles before entering the w rist
41
42
~imeRemaini~ ~ 0
I Tuto' Feedback Suspend End Block
---------------------------------------------------------------------------------------------------------------------------------------------
1
2
" Item: 6 of 46
Q. ld: 1749 [
111 f> Mark .<:J
Previous
[::>
Next
il
Lab Values
~
Notes
~
Calculator
3
4 \.-1 C . VUtJIIICilUI IIIU~I..I~ CIIIU lilt: IICCIU U l lll~ I dUIU~ l I V 70J
5
Explanation: User ld:
7
8 The median nerve receives contributions from the C6 through T1 spinal nerves and emerges as a distinct
9 structure from a combination of fibers from the lateral and medial cords of the brachial plexus. From this
10 location, the median nerve courses w ith the brachial artery in the groove between the biceps brachii and the
11 brachialis muscles. It gains access to the forearm in the medial aspect of the antecubital fossa and
12 immediately courses between the humeral and ulnar heads of the pronator teres muscle. It then courses
13 between the flexor digitorum superficialis and the flexor digitorum profundus muscles before entering the w rist
14 and hand w ithin the flexor retinaculum. Injury of the median nerve at the w rist can result from suicide
15 attempts, carpal tunnel syndrome, and other traumatic insults to the anterior w rist. Denervation atrophy leads
16 to loss of the thenar eminence and an "ape hand" deformity. Sensation is also lost on the palmar surface of
17 the first three and one-half fingers (including the thumb).
18
19 (Choice A) The musculocutaneous nerve courses directly between the biceps brachii and the
20 coracobrachialis muscles. Injury to this nerve leads to paralysis of the biceps and the brachialis, w hich
21 results in an inability to flex the forearm.
22
23 (Choice B) The ulnar nerve courses between the flexor carpi ulnaris and flexor digitorum profundus in the
24 forearm . Injury to this nerve leads to a "claw hand" deformity due to paralysis of most of the intrinsic muscles
25 of the hand.
26
27 (Choice D) The ulnar nerve also courses between the olecranon and the medial epicondyle of the humerus.
28 This is the location of the "funnybone" and is a common site of injury to the ulnar nerve due to the superficial
29 location of the nerve in this region.
30
(Choice E) The radial nerve courses through the supinator muscle near the head of the radius. Injury to this
31
nerve causes "w rist drop" due to the fact that the radial nerve innervates the extensors of the hand at the
32
forearm .
33
34 Educational Objective:
35 The median nerve courses between the humeral and ulnar heads of the pronator teres muscle and then runs
36 between the flexor digitorum superficialis and the flexor digitorum profundus muscles before entering the w rist
37 and hand w ithin the carpal tunnel.
38
39
Time Spent: 1 seconds Copyright USMLEW orld,LLC. Last updated: [11 / 10/2011]
40
41
42
~imeRemaini~ ~ 0
I Tuto' Feedback Suspend End Block
---------------------------------------------------------------------------------------------------------------------------------------------
1
2
" Item: 7 of 46
Q. ld: 1697 [
111 f> Mark .<:J
Previous
[::>
Next
il
Lab Values
~
Notes
~
Calculator
3
4
5
6 A 12-year-old soccer player presents to the emergency room w ith acute right knee pain after sustaining a kick
injury to an extended leg. The trauma caused anterior displacement of her tibia w ith respect to her femur.
8 W hich of the follow ing ligaments w as most likely injured?
9
10
11 0 A. Fibular (lateral) collateral
12 0 B. Tibial (medial) collateral
13
14 0 C. Patellar
15 0 D. Anterior cruciate
16
17 0 E. Posterior cruciate
18
0 F. Oblique popliteal
19
20
21
22
23
24
25
26
27
28
29
30
31
32
33
34
35
36
37
38
39
40
41
42
~imeRemaini~ ~ 0
I Tuto' Feedback Suspend End Block
---------------------------------------------------------------------------------------------------------------------------------------------
1
2
" Item: 7 of 46
Q. ld: 1697 [
111 f> Mark .<:J
Previous
[::>
Next
il
Lab Values
~
Notes
~
Calculator
3
4
5 .; @ D. Anterior cruciate [78%]
6 0 E. Posterior cruciate [15%]
0 F. Oblique popliteal [0%]
8
9
10 Explanation: User ld:
11
12
13 Posterior cruciate ligament
Anterior cruciate ligament
14
15
16 Medial condyle of femu r
17 Lateral condyle of femur
18
19
20
Popliteus tendon
21
22 Medial meniscus
23
24
25 - - Medial collatera l ligament
26 (tibial collateral ligament)
27
28 Lateral collateral ligament Transverse ligament of knee
29 (fibular collateral ligament)
30
31
32
33
34
35
36
USMLEWoM.llC (Q 2011
37
38
39
40 The anterior cruciate liqament (ACL) connects the lateral femoral condyle to the anteromedial tibial head. The
41
42
~imeRemaini~ ~ 0
I Tuto' Feedback Suspend End Block
---------------------------------------------------------------------------------------------------------------------------------------------
1
2
3
4
5

USMLEWo~d. LLC <Q 2011

9
10 The anterior cruciate ligament (ACL) connects the lateral femoral condyle to the anteromedial tibial head. The
11 ACL prevents anterior motion of the tibia w ith respect to the femur. Damage to the ACL can be assessed on
12 physical examination w ith the anterior drawer test, a maneuver that involves flexing the low er extremity to
13 approximately go and pulling the tibia anteriorly w ith the femur held fixed . Laxity indicates ACL injury. This
14 patient appears to have sustained force on the anterior tibia greater than the ACL could bear, causing
15 ligament rupture.
16
17 Athletes w ho play contact sports are at risk for complex knee injuries, w hich include simultaneous damage to
18 the tibial (medial) collateral ligament, menisci, and ACL. These injuries generally result from a forceful blow to
19 the lateral knee w hile the low er extremity is fully extended and the foot is rigidly planted. Extreme force to the
20 lateral knee causes tearing of the tibial (medial) collateral ligament. A meniscal tear also occurs. The ACL
21 can be injured by this mechanism due to anterior and medial movement of the tibia w ith respect to the femur.
22
23 (Choice A) The fibular (lateral) collateral ligament is extremely strong and is rarely injured. Injury to this
24 ligament w ould require a forceful blow to the medial knee w hile the leg is extended.
25
26 (Choice C) The patellar ligament is the inferior projection of the quadriceps femoris tendon. Injury causes
27 difficulty w ith leg extension.
28
29 (Choice E) The posterior cruciate ligament extends from the medial condyle of the femur to the posterior
30 head of the tibia . It functions to prevent posterior movement of the tibia w ith respect to the femur. Damage is
31 assessed using the posterior drawer test.
32
(Choice F) The oblique popliteal ligament is a w ide fibrous band that connects the posterior distal femur to the
33
posterior proximal tibia . It has small openings for nerves, vessels, and muscle insertions.
34
35 Educational Objective:
36 The anterior cruciate ligament (ACL) can be damaged by forceful anterior motion of the tibia w ith respect to
37 the femur.
38
39
Time Spent: 1 seconds Copyright USMLEW orld,LLC. Last updated: [1 /7/2014]
40
41
42
~imeRemaini~ ~ 0
I Tuto' Feedback Suspend End Block
---------------------------------------------------------------------------------------------------------------------------------------------
1
2
" Item: 8 of 46
Q. ld: 1930 [
111 f> Mark .<:J
Previous
[::>
Next
il
Lab Values
~
Notes
~
Calculator
3
4 A 23-year-old male is evaluated after sustaining a neck and shoulder injury. The nerve indicated by the blue
5 arrow on the illustration below is injured in this patient.
6
7

9
10
11
12
13
14
15
16
17
18
19
20
21
22
23
24
25
26
27
28
29
30
31
32
Which of the following is most likely to be weakened as a result of this injury?
33
34
35 0 A. Flexion of the hand
36 0 B. Abduction of the fingers
37
38 0 C. Extension of the hand
39 0 D. Adduction of the thumb
40

~
41
42 F!!ck suW.nd
---------------------------------------------------------------------------------------------------------------------------------------------
EnQck
1
2
" Item: 8 of 46
Q. ld: 1930 [
111 f> Mark .<:J
Previous
[::>
Next
il
Lab Values
~
Notes
~
Calculator
3
4 arrow on the tllustratton below ts tnjured tn thts patten!.
5
6
7

9
10
11
12
13
14
15
16
17
18
19
20
21
22
23
24
25
26
27
28
29
30
31
Which of the following is most likely to be weakened as a result of this injury?
32
33
34 0 A. Flexion of the hand
35 0 B. Abduction of the fingers
36
37 0 C. Extension of the hand
38 0 D. Adduction of the thumb
39 0 E. Opposition of the thumb
40
41
42
~imeRemaini~ ~ 0
I Tuto' Feedback Suspend End Block
---------------------------------------------------------------------------------------------------------------------------------------------
1
2
" Item: 8 of 46
Q. ld: 1930 [
111 f> Mark .<:J
Previous
[::>
Next
il
Lab Values
~
Notes
~
Calculator
3
4 v @ C. Extension of the hand [73%1
5
6 0 D. Adduction of the thumb [4%1
7 0 E. Opposition of the thumb [8%1

9
10 Explanation: User ld:
11
12
13
14
15
16
17
18
19
20
21
22
23
24 Musculocutaneous
25 ..j..
26
27
28
29
t
Long thoracic nerve
30
31
32
33
34 Medial cutaneous nerve of the arm
35
36
37 Medial cutaneous nerve of the foream1
38
39 The structure identified in this illustration is the radial nerve, which receives fibers from C5-C8 and T1 . The
40

~
41
42 F!!ck
---------------------------------------------------------------------------------------------------------------------------------------------
suW.nd EnQck
1
2
3
4
5
6
7

9
10
11
12
t
Long thoracic nerve
13
14
15
16
17
18
19
20 Medial cutaneous nerve of the forearm
21
22 The structure identified in this illustration is the radial nerve, w hich receives fibers from C5-C8 and T1. The
23 radial nerve is the largest branch of the brachial plexus. In addition to supplying the skin of the posterior arm
24 and forearm, the radial nerve innervates the ex1ensor muscles of the upper limb. Because this nerve tracks
25 w ithin the radial groove on the humerus, it is vulnerable to injury should the humerus fracture. Damage to the
26 radial nerve results in "wrist drop," a condition in w hich the w rist and fingers cannot be ex1ended .
27
28 (Choice A) Damage to the median nerve w ould limit hand flexion .
29
30 (Choice B) Damage to the recurrent branch of the median nerve or deep branch of the ulnar nerve w ould limit
31 finger abduction.
32
33 (Choice D) Damage to the deep branch of the ulnar nerve w ould limit thumb adduction .
34
(Choice E) Damage to the median nerve affects opposition of the thumb.
35
36 Educational Objective:
37 The radial nerve is responsible for ex1ension of the hand. Damage to the radial nerve results in w rist drop.
38
39
Time Spent: 6 seconds Copyright USMLEW orld,LLC. Last updated: [11 / 10/2011]
40
41
42
~imeRemaini~ ~ 0
I Tuto' Feedback Suspend End Block
--------------------------------------------------------------------------------------------------------------------------------------
1
2
" Item: 9 of 46
Q. ld: 7621 [
111 f> Mark .<:J
Previous
[::>
Next
il
Lab Values
~
Notes
~
Calculator
3
4
5
6 A 34-year-old w oman comes to the physician complaining of easy fatigability. She lives alone and w orks as
7 an accountant. Her menstrual cycles are regular and her last menstrual period w as 2 w eeks ago. She does
8 not smoke or consume alcohol. Her blood pressure is 150/90 mm Hg and pulse is 82/min. During her
physical examination, she sits up from the supine position without using her hands. Which of the following
10 muscles contributes most to the described movement?
11
12
13 0 A. Adductor magnus
14
0 B. Biceps femoris, long head
15
16 0 C. Gluteus medius
17 0 D. Obturator externus
18
19 0 E. Psoas major
20 0 F. Vastus medialis
21
22
23
24
25
26
27
28
29
30
31
32
33
34
35
36
37
38
39
40
41
42
~imeRemaini~ ~ 0
I Tuto' Feedback Suspend End Block
---------------------------------------------------------------------------------------------------------------------------------------------
1
2
" Item: 9 of 46
Q. ld: 7621 [
111 f> Mark .<:J
Previous
[::>
Next
il
Lab Values
~
Notes
~
Calculator
3
4 v @ E. Psoas major [67%]
5
6 0 F. Vastus medialis [5%]
7
8 ExJ>Ianation User ld

10 Major muscles responsible for motion at the hip


11 Flexion Extension Abduction Adduction
12
13 Gluteus maximus
Iliopsoas Adductor brevis
14 Semitendinosus Gluteus medius
Rectus femoris Adductor longus
Semimembranosus Gluteus minimus
15 Tensor fascia lata Adductor magnus
Biceps femoris - long head
16
17
18 Major muscles used w hen sitting up from the supine position include the external abdominal obliques, the
19 rectus abdominis, and the hip flexors. The psoas major and iliacus contribute most significantly to the hip
20 flexion; they, along w ith the psoas minor, are collectively know n as the iliopsoas. The capacity of these
21 muscles to carry out this motion can be deduced from the fact that they originate on the pelvis and spinal
22 column, cross the hip joint, and insert on the femur.
23
24 The psoas major muscle arises from the bodies and intervertebral discs of the inferior-most thoracic and all of
the lumbar vertebrae. The iliacus originates from the iliac fossa, a large concave surface found on the inner
25
aspect of the ilium. Both muscles insert via the common iliopsoas tendon into the lesser trochanter of the
26
femur.
27
28 (Choice A) The adductor portion of the adductor magnus originates from the inferior ramus of the ischium
29 and inserts into the proximal two-thirds of the posteromedial femur. It is a pow erful adductor of the thigh.
30
31 (Choice B) The long head of the biceps femoris originates from the ischial tuberosity and inserts into the
32 lateral head of the fibula; it functions as a thigh extensor and leg flexor.
33
34 (Choice C) The gluteus medius abducts the femur and is important in keeping the hip level during gait w hen
35 the body's w eight rests on one leg.
36
37 (Choice D) The obturator externus arises from the anteromedial surface of the obturator foramen and inserts
38 near the greater trochanter of the femur. This muscle is a lateral rotator of the thigh.
39
40 (Choice F) The vastus medialis is a component of the quadriceps femoris. It arises from the proximal femur
41
42
~imeRemaini~ ~ 0
I Tuto' Feedback Suspend End Block
---------------------------------------------------------------------------------------------------------------------------------------------
1
2
" Item: 9 of 46
Q. ld: 7621 [
111 f> Mark .<:J
Previous
[::>
Next
il
Lab Values
~
Notes
~
Calculator
3
4 lltopsoas Adductor brevts
Semitendinosus Gluteus medius
5 Rectus femoris Adductor longus
Semimembranosus Gluteus minimus
6 Tensor fascia lata Adductor magnus
Biceps femoris - long head
7
8
Major muscles used w hen sitting up from the supine position include the external abdominal obliques, the
rectus abdominis, and the hip flexors. The psoas major and iliacus contribute most significantly to the hip
10
flexion; they, along w ith the psoas minor, are collectively know n as the iliopsoas. The capacity of these
11
muscles to carry out this motion can be deduced from the fact that they originate on the pelvis and spinal
12
column, cross the hip joint, and insert on the femur.
13
14 The psoas major muscle arises from the bodies and intervertebral discs of the inferior-most thoracic and all of
15 the lumbar vertebrae. The iliacus originates from the iliac fossa, a large concave surface found on the inner
16 aspect of the ilium. Both muscles insert via the common iliopsoas tendon into the lesser trochanter of the
17 femur.
18
19 (Choice A) The adductor portion of the adductor magnus originates from the inferior ramus of the ischium
20 and inserts into the proximal two-thirds of the posteromedial femur. It is a pow erful adductor of the thigh.
21
22 (Choice B) The long head of the biceps femoris originates from the ischial tuberosity and inserts into the
23 lateral head of the fibula; it functions as a thigh extensor and leg flexor.
24
25 (Choice C) The gluteus medius abducts the femur and is important in keeping the hip level during gait w hen
26 the body's w eight rests on one leg.
27
28 (Choice D) The obturator externus arises from the anteromedial surface of the obturator foramen and inserts
29 near the greater trochanter of the femur. This muscle is a lateral rotator of the thigh.
30
(Choice F) The vastus medialis is a component of the quadriceps femoris. It arises from the proximal femur
31
and ultimately inserts into the tibial tuberosity by means of the patella . It functions as a knee extensor.
32
33 Educational objective:
34 Muscles used w hen sitting up from the supine position include the external abdominal obliques, the rectus
35 abdominis, and the hip flexors. The iliopsoas muscle is the most important of the hip flexors and includes the
36 psoas major, psoas minor, and iliacus. The rectus femoris, sartorius, tensor fascia lata, and the medial
37 compartment of the thigh also contribute to hip flexion .
38
39
Time Spent: 1 seconds Copyright USMLEW orld,LLC. Last updated: [1 / 14/2014]
40 "
41
42
~imeRemaini~ ~ 0
I Tuto' Feedback Suspend End Block
---------------------------------------------------------------------------------------------------------------------------------------------
1
2
" Item: 10 of 46
Q.ld: 1924 [
lil f> Mark .<:J
Previous
[::>
Next
il
Lab Values
~
Notes
~
Calculator
3
4
5
6 A 33-year-old male slips w hile working in his yard and falls on outstretched hands. He later presents to the
7 emergency room complaining of right shoulder pain. On physical examination, there is flattening of the right
8 deltoid muscle and insensitivity of the overlying skin to pinprick. W hich of the follow ing injuries is most likely
9 responsible?

11
12 0 A. Anterior dislocation of the humerus
13 0 B. Acromioclavicular joint subluxation
14
15 0 C. Clavicular fracture
16 0 D. Spiral fracture of the humeral midshaft
17
18 0 E. Fracture of the coracoid process
19 0 F. Rotator cuff tear
20
21
22
23
24
25
26
27
28
29
30
31
32
33
34
35
36
37
38
39
40
41
42
~imeRemaini~ ~ 0
I Tuto' Feedback Suspend End Block
---------------------------------------------------------------------------------------------------------------------------------------------
1
2
" Item: 10 of 46
Q.ld: 1924 [
lil f> Mark .<:J
Previous
[::>
Next
il
Lab Values
~
Notes
~
Calculator
3
4
"' @ A. Anterior dislocation of the humerus [65%]
5
6 0 B. Acromioclavicular joint subluxation [1 3%]
7 0 C. Clavicular fracture [5%]
8
0 D. Spiral fracture of the humeral midshaft [7%]
9
0 E. Fracture of the coracoid process [4%]
11 0 F. Rotator cuff tear [6%]
12
13
14 Explanation: User ld:
15
16 The glenohumeral joint is the most commonly dislocated joint in the body ow ing to the shallow articulation
17 between the humeral head and the glenoid fossa of the scapula . The shoulder may dislocate anteriorly,
18 inferiorly or posteriorly, but anterior dislocations are by far the most common . Anterior dislocations of the
19 humerus classically follow injuries involving forceful external rotation and abduction of the arm at the
20 shoulder. W hen the head of the humerus is displaced anteriorly, there is flattening of the deltoid prominence,
21 protrusion of the acromion, and anterior axillary fullness (due to the humeral head's movement into this
22 location}. The axillary nerve is the nerve most commonly injured by anterior shoulder dislocations. It
23 innervates the deltoid and teres minor muscles and provides sensory innervation to the lateral arm, including
24 the skin overlying the deltoid.
25
(Choice B) Acromioclavicular joint subluxation typically results from a dow nward blow on the tip of the
26
shoulder and produces sw elling and upw ard displacement of the clavicle. It is not associated w ith specific
27
major nerve injuries or sensory deficits.
28
29 (Choice C) Clavicular fractures usually occur follow ing direct trauma to the clavicle. Most fractures are in the
30 middle third of the clavicle and produce local sw elling and tenderness. Associated neurovascular damage is
31
rare.
32
33 (Choice D) A spiral humeral midshaft fracture may result from torsion produced during a fall on an
34 outstretched hand. Patients present w ith sw elling, crepitus and ecchymoses of the arm . The radial nerve is
35 commonly injured in fractures of the humeral midshaft.
36
37 (Choice E) Fracture of the coracoid process of the scapula is rare. Most commonly affected are individuals
38 w ho engage in shotgun or rifle-related sports .
39
40 !Choice Fl Rotator cuff tears mav occur in shoulder dislocations. but do not cause nerve iniurv. The rotator

~
41
42 F!!ck
---------------------------------------------------------------------------------------------------------------------------------------------
suW.nd EnQck
1
2
" Item: 10 of 46
Q.ld: 1924 [
lil f> Mark .<:J
Previous
[::>
Next
il
Lab Values
~
Notes
~
Calculator
3
4
5
Explanation: User ld:
6
7 The glenohumeral joint is the most commonly dislocated joint in the body ow ing to the shallow articulation
8 between the humeral head and the glenoid fossa of the scapula. The shoulder may dislocate anteriorly,
9 inferiorly or posteriorly, but anterior dislocations are by far the most common. Anterior dislocations of the
humerus classically follow injuries involving forceful external rotation and abduction of the arm at the
11 shoulder. W hen the head of the humerus is displaced anteriorly, there is flattening of the deltoid prominence,
12 protrusion of the acromion, and anterior axillary fullness (due to the humeral head's movement into this
13 location). The axillary nerve is the nerve most commonly injured by anterior shoulder dislocations. It
14 innervates the deltoid and teres minor muscles and provides sensory innervation to the lateral arm, including
15 the skin overlying the deltoid.
16
17 (Choice B) Acromioclavicular joint subluxation typically results from a dow nward blow on the tip of the
18 shoulder and produces sw elling and upw ard displacement of the clavicle. It is not associated w ith specific
19 major nerve injuries or sensory deficits.
20
21 (Choice C) Clavicular fractures usually occur follow ing direct trauma to the clavicle. Most fractures are in the
22 middle third of the clavicle and produce local sw elling and tenderness. Associated neurovascular damage is
23 rare.
24
25 (Choice D) A spiral humeral midshaft fracture may result from torsion produced during a fall on an
26 outstretched hand. Patients present w ith sw elling, crepitus and ecchymoses of the arm . The radial nerve is
27 commonly injured in fractures of the humeral midshaft.
28
29 (Choice E) Fracture of the coracoid process of the scapula is rare. Most commonly affected are individuals
30 w ho engage in shotgun or rifle-related sports.
31
(Choice F) Rotator cuff tears may occur in shoulder dislocations, but do not cause nerve injury. The rotator
32
cuff is made up of the subscapularis, supraspinatus, infraspinatus and teres minor muscles and tendons.
33
34 Educational Objective:
35 Flattening of the deltoid muscle after a shoulder injury suggests anterior shoulder dislocation . This injury most
36 commonly results from forceful external rotation and abduction at the shoulder joint. Axillary nerve injury,
37 resulting in deltoid paralysis and loss of sensation over the lateral arm, is often associated .
38
39
Time Spent: 1 seconds Copyright USMLEW orld,LLC. Last updated: [7/7/201 0]
40
41
42
~imeRemaini~ ~ 0
I Tuto' Feedback Suspend End Block
---------------------------------------------------------------------------------------------------------------------------------------------
1
2
" Item: 11 of 46
Q.ld: 1885 [
111 f> Mark .<:J
Previous
[::>
Next
il
Lab Values
~
Notes
~
Calculator
3
4
5
6 A 52-year-old male is being evaluated for vague back and abdominal pain . He has lost 4 lbs over the past
7 three w eeks. An axial CT scan of his abdomen is shown below.
8
9
10

12
13
14
15
16
17
18
19
20
21
22
23 The arrow points to which structure?
24
25 0 A. Quadratus lumborum muscle
26 0 B. Erector spinae muscle
27
28 0 C. Psoas major muscle
29 0 D. Transversus abdominis muscle
30
31 0 E. Ligamentum flavum
32
33
34
35
36
37
38
39
40
41
42
~imeRemaini~ ~ 0
I Tuto' Feedback Suspend End Block
---------------------------------------------------------------------------------------------------------------------------------------------
1
2
" Item: 11 of 46
Q.ld: 1885 [
111 f> Mark .<:J
Previous
[::>
Next
il
Lab Values
~
Notes
~
Calculator
3
4 .; @ C. Psoas major muscle [64%]
5 0 D. Transversus abdominis muscle [3%]
6
7 0 E. Ligamentum flavum [3%]
8
9 Explanation: User ld:
10
The above arrow points to a bilaterally symmetric structure that lies in close association to the vertebral body
12 and vertebral transverse process. This is the psoas muscle. Inferiorly, the psoas muscle combines w ith the
13 iliacus muscle to form the iliopsoas muscle, w hich functions in hip flexion.
14
15
16
17
18
19
20
21
22
23
24
25
26
27
28
29 (Choice A) The quadratus lumborum is a muscle of the posterior abdominal w all. It is not seen in the image
30 above.
31
(Choice B) The erector spinae is a large muscle group of the back that courses longitudinally along the
32
spinous processes. Bilateral contraction causes spine extension .
33
34
(Choice D) The transversus abdominis (transversalis) muscle is the most internal component of the anterior
35 abdominal w all musculature. It lies immediately deep to the internal oblique. Since this muscle is not
36 attached to the ribs, it is able to provide support to the abdominal contents during respiration w ithout interfering
37 w ith ribcage movement.
38
39 (Choice E) The ligamenta flava are paired elastic ligaments that connect the vertebral laminae. The
40

~
41
42 F!!ck
---------------------------------------------------------------------------------------------------------------------------------------------
suW.nd EnQck
1
2
" Item: 11 of 46
Q.ld: 1885 [
111 f> Mark .<:J
Previous
[::>
Next
il
Lab Values
~
Notes
~
Calculator
3
I I 1'-' UUVV.._, ( ,U I V YI' !-'VII~~~ ~V U UIIU~'-' 1 UIIJ ~ J I I II I 1'-'U II.. ~ U u .... U .U'-' U IU~ "'-'"'II I ""'V "' '-' U "' "'V"-'1UUVI I ~V U 1'-' .,.._,, ~'-'U I t.U UVUJ
4
and vertebral transverse process. This is the psoas muscle. Inferiorly, the psoas muscle combines w ith the
5
iliacus muscle to form the iliopsoas muscle, w hich functions in hip flexion .
6
7
8
9
10

12
13
14
15
16
17
18
19
20
21 (Choice A) The quadratus lumborum is a muscle of the posterior abdominal w all. It is not seen in the image
22 above.
23
24 (Choice B) The erector spinae is a large muscle group of the back that courses longitudinally along the
25 spinous processes. Bilateral contraction causes spine extension .
26
27 (Choice D) The transversus abdominis (transversalis) muscle is the most internal component of the anterior
28 abdominal w all musculature. It lies immediately deep to the internal oblique. Since this muscle is not
29 attached to the ribs, it is able to provide support to the abdominal contents during respiration w ithout interfering
30 w ith ribcage movement.
31
(Choice E) The ligamenta flava are paired elastic ligaments that connect the vertebral laminae. The
32
ligamenta flava form the posterior w all of the spinal canal and help hold the body erect.
33
34 Educational Objective:
35 The psoas muscle originates from the anterior surface of the transverse processes and lateral surface of the
36 corresponding vertebral bodies T12-L5. The psoas muscle acts primarily to flex the thigh at the hip. It
37 contributes somew hat to lateral rotation and abduction of the thigh as w ell.
38
39
Time Spent: 3 seconds Copyright USMLEW orld,LLC. Last updated: [11 / 10/2011]
40

~
41
42
---------------------------------------------------------------------------------------------------------------------------------------------
F!!ck suW.nd EnQck
1
2
" Item: 12 of 46
Q.ld: 1685 [
lil f> Mark .<:J
Previous
[::>
Next
il
Lab Values
~
Notes
~
Calculator
3
4
5
6 A 25-year-old male fractures his right tibia in a motor vehicle accident. His right leg is fixed in a cast, and he
7 requires crutches to ambulate. Tw o w eeks later, he presents w ith difficulty extending his w rist. Injury to
8 w hich of the follow ing nerves is most likely responsible?
9
10
11 0 A. Suprascapular nerve
0 B. Long thoracic nerve
13
14 0 C. Axillary nerve
15
0 D. Accessory nerve
16
17 0 E. Radial nerve
18
19
20
21
22
23
24
25
26
27
28
29
30
31
32
33
34
35
36
37
38
39
40

~
41
42 F!!ck
---------------------------------------------------------------------------------------------------------------------------------------------
suW.nd EnQck
1
2
" Item: 12 of 46
Q.ld: 1685 [
lil f> Mark .<:J
Previous
[::>
Next
il
Lab Values
~
Notes
~
Calculator
3
4
5
I " @ E. Radial nerve [82%]
6
7 Explanation: User ld:
8
9
10
11

13
14
15
16
17
18
19
20 Musculocutaneou
21
22
""
23
24 Long tho1acic nerve
25
26
27 t
28 i Medial cutaneous nerve of the arm
29 Ulnar ~erve I
30 Medial cutaneous nerve of the forearm
31
32 This patient is suffering from a pressure-induced radial nerve injury, most likely caused by improperly fitted
33 crutches. The radial nerve is derived from the C5-T1 spinal nerves, and is one of the two final branches of the
34 posterior cord of the brachial plexus. The radial nerve initially courses medial to the surgical neck of the
35 humerus inferior to the teres major muscle within the axilla before entering the posterior arm to course
36 between the long head of the triceps brachii and the posterior humerus.
37
38 The radial nerve can be injured at its superficial location within the axilla by repetitive pressure and trauma
39 caused by an ill-fitting crutch ("crutch palsy"). The radial nerve and deep brachial artery can also be injured by
40

~
41
42 F!!ck
---------------------------------------------------------------------------------------------------------------------------------------------
suW.nd EnQck
1
2
" Item: 12 of 46
Q.ld: 1685 [
lil f> Mark .<:J
Previous
[::>
Next
il
Lab Values
~
Notes
~
Calculator
3

t</::7~1{
4
5
6 '-Y~ 6f"
~edial cutaneous nerve of the arm
7 Ulnar ~erve I
8 Medial cutaneous nerve of the forearm
9
10 This patient is suffering from a pressure-induced radial nerve injury, most likely caused by improperly fitted
11 crutches. The radial nerve is derived from the C5-T1 spinal nerves, and is one of the two final branches of the
posterior cord of the brachial plexus. The radial nerve initially courses medial to the surgical neck of the
13 humerus inferior to the teres major muscle within the axilla before entering the posterior arm to course
14 between the long head of the triceps brachii and the posterior humerus.
15
16 The radial nerve can be injured at its superficial location within the axilla by repetitive pressure and trauma
17 caused by an ill-fitting crutch ("crutch palsy"). The radial nerve and deep brachial artery can also be injured by
18 midshaft humerus fractures. Injury to the radial nerve at either location causes w eakness or paralysis of the
19 extensor muscles of the forearm and w rist as w ell as to the extrinsic extensors of the digits, a motor deficit
20 described clinically as "wrist drop."
21
22 (Choice A) The suprascapular nerve innervates the supraspinatus and infraspinatus muscles. These
23 muscles function to abduct and laterally rotate the arm, respectively.
24
25 (Choice B) Damage to the long thoracic nerve causes paralysis of the serratus anterior muscle and winging
26 of the scapula . This nerve is often injured during lymph node dissection in patients undergoing radical
27 mastectomy.
28
29 (Choice C) Fracture of the surgical neck of the humerus and anterior dislocation of the glenohumeral joint can
30 cause injury to the axillary nerve. The resultant paralysis of the deltoid and teres minor muscles causes
31 w eakness of abduction, extension, flexion and lateral rotation of the arm at the shoulder.
32
(Choice D) The spinal accessory nerve (CN XI} exits the jugular foramen and courses over the levator
33
scapulae muscle to innervate the sternocleidomastoid and trapezius muscles bilaterally.
34
35 Educational Objective:
36 Improperly fitted crutches can cause radial nerve injury, resulting in w eakness of all forearm, wrist and finger
37 extensors ("wristdrop").
38
39
Time Spent: 1 seconds Copyright USMLEW orld,LLC. Last updated: [7/7/201 0]
40
41
42
~imeRemaini~ ~ 0
I Tuto' Feedback Suspend End Block
---------------------------------------------------------------------------------------------------------------------------------------------
1
2
" Item: 13 of 46
Q.ld: 1636 [
lil f> Mark .<:J
Previous
[::>
Next
il
Lab Values
~
Notes
~
Calculator
3
4
5
6 A young athlete presents to your office after sustaining an injury to the right upper extremity. Physical
7 examination reveals w eak right forearm flexion and an absent biceps reflex. Given these findings, sensation
8 loss in w hich of the following areas is most likely to be found in this patient?
9
10
11 0 A. Posterior arm
12 0 B. Posterior forearm

14 0 C. Lateral forearm
15 0 D. Medial forearm
16
17 0 E. Thenar eminence
18
19
20
21
22
23
24
25
26
27
28
29
30
31
32
33
34
35
36
37
38
39
40
41
42
~imeRemaini~ ~ 0
I Tuto' Feedback Suspend End Block
---------------------------------------------------------------------------------------------------------------------------------------------
1
2
" Item: 13 of 46
Q.ld: 1636 [
lil f> Mark .<:J
Previous
[::>
Next
il
Lab Values
~
Notes
~
Calculator
3
4 examination reveals weak right forearm flexion and an absent biceps reflex. Given these findings, sensation
5 loss in which of the following areas is most likely to be found in this patient?
6
7 0 A. Posterior arm [8%1
8
9 0 B. Posterior forearm [7%1
10 ~ @ C. Lateral forearm [51 %1
11 0 D. Medial forearm [24%1
12
0 E. Thenar eminence [8%1
14
15 Explanation: User ld:
16
17 This patient has symptoms suggestive of injury to the musculocutaneous nerve. The musculocutaneous
18 nerve is derived from the C5-C7 ventral rami . As its name implies, the musculocutaneous nerve provides
19 both motor innervation and somatic sensory innervation to the arm . It innervates the major upper arm flexors
20 including the biceps brachii, coracobrachialis, and brachialis muscles. After innervating these muscles, the
21 remaining fibers continue as the lateral cutaneous nerve of the forearm, providing sensory innervation to the
22 skin of the lateral forearm .
23
24 (Choice A) The posterior arm is innervated by a branch of the radial nerve (posterior cutaneous nerve of the
25 arm).
26
27 (Choice B) The posterior forearm is innervated by a branch of the radial nerve (posterior cutaneous nerve of
28 the forearm).
29
30 (Choice D) The medial forearm is innervated by a branch of the ulnar nerve (medial cutaneous nerve of the
31 forearm).
32
(Choice E) The thenar eminence is innervated by the recurrent branch of the median nerve.
33
34 Educational objective:
35 The musculocutaneous nerve innervates the flexor muscles of the upper arm and provides sensory
36 innervation to the lateral forearm. The musculocutaneous nerve is derived from the upper trunk of the brachial
37 plexus and can be injured by forceful injuries that cause separation of the neck and shoulder.
38
39
Time Spent: 1 seconds Copyright USMLEWorld,LLC. Last updated: [1 / 1/20141
40
41
42
~imeRemaini~ ~ 0
I Tuto' Feedback Suspend End Block
---------------------------------------------------------------------------------------------------------------------------------------------
1
2
" Item: 14 of 46
Q.ld: 1703 [
lil f> Mark .<:J
Previous
[::>
Next
il
Lab Values
~
Notes
~
Calculator
3
4
5
6 A 62-year-old man comes to the physician complaining of double vision. He first noticed mild difficulty
7 focusing his eyes about 3 w eeks ago and his symptoms have progressively w orsened. The patient's other
8 medical problems include metastatic prostate cancer. On neurologic examination, he is unable to adduct his
9 left eye, and stimulation of the left cornea does not elicit a corneal reflex. A lesion involving w hich of the
10 follow ing anatomical structures is most likely responsible for this patient's symptoms?
11
12
13 O A. Inferior orbital fissure
0 B. Optic canal
15
16 o c. Superior orbital fissure
17 O D. Foramen rotundum
18
19 O E. Foramen lacerum
20 0 F. Foramen ovale
21
22
23
24
25
26
27
28
29
30
31
32
33
34
35
36
37
38
39
40
41
42
~imeRemaini~ ~ 0
I Tuto' Feedback Suspend End Block
---------------------------------------------------------------------------------------------------------------------------------------------
1
2
" Item: 14 of 46
Q.ld: 1703 [
lil f> Mark .<:J
Previous
[::>
Next
il
Lab Values
~
Notes
~
Calculator
3
4 ~ @ C. Superior orbital fissure [69%]
5
0 D. Foramen rotundum [9%]
6
7 0 E. Foramen lacerum [2%]
8 0 F. Foramen ovale [4%]
9
10
11 Explanation: User ld:
12
13 This patient's diplopia is caused by his inability to adduct his left eye. Eye adduction depends on the
oculomotor nerve (CN Ill) and the medial rectus muscle. The oculomotor nerve originates in the oculomotor
15 nucleus of the midbrain, emerges from the anterior midbrain, and enters the orbit through the superior orbital
16 fissure. The patient also has loss of the corneal reflex on the left side. The sensory limb of the corneal reflex
17 is mediated by the nasociliary branch of the first division of the trigeminal nerve (CN V,). The motor
18 component of the corneal reflex is carried primarily by the temporal branch of the facial nerve (CN VII}. Like
19 the oculomotor nerve, the nasociliary nerve enters the orbit through the superior orbital fissure. Thus, a lesion
20 involving the superior orbital fissure w ould cause the described deficits.
21
22 The trochlear nerve (CN IV), abducens nerve (CN VI}, and superior ophthalmic vein also enter the orbit via the
23 superior orbital fissure.
24
(Choice A} The maxillary division of the trigeminal nerve (CN V2}, the infraorbital vessels, and branches from
25
26 the sphenopalatine ganglion pass through the inferior orbital fissure but do not enter the orbit.
27
(Choice B) The optic canal is medial to the superior orbital fissure and transmits only the optic nerve (CN II}
28
29 and ophthalmic artery.
30 (Choice D) The foramen rotundum transmits the maxillary division of the trigeminal nerve (CN V2} from w ithin
31
the skull to the pterygopalatine fossa . From there, it courses through the inferior orbital fissure to appear on
32
the face at the infraorbital foramen as the infraorbital nerve.
33
34
(Choice E) The foramen lacerum is occluded by cartilage. The internal carotid artery courses just superior
35 to the foramen lacerum in the lacerum portion of the carotid canal.
36
37 (Choice F) The mandibular branch of the trigeminal nerve (CN V3} passes through the foramen ovale.
38
39 Educational objective:
40 .......... ...... . .. .... ....... ....4-..... .... ....... . .... /l""t.. l Ill \ ............................... : ............... . .... ........... \ I \ ................................ 4-.......................... ................. , ...... ,. . 1\ / \ .................................... ....... . ....

~
41
42
---------------------------------------------------------------------------------------------------------------------------------------------
F!!ck suW.nd EnQck
1
2
" Item: 14 of 46
Q.ld: 1703 [
lil f> Mark .<:J
Previous
[::>
Next
il
Lab Values
~
Notes
~
Calculator
3
4 0 F. Foramen ovale [4%]
5
6
7 Explanation: User ld:
8
9 This patient's diplopia is caused by his inability to adduct his left eye. Eye adduction depends on the
10 oculomotor nerve (CN Ill) and the medial rectus muscle. The oculomotor nerve originates in the oculomotor
11 nucleus of the midbrain, emerges from the anterior midbrain, and enters the orbit through the superior orbital
12 fissure. The patient also has loss of the corneal reflex on the left side. The sensory limb of the corneal reflex
13 is mediated by the nasociliary branch of the first division of the trigeminal nerve (CN V,). The motor
component of the corneal reflex is carried primarily by the temporal branch of the facial nerve (CN VII). Like
15 the oculomotor nerve, the nasociliary nerve enters the orbit through the superior orbital fissure. Thus, a lesion
16 involving the superior orbital fissure w ould cause the described deficits.
17
18 The trochlear nerve (CN IV), abducens nerve (CN VI), and superior ophthalmic vein also enter the orbit via the
19 superior orbital fissure.
20
(Choice A) The maxillary division of the trigeminal nerve (CN V2) , the infraorbital vessels, and branches from
21
22 the sphenopalatine ganglion pass through the inferior orbital fissure but do not enter the orbit.
23
(Choice B) The optic canal is medial to the superior orbital fissure and transmits only the optic nerve (CN II)
24
and ophthalmic artery.
25
26 (Choice D) The foramen rotundum transmits the maxillary division of the trigeminal nerve (CN V2) from w ithin
27
the skull to the pterygopalatine fossa . From there, it courses through the inferior orbital fissure to appear on
28
the face at the infraorbital foramen as the infraorbital nerve.
29
30 (Choice E) The foramen lacerum is occluded by cartilage. The internal carotid artery courses just superior
31 to the foramen lacerum in the lacerum portion of the carotid canal.
32
33 (Choice F) The mandibular branch of the trigeminal nerve (CN V) passes through the foramen ovale.
34
35 Educational objective:
36 The oculomotor nerve (CN Ill), ophthalmic nerve (CN V,) branches, trochlear nerve (CN IV), abducens nerve
37 (CN VI), and superior ophthalmic vein enter the orbit via the superior orbital fissure.
38
39
Time Spent: 1 seconds Copyright USMLEW orld,LLC. Last updated: [11 / 11/2013]
40
41
42
~imeRemaini~ ~ 0
I Tuto' Feedback Suspend End Block
---------------------------------------------------------------------------------------------------------------------------------------------
1
2
" Item: 15 of 46
Q.ld: 1803 [
lil f> Mark .<:J
Previous
[::>
Next
il
Lab Values
~
Notes
~
Calculator
3
4
5
6 A 17-year-old football player comes to the ER hopping on his right foot. An X-ray of the left leg show s a
7 fracture of the neck of the fibula . W hich of the follow ing findings do you most expect on physical examination?
8
9
10 0 A. Loss of plantar flexion of the left foot
11 0 B. W eakened inversion of the left foot
12
13 0 C. Loss of sensation on the sole of the left foot
14 0 D. Loss of sensation on the medial aspect of the left leg

16 0 E. Loss of sensation on the dorsum of the left foot


17
18
19
20
21
22
23
24
25
26
27
28
29
30
31
32
33
34
35
36
37
38
39
40
41
42
~imeRemaini~ ~ 0
I Tuto' Feedback Suspend End Block
---------------------------------------------------------------------------------------------------------------------------------------------
1
2
3
4
.; @ E. Loss of sensation on the dorsum of the left foot [61%]
5
6
7 Explanation: User ld:
8
9 Sciatic nerve
10
11
12
13
14
on peroneal
nerve
16
17
Saohenous nerve
18 (branch of
19 femoral nerve)
20
21
22 peroneal nerve
23
24
25
26
27
28
29
30
31
32 Deep peroneal nerve
33
34 The common peroneal nerve is the most commonly injured nerve in the leg due to its superficial location
35 w here it courses laterally around the neck of the fibula . This location makes the common peroneal nerve
36 susceptible to injury w hen a traumatic insult results in fracture of the neck of the fibula . The sciatic nerve
37 branches into the common peroneal (fibular) nerve and the tibial nerve posteriorly on the thigh just proximal to
38 the popliteal fossa . After coursing around the neck of the fibula, the common peroneal nerve divides into
39 superficial and deep branches. The superficial branch innervates the muscles of the lateral compartment of
40 the lea. w hich function orimarilv to evert the foot. The deeo oeroneal nerve innervates the anterior

~
41
42 F!!ck suW.nd EnQck
---------------------------------------------------------------------------------------------------------------------------------------------
1
2
3
4
5
6
7
8
9
10
11 Deep peroneal nerve
12
13 The common peroneal nerve is the most commonly injured nerve in the leg due to its superficial location
14 w here it courses laterally around the neck of the fibula . This location makes the common peroneal nerve
susceptible to injury w hen a traumatic insult results in fracture of the neck of the fibula. The sciatic nerve
16 branches into the common peroneal (fibular} nerve and the tibial nerve posteriorly on the thigh just proximal to
17 the popliteal fossa . After coursing around the neck of the fibula, the common peroneal nerve divides into
18 superficial and deep branches. The superficial branch innervates the muscles of the lateral compartment of
19 the leg, w hich function primarily to evert the foot. The deep peroneal nerve innervates the anterior
20 compartment of the leg, w hose muscles mainly act as dorsiflexors of the foot and toes, though they also
21 participate in some inversion. The superficial peroneal nerve gives off branches that provide sensory
22 innervation to the majority of the dorsum of the foot w hile the deep peroneal nerve provides sensory
23 innervation only to the region between the first and second digits of the foot. Injury to the common peroneal
24 nerve from a proximal fibula fracture w ould cause loss of sensation in both of these regions as w ell as motor
25 deficits resulting in a clinical presentation w ith "foot drop".
26
27 (Choices A, B and C) l oss of plantar flexion of the left foot could result from injury of the tibial nerve as it
28 courses through the middle of the popliteal fossa . Such an injury w ould also cause w eakness of inversion of
29 the foot as w ell as sensory loss on the sole.
30
(Choice D) l oss of sensation on the medial aspect of the left leg w ould result from a lesion of the saphenous
31
nerve. The saphenous nerve is the largest purely sensory branch of the femoral nerve.
32
33 Educational Objective:
34 The common peroneal nerve is vulnerable to injury w here it courses around the neck of the fibula . Fibular
35 neck fractures can lesion this nerve, causing w eakness of dorsiflexion (deep peroneal nerve} and eversion
36 (superficial peroneal nerve} of the foot as w ell as loss of sensation over the dorsum of the foot. Inversion and
37 plantarflexion w ould remain intact due to the action of the tibial nerve.
38
39
Time Spent: 1 seconds Copyright USMl EW orld,l lC. l ast updated: [1 / 16/2013]
40
41
42
~imeRemaini~ ~ 0
I Tuto' Feedback Suspend End Block
---------------------------------------------------------------------------------------------------------------------------------------------
1
2
" Item: 16 of 46
Q.ld: 1802 [
lil f> Mark .<:J
Previous
[::>
Next
il
Lab Values
~
Notes
~
Calculator
3
4
5
6 A 56-year-old male w ith advanced bladder carcinoma suffers from compression of a nerve that passes
7 through the obturator foramen . W hich of the follow ing functions would be most affected in this patient?
8
9
10 0 A. Abduction of the thigh
11 0 B. Adduction of the thigh
12
13 0 C. Flexion of the thigh
14 0 D. Extension of the thigh
15
0 E. Extension of the leg
17
18
19
20
21
22
23
24
25
26
27
28
29
30
31
32
33
34
35
36
37
38
39
40
41
42
~imeRemaini~ ~ 0
I Tuto' Feedback Suspend End Block
---------------------------------------------------------------------------------------------------------------------------------------------
1
2
" Item: 16 of 46
Q.ld: 1802 [
lil f> Mark .<:J
Previous
[::>
Next
il
Lab Values
~
Notes
~
Calculator
3
4
~ @ B. Adduction of the thigh [64%]
5
6 0 C. Flexion of the thigh [1 0%]
7 0 D. Extension of the thigh [8%]
8
9 0 E. Extension of the leg [3%]
10
11
Explanation: User ld:
12
13 The obturator nerve is the only nerve that exits the pelvis via the obturator foramen. This nerve innervates the
14 adductor compartment of the thigh after dividing into anterior and posterior branches subsequent to leaving
15 the pelvis. The anterior branch innervates the gracilis, pectineus, and the adductors longus and brevis. The
posterior branch innervates the obturator externus and the adductor magnus. Obturator nerve injury w ould
17 cause w eakness and spasm of the adductor compartment muscles.
18
19 (Choice A) Abduction of the thigh is accomplished by the tensor fascia lata and the sartorius muscles. These
20 muscles are supplied by the superior gluteal and femoral nerves, respectively, and lie in the anterior
21 compartment of the thigh.
22
23 (Choice C) Flexion of the thigh is accomplished by the psoas, iliacus, tensor fascia lata, and the sartorius
24 muscles. The psoas is directly innervated by the lumbar plexus, and the iliacus is innervated by the femoral
25 nerve.
26
27 (Choice D) Extension of the thigh is accomplished primarily by the gluteus maximus muscle, w hich is
28 supplied by the inferior gluteal nerve. This nerve exits the pelvis through the greater sciatic foramen below the
29 piriformis.
30
(Choice E) Extension of the leg is primarily accomplished by the quadriceps femoris muscle group. These
31
muscles are innervated by the femoral nerve, w hich gains access to the thigh deep to the center of the
32
inguinal ligament.
33
34 Educational Objective:
35 The obturator nerve is the only major nerve that exits the pelvis through the obturator foramen . This nerve
36 supplies the muscles of the medial (adductor) compartment of the thigh and can be damaged during pelvic
37 surgery, especially in procedures such as lymph node dissection.
38
39
Time Spent: 1 seconds Copyright USMLEW orld,LLC. Last updated: [7/7/201 OJ
40
41
42
~imeRemaini~ ~ 0
I Tuto' Feedback Suspend End Block
---------------------------------------------------------------------------------------------------------------------------------------------
1
2
" Item: 17 of 46
Q.ld: 1686 [
111 f> Mark .<:J
Previous
[::>
Next
il
Lab Values
~
Notes
~
Calculator
3
4
5
6 A 34-year-old male w ith a squamous cell carcinoma undergoes surgical neck dissection. W hile attempting to
7 ligate the inferior thyroid artery, the surgeon accidentally damages a nerve that lies in close proximity to it.
8 W hich of the follow ing nerves w as most likely damaged?
9
10
11 0 A. Superior laryngeal nerve
12 0 B. Recurrent laryngeal nerve
13
14 0 C. Ansa cervicalis
15 0 D. Hypoglossal nerve
16
0 E. Accessory nerve
18
19
20
21
22
23
24
25
26
27
28
29
30
31
32
33
34
35
36
37
38
39
40
41
42
~imeRemaini~ ~ 0
I Tuto' Feedback Suspend End Block
---------------------------------------------------------------------------------------------------------------------------------------------
1
2
" Item: 17 of 46
Q.ld: 1686 [
111 f> Mark .<:J
Previous
[::>
Next
il
Lab Values
~
Notes
~
Calculator
3
4 v @ B. Recurrent laryngeal nerve [80%]
5
6 0 C. Ansa cervicalis [3%]
7 0 D. Hypoglossal nerve [2%]
8 0 E. Accessory nerve [2%]
9
10
11 Explanation: User ld:
12
13 The inferior thyroid artery arises from the thyrocervical trunk, a branch of the subclavian artery. Along with the
14 superior thyroid arteries (branches of the external carotid arteries), the inferior thyroid arteries provide blood to
15 the thyroid gland and other structures of the anterior neck.
16
From the thyrocervical trunk, the inferior thyroid artery courses deep to the vagus nerve, common carotid
18 artery and internal jugular vein before turning tow ard the thyroid. Just before meeting the thyroid gland, this
19 artery courses just superficial to the recurrent laryngeal nerve. The recurrent laryngeal nerve is a branch of
20 the vagus nerve that loops below the aortic arch on the left and below the subclavian artery on the right to
21 provide motor innervation to the muscles of the larynx (all except the cricothyroid muscle). Unilateral injury to
22 this nerve causes hoarseness; bilateral injury causes respiratory difficulty due to airway obstruction by
23 immobile vocal folds.
24
(Choice A) The superior laryngeal nerve is a branch of the vagus nerve that gives off internal and external
25
branches. The superior thyroid artery courses w ith the external branch of this nerve; ligation of this artery
26
during surgery may damage the external laryngeal nerve.
27
28 (Choice C) The ansa cervicalis is a loop of the cervical plexus that receives contributions from C1 , C2 and
29 C3. It courses deep to the sternocleidomastoid muscle and loops around the internal jugular vein . Branches
30 from the ansa cervicalis innervate the sternohyoid, sternothyroid and omohyoid muscles.
31
32 (Choice D) The hypoglossal nerve (CN XII) exits the cranium through the hypoglossal canal and innervates all
33 of the intrinsic muscles of the tongue except the palatoglossus.
34
35 (Choice E) The spinal accessory nerve (CN XI) exits the jugular foramen and courses on top of the levator
36 scapulae muscle to innervate the sternocleidomastoid and trapezius muscles on either side.
37
38 Educational Objective:
39 The recurrent laryngeal nerve travels in close approximation to the inferior thyroid artery and can be injured in
40 surqical procedures of the anterior neck (e.q. thyroidectomy), resultinq in larvnqeal muscle paralysis,
41
42
~imeRemaini~ ~ 0
I Tuto' Feedback
---------------------------------------------------------------------------------------------------------------------------------------------
Suspend End Block
1
2
" Item: 17 of 46
Q.ld: 1686 [
111 f> Mark .<:J
Previous
[::>
Next
il
Lab Values
~
Notes
~
Calculator
3
..._., ..... . . ' 1 1"'"'~'"''-''-' .... ' .............. l'- ' "'J
4
5 0 E. Accessory nerve [2%]
6
7
Explanation: User ld:
8
9 The inferior thyroid artery arises from the thyrocervical trunk, a branch of the subclavian artery. Along w ith the
10 superior thyroid arteries (branches of the external carotid arteries}, the inferior thyroid arteries provide blood to
11 the thyroid gland and other structures of the anterior neck.
12
13 From the thyrocervical trunk, the inferior thyroid artery courses deep to the vagus nerve, common carotid
14 artery and internal jugular vein before turning tow ard the thyroid. Just before meeting the thyroid gland, this
15 artery courses just superficial to the recurrent laryngeal nerve. The recurrent laryngeal nerve is a branch of
16 the vagus nerve that loops below the aortic arch on the left and below the subclavian artery on the right to
provide motor innervation to the muscles of the larynx (all except the cricothyroid muscle}. Unilateral injury to
18 this nerve causes hoarseness; bilateral injury causes respiratory difficulty due to airway obstruction by
19 immobile vocal folds.
20
21 (Choice A) The superior laryngeal nerve is a branch of the vagus nerve that gives off internal and external
22 branches. The superior thyroid artery courses w ith the external branch of this nerve; ligation of this artery
23 during surgery may damage the external laryngeal nerve.
24
25 (Choice C) The ansa cervicalis is a loop of the cervical plexus that receives contributions from C1 , C2 and
26 C3. It courses deep to the sternocleidomastoid muscle and loops around the internal jugular vein . Branches
27 from the ansa cervicalis innervate the sternohyoid, sternothyroid and omohyoid muscles.
28
29 (Choice D) The hypoglossal nerve (CN XII} exits the cranium through the hypoglossal canal and innervates all
30 of the intrinsic muscles of the tongue except the palatoglossus.
31
(Choice E) The spinal accessory nerve (CN XI} exits the jugular foramen and courses on top of the levator
32
scapulae muscle to innervate the sternocleidomastoid and trapezius muscles on either side.
33
34 Educational Objective:
35 The recurrent laryngeal nerve travels in close approximation to the inferior thyroid artery and can be injured in
36 surgical procedures of the anterior neck (e.g. thyroidectomy}, resulting in laryngeal muscle paralysis,
37 hoarseness and dyspnea .
38
39
Time Spent: 1 seconds Copyright USMLEW orld,LLC. Last updated: [7/7/201 OJ
40
41
42
~imeRemaini~ ~ 0
I Tuto' Feedback Suspend End Block
---------------------------------------------------------------------------------------------------------------------------------------------
1
2
" Item: 18 of 46
Q.ld: 1826 [
lil f> Mark .<:J
Previous
[::>
Next
il
Lab Values
~
Notes
~
Calculator
3
4
5
6 A patient drinking hot tea at a restaurant experiences a burning sensation at the tip of his tongue. This
7 sensation is transmitted by w hich of the following?
8
9
10 0 A. Trigerninal nerve
11 0 B. Chorda tympani
12
13 0 C. Glossopharyngeal nerve
14 0 D. Hypoglossal nerve
15
16 0 E. Vagus nerve
17

19
20
21
22
23
24
25
26
27
28
29
30
31
32
33
34
35
36
37
38
39
40
41
42
~imeRemaini~ ~ 0
I Tuto' Feedback Suspend End Block
---------------------------------------------------------------------------------------------------------------------------------------------
1
2
" Item: 18 of 46
Q.ld: 1826 [
lil f> Mark .<:J
Previous
[::>
Next
il
Lab Values
~
Notes
~
Calculator
3
4
"' @ A. Trigeminal nerve [67%1
5
6 0 B. Chorda tympani [ 17%1
7 0 C. Glossopharyngeal nerve [9%1
8
9
0 D. Hypoglossal nerve (5%1
10 0 E. Vagus nerve [2%1
11
12
13 Explanation: User ld:
14
The innervation of the tongue is complex because it involves motor, general somatic sensory and special
15 sensory functions. Somatic sensation includes pain, touch, temperature and pressure, the same sensations
16 felt by the skin . The lingual nerve, a branch of the mandibular division of the trigeminal nerve (CN V a),
17
provides somatic sensory innervation to the anterior 2/3 of the tongue. CN V 3 also provides somatic sensory
19 innervation to the skin of the face. The glossopharyngeal (CN IX) nerve provides somatic sensory innervation
20 to the posterior 1/3 of the tongue.
21
22 (Choice B) The chorda tympani is a branch of the facial nerve that passes along the deep surface of the
23 tympanic membrane and through the petrotympanic fissure before merging w ith the lingual nerve to deliver
24 taste sensation to the anterior 2/3 of the tongue.
25
(Choice C) The glossopharyngeal nerve (CN IX) delivers somatic sensory and taste innervation to the
26
posterior 1/3 of the tongue.
27
28 (Choice D) The hypoglossal nerve (CN XII) innervates all intrinsic muscles of the tongue except the
29 palatoglossus muscle, w hich is innervated by the vagus nerve (CN X).
30
31 (Choice E) The vagus nerve (CN X) innervates the palatoglossus muscle and carries taste signals from the
32 pharynx and epiglottis. It does not carry taste signals from the tongue, how ever.
33
34 Educational Objective:
35 A branch of the mandibular division of the trigeminal nerve (CN V 3 ) provides somatic sensory innervation to
36 the anterior 2/3 of the tongue. The chorda tympani branch of the facial nerve provides taste sensation from
37 the anterior 2/3 of the tongue. The glossopharyngeal nerve (CN IX} provides somatic sensory and taste
38 innervation to the posterior portion of the tongue.
39
40
41
42
~imeRemaini~ ~ 0
I Tuto' Feedback
---------------------------------------------------------------------------------------------------------------------------------------------
Suspend End Block
1
2
" Item: 19 of 46
Q.ld: 1969 [
lil f> Mark .<:J
Previous
[::>
Next
il
Lab Values
~
Notes
~
Calculator
3
4
5
6 A 23-year-old athlete presents to your office w ith right knee pain after playing basketball. A coronal MRI image
7 of his knee is show n below .
8
9
10
11
12
13
14
15
16
17
18

20
21
22
23
24
25
26
27 The asterisk marks the attachment site of w hich of the follow ing ligaments?
28
29 0 A. Anterior cruciate
30
31 0 B. Tibial collateral
32 0 C. Transverse genicular
33 0 D. Posterior cruciate
34
35
0 E. Patellar
36
37
38
39
40

~
41
42 F!!ck
---------------------------------------------------------------------------------------------------------------------------------------------
suW.nd EnQck
1
2
" Item: 19 of 46
Q.ld: 1969 [
lil f> Mark .<:J
Previous
[::>
Next
il
Lab Values
~
Notes
~
Calculator
3
4 v @ D. Posterior cruciate [45%]
5
6 0 E. Patellar [2%]
7
8 Explanation: User ld:
9
10 In this coronal MRI image of the right knee, the opaque (white) regions, from top to bottom, correspond to: the
11 lateral and medial condyles of the femur (the medial condyle is adjacent to the asterisk), the lateral and medial
12 condyles of the tibia, and the head of the fibula. The anterior and posterior cruciate ligaments are ligaments
13 w ithin the articular capsule of the knee joint that cross one another as each spans from the tibia to the femur.
14 The posterior cruciate ligament attaches to the posterior part of the intercondylar area of the tibia and the
15 anterior lateral surface of the medial epicondyle of the femur (asterisk).
16
Anterior view Posterior view
17
18

20
21 Posterior cruciate ligament
22
Lateral collateral ligament
23 (fibular collateral ligament)
24
25
Medial collateral ligament_ Popliteus tendon
26
(tibial collateral ligament)
27
28
29
30
Anterior cruciate ligament
31
32
33
34
35 USMUWOrld.lLC () 201 I
36
37
(Choice A) The anterior cruciate ligament is more commonly injured than the posterior cruciate ligament. It
38
spans from the anterior portion of the intercondylar tibia to the posterior medial side of the lateral femoral
39
40 condyle. It prevents posterior displacement of the femur relative to the tibia w hen the knee is fully extended.

~
41
42 F!!ck suW.nd EnQ ck
---------------------------------------------------------------------------------------------------------------------------------------------
1
2
3
4
5
6 Posterior cruciate ligament
7 Lateral collateral ligament
8 (fibular collateral ligament)
9
10 Medial collateral ligament_ Popliteus tendon
11 (tibial collateral ligament)
12
13
14
15 Anterior cruciate ligament
16
17
18

USMUWOrld.lLC () 201 I
20
21
22 (Choice A) The anterior cruciate ligament is more commonly injured than the posterior cruciate ligament. It
23 spans from the anterior portion of the intercondylar tibia to the posterior media! side of the latera! femora!
24 condyle. It prevents posterior displacement of the femur relative to the tibia w hen the knee is fully extended.
25
26 (Choice B) The tibial collateral ligament is a very strong band of tissue that spans from the medial femoral
27 epicondyle to the medial condyle of the tibia . It also attaches to the medial meniscus.
28
29 (Choice C) The transverse genicular ligament attaches the medial and lateral menisci anteriorly on the
30 proximal articular surface of the tibia .
31
(Choice E) The patellar ligament spans from the patella to the tibial tuberosity. It is the continuation of the
32
quadriceps femoris tendon.
33
34 Educational Objective:
35 The posterior cruciate ligament prevents anterior displacement of the femur relative to the tibia w hen the knee
36 is flexed. It attaches to the posterior part of the intercondylar area of the tibia and the anterior part of the lateral
37 surface of the medial epicondyle of the femur.
38
39
Time Spent: 2 seconds Copyright USMLEW orld,LLC. Last updated: [1 /8/2014]
40

~
41
42 F!!ck suW.nd
---------------------------------------------------------------------------------------------------------------------------------------------
EnQck
1
2
" Item: 20 of 46
Q.ld: 1943 [
111 f> Mark .<:J
Previous
[::>
Next
il
Lab Values
~
Notes
~
Calculator
3
4
5
6 A 52-year-old male has had a chronic cough w ith occasional low -volume hemoptysis for the past three
7 w eeks. He is a two pack-per-day cigarette smoker, and drinks three to four cans of beer on w eekends.
8 Physical examination reveals right-sided face and right arm sw elling and engorgement of subcutaneous veins
9 on the right side of the neck. W hich of the follow ing veins is most likely obstructed in this patient?
10
11
12 0 A. Internal jugular
13 0 B. External jugular
14
15 0 C. Subclavian
16 0 D. Brachiocephalic
17
18 0 E. Superior vena cava
19 0 F. Axillary

21
22
23
24
25
26
27
28
29
30
31
32
33
34
35
36
37
38
39
40

~
41
42 F!!ck suW.nd
---------------------------------------------------------------------------------------------------------------------------------------------
EnQck
1
2
" Item: 20 of 46
Q.ld: 1943 [
111 f> Mark .<:J
Previous
[::>
Next
il
Lab Values
~
Notes
~
Calculator
3
4 0 B. External jugular [7%]
5
6 0 C. Subclavian [19%]
7 v @ D. Brachiocephalic [32%]
8 0 E. Superior vena cava [34%]
9
10 0 F. Axillary [2%]
11
12 Explanation: User ld:
13
14 This patient has symptoms consistent w ith an obstructed right brachiocephalic (innominate) vein . This rnay
15 be the result of external compression, as by an apical lung tumor, or thrombotic occlusion, as can occur w hen
16 a central catheter has been in place for an extended period of time. The right brachiocephalic vein is formed
17 by the union of the right subclavian vein and the right internal jugular vein. The right external jugular vein drains
18 into the right subclavian, so obstruction of the right brachiocephalic w ill cause venous congestion of structures
19 drained by the external jugular. It is important to note that the right brachiocephalic vein also drains the right
lymphatic duct, w hich drains lymph from the right upper extremity, the right face and neck, the right
21 hemithorax, and the right upper quadrant of the abdomen .
22
23 (Choice A) The internal jugular vein drains the brain and superficial face and neck. Obstruction of the internal
24 jugular w ould not cause arm swelling .
25
26 (Choice B) The external jugular vein drains the scalp and portions of the lateral face.
27
28 (Choices C and F) The subclavian vein is the continuation of the axillary vein . Both drain blood from the
29 upper extremity.
30
(Choice E) Superior vena cava syndrome results w hen the superior vena cava is obstructed . The signs and
31
symptoms of SVC syndrome are similar to those described here, except that both sides of the face, neck and
32
chest and both arms w ould be involved.
33
34 Educational Objective:
35 The brachiocephalic vein drains the ipsilateral jugular and subclavian veins. The bilateral brachiocephalic
36 veins combine to form the SVC. Brachiocephalic vein obstruction causes symptoms similar to those seen in
37 SVC syndrome, but on just one side of the body.
38
39
Time Spent: 1 seconds Copyright USMLEW orld,LLC. Last updated: [7/7/201 OJ
40
41
42
~imeRemaini~ ~ 0
I Tuto' Feedback Suspend End Block
---------------------------------------------------------------------------------------------------------------------------------------------
1
2
" Item: 21 of 46
Q.ld : 8589 [
111 f> Mark .<:J
Previous
[::>
Next
il
Lab Values
~
Notes
~
Calculator
3
4
5
6 A 54-year-old w oman comes to the physician complaining of difficulty hearing for the past few w eeks. During
7 the neurologic examination, the physician assesses her hearing using a vibrating tuning fork. She appreciates
8 a loud tone w hen the handle of the tuning fork is placed on her left mastoid process but hears a softer tone
9 w hen the tines are placed near her left external auditory meatus. W hen the handle is placed on the middle of
10 her forehead, she hears the vibration better in her left ear than her right. This patient is most likely
11 experiencing w hich of the follow ing types of hearing loss?
12
13
14 0 A. Conductional loss in both ears
15 0 B. Conductional loss in left ear
16
17 0 C. Conductional loss in right ear
18 0 D. Sensorineural loss in both ears
19
20 0 E. Sensorineural loss in left ear
0 F. Sensorineural loss in right ear
22
23
24
25
26
27
28
29
30
31
32
33
34
35
36
37
38
39
40
41
42
~imeRemaini~ ~ 0
I Tuto' Feedback Suspend End Block
---------------------------------------------------------------------------------------------------------------------------------------------
1
2
" Item: 21 of 46
Q.ld: 8589 [
111 f> Mark .<:J
Previous
[::>
Next
il
Lab Values
~
Notes
~
Calculator
3
4 ~ @ B. Conductionalloss in left ear [63% 1
5
6 0 C. Conductional loss in right ear [8%1
7 0 D. Sensorineural loss in both ears [2%1
8 0 E. Sensorineural loss in left ear [ 18%1
9
10 0 F. Sensorineural loss in right ear [7%1
11
12 Explanation: User ld:
13
14
15 Type of
16 Rinne test Weber test Possible causes
17 hearing loss
18
19 Cerumen impaction, cholesteatoma,
20 Abnormal in
Localizes to otosclerosis, external or middle ear
Conductive affected ear
22 affected ear tumors, tympanic membrane
(bone > air)
23 rupture, severe otitis media
24
25
26 Normal in Loca Iizes to Meniere's disease, acoustic neuroma,
27 Sensori neura I bot h ears unaffected presbycusis, ototoxic drugs
28 (air > bone) (eg, aminoglycosides)
ear
29
30 >US-MlfW<II1d. l LC
31
32 Hearing loss is classified as either conductive (obstruction of external sound vibrations to the inner ear) or
33 sensorineural (involving the inner ear, cochlea, or auditory nerve). The Rinne and W eber tests can be used to
34 help determine the type of hearing loss.
35
36 The Rinne test compares the perceived sound produced by a vibrating tuning fork w hen the stem is placed on
37 the mastoid bone (bone conduction) versus w hen the tines are held near the external auditory meatus (air
38 conduction). Air -conducted sound is normally louder and heard longer than bone-conducted sound. The
39 Rinne test is considered positive (normal) if the sound is heard best at the external auditory meatus and
40 nPn:::~tivP f :::~hnnrm:::. l) if t hP n:::.tipnt hp:::.rc:::: t hP v ihr:::.tinn hPttPr :::.t thp m:::.c;:;t n irl An :::.hnnrm:::~ l R innp tpc:::t C::::l lnnpc:::tc::::

~
41
42
---------------------------------------------------------------------------------------------------------------------------------------------
F!!ck suW.nd EnQck
1
2
" Item: 21 of 46
Q.ld: 8589 [
111 f> Mark .<:J
Previous
[::>
Next
il
Lab Values
~
Notes
~
Calculator
3
4
Hearing loss is classified as either conductive (obstruction of external sound vibrations to the inner ear) or
5 sensorineural (involving the inner ear, cochlea, or auditory nerve). The Rinne and W eber tests can be used to
6 help determine the type of hearing loss.
7
8 The Rinne test compares the perceived sound produced by a vibrating tuning fork w hen the stern is placed on
9 the mastoid bone (bone conduction) versus w hen the tines are held near the external auditory meatus (air
10 conduction). Air-conducted sound is normally louder and heard longer than bone-conducted sound. The
11 Rinne test is considered positive (normal) if the sound is heard best at the external auditory meatus and
12 negative (abnormal) if the patient hears the vibration better at the mastoid. An abnormal Rinne test suggests
13 conductive hearing loss.
14
15 The Weber test also helps differentiate between conductive and sensorineural hearing loss. It is performed by
16 placing a vibrating tuning fork on the middle of forehead equidistant from both ears. The vibration is normally
17 heard equally in both ears; vibration heard louder in 1 ear indicates an abnormal test. Conductive hearing loss
18 causes lateralization to the affected ear as the conduction deficit masks the ambient noise in the room,
19 allow ing the vibration to be better heard. In contrast, sensorineural hearing loss causes lateralization to the
20 unaffected ear as the unimpaired inner ear can better sense the vibration.

22 The Rinne test is abnormal in this patient's left ear, and the W eber test lateralizes to her left ear. These
23 findings suggest conductive hearing loss in the left ear.
24
25 (Choice A) Conductional loss in both ears would result in an abnormal Rinne test bilaterally and a normal
26 W eber test.
27
(Choice C) Conductional loss in the right ear would result in an abnormal Rinne test on the right and a
28
W eber test that lateralizes to the right.
29
30 (Choice D) Sensorineural loss in both ears would result in a normal Rinne test bilaterally and a normal
31
W eber test.
32
33 (Choice E) Sensorineural loss in the left ear would result in a normal Rinne test bilaterally and a W eber test
34 that lateralizes to the right.
35
36 (Choice F) Sensorineural loss in the right ear would result in a normal Rinne test bilaterally and a W eber test
37 that lateralizes to the left.
38
39 Educational objective:
40 Conductive hearina loss (bone conduction > air conduction\: abnormal Rinne test (in affected ear\ and W eber

~
41
42 F!!ck
---------------------------------------------------------------------------------------------------------------------------------------------
suW.nd EnQck
1
2
" Item: 21 of 46
Q.ld : 8589 [
111 f> Mark .<:J
Previous
[::>
Next
il
Lab Values
~
Notes
~
Calculator
3
4 conductton). Air-conducted sound IS normally louder and heard longer than bone-conducted sound . 1he
5 Rinne test is considered positive (normal) if the sound is heard best at the external auditory meatus and
6 negative (abnormal) if the patient hears the vibration better at the mastoid. An abnormal Rinne test suggests
7 conductive hearing loss.
8
9 The W eber test also helps differentiate between conductive and sensorineural hearing loss. It is performed by
10 placing a vibrating tuning fork on the middle of forehead equidistant from both ears. The vibration is normally
11 heard equally in both ears; vibration heard louder in 1 ear indicates an abnormal test. Conductive hearing loss
12 causes lateralization to the affected ear as the conduction deficit masks the ambient noise in the room,
13 allow ing the vibration to be better heard. In contrast, sensorineural hearing loss causes lateralization to the
14 unaffected ear as the unimpaired inner ear can better sense the vibration.
15
The Rinne test is abnormal in this patient's left ear, and the W eber test lateralizes to her left ear. These
16
findings suggest conductive hearing loss in the left ear.
17
18 (Choice A) Conductional loss in both ears w ould result in an abnormal Rinne test bilaterally and a normal
19
W eber test.
20
(Choice C) Conductional loss in the right ear w ould result in an abnormal Rinne test on the right and a
22 W eber test that lateralizes to the right.
23
24 (Choice D) Sensorineural loss in both ears w ould result in a normal Rinne test bilaterally and a normal
25 W eber test.
26
27 (Choice E) Sensorineural loss in the left ear w ould result in a normal Rinne test bilaterally and a W eber test
28 that lateralizes to the right.
29
30 (Choice F) Sensorineural loss in the right ear w ould result in a normal Rinne test bilaterally and a W eber test
31 that lateralizes to the left.
32
Educational objective:
33
Conductive hearing loss (bone conduction> air conduction): abnormal Rinne test (in affected ear) and W eber
34
test localizes to affected ear.
35
36 Sensorineural hearing loss (air conduction> bone conduction): normal Rinne test and W eber test localizes to
37 unaffected ear.
38
39
Time Spent: 1 seconds Copyright USMLEW orld,LLC. Last updated: [1 2124/2013]
40
41
42
~imeRemaini~ ~ 0
I Tuto' Feedback Suspend End Block
---------------------------------------------------------------------------------------------------------------------------------------------
1
2
" Item: 22 of 46
Q.ld: 1748 [
111 f> Mark .<:J
Previous
[::>
Next
il
Lab Values
~
Notes
~
Calculator
3
4
5
6 A 7-year-old male has a right leg cast after a bicycle accident. After w earing the cast for some time, he
7 complains of pain and numbness of the dorsum of his right foot and cannot dorsiflex his right ankle. W hich of
8 the follow ing is the most likely site of nerve compression in this patient?
9
10
11 0 A. Popliteal fossa
12 0 B. Fibular head
13
14 0 C. Lateral compartment of the leg
15 0 D. Anterior compartment of the leg
16
17 0 E. Medial malleolus
18
19
20
21

23
24
25
26
27
28
29
30
31
32
33
34
35
36
37
38
39
40
41
42
~imeRemaini~ ~ 0
I Tuto' Feedback Suspend End Block
---------------------------------------------------------------------------------------------------------------------------------------------
1
2
" Item: 22 of 46
Q.ld: 1748 [
111 f> Mark .<:J
Previous
[::>
Next
il
Lab Values
~
Notes
~
Calculator
3
4 .; @ B. Fibular head [55%]
5
0 C. Lateral compartment of the leg [18%]
6
7 0 D. Anterior compartment of the leg [15%]
8 0 E. Medial malleolus [4%]
9
10
11 Explanation: User ld:
12
13 The common peroneal (fibular) nerve is the most commonly injured nerve in the leg due to its superficial
14 location w here it courses laterally around the neck of the fibula . This location makes this nerve susceptible to
15 trauma resulting from lateral blow s to the knee as w ell as from tightly applied plaster casts and other devices.
16 The sciatic nerve branches into the common peroneal nerve and the tibial nerve posteriorly in the thigh just
17 proximal to the popliteal fossa . After coursing around the neck of the fibula, the common peroneal nerve
18 divides into superficial and deep branches . The superficial branch innervates the muscles of the lateral
19 compartment of the leg, w hich function primarily to evert the foot. The deep peroneal nerve innervates the
20 anterior compartment of the leg, w hose muscles act as dorsiflexors of the foot and toes. The superficial
21 peroneal nerve gives off branches that provide sensory innervation to the majority of the foot w hile the deep
peroneal nerve provides sensory innervation only to the region between the first and second digits of the foot.
23 Injury to the common peroneal nerve w ould cause a clinical presentation of foot drop due to w eakness of the
24 dorsiflexors. The foot w ould also be held in inversion due to w eakness of the muscles of the lateral
25 compartment.
26
(Choice A) The popliteal artery and vein, as w ell as the tibial nerve, course through the middle of the popliteal
27
fossa . The tibial nerve provides plantar flexion and inversion; therefore, injury to the tibial nerve w ould result in
28
unopposed dorsiflexion and eversion of the foot.
29
30 (Choices C and D) The lateral and anterior compartments of the leg are innervated by the superficial and
31 deep peroneal nerves, respectively. These nerves lie deep w ithin their respective compartments and are
32 unlikely to be injured by superficial pressure.
33
34 (Choice E) Several important structures course near the medial malleolus. The saphenous nerve and great
35 saphenous vein lie anteriorly w hile the posterior tibial artery, the tibial nerve, and the tendons of the flexor
36 digitorum longus, flexor hallucis longus, and tibialis posterior course posteriorly.
37
38 Educational Objective:
39 Trauma or sustained pressure to the neck of the fibula can cause injury to the common peroneal nerve as it
40 r.o11r!':P.!': !':IIOP.rfir.irJIIv rmd lr~tP.rr~llv to thi!': !':In J<:tJ JrP.

~
41
42 F!!ck
---------------------------------------------------------------------------------------------------------------------------------------------
suW.nd EnQck
1
2
" Item: 22 of 46
Q.ld: 1748 [
111 f> Mark .<:J
Previous
[::>
Next
il
Lab Values
~
Notes
~
Calculator
3
4 0 D. Anterior compartment of the leg [15%]
5
0 E. Medial malleolus [4%]
6
7
8 Explanation: User ld:
9
10 The common peroneal (fibular} nerve is the most commonly injured nerve in the leg due to its superficial
11 location w here it courses laterally around the neck of the fibula . This location makes this nerve susceptible to
12 trauma resulting from lateral blow s to the knee as w ell as from tightly applied plaster casts and other devices.
13 The sciatic nerve branches into the common peroneal nerve and the tibial nerve posteriorly in the thigh just
14 proximal to the popliteal fossa . After coursing around the neck of the fibula, the common peroneal nerve
15 divides into superficial and deep branches. The superficial branch innervates the muscles of the lateral
16 compartment of the leg, w hich function primarily to evert the foot. The deep peroneal nerve innervates the
17 anterior compartment of the leg, w hose muscles act as dorsiflexors of the foot and toes. The superficial
18 peroneal nerve gives off branches that provide sensory innervation to the majority of the foot w hile the deep
19 peroneal nerve provides sensory innervation only to the region between the first and second digits of the foot.
20 Injury to the common peroneal nerve w ould cause a clinical presentation of foot drop due to w eakness of the
21 dorsiflexors. The foot w ould also be held in inversion due to w eakness of the muscles of the lateral
compartment.
23
24 (Choice A) The popliteal artery and vein, as w ell as the tibial nerve, course through the middle of the popliteal
25 fossa . The tibial nerve provides plantar flexion and inversion; therefore, injury to the tibial nerve w ould result in
26 unopposed dorsiflexion and eversion of the foot.
27
28 (Choices C and D) The lateral and anterior compartments of the leg are innervated by the superficial and
29 deep peroneal nerves, respectively. These nerves lie deep w ithin their respective compartments and are
30 unlikely to be injured by superficial pressure.
31
(Choice E) Several important structures course near the medial malleolus. The saphenous nerve and great
32
saphenous vein lie anteriorly w hile the posterior tibial artery, the tibial nerve, and the tendons of the flexor
33
digitorum longus, flexor hallucis longus, and tibialis posterior course posteriorly.
34
35 Educational Objective:
36 Trauma or sustained pressure to the neck of the fibula can cause injury to the common peroneal nerve as it
37 courses superficially and laterally to this structure.
38
39
Time Spent: 1 seconds Copyright USMLEW orld,LLC. Last updated: [7/7/201 OJ
40
41
42
~imeRemaini~ ~ 0
I Tuto' Feedback Suspend End Block
---------------------------------------------------------------------------------------------------------------------------------------------
1
2
" Item: 23 of 46
Q.ld: 1811 [
111 f> Mark .<:J
Previous
[::>
Next
il
Lab Values
~
Notes
~
Calculator
3
4
5
6 A 24-year-old male complains of 'clumsiness' of his right arm . The patient is asked to face a w all and push
7 against it with both hands. Abnormal prominence of the right inferior scapular angle is observed during the
8 maneuver. Which of the following muscles is most likely denervated in this patient?
9
10
11 0 A. Serratus anterior
12 0 B. Latissimus dorsi
13
14 0 C. Deltoid
15 0 D. Rhomboid major
16
17 0 E. Teres major
18
19
20
21
22

24
25
26
27
28
29
30
31
32
33
34
35
36
37
38
39
40
41
42
~imeRemaini~ ~ 0
I Tuto' Feedback Suspend End Block
---------------------------------------------------------------------------------------------------------------------------------------------
1
2
" Item: 23 of 46
Q.ld: 1811 [
111 f> Mark .<:J
Previous
[::>
Next
il
Lab Values
~
Notes
~
Calculator
3
4
5 A 24-year-old male complains of 'clumsiness' of his right arm . The patient is asked to face a w all and push
6 against it w ith both hands. Abnormal prominence of the right inferior scapular angle is observed during the
7 maneuver. W hich of the follow ing muscles is most likely denervated in this patient?
8
9 .; @ A. Serratus anterior [79%]
10
11 0 B. Latissimus dorsi [9%]
12 0 C. Deltoid [4%]
13 0 D. Rhomboid major [3%]
14
15 0 E. Teres major [4%]
16
17 Explanation: User ld:
18
19 Injury to the long thoracic nerve results in paralysis of the serratus anterior. This presents as a w inged
20 scapula, w hich means that the medial border and inferior angle sticks out posteriorly like a bird's w ing w hen
21 the patient presses anteriorly against a w all.
22
(Choice B) The latissimus dorsi muscle is innervated by the thoracodorsal nerve and serves to adduct and
24 extend the humerus.
25
26 (Choice C) The deltoid is innervated by the axillary nerve and has three segments that act to flex, extend, and
27 abduct the arm.
28
29 (Choice D) The rhomboid major is innervated by the dorsal scapular nerve and acts to draw the scapula
30 medially at its medial border.
31
(Choice E) The teres major is innervated by the subscapular nerve and acts to adduct and medially rotate the
32
arm . This muscle is not a part of the rotator cuff.
33
34 Educational Objective:
35 The serratus anterior muscle serves to fix the scapula against the posterior chest w all and rotate the scapula
36 to allow abduction of the arm over the head. Paralysis of this muscle occurs w ith injury to the long thoracic
37 nerve and results in w inging of the scapula .
38
39
Time Spent: 2 seconds Copyright USMLEW orld,LLC. Last updated: [7/7/201 OJ
40
41
42
~imeRemaini~ ~ 0
I Tuto' Feedback Suspend End Block
---------------------------------------------------------------------------------------------------------------------------------------------
1
2
" Item: 24 of 46
Q.ld : 8631 [
111 f> Mark .<:J
Previous
[::>
Next
il
Lab Values
~
Notes
~
Calculator
3
4
5
6 A 36-year-old man is rushed to the emergency department after the sudden onset of shortness of breath and
7 difficulty sw allow ing. He has visited the emergency department several times before for food and skin
8 allergies. He is unconscious on arrival. Examination show s excessive accessory respiratory muscle use and
9 edematous sw elling of his face, lips, and tongue. There is also scattered urticaria over his upper body.
10 Attempts at intubation are unsuccessful due to massive soft tissue edema involving his pharynx. A decision is
11 made to perform an emergency cricothyrotomy. The incision made during this procedure w ill most likely pass
12 through w hich of the follow ing structures?
13
14
15 0 A. Buccopharyngeal fascia and platysma
16 0 B. Platysma and thyroid isthmus
17
18 0 C. Pretracheal fascia and cricoid cartilage
19 0 D. Pretracheal fascia and prevertebral fascia
20
21 0 E. Superficial cervical fascia and cricothyroid membrane
22
23

25
26
27
28
29
30
31
32
33
34
35
36
37
38
39
40
41
42
~imeRemaini~ ~ 0
I Tuto' Feedback Suspend End Block
---------------------------------------------------------------------------------------------------------------------------------------------
1
2
3
4
5
6
7 Explanation: User ld:
8
9 Palpation of cricothyroid membrane
10
11
12
13 Laryngeal prominence
14 (Adam's apple)
15
16
17
18
19
20
21
22
23

25
26
27
28
29
30
31
32
Thyroid
33
34 cartilage
35 Trachea cartilage
36 Thyroid
37 gland
38 membrane
39
40
41
42
~imeRemaini~ ~ 0
I Tuto' Feedback Suspend End Block
---------------------------------------------------------------------------------------------------------------------------------------------
1
2
" Item: 24 of 46
Q.ld : 8631 [
111 f> Mark .<:J
Previous
[::>
Next
il
Lab Values
~
Notes
~
Calculator
3
4
5 Cricothyrotomy
6
7
8
9
10
11
12
13
14
15
16
17 Thyroid cartilage
18
19
20 Cricothyroid
21 membrane
22
23

25
26
27
28
29
30
31
32
33
34
35
36
37
38
@ VSMLEWorld. LLC
39
40
41
42
~imeRemaini~ ~ 0
I Tuto' Feedback Suspend End Block
---------------------------------------------------------------------------------------------------------------------------------------------
1
2
" Item: 24 of 46
Q.ld : 8631 [
111 f> Mark .<:J
Previous
[::>
Next
il
Lab Values
~
Notes
~
Calculator
3
4
5
6 USMLEWorid.lLC
7
8 This patient is experiencing acute, life-threatening laryngeal edema as part of an anaphylactic reaction
9 precipitated by exposure to an allergen. Laryngeal edema can initially present w ith throat tightness, difficulty
10 sw allow ing, dyspnea, and hoarseness. How ever, the condition can quickly progress to compromise the
11 airway and cause asphyxiation .
12
13 Cricothyrotomy is indicated w hen an emergency airway is required and orotracheal or nasotracheal intubation
14 is either unsuccessful or contraindicated (eg, due to massive hemorrhage, vomiting, facial trauma, or airway
15 obstruction). The procedure establishes an airway through the placement of a tube between the cricoid and
16 thyroid cartilages and requires an incision though the follow ing structures:
17
1. Skin
18
2. Superficial cervical fascia (including subcutaneous fat and platysma muscle)
19
3. Investing and pretracheallayers of the deep cervical fascia
20
4. Cricothyroid membrane
21
22
23 (Choice A) The buccopharyngeal fascia extends from the carotid sheath to invest the pharyngeal constrictor
muscles. This fascial layer lies anterior to the prevertebral fascia, forming the retropharyngeal space between
25 them . Infections involving the retropharyngeal space can extend directly into the superior mediastinum and
26 cause acute necrotizing mediastinitis.
27
28 (Choices B and C) The cricoid cartilage lies inferior to the thyroid cartilage at the level of C6 vertebra and the
29 thyroid isthmus lies inferior to the cricoid cartilage. The cricothyrotomy incision does not pierce these
30 structures.
31
(Choice D) The deep cervical fascia is composed of three layers: investing, pretracheal, and prevertebral.
32
The prevertebrallayer of the deep fascia is not penetrated during cricothyrotomy.
33
34 Educational objective:
35 Cricothyrotomy is indicated w hen an emergency airway is required and orotracheal or nasotracheal intubation
36 is either unsuccessful or contraindicated. The cricothyrotomy incision passes through the superficial cervical
37 fascia, pretracheal fascia, and the cricothyroid membrane.
38
39
Time Spent: 1 seconds Copyright USMLEW orld,LLC. Last updated: [3/2212014]
40
41
42
~imeRemaini~ ~ 0
I Tuto' Feedback Suspend End Block
---------------------------------------------------------------------------------------------------------------------------------------------
1
2
" Item: 25 of 46
Q.ld: 1639 [
111 f> Mark .<:J
Previous
[::>
Next
il
Lab Values
~
Notes
~
Calculator
3
4
5
6 A 43-year-old male presents to your office w ith difficulty w alking. On gait examination, you note that he leans
7 to the right side w hile w alking. W hen he is asked to stand on his left foot, his right hip tilts significantly
8 dow nward. W hich of the follow ing ne!Ves is most likely injured in this patient?
9
10
11 0 A. Superior gluteal
12 0 B. Inferior gluteal
13
14 0 C. Obturator
15 0 D. Femoral
16
17 0 E. Sciatic
18
19
20
21
22
23
24

26
27
28
29
30
31
32
33
34
35
36
37
38
39
40
41
42
~imeRemaini~ ~ 0
I Tuto' Feedback Suspend End Block
---------------------------------------------------------------------------------------------------------------------------------------------
1
2
" Item: 25 of 46
Q.ld: 1639 [
111 f> Mark .<:J
Previous
[::>
Next
il
Lab Values
~
Notes
~
Calculator
3
4
5 .; @ A. Superior gluteal [63%]
6 0 B. Inferior gluteal [21 %]
7
0 C. Obturator [8%]
8
9 0 D. Femoral [2%]
10 0 E. Sciatic [5%]
11
12
13 Explanation: User ld:
14
15 This patient demonstrates a positive Trendelenburg sign . The Trendelenburg test involves observing a patient
16 standing facing aw ay from the examiner w hile lifting each foot off of the ground. The sign is positive w hen the
17 hip dips toward the unaffected side w hen the patient stands on the affected leg. This can occur due to
18 w eakness of the gluteus medius and gluteus minimus muscles because these muscles function to pull the
19 pelvis dow n and abduct the thigh. These muscles are supplied by the superior gluteal nerve.
20
Patients w ith injury of the superior gluteal nerve may exhibit a w addling gait or a characteristic limp know n as
21
the "gluteus medius limp." The Trendelenburg sign and gluteus medius limp can also be seen in patients w ith
22
injuries to the gluteus medius and gluteus minimus muscles themselves as w ell as in patients w ith injuries to
23
24 the hip joint.

(Choice B) The inferior gluteal nerve innervates the gluteus maximus muscle. This muscle is responsible for
26 extension of the thigh at the hip and external rotation of the thigh. Injury of the inferior gluteal nerve causes
27 difficulty rising from the seated position and climbing stairs.
28
29 (Choice C) Injury of the obturator nerve causes deficits of thigh adduction as w ell as medial thigh sensory
30 loss.
31
32 (Choice D) The femoral nerve innervates the muscles responsible for flexion of the thigh at the hip (iliacus
33 and sartorius muscles) and extension of the leg at the knee (quadriceps femoris). It also supplies sensory
34 innervation to the skin on the anterior thigh and medial leg. Injury results in loss of the knee reflex and anterior
35 thigh sensory loss.
36
37 (Choice E) The sciatic nerve innervates all of the knee flexors (hamstrings) except the short head of the
38 biceps femoris. It subsequently divides into the tibial and common peroneal nerves, w hich each go on to
39 provide motor and sensory innervation to the leg and foot.
40
41
42
~imeRemaini~ ~ 0
I Tuto' Feedback Suspend End Block
---------------------------------------------------------------------------------------------------------------------------------------------
1
2
" Item: 25 of 46
Q.ld: 1639 [
111 f> Mark .<:J
Previous
[::>
Next
il
Lab Values
~
Notes
~
Calculator
3
4
5
Explanation: User ld:
6
7 This patient demonstrates a positive Trendelenburg sign . The Trendelenburg test involves observing a patient
8 standing facing aw ay from the examiner w hile lifting each foot off of the ground. The sign is positive w hen the
9 hip dips tow ard the unaffected side w hen the patient stands on the affected leg. This can occur due to
10 w eakness of the gluteus medius and gluteus minimus muscles because these muscles function to pull the
11 pelvis dow n and abduct the thigh . These muscles are supplied by the superior gluteal nerve.
12
13 Patients w ith injury of the superior gluteal nerve may exhibit a w addling gait or a characteristic limp know n as
14 the "gluteus medius limp." The Trendelenburg sign and gluteus medius limp can also be seen in patients w ith
15 injuries to the gluteus medius and gluteus minimus muscles themselves as w ell as in patients w ith injuries to
16 the hip joint.
17
18 (Choice B) The inferior gluteal nerve innervates the gluteus maximus muscle. This muscle is responsible for
19 extension of the thigh at the hip and external rotation of the thigh . Injury of the inferior gluteal nerve causes
20 difficulty rising from the seated position and climbing stairs.
21
22 (Choice C) Injury of the obturator nerve causes deficits of thigh adduction as w ell as medial thigh sensory
23 loss.
24
(Choice D) The femoral nerve innervates the muscles responsible for flexion of the thigh at the hip (iliacus
26 and sartorius muscles) and extension of the leg at the knee (quadriceps femoris). It also supplies sensory
27 innervation to the skin on the anterior thigh and medial leg. Injury results in loss of the knee reflex and anterior
28 thigh sensory loss.
29
(Choice E) The sciatic nerve innervates all of the knee flexors (hamstrings) except the short head of the
30
biceps femoris . It subsequently divides into the tibial and common peroneal nerves, w hich each go on to
31
provide motor and sensory innervation to the leg and foot.
32
33 Educational Objective:
34 1) Injury to the superior gluteal nerve causes w eakness of the gluteus medius and gluteus minimus muscles,
35 producing a positive Trendelenburg test.
36 2) The inferior gluteal nerve innervates the gluteus maximus muscle. Injury of the inferior gluteal nerve causes
37 difficulty rising from the seated position and climbing stairs.
38
39
Time Spent: 1 seconds Copyright USMLEW orld,LLC. Last updated: [11 / 10/2011]
40
41
42
~imeRemaini~ ~ 0
I Tuto' Feedback Suspend End Block
---------------------------------------------------------------------------------------------------------------------------------------------
1
2
" Item: 26 of 46
Q.ld: 1102 [
111 f> Mark .<:J
Previous
[::>
Next
il
Lab Values
~
Notes
~
Calculator
3
4
5
6 A 34-year-old male presents to the emergency room complaining of severe right shoulder pain after falling
7 from a tree. X-ray reveals a fracture in the middle third of the right clavicle. The medial end of the fractured
8 clavicle is displaced upw ard due to traction by the:
9
10
11 0 A. Rhomboid muscles
12 0 B. Sternocleidomastoid muscle
13
14 0 C. Pectoralis minor muscle
15 0 D. Serratus anterior muscle
16
17 0 E. Subclavius muscle
18
0 F. Sternoclavicular ligaments
19
20
21
22
23
24
25

27
28
29
30
31
32
33
34
35
36
37
38
39
40
41
42
~imeRemaini~ ~ 0
I Tuto' Feedback Suspend End Block
---------------------------------------------------------------------------------------------------------------------------------------------
1
2
" Item: 26 of 46
Q.ld: 1102 [
111 f> Mark .<:J
Previous
[::>
Next
il
Lab Values
~
Notes
~
Calculator
3
4 ~ @ B. Sternocleidomastoid muscle [73%]
5
6 0 C. Pectoralis minor muscle [4%]
7 0 D. Serratus anterior muscle [5%]
8 0 E. Subclavius muscle [9%]
9
10 0 F. Sternoclavicular ligaments [7%]
11
12 Explanation: User ld:
13
14 Several muscles insert on the clavicle, including the pectoralis major muscle on the inferior medial aspect, the
15 deltoid muscle on the inferior lateral aspect, the subclavius muscle on the inferior lateral aspect, the trapezius
16 muscle on the superior lateral aspect, and the sternocleidomastoid muscle on the superior medial aspect.
17 The sternocleidomastoid muscle is responsible for the upw ard traction on the medial section of the fractured
18 clavicle described above. Recall that the sternocleidomastoid muscle has three points of insertion (described
19 in its name): "sterno" - the muscle inserts superiorly on the manubrium of the sternum, "cleido" - the muscle
20 inserts on the medial part of the clavicle, "mastoid" - the muscle inserts on the mastoid process of the skull.
21
22 (Choice A) Both the major and the minor rhomboid muscles originate on the vertebral bodies/spines and
23 insert on the medial border of the scapula .
24
25 (Choice C) The pectoralis minor muscle originates on 3"', 4~ and 5"' ribs and inserts on the the coracoid
process of the scapula.
27
28 (Choice D) The serratus anterior muscle originates laterally on the 1' through 8"' ribs and inserts onto the
29 medial border of the scapula . This muscle is innervated by the long thoracic nerve, injury to w hich causes
30 w inging of the scapula .
31
(Choice E) The subclavius muscle originates on the 101 rib and inserts on the inferior lateral aspect of the
32
clavicle.
33
34 (Choice F) The sternoclavicular ligaments join the clavicle to the manubrium. These ligaments would tend to
35 hold the medial fragment of the clavicle in place.
36
37 Educational Objective:
38 The sternocleidomastoid muscle originates on the medial clavicle and manubrium and inserts on the mastoid
39 process of the skull. It is innervated by CN XI and functions to turn the head in the opposite direction.
40
41
42
~imeRemaini~ ~ 0
I Tuto' Feedback Suspend End Block
---------------------------------------------------------------------------------------------------------------------------------------------
1
2
" Item: 26 of 46
Q.ld: 1102 [
111 f> Mark .<:J
Previous
[::>
Next
il
Lab Values
~
Notes
~
Calculator
3
4 u L:. t'ectoraus mtnor muscle [4'7oJ
5 0 D. Serratus anterior muscle [5%]
6
0 E. Subclavius muscle [9%]
7
8 0 F. Sternoclavicular ligaments [7%]
9
10
11 Explanation: User ld:
12 Several muscles insert on the clavicle, including the pectoralis major muscle on the inferior medial aspect, the
13 deltoid muscle on the inferior lateral aspect, the subclavius muscle on the inferior lateral aspect, the trapezius
14 muscle on the superior lateral aspect, and the sternocleidomastoid muscle on the superior medial aspect.
15 The sternocleidomastoid muscle is responsible for the upw ard traction on the medial section of the fractured
16 clavicle described above. Recall that the sternocleidomastoid muscle has three points of insertion (described
17 in its name}: "sterno" - the muscle inserts superiorly on the manubrium of the sternum, "cleido" - the muscle
18 inserts on the medial part of the clavicle, "mastoid"- the muscle inserts on the mastoid process of the skull.
19
20 (Choice A) Both the major and the minor rhomboid muscles originate on the vertebral bodies/spines and
21 insert on the medial border of the scapula.
22
23 (Choice C) The pectoralis minor muscle originates on 310, 4~ and 5" ribs and inserts on the the coracoid
24 process of the scapula .
25
(Choice D) The serratus anterior muscle originates laterally on the 101 through s~ ribs and inserts onto the
27 medial border of the scapula . This muscle is innervated by the long thoracic nerve, injury to w hich causes
28 w inging of the scapula .
29
30 (Choice E) The subclavius muscle originates on the 101 rib and inserts on the inferior lateral aspect of the
31 clavicle.
32
(Choice F) The sternoclavicular ligaments join the clavicle to the manubrium. These ligaments w ould tend to
33
hold the medial fragment of the clavicle in place.
34
35 Educational Objective:
36 The sternocleidomastoid muscle originates on the medial clavicle and manubrium and inserts on the mastoid
37 process of the skull. It is innervated by CN XI and functions to turn the head in the opposite direction.
38
39
Time Spent: 1 seconds Copyright USMLEWorld,LLC. Last updated: [1 / 10/2013]
40
41
42
~imeRemaini~ ~ 0
I Tuto' Feedback Suspend End Block
---------------------------------------------------------------------------------------------------------------------------------------------
1
2
" Item: 27 of 46
Q.ld: 1491 [
111 f> Mark .<:J
Previous
[::>
Next
il
Lab Values
~
Notes
~
Calculator
3
4
5
6 A 46-year-old gardener presents to your office w ith knee pain . He reports that he spends hours on his knees
7 several times per w eek w hile gardening. W hich of the follow ing bursae do you most expect to be affected in
8 this patient?
9
10
11 0 A. Suprapatellar bursa
12 0 B. Prepatellar bursa
13
14 0 C. Anserine bursa
15 0 D. Semimembranous (popliteal) bursa
16
17 0 E. Gastrocnemius bursa
18
19
20
21
22
23
24
25
26

28
29
30
31
32
33
34
35
36
37
38
39
40
41
42
~imeRemaini~ ~ 0
I Tuto' Feedback Suspend End Block
---------------------------------------------------------------------------------------------------------------------------------------------
1
2
" Item: 27 of 46
Q.ld: 1491 [
111 f> Mark .<:J
Previous
[::>
Next
il
Lab Values
~
Notes
~
Calculator
3
4 A 'lo-year-ota garaener presents to your omce w nn Knee pa1n . He reports mat ne spenas nours on ms Knees
5 several times per w eek w hile gardening. W hich of the follow ing bursae do you most expect to be affected in
6 this patient?
7
8 0 A. Suprapatellar bursa [25%]
9
~ @ B. Prepatellar bursa [56%]
10
11 0 C. Anserine bursa [6%]
12 0 D. Semimembranous (popliteal) bursa [11 %]
13
14
0 E. Gastrocnemius bursa [2%]
15
16 Explanation: User ld:
17
18 The prepatellar bursa is located between the patella and the overlying skin and prepatellar tendon . It is lined
19 by synovium and contains very little fluid. Its function is to diminish friction and ensure maximal range of
20 motion at the knee. Chronic trauma from repeated kneeling is one cause of prepatellar bursitis, also called
21 "housemaid's knee." It is common in roofers, plumbers and carpet layers.
22
23 Symptoms of prepatellar bursitis include knee pain, sw elling, redness and inability to flex the knee on the
24 affected side. The symptoms are usually relieved by rest. Physical examination reveals tenderness to
25 palpation, erythema, crepitance, and fluctuant edema over the low er pole of the patella .
26
(Choices A, D and E) Prolonged kneeling is not associated w ith inflammation of the suprapatellar, popliteal or
28 gastrocnemius bursae. Popliteal and gastrocnemius bursitis can be associated w ith the formation of Baker's
29 cysts.
30
(Choice C) Anserine bursitis (pes anserinus bursitis) presents w ith pain along the medial aspect of the
31
knee. It results from overuse in athletes or from chronic trauma in patients w ith heavy body habitus. There is
32
typically tenderness to palpation approximately 4 em distal to the anteromedial joint margin of the knee.
33
34 Educational Objective:
35 Repeated and prolonged kneeling can cause prepatellar bursitis. Dubbed "housemaid's knee," today it is
36 most commonly seen in roofers, carpenters and plumbers. Signs and symptoms of prepatellar bursitis
37 include knee pain, erythema, sw elling and inability to kneel on the affected side.
38
39
Time Spent: 2 seconds Copyright USMLEW orld,LLC. Last updated: [1 /10/2013]
40
41
42
~imeRemaini~ ~ 0
I Tuto' Feedback Suspend End Block
---------------------------------------------------------------------------------------------------------------------------------------------
1
2
" Item: 28 of 46
Q.ld: 8671 [
111 f> Mark .<:J
Previous
[::>
Next
il
Lab Values
~
Notes
~
Calculator
3
4 A 65-year-old man comes to the physician because of right low er abdominal and groin pain and difficulty
5 w alking for the past w eek. Climbing stairs has been particularly difficult for him, and he has fallen several
6 times due to "knee buckling." His past medical history is significant for diabetes mellitus, hypertension, and
7 atrial fibrillation. He takes w arfarin for chronic anticoagulation. On physical examination, the right patellar
8 reflex is decreased compared to the left. His abdominal CT is show n below .
9
10
11
12
13
14
15
16
17
18
19
20
21
22
23
24
25
26
27

29
30
31 W hich of the follow ing additional findings w ould you most expect on physical examination?
32
33
34 0 A. Impaired adduction of the right thigh
35 0 B. Loss of sensation on the anterior aspect of the right thigh
36
37 0 C. Loss of sensation on the dorsum of the right foot
38
39 0 D. Loss of sensation on the sole of the right foot
40

~
41
42 F!!ck
---------------------------------------------------------------------------------------------------------------------------------------------
suW.nd EnQck
1
2
" Item: 28 of 46
Q.ld: 8671 [
111 f> Mark .<:J
Previous
[::>
Next
il
Lab Values
~
Notes
~
Calculator
3
4 W i:::IIK.JriY I U l U I~ f.JCI~l W ~~K . VIU riUU ry :SlCIIf :S rli:::I:S U~t'rl fJi:::IIUC.UICIII Y UIIIIC.Uil I Ul rill rl, i:::IIIU rlt' rli:::I:S I i:::lllt'rl :St'Vt'l i:::ll

5 times due to "knee buckling." His past medical history is significant for diabetes mellitus, hypertension, and
6 atrial fibrillation . He takes w arfarin for chronic anticoagulation. On physical examination, the right patellar
7 reflex is decreased compared to the left. His abdominal CT is show n below.
8
9
10
11
12
13
14
15
16
17
18
19
20
21
22
23
24
25
26
27

29
W hich of the follow ing additional findings w ould you most expect on physical examination?
30
31
32 0 A. Impaired adduction of the right thigh
33
34 0 B. Loss of sensation on the anterior aspect of the right thigh
35
0 C. Loss of sensation on the dorsum of the right foot
36
37 0 D. Loss of sensation on the sole of the right foot
38
39 0 E. W eakened abduction of the right thigh
40

~
41
42
---------------------------------------------------------------------------------------------------------------------------------------------
F!!ck suW.nd EnQck
1
2
" Item: 28 of 46
Q.ld: 8671 [
111 f> Mark .<:J
Previous
[::>
Next
il
Lab Values
~
Notes
~
Calculator
3
4 ~ @ B. l oss of sensation on the anterior aspect of the right thigh [45%]
5
6 0 C. Loss of sensation on the dorsum of the right foot [14%]
7 0 D. Loss of sensation on the sole of the right foot [9%]
8 0 E. Weakened abduction of the right thigh [17%]
9
10
11 Explanation: User ld:
12
13
14 Lower extremity Nerve
Cause of injury Sensory deficit Motor deficit
15 nerve roots
16
17
18
19 t
20 Anterior hip dislocation,
21 ) Thigh adduction
22 Obturator L2L4 iatrogenic
(Choice A)
23 (eg, pelvic surgery)
24
25
26
27
r-.., ,- -
29
30
31 Pelvic fracture or mass \
32
Femoral L2L4
involving iliopsoas/ iliacus ~ Flexion of thigh,
33 muscle (eg, hematoma or -~ extension of leg

~-
34
abscess)
35
36
37 \,
38
39
40

~
41
42 F!!ck
---------------------------------------------------------------------------------------------------------------------------------------------
suW.nd EnQck
1
2
" Item: 28 of 46
Q.ld: 8671 [
111 f> Mark .<:J
Previous
[::>
Next
il
Lab Values
~
Notes
~
Calculator
3
4
5 \
6
7
8 ~
Fibula neck fractu re or \ Foot eversion,
9
10
Common
l4-S2 nerve compression at l dorsiflexion,
peroneal
11 fibular neck toe extension
12
13
14
~
15 (Choice C)
16
17
18 II
19
20
( Foot inversion,
:'-
21 Tibial l4-S3 Trauma to the knee plantar flexion
22 & toe flexion
23 1'1
H
24 ...}
25
26 (Choice D)
27 Thigh abduction
Superior gluteal l4-S1 Iatrogenic (eg, posterior None
(Choice E)
29 hip dislocation or buttocks
30
31 Inferior gluteal l5-S2 injection) None Thigh extension
32
( >uSMtEWodd, l l C
33
34
35 The CT image reveals a large fluid collection in the right retroperitoneum lying anterior to the psoas muscle.
36 The fluid is isodense with muscle and displaces the right kidney anteriorly. These findings are consistent with
37 a spontaneous retroperitoneal hematoma, most likely secondary to w arfarin use. The risk of bleeding while on
38 w arfarin therapy is greatest in patients with risk factors such as increased age, diabetes mellitus,
39 hypertension, and alcoholism.
40

~
41
42 F!!ck
---------------------------------------------------------------------------------------------------------------------------------------------
suW.nd EnQck
1
2
3
4
5 Tibial L4-S3 Trauma to the knee plantar flexion
6 & toe flexion
7
8
9
10 (Choice D)
11 Thigh abduction
12 Superior gluteal L4-S1 Iatrogenic (eg, posterior None
(Choice E)
13 hip dislocation or buttocks
14
Inferior gluteal LS-52 injection) None Thigh extension
15
16
()USMLWottd, LLC
17
18
19 The CT image reveals a large fluid collection in the right retroperitoneum lying anterior to the psoas muscle.
20 The fluid is isodense w ith muscle and displaces the right kidney anteriorly. These findings are consistent w ith
21 a spontaneous retroperitoneal hematoma, most likely secondary to w arfarin use. The risk of bleeding w hile on
22 w arfarin therapy is greatest in patients w ith risk factors such as increased age, diabetes mellitus,
23 hypertension, and alcoholism.
24
The femoral nerve descends through the fibers of the psoas major muscle, emerges laterally between the
25
psoas and iliacus muscle, and then runs beneath the inguinal ligament into the thigh. Femoral nerve
26
mononeuropathy can occur due to trauma (eg, pelvic fracture), compression from a hematoma or abscess,
27
stretch injury, or ischemia. Patients w ith femoral neuropathy develop w eakness involving the quadriceps
muscle group and may have w eakening of the iliopsoas w ith more proximal nerve injuries. They often
29
complain of difficulty w ith stairs and frequent falling secondary to "knee buckling." On examination, the patellar
30
reflex is generally diminished. In addition, sensory loss over the anterior and medial thigh and medial leg is
31
typical. Acute, severe pain in the groin, lower abdomen, or back may also occur if the neuropathy is caused
32
by a retroperitoneal hematoma.
33
34 Educational objective:
35 Lesions of the femoral nerve can occur due to trauma, nerve compression, stretch injury, or ischemia.
36 Patients develop weakness of the quadriceps muscle, loss of the patellar reflex, and loss of sensation over
37 the anterior and medial thigh and medial leg.
38
39
Time Spent: 5 seconds Copyright USMLEWorld,LLC. Last updated: [2112/2014]
40

~
41
42 F!!ck suW.nd EnQck
---------------------------------------------------------------------------------------------------------------------------------------------
1
2
" Item: 29 of 46
Q.ld: 1130 [
111 f> Mark .<:J
Previous
[::>
Next
il
Lab Values
~
Notes
~
Calculator
3
4
5
6 A 32-year-old male presents to the ER w ith sudden-onset heart palpitations. His blood pressure is 100/70
7 mmHg, and his heart rate is 160/min w ith regular rhy1hm. The physician instructs the patient to do the
8 'Valsalva maneuver" to relieve these symptoms. W hich of the follow ing structures indicated on the pelvic CT
9 image below is most important in performing the Valsalva maneuver?
10
11
12
13
14
15
16
17
18
19
20
21
22
23
24
25
26
27
28

30
31
32
33
0 A. A
34
35 0 B. B
36 0 C. C
37
0 D. D
38
39 0 E. E
40
41
42
~imeRemaini~ ~ 0
I Tuto' Feedback Suspend End Block
---------------------------------------------------------------------------------------------------------------------------------------------
1
2
" Item: 29 of 46
Q.ld: 1130 [
111 f> Mark .<:J
Previous
[::>
Next
il
Lab Values
~
Notes
~
Calculator
3
4
5 v @ A. A [74%]
6 0 B. B [1 5%]
7
8 0 C. C [3%]
9 0 D. D [3%]
10 0 E. E [3%]
11
12
13 Explanation: User ld:
14
15 The patient described in the question stem is experiencing paroxysmal supraventricular tachycardia .
16 Treatment can be with vagal stimulation, such as carotid sinus massage or the Valsalva maneuver.
17 Maneuvers that increase vagal tone increase the refractory period in the AV node and help prevent a reentrant
18 circuit from conducting. If Valsalva measures fail, intravenous administration of adenosine is recommended.
19
20 The Valsalva maneuver is executed by forcibly exhaling against a closed glottis. This is done by taking a full
21 inhalation, closing the glottis (i.e. holding one's breath), and subsequently bearing down-without exhaling-as
22 one would during a bowel movement. The rectus muscles are recruited in this process, and they (Choice A)
23 play the largest role in the development of the resultant elevated intraabdominal and intrathoracic pressure
24 during this maneuver.
25
(Choice B) This structure is the iliacus muscle, a flexor of the hip that lies over the iliac fossa .
26
27 (Choice C) This structure is the gluteus minimus muscle. This muscle is innervated by the superior gluteal
28 nerve and has the subtle, but important function of preventing the contralateral (non-weight-bearing) side of
the pelvis from dipping when that leg is elevated off of the ground, as when one walks.
30
31 (Choice D) This structure is the gluteus medius muscle. This muscle has the same innervation and function
32 as the gluteus minimus.
33
34 (Choice E) This structure is the gluteus maximus muscle. This muscle is innervated by the inferior gluteal
35 nerve and is the major ex1ensor of the thigh at the hip.
36
37 Educational Objective:
38 The Valsalva maneuver increases vagal tone and can be used to abolish paroxysmal supraventricular
39 tachycardia. The rectus abdominis is the most important muscle in achieving the increased intraabdominal
40 and intrathoracic pressure of the Valsalva maneuver.
41
42
~imeRemaini~ ~ 0
I Tuto' Feedback Suspend End Block
---------------------------------------------------------------------------------------------------------------------------------------------
1
2
" Item: 29 of 46
Q.ld: 1130 [
111 f> Mark .<:J
Previous
[::>
Next
il
Lab Values
~
Notes
~
Calculator
3
4 u I::S. I::S [1 0 '10J
5 0 C. C [3%]
6
0 D. D [3%]
7
8 0 E. E [3%]
9
10
11 Explanation: User ld:
12 The patient described in the question stem is experiencing paroxysmal supraventricular tachycardia .
13 Treatment can be w ith vagal stimulation, such as carotid sinus massage or the Valsalva maneuver.
14 Maneuvers that increase vagal tone increase the refractory period in the AV node and help prevent a reentrant
15 circuit from conducting. If Valsalva measures fail, intravenous administration of adenosine is recommended.
16
17 The Valsalva maneuver is executed by forcibly exhaling against a closed glottis. This is done by taking a full
18 inhalation, closing the glottis (i.e. holding one's breath), and subsequently bearing dow n-w ithout exhaling-as
19 one w ould during a bow el movement. The rectus muscles are recruited in this process, and they (Choice A)
20 play the largest role in the development of the resultant elevated intraabdominal and intrathoracic pressure
21 during this maneuver.
22
23 (Choice B) This structure is the iliacus muscle, a flexor of the hip that lies over the iliac fossa .
24
25 (Choice C) This structure is the gluteus minimus muscle. This muscle is innervated by the superior gluteal
26 nerve and has the subtle, but important function of preventing the contralateral (non-w eight-bearing) side of
27 the pelvis from dipping w hen that leg is elevated off of the ground, as w hen one w alks.
28
(Choice D) This structure is the gluteus medius muscle. This muscle has the same innervation and function
30 as the gluteus minimus.
31
(Choice E) This structure is the gluteus maximus muscle. This muscle is innervated by the inferior gluteal
32
nerve and is the major extensor of the thigh at the hip.
33
34 Educational Objective:
35 The Valsalva maneuver increases vagal tone and can be used to abolish paroxysmal supraventricular
36 tachycardia. The rectus abdominis is the most important muscle in achieving the increased intraabdominal
37 and intrathoracic pressure of the Valsalva maneuver.
38
39
Time Spent: 3 seconds Copyright USMLEW orld,LLC. Last updated: [1 / 10/2013]
40
41
42
~imeRemaini~ ~ 0
I Tuto' Feedback Suspend End Block
---------------------------------------------------------------------------------------------------------------------------------------------
1
2
" Item: 30 of 46
Q.ld: 1698 [
111 f> Mark .<:J
Previous
[::>
Next
il
Lab Values
~
Notes
~
Calculator
3
4
5
6 A 30-year-old male presents to the emergency room after being involved in a street fight. There is significant
7 soft tissue sw elling around the left eye.
8
9
10
11
12
13
14
15
16
17
18
19
20
21
22
23
24
25
26
27
28 The imaging study above reveals a left orbital floor fracture w ith fluid accumulation in the:
29

31
0 A. Ethmoid cells
32 0 B. Maxillary sinus
33 0 C. Frontal sinus
34
0 D. Sphenoid sinus
35
36 0 E. Inferior conchae
37
38
39
40
41
42
~imeRemaini~ ~ 0
I Tuto' Feedback Suspend End Block
---------------------------------------------------------------------------------------------------------------------------------------------
1
2
" Item: 30 of 46
Q.ld: 1698 [
111 f> Mark .<:J
Previous
[::>
Next
il
Lab Values
~
Notes
~
Calculator
3
4
5
0 A. Ethmoid cells [3%]
6
7 " @ B. Maxillary sinus [89%]
8 0 C. Frontal sinus [2%]
9
0 D. Sphenoid sinus [4%]
10
11 0 E. Inferior conchae [1 %]
12
13
Explanation: User ld:
14
15 The orbit is bound superiorly by the thick orbital plate of the frontal bone and laterally by the thick bone of the
16 zygoma and greater and lesser sphenoid w ings. In contrast, the orbital floor is composed of a very thin layer
17 of bone that separates the orbit from the air-filled maxillary sinus. Similarly, the orbit's medial w all is
18 composed of the thin ethmoid and lacrimal bones, w hich separate it from the ethmoid air cells.
19
20 Blunt trauma to the eye causes a rapid increase in pressure that typically does not rupture the globe but is
21 transmitted posteriorly into the orbit. The w eakest plates of bone in the orbit, the medial and inferior w alls, are
22 common sites of fracture. Fracture is typically evident on radiographic imaging, and fluid (blood) can usually
23 be visualized in the normally air-filled adjacent sinuses. On the right side of the image above (corresponding
24 to the patient's left side), the maxillary sinus (below the orbital floor) is filled w ith fluid.
25
26 (Choice A) The ethmoid air cells are medial to the orbit. Here they are unaffected, but they can be affected in
27 blow out fractures of the orbit's medial w all.
28
29 (Choice C) The frontal sinus is not pictured above. It lies in the medial part of the supraorbital frontal bone.

31 (Choice D) The sphenoid sinuses (not pictured above) lie anterior to the optic chiasm and are not in close
32 relation to the orbit.
33
(Choice E) The inferior nasal conchae form part of the nasal turbinate system . They are not air-filled sinuses.
34
35 Educational Objective:
36 Blunt trauma to the globe can cause orbital blow out fractures. These fractures most commonly involve the
37 medial or inferior orbital w alls, because the bone bordering the ethmoid and maxillary sinuses is thin .
38
39
Time Spent: 25 seconds Copyright USMLEWorld,LLC. Last updated: [7/7/201OJ
40
41
42
~imeRemaini~ ~ 0
I Tuto' Feedback Suspend End Block
--------------------------------------------------------------------------------------------------------------------------------------
1
2
" Item: 31 of 46
Q.ld : 8579 [
lil f> Mark .<:J
Previous
[::>
Next
il
Lab Values
~
Notes
~
Calculator
3
4
5
6 A 3-year-old boy is brought to the emergency department for refusal to move his right upper extremity. His
7 mother states that the child w as fine this morning. She recalls that he fell w hile holding hands and w alking
8 w ith his older sister 2 hours earlier. On physical examination, he holds his right upper extremity at his side
9 w ith his elbow extended and forearm pronated. Any motion of the right elbow produces pain. There are no
10 visible bruises or edema . W hich of the follow ing structures is most likely injured in this patient?
11
12
13 0 A. Annular ligament
14 0 B. Biceps tendon
15
16 0 C. Interosseous membrane
17 0 D. Radial collateral ligament
18
19 0 E. Ulnar collateral ligament
20
21
22
23
24
25
26
27
28
29
30

32
33
34
35
36
37
38
39
40
41
42
~imeRemaini~ ~ 0
I Tuto' Feedback Suspend End Block
---------------------------------------------------------------------------------------------------------------------------------------------
1
2
" Item: 31 of 46
Q.ld : 8579 [
lil f> Mark .<:J
Previous
[::>
Next
il
Lab Values
~
Notes
~
Calculator
3
4
.; @ A. Annular ligament[33%]
5
6 0 B. Biceps tendon [27%]
7 0 C. Interosseous membrane [6%]
8
9
0 D. Radial collateral ligament [23%]
10 0 E. Ulnar collateral ligament [12%]
11
12
13 Explanation: User ld:
14
15 Radial head subluxation (nursemaid's elbow)
16
17
Annular ligament
18 Normal Traction
19 displacement
20
21
22
23
Distal
humerus
_j
24
25
26
27
28
29
30

32
33
34 Annular
35 ligament ---
36
37
38
39
40
41
42
1
2
" Item: 31 of 46
Q.ld: 8579 [
lil f> Mark .<:J
Previous
[::>
Next
il
Lab Values
~
Notes
~
Calculator
3
4
5 Radial head subluxation (nursemaid's elbow)
6
7
Annular ligament
8 Normal Traction
9 displacement
10
11
12
13
Distal
humerus
_j
14
15
16
17
18
19
20
21
22
23
24
25
26
27
28
29
30
Radius---- - - Ulna
32
33
34 @VSMlEWorld.ltC
35
36 This patient is experiencing radial head subluxation (nursemaid's elbow}, the most common elbow injury in
37 children. It occurs most frequently between the ages of 1 to 4 years. This injury often results from a sharp
38 pull on the hand w hile the forearm is pronated and the elbow is extended. The sudden increase in axial
39 traction on the proximal radius causes the annular ligament to tear from its periosteal attachment at the radial
40 nP.r.k It thP.n !'lin!': ovP.r thP. hP.<'!rl of thP. r<'!rlill!': <'!nrl !':lirlP.!<: into thP. r<'!rliohllmP.r<'!l ioint w hP.rP. it hP.r.omP.!':

~
41
42
---------------------------------------------------------------------------------------------------------------------------------------------
F!!ck suW.nd EnQck
1
2
" Item: 31 of 46
Q.ld : 8579 [
lil f> Mark .<:J
Previous
[::>
Next
il
Lab Values
~
Notes
~
Calculator
3
4 @ USMlEWOrld, UC

5
6 This patient is experiencing radial head subluxation (nursemaid's elbow}, the most common elbow injury in
7 children . It occurs most frequently between the ages of 1 to 4 years. This injury often results from a sharp
8 pull on the hand w hile the forearm is pronated and the elbow is extended. The sudden increase in axial
9 traction on the proximal radius causes the annular ligament to tear from its periosteal attachment at the radial
10 neck. It then slips over the head of the radius and slides into the radiohumeral joint, w here it becomes
11 trapped. By age 5, the annular ligament becomes thick and strong, decreasing the likelihood that it can be
12 torn or displaced.
13
14 Affected children present w ith the injured arm held close to the body w ith the elbow extended (or slightly
flexed} and the forearm pronated. The child is typically in little distress until attempts are made to move the
15
elbow . Mild tenderness can be elicited on palpation of the subluxed radial head. Reduction can be
16
accomplished by fully supinating the child's forearm follow ed by fully flexing the elbow .
17
18
(Choice B) The long head tendon of the biceps brachii courses intra-articularly to insert into the supraglenoid
19
tubercle, w hile the short head tendon attaches to the tip of the coracoid process. The common distal tendon
20 inserts into the radial tuberosity and the fascia of the forearm . Biceps tendon ruptures can occur both
21 proximally and distally, and present w ith a visible or palpable mass in the mid-upper arm ("Popeye" deformity}.
22
23 (Choice C) The interosseous membrane is a broad sheet of connective tissue that spans the space
24 between the radius and ulna, connecting these 2 bones at their midsections and serving as a site of
25 attachment for muscles of the forearm.
26
27 (Choice D) The radial collateral ligament lies on the lateral side of the elbow joint and strengthens the
28 radiohumeral joint.
29
30 (Choice E) The ulnar collateral ligament lies on the medial side of the elbow joint and strengthens the
ulnohumeral joint. Ulnar collateral ligament injuries occur most commonly in throw ers (eg, baseball pitchers}
32 due to intense valgus stress at the elbow . Treatment often involves ulnar collateral ligament reconstruction
33 (''Tom m y John" surgery}.
34
35 Educational objective:
36 Radial head subluxation (nursemaid's elbow} results from sudden traction on the outstretched and pronated
37 arm of a child. Affected children are usually in little distress unless attempts are made to move the
38 elbow . The annular ligament is torn and displaced in this injury.
39
40 Time Soent: 1 seconds Coovriaht USMLEW orld.LLC. Last uodated: 11 2128/20131
41
42
~imeRemaini~ ~ 0
I Tuto' Feedback Suspend End Block
---------------------------------------------------------------------------------------------------------------------------------------------
1
2
" Item: 32 of 46
Q.ld: 8329 [
111 f> Mark .<:J
Previous
[::>
Next
il
Lab Values
~
Notes
~
Calculator
3
4
5
6 A 51-year-old w oman comes to the physician complaining that the right side of her face has felt "funny" for the
7 past 12 hours. Her past medical history is significant for type 2 diabetes mellitus. Physical examination
8 reveals asymmetry of her face w hen she smiles, puffs out her cheeks, and closes her eyes tightly. There is
9 also effacement of the right nasolabial fold, and her mouth is drawn tow ard the left side. W hich of the
10 follow ing additional findings is most likely associated w ith this patient's condition?
11
12
13 0 A. Attenuated sense of touch on the right side of face
14 0 B. Decreased tearing from the right eye
15
16 0 C. Failure to elevate the palate w hen saying "ah"
17 0 D. Inability to sw eat on the right side of face
18
19 0 E. Loss of taste sensation from the posterior 1/3 of the tongue
20
21
22
23
24
25
26
27
28
29
30
31

33
34
35
36
37
38
39
40
41
42
~imeRemaini~ ~ 0
I Tuto' Feedback Suspend End Block
---------------------------------------------------------------------------------------------------------------------------------------------
1
2
" Item: 32 of 46
Q.ld: 8329 [
111 f> Mark .<:J
Previous
[::>
Next
il
Lab Values
~
Notes
~
Calculator
3
4
~ @ B. Decreased tearing from the right eye [51 %]
5
6 0 C. Failure to elevate the palate when saying "ah" [9%]
7 0 D. Inability to sweat on the right side of face [1 6%]
8
9
0 E. Loss of taste sensation from the posterior 1/3 of the tongue [9%]
10
11 Explanation: User ld:
12
13 This patient's presentation is consistent with Bell's palsy, an idiopathic paresis of the facial nerve. To
14 understand the pathophysiology of Bell's palsy, it is important to understand the function of the facial nerve
15 (CN VII). The facial nerve is a mixed nerve, consisting of the following:
16
17 1. Motor output to the facial muscles
18 2. Parasympathetic innervation to the lacrimal, submandibular, and sublingual salivary glands
19 3. Special afferent fibers for taste from the anterior 2/3 of the tongue
20 4. Somatic afferents from the pinna and external auditory canal
21
22 Patients with Bell's palsy typically present with sudden onset of unilateral facial paralysis. Specific findings
23 include impaired eye closure, eyebrow sagging, inability to smile and frown on the affected side,
24 disappearance of the nasolabial fold, and the mouth being drawn to the non-affected side. Patients afflicted
25 with Bell's palsy may also have decreased tearing, hyperacusis, and/or loss of taste sensation over the
26 anterior two-thirds of the tongue.
27
28 (Choice A) Trigeminal nerve (CN V) somatic afferents are responsible for providing general sensation over
29 the face; the motor branch of the trigeminal nerve (V3} innervates the muscles of mastication and the tensor
30 tympani.
31
(Choice C) Palate elevation and the gag reflex involve the glossopharyngeal (CN IX) and vagus (CN X)
33 nerves.
34
35 (Choice D) Horner's syndrome occurs due to interruption of sympathetic innervation and clinically presents
36 with ptosis, miosis, anhydrosis, and enophthalmos. Post-synaptic sympathetic axons travel in a perivascular
37 plexus along the carotid arteries to reach and innervate the face.
38
(Choice E) Taste sensation from the posterior 1/3 of the tongue is provided by the glossopharyngeal nerve
39
40 (CN IX).
41
42
~imeRemaini~ ~ 0
I Tuto' Feedback Suspend End Block
---------------------------------------------------------------------------------------------------------------------------------------------
1
2
" Item: 32 of 46
Q.ld: 8329 [
111 f> Mark .<:J
Previous
[::>
Next
il
Lab Values
~
Notes
~
Calculator
3
4
. . .
(CN VII). The facial nerve is a mixed nerve, consisting of the following:
5
6 1. Motor output to the facial muscles
7 2. Parasympathetic innervation to the lacrimal, submandibular, and sublingual salivary glands
8 3. Special afferent fibers for taste from the anterior 2/3 of the tongue
9 4. Somatic afferents from the pinna and external auditory canal
10
11 Patients with Bell's palsy typically present with sudden onset of unilateral facial paralysis. Specific findings
12 include impaired eye closure, eyebrow sagging, inability to smile and frown on the affected side,
13 disappearance of the nasolabial fold, and the mouth being drawn to the non-affected side. Patients afflicted
14 with Bell's palsy may also have decreased tearing, hyperacusis, and/or loss of taste sensation over the
15 anterior two-thirds of the tongue.
16
17 (Choice A) Trigeminal nerve (CN V) somatic afferents are responsible for providing general sensation over
18 the face; the motor branch of the trigeminal nerve (V3} innervates the muscles of mastication and the tensor
19 tympani.
20
21 (Choice C) Palate elevation and the gag reflex involve the glossopharyngeal (CN IX) and vagus (CN X)
22 nerves.
23
24 (Choice D) Horner's syndrome occurs due to interruption of sympathetic innervation and clinically presents
25 with ptosis, miosis, anhydrosis, and enophthalmos. Post-synaptic sympathetic axons travel in a perivascular
26 plexus along the carotid arteries to reach and innervate the face.
27
28 (Choice E) Taste sensation from the posterior 113 of the tongue is provided by the glossopharyngeal nerve
29 (CN IX).
30
Educational objective:
31
In addition to unilateral facial paralysis, patients with Bell's palsy may experience decreased tearing,
hyperacusis, and/or loss of taste sensation over the anterior two-thirds of the tongue.
33
34
35 References:
36 1. Seventh cranial neuropathy.
37
38
39
Time Spent: 2 seconds Copyright USMLEWorld,LLC. Last updated: [12/2312013]
40

~
41
42 F!!ck
---------------------------------------------------------------------------------------------------------------------------------------------
suW.nd EnQck
1
2
" Item: 33 of 46
Q.ld: 8636 [
111 f> Mark .<:J
Previous
[::>
Next
il
Lab Values
~
Notes
~
Calculator
3
4
A 67 -year-old w oman comes to the physician complaining of a decrease in vision in her right eye that she
5
noticed a w eek ago. She says that her sight in her right eye is "narrow er" than in her left and seems to be
6
"directed more tow ards the right." Her past medical history is significant for hypertension, type 2 diabetes
7
mellitus, and hyperlipidemia. She also suffers from intermittent claudication and has had 2 prior episodes of
8
transient vision loss in her right eye. Visual field testing reveals a right nasal hemianopia. CT scan of the
9
10 head show s a calcified right internal carotid artery. W hich of the follow ing portions of the visual pathway is
11 most likely disrupted in this patient?
12
13
14
15
16
17
18
19
20
21
22
23
24
25
26
27
28
29
30
31
32

34
35
36
37
38
39
40
41
42
1
2
" Item: 33 of 46
Q.ld: 8636 [
111 f> Mark .<:J
Previous
[::>
Next
il
Lab Values
~
Notes
~
Calculator
3
4
5
6
7
8
9
10
11
12
13
14
15
16
17
18
19
20
21
22
23
24
25
26
27
28
29
30
31
32

34
35
36
37
38
39
40
41
42
1
2
3
4
5
6
7
8
9
10
11
12
13
14
15
16
17
18
19
20
21
22
23
24
25
26
27 \<) USMlEWorld. l lC H
28
29
30 0 A. A
31 0 B. B
32
0 C. C
34 0 D. D
35 0 E. E
36
37 0 F. F
38 0 G. G
39 0 H. H
40
41
42
1
2
" Item: 33 of 46
Q.ld: 8636 [
111 f> Mark .<:J
Previous
[::>
Next
il
Lab Values
~
Notes
~
Calculator
3
4 ~ @D. D [34%]
5
6
0 E. E [8%]
7 0 F. F [3%]
8 0 G. G [1 %]
9
10
0 H. H [2%]
11
12 Explanation: User ld:
13
14
15 Visual field defect Location of lesion Possible causes
16
17
18 A. Monocular scotoma Partial lesion in the Macular degeneration, optic
19 retina, optic disk, neuritis

0~
20 optic nerve
21
22
23
24
B. Right anopia Right optic nerve Retinal artery or central retinal
25
vein occlusion

0
26
27
28
29
30
31 C. Bitemporal hemianopia Optic chiasm Pressure exerted by a pituitary
32
tumor, craniopharyngioma,
34 aneurysm of anterior
35 commun icating artery
36
37
38 D. Right nasal hemianopia Right peri-chiasma) Calcification or aneurysm of
39 lesion the internal carotid artery
40

~
41
42 F!!ck
--------------------------------------------------------------------------------------------------------------------------------------
suW.nd EnQck
1
2
" Item: 33 of 46
Q.ld : 8636 [
111 f> Mark .<:J
Previous
[::>
Next
il
Lab Values
~
Notes
~
Calculator
3
4
5 Visual field defect Location of lesion Possible causes "
6
7
A. Monocular scotoma Partial lesion in the Macular degeneration, optic
8
retina, optic disk, neuritis

0 ~
9
10 optic nerve
11
12
13
14 B. Right anopia Right optic nerve Retinal artery or central retinal
15
vein occlusion

0
16
17
18
19
20
21 C. Bitemporal hemianopia Optic chiasm Pressure exerted by a pituitary
22 tumor, craniopharyngioma,
23
24 aneurysm of anterior
25 communicating artery
26
27
28 D. Right nasal hemianopia Right peri-chiasma! Calcification or aneurysm of
29 lesion the internal carotid artery
30
31
32

34
35
0
E. Left homonymous hemianopia Right optic tract or
impinging on uncrossed, lateral
retinal fibers

Optic tract: occlusion of anterior


36 optic radiation choroidal artery
37
38 Optic radiation: occlusion of a
39 middle cerebral artery branch
40
41
42
~imeRemaini~ ~ 0
I Tuto' Feedback
---------------------------------------------------------------------------------------------------------------------------------------------
Suspend End Block
1
2
" Item: 33 of 46
Q.ld : 8636 [
111 f> Mark .<:J
Previous
[::>
Next
il
Lab Values
~
Notes
~
Calculator
3
4
5 E. Left homonymous hemianopia Right optic tract or Optic tract: occlusion of anterior
6 optic radiation choroidal artery
7
8 Optic radiation: occlusion of a
9
middle cerebral artery branch
10
11 or lesion involving posterior
12 limb of internal capsule
13
14
F. Left homonymous superior Right temporal lobe Lesion or stroke involving
15
16 quadrantanopia ("pie in the sky") (Meyer's loop) temporal lobe
17
18
19
20
21
22
23 G. Left homonymous inferior Right parietal lobe Lesion or stroke involving
24
quadrantanopia ("pie on the floor") (dorsal optic parietal lobe
25
26 radiation)
27
28
29
30
31
32 H. Left homonymous hemianopia Right pri mary visual Occlusion of posterior cerebral
with macular sparing cortex (occipital lobe) artery. Macula is spared due to
34
collateral blood from the
35
36 middle cerebral artery
37
38
39
40 ~ I I ClA I I:W...,~I A I I r
41
42
~imeRemaini~ ~ 0
I Tuto' Feedback Suspend End Block
--------------------------------------------------------------------------------------------------------------------------------------
1
2
" Item: 33 of 46
Q.ld : 8636 [
111 f> Mark .<:J
Previous
[::>
Next
il
Lab Values
~
Notes
~
Calculator
3
4
5
6 H. Left homonymous hemianopia Right primary visual Occlusion of posterior cerebral
7 with macular sparing cortex (occipita l lobe) artery. Macula is spared due to
8 collateral blood from the
9
10 middle cerebral artery
11
12
13
14 @ USMl.EWorld.llC
15
16 Humans have a maximum horizontal field of view of about 200 degrees with both eyes. The central 120
17 degrees make up the binocular field of view (stereoscopic vision), with 2 side fields of about 40 degrees seen
18 by only 1 eye (monoscopic vision). Unilateral visual defects within the binocular field of view (such as
19 unilateral nasal hemianopia) may be masked due to compensation by the normal eye and go unnoticed by the
20 patient.
21
22 Light from the nasal visual field strikes the temporal side of each retina, and optic nerve fibers from the
23 temporal part of the retina travel laterally through the optic chiasm to pass into the ipsilateral optic tract.
24 Lesions involving the lateral aspects of the optic chiasm can cause nasal hemianopia by damaging these
25 uncrossed, temporal retinal fibers. Aneurysm or atherosclerotic calcification of the internal carotid artery can
26 induce a nasal defect by laterally impinging on the optic chiasm .
27
28 This patient's history of transient right-sided vision loss and head CT findings are consistent with significant
29 right internal carotid artery atherosclerosis. Thus, extrinsic compression of the optic chiasm by the right
30 internal carotid artery is the most likely cause of her symptoms.
31
(Choice A) A lesion involving the retina is more likely to produce a focal monocular scotoma than nasal
32
hemianopia, since the lesion would need to involve the entire temporal hemiretina in the latter.
34 Educational objective:
35 Aneurysm or atherosclerotic calcification of the internal carotid artery can laterally impinge on the optic
36 chiasm. This can cause nasal hemianopia by damaging uncrossed optic nerve fibers from the temporal
37 portion of the ipsilateral retina .
38
39
Time Spent: 7 seconds Copyright USMLEWorld,LLC. Last updated: [11 /6/201 3]
40
41
42
~imeRemaini~ ~ 0
I Tutor Feedback Suspend End Block
---------------------------------------------------------------------------------------------------------------------------------------------
1
2
" Item: 34 of 46
Q.ld: 8670 [
111 f> Mark .<:J
Previous
[::>
Next
il
Lab Values
~
Notes
~
Calculator
3
4
5
6 A 56-year-old man comes to the emergency department because of persistent left w rist pain. W hile w alking
7 his dog 5 hours ago, he fell forward to the ground and landed on his outstretched left hand. Examination
8 show s mild sw elling over the dorsum of the left hand w ith preserved range of motion. There is point
9 tenderness present over the anatomic snuffbox. An x-ray of his left w rist is show n below .
10
11
12
13
14
15
16
17
18
19
20
21
22
23
24
25
26
27
28
29
30
31
32
33

35
36
37
38
39
40
41
42
~imeRemaini~ ~ 0
I Tuto' Feedback
---------------------------------------------------------------------------------------------------------------------------------------------
Suspend End Block
1
2
3
4
5
6
7
8
9
10
11
12
13
14
15
16
17
18
19
20
21
22
23
24
25
26
27
28
29
30
31
This patient is at greatest risk for developing w hich of the follow ing complications?
32
33
0 A. Avascular necrosis
35 0 B. Carpal tunnel syndrome
36
37 0 C. Compartment syndrome
38 0 D. Dupuytren's contracture
39 0 E. Osteomyelitis
40
41
42
~imeRemaini~ ~ 0
I Tuto' Feedback Suspend End Block
---------------------------------------------------------------------------------------------------------------------------------------------
1
2
" Item: 34 of 46
Q.ld: 8670 [
111 f> Mark .<:J
Previous
[::>
Next
il
Lab Values
~
Notes
~
Calculator
3
4
5 .; @ A. Avascular necrosis [76%]
6 0 B. Carpal tunnel syndrome [9%]
7
0 C. Compartment syndrome [8%]
8
9 0 D. Dupuytren's contracture [6%]
10 0 E. Osteomyelitis (1 %]
11
12
13 Explanation: User ld:
14
15
16 Scaphoid avascular necrosis
17
18
19
20
21
22
23
24
25
26
27
28
29
30
31 ,---"~-------=~--- Palmar scaphoid
branch
32 Superficial
33 ,--___:.--:,.:.:._--!:--- - Dorsal carpal branch
palmar branch
35
36 Dorsal scaphoid _ __ ~,....,.........,....----<~~=------ Scaphoid fracture
37 branch
38 Necrosis of proximal
--~-------
39 pole of scaphoid
40
41
42
1
2
" Item: 34 of 46
Q.ld: 8670 [
111 f> Mark .<:J
Previous
[::>
Next
il
Lab Values
~
Notes
~
Calculator
3
4
5 Scaphoid avascular necrosis
6
7
8
9
10
11
12
13
14
15
16
17
18
19
20 r---'~-----.,..----=>;.---- Palmar scaphoid
21 branch
22
Superficial
r--__:.-..;;.:._-~-- Dorsal carpal branch
23 palmar branch
24
25 Dorsal scaphoid _ __ ~,....,.-.._,~~~=------Scaphoid fracture
26 branch
27 Necrosis of proximal
28 --~------- pole of scaphoid
29 Radial artery
30
31
32
33

35
36
37
38
39
40 <!:) USMLoWorld. LLC

41
42
~imeRemaini~ ~ 0
I Tuto' Feedback Suspend End Block
--------------------------------------------------------------------------------------------------------------------------------------
1
2
" Item: 34 of 46
Q.ld: 8670 [
111 f> Mark .<:J
Previous
[::>
Next
il
Lab Values
~
Notes
~
Calculator
3
4
5 Scaphoid fractures are the most common of the carpal bone fractures. They frequently result from falls
6 onto an outstretched arm that cause direct axial compression or wrist hyperextension. A scaphoid fracture
7 should be suspected in any patient with persistent wrist pain and tenderness in the anatomical snuff box
8 following a fall. The snuffbox is a triangular deepening on the dorsoradial aspect of the hand at the level of the
9 carpal bones. The scaphoid and trapezium bones form the floor of the snuffbox.
10
11 The scaphoid is the largest bone of the proximal carpal row and is located on the radial aspect of the hand just
12 distal to the radius itself. The dorsal scaphoid branch of the radial artery supplies the majority of the scaphoid
13 after entering near the bone's distal pole. Blood supply to the proximal pole proceeds in a retrograde manner
14 and can be easily interrupted by a fracture. Thus, scaphoid fractures are at risk for avascular necrosis and
15 nonunion.
16
(Choice B) Carpal tunnel syndrome is caused by median nerve compression as it travels through the carpal
17
18 tunnel. It is associated with repetitive wrist movements, hypothyroidism, diabetes mellitus, and rheumatoid
19 arthritis.
20 (Choice C) Acute compartment syndrome occurs when increased pressure within a fascial compartment
21 compromises blood circulation and tissue function within that space. It develops after significant trauma,
22 particularly long-bone fractures of the leg or forearm.
23
24 (Choice D) Dupuytren's contracture is a benign, slowly progressive fibroproliferative disease of the palmar
25 fascia. As the scarring progresses, nodules form on the palmar fascia and the fingers gradually lose their
26 flexibility, eventually resulting in contractures that draw the fingers into flexion.
27
28 (Choice E) Osteomyelitis commonly results from either hematogenous seeding of bone with bacteria or by
29 contiguous spread of organisms from a nearby wound. However, unexposed bone is unlikely to develop
30 osteomyelitis. Staphy lococcus aureus is the most common cause of hematogenous osteomyelitis.
31
32 Educational objective:
33 Fall on an outstretched hand may cause fracture of the scaphoid bone. Examination shows tenderness in the
anatomical snuff box. The scaphoid bone is vulnerable to avascular necrosis and nonunion due to its tenuous
35 blood supply.
36
37 References:
38
39 1. Fractures in brief: scaphoid fractures.
40
41
42
~imeRemaini~ ~ 0
I Tuto' Feedback Suspend End Block
---------------------------------------------------------------------------------------------------------------------------------------------
1
2
" Item: 34 of 46
Q.ld: 8670 [
111 f> Mark .<:J
Previous
[::>
Next
il
Lab Values
~
Notes
~
Calculator
3
V II ~V (,UI VU~~ U'-' ~"'11'-'U U l l l l UIU~ .... UU"''-' Ull'-' .... ~ U./\I(,U I..V IIIiJI '-'"'"''VII VI 1111 11"'~ IIJtJ'-'I'-'A~'-'II..,IVII . I"\ "''"-UtJIIVIU I I U .... U .U'-'
4
should be suspected in any patient w ith persistent w rist pain and tenderness in the anatomical snuff box
5
follow ing a fall. The snuffbox is a triangular deepening on the dorsoradial aspect of the hand at the level of the
6
carpal bones. The scaphoid and trapezium bones forrn the floor of the snuffbox.
7
8 The scaphoid is the largest bone of the proximal carpal row and is located on the radial aspect of the hand just
9 distal to the radius itself. The dorsal scaphoid branch of the radial artery supplies the majority of the scaphoid
10 after entering near the bone's distal pole. Blood supply to the proximal pole proceeds in a retrograde rnanner
11 and can be easily interrupted by a fracture. Thus, scaphoid fractures are at risk for avascular necrosis and
12 nonunion.
13
14 (Choice B) Carpal tunnel syndrome is caused by rnedian nerve compression as it travels through the carpal
15 tunnel. It is associated w ith repetitive w rist rnovernents, hypothyroidism, diabetes mellitus, and rheumatoid
16 arthritis.
17
18 (Choice C) Acute cornpartrnent syndrome occurs w hen increased pressure w ithin a fascial cornpartrnent
19 cornprornises blood circulation and tissue function w ithin that space. It develops after significant traurna,
20 particularly long-bone fractures of the leg or forearm .
21
22 (Choice D) Dupuytren's contracture is a benign, slow ly progressive fibroproliferative disease of the palrnar
23 fascia. As the scarring progresses, nodules forrn on the palrnar fascia and the fingers gradually lose their
24 flexibility, eventually resulting in contractures that draw the fingers into flexion.
25
26 (Choice E) Osteomyelitis cornrnonly results frorn either hematogenous seeding of bone w ith bacteria or by
27 contiguous spread of organisms frorn a nearby w ound. How ever, unexposed bone is unlikely to develop
28 osteomyelitis. Staphylococcus aureus is the rnost cornrnon cause of hematogenous osteomyelitis.
29
Educational objective:
30
Fall on an outstretched hand rnay cause fracture of the scaphoid bone. Examination show s tenderness in the
31
anatomical snuff box. The scaphoid bone is vulnerable to avascular necrosis and nonunion due to its tenuous
32
blood supply.
33

35 References:
36 1. Fractures in brief: scaphoid fractures.
37
38
39
Tirne Spent: 4 seconds Copyright USMLEW orld,LLC. Last updated: [1 0/2212013]
40
41
42
~imeRemaini~ ~ 0
I Tuto' Feedback Suspend End Block
---------------------------------------------------------------------------------------------------------------------------------------------
1
2
" Item: 35 of 46
Q.ld: 1704 [
111 f> Mark .<:J
Previous
[::>
Next
il
Lab Values
~
Notes
~
Calculator
3
4
5
6 A 24-year-old mountain climber presents to the emergency room w ith right shoulder pain follow ing a fall. He
7 w as climbing a cliff at a nearby national park w hen he lost his grip and fell approximately 5 meters onto the
8 boulders below. He says that he "landed on his right arm and heard the bone snap". Physical examination
9 reveals total inability to extend the right w rist. X-ray reveals a fracture of the right humeral shaft. W hich of the
10 follow ing arteries is most likely to be injured in this patient?
11
12
13 0 A. Anterior circumflex humeral artery
14
0 B. Brachial artery
15
16 0 C. Common interosseous artery
17
0 D. Deep brachial artery
18
19 0 E. Radial collateral artery
20
21
22
23
24
25
26
27
28
29
30
31
32
33
34

36
37
38
39
40
41
42
~imeRemaini~ ~ 0
I Tuto' Feedback Suspend End Block
---------------------------------------------------------------------------------------------------------------------------------------------
1
2
" Item: 35 of 46
Q.ld: 1704 [
111 f> Mark .<:J
Previous
[::>
Next
il
Lab Values
~
Notes
~
Calculator
3
4 .; @ D. Deep brachial artery [43%]
5 0 E. Radial collateral artery [14%]
6
7
8 Explanation: User ld:
9 Axillary artery
10
11
12
Anterior & Posterior _.
13 circumftex humeral arteries
14
15 Deep brachial artery
16 (runs with the radial nerve)
17
18
19
20 Brachial artery
21
22
23
24
25
26
27
28
Common interosseous artery
29
30
31 Radial artery
32
33
34

36
37
38
39
W hen the midshaft of the humerus is fractured, there is significant associated risk of injury to the radial nerve
40
41
42
~imeRemaini~ ~ 0
I Tuto' Feedback Suspend End Block
---------------------------------------------------------------------------------------------------------------------------------------------
1
2
" Item: 35 of 46
Q.ld: 1704 [
111 f> Mark .<:J
Previous
[::>
Next
il
Lab Values
~
Notes
~
Calculator
3
4
5
6 W hen the midshaft of the humerus is fractured, there is significant associated risk of injury to the radial nerve
7 and deep brachial artery. The radial nerve innervates most of the forearm extensors at the elbow and most of
8 the hand extensors at the w rist. It also innervates the extrinsic extensors of the digits, and the brachioradialis
9 and supinator muscles. The radial nerve also provides cutaneous sensory innervation to the dorsal hand,
10 forearm and upper arm . The deep brachial artery branches off of the brachial artery high in the arm, passes
11 inferior to the teres major muscle, and courses posteriorly along the humerus in close association w ith the
12 radial nerve.
13
14 The patient described above is exhibiting signs of radial nerve injury. The combination of radial nerve deficits
15 w ith radiographic evidence of a humeral fracture should raise concern for an associated injury to the deep
16 brachial artery.
17
18 (Choice A) The anterior circumflex humeral artery is a branch of the axillary artery that passes anterior to the
19 surgical neck of the humerus and anastomoses w ith the posterior circumflex humeral artery. A fracture to the
20 surgical neck of the humerus may damage this vessel and the axillary nerve.
21
22 (Choice B) The axillary artery gives off the posterior circumflex humeral artery and becomes the brachial
23 artery. The brachial artery courses anteromedially in the arm w ithin the bicipital groove and branches into the
24 radial and ulnar arteries after entering the forearm. Supracondylar fractures of the humerus may injure this
25 artery.
26
27 (Choice C) The common interosseous artery is a short branch of the ulnar artery that gives rise to anterior,
28 posterior and recurrent branches w ithin the proximal forearm.
29
(Choice E) The deep brachial artery divides into the radial collateral artery (the continuation of the deep
30
brachial artery) and middle collateral artery at the midpoint of the humerus. The radial collateral artery also
31
courses w ith the radial nerve. How ever, injury to the radial collateral artery is less likely because it originates
32
at the low er end of the spiral groove w here the deep brachial artery pierces the lateral intramuscular septum .
33
34 Educational Objective:
The deep brachial artery and radial nerve course along the posterior aspect of the humerus. Midshaft
36 fractures of the humerus risk injury to these structures. Supracondylar fractures are associated w ith injury to
37 the brachial artery.
38
39
Time Spent: 2 seconds Copyright USMLEW orld,LLC. Last updated: [219/2014]
40
41
42
~imeRemaini~ ~ 0
I Tuto' Feedback Suspend End Block
---------------------------------------------------------------------------------------------------------------------------------------------
1
2
" Item: 36 of 46
Q.ld: 1149 [
111 f> Mark .<:J
Previous
[::>
Next
il
Lab Values
~
Notes
~
Calculator
3
4
5
6 A 54-year-old alcoholic w ho has been sober for 6 months "falls off the w agon" and imbibes a large amount of
7 isopropyl alcohol. After an unknow n period of time she is found by her neighbor and brought to the emergency
8 room . The patient eventually arouses but is troubled by a persistent right foot drop. Prolonged compression
9 of w hat nerve most likely accounts for her symptoms?
10
11
12 O A. Common peroneal
13 0 B. Femoral
14
15 o c. Tibial
16 O D. Radial
17
18 O E. Pudendal
19 0 F. Lateral sural cutaneous
20
21
22
23
24
25
26
27
28
29
30
31
32
33
34
35

37
38
39
40
41
42
~imeRemaini~ ~ 0
I Tuto' Feedback Suspend End Block
---------------------------------------------------------------------------------------------------------------------------------------------
1
2
" Item: 36 of 46
Q.ld: 1149 [
111 f> Mark .<:J
Previous
[::>
Next
il
Lab Values
~
Notes
~
Calculator
3
4 ~ @ A. Common peroneal [82%]
5
6 0 B. Femoral [2%]
7 0 C. Tibial [12%]
8 0 D. Radial [1 %]
9
10 0 E. Pudendal [2%]
11 0 F. Lateral sural cutaneous [1 %]
12
13
14 Explanation: User ld:
15
The common peroneal nerve is the lateral branch of the sciatic nerve and originates at the superior aspect of
16
the popliteal fossa . It then traces the lateral aspect of the fossa, crosses the head of the fibula, and comes
17
18 into direct contact w ith the lateral neck of the fibula as it enters the leg. At this point it divides into the deep
19 peroneal nerve and superficial peroneal nerve. The deep peroneal innervates the extensor and the great
dorsiflexors. The superficial peroneal branch supplies the peroneal muscles and the skin of most of the toes.
20
The common peroneal nerve is particularly susceptible to damage as it traces the lateral neck of the fibula .
21
Damage typically occurs via compression or leg fracture. The result is inability to dorsiflex the foot.
22
23 (Choice B) The femoral nerve (L2-L4) is the largest nerve in the lumbar plexus and innervates the muscles of
24
the anterior and medial thigh . Damage to the femoral nerve w ill result in loss of knee jerk.
25
26 (Choice C) The tibial nerve is the larger of the two terminal branches of the sciatic nerve and it provides
27 motor innervation for the popliteus and the flexors of the foot. Damage of this nerve w ould yield difficulty w ith
28 plantar flexion .
29
30 (Choice D) The radial nerve innervates the upper extremity and damage results in wrist drop.
31
32 (Choice E) The pudendal nerve provides the majority of the motor and sensory input to the pelvic floor.
33
34 (Choice F) The lateral sural cutaneous is a cutaneous branch of the tibial nerve and supplies sensation to the
35 calf.

Educational Objective:
37
Common peroneal nerve injury as it traces the lateral aspect of the fibular neck is common . Bony fracture
38
and compression are the most common causes. Clinically this manifests as foot drop.
39
40
41
42
~imeRemaini~ ~ 0
I Tuto' Feedback Suspend End Block
---------------------------------------------------------------------------------------------------------------------------------------------
1
2
" Item: 37 of 46
Q.ld: 1682 [
111 f> Mark .<:J
Previous
[::>
Next
il
Lab Values
~
Notes
~
Calculator
3
4 A 34-year-old female presents to the emergency room after falling in the bathroom . She complains of w rist
5 pain and numbness in her right palm. Based on her symptoms, you suspect that she has dislocated the
6 lunate bone in the w rist. Identify the lunate bone on the X-ray below .
7
8
9
10
11
12
13
14
15
16
17
18
19
20
21
22
23
24
25
26
27
28
29
30
31
32
33
34 0 A. A
35
0 B. B
36
0 C. C
38 0 D. D
39
40
0 E. E
41
42
~imeRemaini~ ~ 0
I Tuto' Feedback Suspend End Block
---------------------------------------------------------------------------------------------------------------------------------------------
1
2
3
4
5
6
7
8
9
10
11 0 A.A [7%]
12
0 B. B [1 3%]
13
14 0 C. C [31%]
15 -' @ D. D [41 %]
16
0 E. E [8%]
17
18
19 Explanation: User ld:
20
21 A fall on an outstretched, dorsiflexed hand is the classic injury that causes lunate dislocation. The lunate
22 (Choice D) is one of the proximal carpal bones. It can be identified on an X-ray of the hand as the more
23 medial of the two bones that articulate with the radius. The scaphoid is the other (more lateral) bone that
24 articulates with the radius (Choice C). Injuries that cause lunate dislocation also often cause fracture of the
25 scaphoid. A potential late sequela of such an injury is avascular necrosis of the proximal scaphoid fracture
26 fragment.
27
28 (Choice A) The trapezium is the most lateral of the distal carpal bones. Recall that the "thumb swings on the
29 trapezium ."
30
31 (Choice B) The capitate bone lies in the center of the wrist.
32
(Choice E) The triquetrum bone is located in the proximal medial wrist and together with the lunate forms the
33
small articular surface between the distal ulna and the carpal bones.
34
35 Educational Objective:
36 The lunate bone can be identified on a hand X-ray as the more medial of the two carpal bones that articulate
with the radius. It lies immediately medial to the scaphoid bone.
38
39
Time Spent: 4 seconds Copyright USMLEWorld,LLC. Last updated: [7/7/201 OJ
40
41
42
~imeRemaini~ ~ 0
I Tuto' Feedback Suspend End Block
--------------------------------------------------------------------------------------------------------------------------------------
1
2
" Item: 38 of 46
Q.ld: 1829 [
111 f> Mark .<:J
Previous
[::>
Next
il
Lab Values
~
Notes
~
Calculator
3
4
5
6 A 12-year-old boy falls from a tree he is climbing, but catches himself on a branch w ith his right hand. He
7 sw ings by his right arm and jumps to the ground. Several hours later he presents to the emergency room
8 w ith right hand clumsiness. W hich of the follow ing structures has he most likely injured?
9
10
11 0 A. Axillary nerve
12
0 B. Radial nerve
13
14 0 C. Musculocutaneous nerve
15
16 0 D. Long thoracic nerve
17 0 E. Brachial plexus, low er trunk
18
19
20
21
22
23
24
25
26
27
28
29
30
31
32
33
34
35
36
37

39
40

~
41
42 F!!ck suW.nd
---------------------------------------------------------------------------------------------------------------------------------------------
EnQck
1
2
3
4
5
6
7 Explanation: User ld:
8
9
10
11
12
13
14
15
16
17
18
19
20
21
22
23
24
25 t
Long thoracic nerve
26
27
28
29
30 Medial cutaneous nerve of the arm
31
32
33 Medial cutaneous nerve of the forearm
34
35 Of the choices listed, only injury to the low er trunk of the brachial plexus would cause hand weakness.
36 Sudden upw ard jerking of the arm at the shoulder, as occurred in this patient, can cause injury to the low er
37 trunk of the brachial plexus. The lower trunk of the brachial plexus carries nerve fibers from the CS and T1
spinal levels that ultimately contribute to the median and ulnar nerves. Together, these nerves innervate all of
39 the intrinsic muscles of the hand. Thus injury to the low er trunk of the brachial plexus can cause hand
40 clumsiness or oaralvsis.

~
41
42 F!!ck suW.nd EnQck
---------------------------------------------------------------------------------------------------------------------------------------------
1
2
3
4
5
6
7
8
9
10
11
12 Me<
dial cutaneous nerve of the forearm
13
14 Of the choices listed, only injury to the low er trunk of the brachial plexus w ould cause hand w eakness.
15 Sudden upw ard jerking of the arm at the shoulder, as occurred in this patient, can cause injury to the low er
16 trunk of the brachial plexus. The low er trunk of the brachial plexus carries nerve fibers from the CS and T1
17 spinal levels that ultimately contribute to the median and ulnar nerves. Together, these nerves innervate all of
18 the intrinsic muscles of the hand. Thus injury to the low er trunk of the brachial plexus can cause hand
19 clumsiness or paralysis.
20
21 (Choice A) Shoulder dislocations or fractures of the proximal humerus can cause axillary nerve injury.
22 W eakness of the deltoid and teres minor muscles may result.
23
24 (Choice B) Fractures at the midshaft of the humerus commonly cause radial nerve injury. Patients may
25 experience posterior arm numbness and paralysis of the arm and forearm ex1ensors.
26
27 (Choice C) The musculocutaneous nerve may be damaged in injuries involving the upper trunk of the brachial
28 plexus. The upper trunk of the brachial plexus is classically injured w hen the head and shoulder are violently
29 stretched apart. The musculocutaneous nerve innervates the biceps brachii and brachialis muscles, so injury
30 to this nerve w ould cause w eakness of forearm flexion at the elbow .
31
(Choice D) Long thoracic nerve injury classically occurs during axillary lymph node dissection and results in
32
paralysis of the serratus anterior muscle leading to scapular "w inging" on physical examination.
33
34 Educational Objective:
35 Sudden upw ard stretching on the arm at the shoulder can damage the low er trunk of the brachial plexus. This
36 trunk carries nerves from the CS and T 1 spinal levels that ultimately form the median and ulnar nerves. These
37 nerves innervate all of the intrinsic muscles of the hand.

39
Time Spent: 3 seconds Copyright USMLEW orld,LLC. Last updated: [1 / 17/2013]
40

~
41
42 F!!ck suW.nd EnQck
---------------------------------------------------------------------------------------------------------------------------------------------
1
2
" Item: 39 of 46
Q.ld: 1640 [
111 f> Mark .<:J
Previous
[::>
Next
il
Lab Values
~
Notes
~
Calculator
3
4
5
6 A 43-year-old male received a deep intramuscular injection tw o days ago and now presents w ith difficulty
7 w alking . You note that his right hip drops every time he raises his right foot. At w hich location did this patient
8 most likely receive his injection?
9
10
11 0 A. Superomedial quadrant of the buttock
12 0 B. Superolateral quadrant of the buttock
13
14 0 C. lnferomedial quadrant of the buttock
15 0 D. lnferolateral quadrant of the buttock
16
17 0 E. Posterior thigh
18
19
20
21
22
23
24
25
26
27
28
29
30
31
32
33
34
35
36
37
38

40
41
42
~imeRemaini~ ~ 0
I Tuto' Feedback Suspend End Block
---------------------------------------------------------------------------------------------------------------------------------------------
1
2
" Item: 39 of 46
Q.ld: 1640 [
111 f> Mark .<:J
Previous
[::>
Next
il
Lab Values
~
Notes
~
Calculator
3
4 .; @ A. Superomedial quadrant of the buttock [43%]
5 0 B. Superolateral quadrant of the buttock [19%]
6
7 0 C. lnferomedial quadrant of the buttock [23%]
8 0 D. lnferolateral quadrant of the buttock [12%]
9 0 E. Posterior thigh [3%]
10
11
12 Explanation: User ld:
13
14
15
16
17
18
19
20
21 rve
22
23
24
25
26
27
28
29
30
31
32
)\
33
34
Sciatic nerve
35
36 All injections into the gluteal region should target the superolateral quadrant, to avoid damage to the gluteal
37 nerves and sciatic nerve. This patient is exhibiting the classic "gluteus medius gait," i.e. the hip dips
38 dow nw ard w hen the ipsilateral foot is lifted off of the ground. This gait is observed w hen there is injury to the
superior gluteal nerve or to the gluteus medius muscle itself. The superior gluteal nerve is derived from the
40 L4-S1 ventral rami and leaves the pelvis through the greater sciatic foramen above the level of the piriformis.
41
42
~imeRemaini~ ~ 0
I Tuto' Feedback Suspend End Block
---------------------------------------------------------------------------------------------------------------------------------------------
1
2
3
4
5
6
7
8
9
10
11
12
13
14
15 Sciatic nerve
16
17 All injections into the gluteal region should target the superolateral quadrant, to avoid damage to the gluteal
18 nerves and sciatic nerve. This patient is exhibiting the classic "gluteus medius gait," i.e. the hip dips
19 dow nw ard w hen the ipsilateral foot is lifted off of the ground. This gait is observed w hen there is injury to the
20 superior gluteal nerve or to the gluteus medius muscle itself. The superior gluteal nerve is derived from the
21 L4-S1 ventral rami and leaves the pelvis through the greater sciatic foramen above the level of the piriformis.
22 Injection in the superomedial quadrant of the buttock is likely to injure this nerve.
23
24 (Choice B) The superolateral quadrant of the buttock is a safe site for intramuscular injections.
25
26 (Choice C) The sciatic nerve courses through the inferomedial quadrant of the buttock, thus nerve damage
27 can occur w ith injections to this site.
28
29 (Choice D) The inferolateral quadrant of the buttock is the site w here many of the muscles of the gluteal
30 region have their tendinous insertions.
31
(Choice E) The sciatic nerve courses just below the biceps femoris muscle in the posterior thigh . The sciatic
32
nerve innervates all of the knee flexors (hamstrings) w ith the exception of the short head of the biceps femoris.
33
34 Educational Objective:
35 Injections given in the superomedial part of the buttock risk injury to the gluteal nerves. Injections given in the
36 inferomedial part of the buttock risk injury to the sciatic nerve. The superolateral quadrant of the buttock is a
37 safe site for intramuscular injections.
38
Time Spent: 1 seconds Copyright USMLEW orld,LLC. Last updated: [7/7/201 0]
40
41
42
~imeRemaini~ ~ 0
I Tuto' Feedback Suspend End Block
---------------------------------------------------------------------------------------------------------------------------------------------
1
2
" Item: 40 of 46
Q.ld: 1520 [
111 f> Mark .<:J
Previous
[::>
Next
il
Lab Values
~
Notes
~
Calculator
3
4
5
6 A 65-year-old male presents to your office complaining of hearing loss. An audiogram show s impaired
7 hearing of low -frequency sounds. Sensory cells close to w hich of the follow ing structures are most likely
8 affected in this patient?
9
10
11 0 A. Cochlear base
12 0 B. Oval w indow
13
14 0 C. Round w indow
15 0 D. Helicotrema
16
17 0 E. Stapes
18
19
20
21
22
23
24
25
26
27
28
29
30
31
32
33
34
35
36
37
38
39

41
42
~imeRemaini~ ~ 0
I Tuto' Feedback Suspend End Block
---------------------------------------------------------------------------------------------------------------------------------------------
1
2
" Item: 40 of 46
Q.ld: 1520 [
111 f> Mark .<:J
Previous
[::>
Next
il
Lab Values
~
Notes
~
Calculator
3
4 ~ @ D. Helicotrema [27%]
5
6
0 E. Stapes [9%]
7
8 Explanation: User ld:
9
10 Oval window Scala vestibuli
11
12
13
14
15
16
17 Round window
Basilar membrane Scala tympani Helicotrema
18
19
20
21
22
23 ,,
24 I \
High frequency
25
26
27
28
29
30
31 ,--- ............
32
33
34
Low frequency
--
35
36 The inner ear is a fluid-filled site encased in bone that houses the cochlea, the semicircular canals, and the
37 vestibule. The cochlea is a cone-shaped spiraling structure. The base of the cochlea is the widest section of
38 the cone, and it lies near the oval and round window s. The apex of the cochlea is the tip of the cone, which
39 lies near the helicotrema. The cochlea is composed of three spiraling fluid-filled ducts known as the scala
vestibuli, the scala media, and the scala tympani. The scala media lies between the scala vestibuli and the
41
42
~imeRemaini~ ~ 0
I Tuto' Feedback Suspend End Block
---------------------------------------------------------------------------------------------------------------------------------------------
1
2
" Item: 40 of 46
Q.ld: 1520 [
111 f> Mark .<:J
Previous
[::>
Next
il
Lab Values
~
Notes
~
Calculator
3
4
5
6 The inner ear is a fluid-filled site encased in bone that houses the cochlea, the semicircular canals, and the
7 vestibule. The cochlea is a cone-shaped spiraling structure. The base of the cochlea is the w idest section of
8 the cone, and it lies near the oval and round w indow s. The apex of the cochlea is the tip of the cone, w hich
9 lies near the helicotrema. The cochlea is composed of three spiraling fluid-filled ducts know n as the scala
10 vestibuli, the scala media, and the scala tympani. The scala media lies between the scala vestibuli and the
11 scala tympani. Both the scala vestibuli and the scala tympani are filled w ith perilymph, w hich has a high
12 sodium concentration, w hile the scala media is filled w ith endolymph having a high potassium concentration .
13 The scala media is separated from the scala tympani by the basilar membrane. The scala media is also
14 unique in that it houses the tectorial membrane and the organ of Corti. The organ of Corti is the specific site
15 w here sound is transduced into the nervous system .
16
17 The transduction of mechanical auditory forces into nervous impulses occurs in the organ of Corti by the
18 follow ing steps. First, sound reaches the middle ear by vibrating the tympanic membrane. This vibration is
19 transferred to the oval w indow by the ossicles. Vibration of the oval w indow causes movement of the
20 perilymph in the scala vestibuli, w hich is transmitted to the scala tympani. Perilymph movement in the scala
21 tympani, in turn, causes the flexible basilar membrane to vibrate. Vibration of the basilar membrane causes
22 bending of the cilia of the hair cells as fluid runs past the rigid tectorial membrane. Hair cell bending causes
23 oscillating hyperpolarization and depolarization, thus creating nervous impulses from sound. At the base of
24 the cochlea, the basilar membrane is thin and rigid and best responds to high frequency sound. The basilar
25 membrane at the apex of the cochlea, near the helicotrema, is large and flexible, so it best responds to low
26 frequency sounds. The helicotrema is the site w here the scala vestibuli and the scala tympani meet.
27
28 (Choices A, B and C) The cochlear base lies near the round and oval w indow s; this segment of the cochlea
29 best responds to high-frequency sound. Older adults do suffer from high-frequency hearing loss much more
30 often than low -frequency loss. The oval w indow transmits vibration to the scala vestibuli, w hich
31 communicates w ith the scala tympani at the helicotrema, w hich then transmits vibration to the round w indow .
32 As the oval w indow moves inw ard, the round w indow moves outward.
33
(Choice E) The stapes is one of the ossicles of the middle ear. The other two are the malleus and incus.
34
35 Educational Objective:
36 Low -frequency sound is best detected at the apex of the cochlea near the helicotrema. High-frequency sound
37 is best detected at the base of the cochlea near the oval and round w indow s.
38
39
Time Spent: 1 seconds Copyright USMLEW orld,LLC. Last updated: [11 / 10/2011]
41
42
~imeRemaini~ ~ 0
I Tuto' Feedback Suspend End Block
---------------------------------------------------------------------------------------------------------------------------------------------
1
2
" Item: 41 of 46
Q.ld: 1638 [
111 f> Mark .<:J
Previous
[::>
Next
il
Lab Values
~
Notes
~
Calculator
3
4
5
6 A 34-year-old male has difficulty w alking after sustaining a traumatic injury to his right leg. Physical
7 examination reveals a right foot that is dorsiflexed and everted. The patient is unable to stand on his tiptoes.
8 W hat is the most likely area of sensory loss in this patient?
9
10
11 0 A. Anterior thigh
12 0 B. Medial leg
13
14 0 C. Medial foot
15 0 D. Dorsal foot
16
17 0 E. Plantar foot
18
19
20
21
22
23
24
25
26
27
28
29
30
31
32
33
34
35
36
37
38
39
40

42
~imeRemaini~ ~ 0
I Tuto' Feedback Suspend End Block
---------------------------------------------------------------------------------------------------------------------------------------------
1
2
" Item: 41 of 46
Q.ld: 1638 [
111 f> Mark .<:J
Previous
[::>
Next
il
Lab Values
~
Notes
~
Calculator
3
4 I .; @ E. Plantar foot[55%]
5
6
7 Explanation: User ld:
8
Sciatic nerve
9
10
11
12
13 Common
+-- peroneal
14 nerve
15
16 Saphenous nerve
17 (blanch of femoral nerve)
18
19 Tibial
20 nerve
21
22
23
24
25
26
27
28 Deep peroneal nerve
29
30 The tibial nerve is the large medial branch of the sciatic nerve that descends through the popliteal fossa
31 together w ith the popliteal vein and artery. Injury to this nerve can occur after deep penetrating trauma to the
32 popliteal fossa and can cause a number of different deficits due to its numerous functions. The tibial nerve
33 innervates the gastrocnemius, soleus and plantaris muscles, w hich are responsible for plantar flexion of the
34 foot. It also supplies the flexor digitorum longus and flexor hallucis longus, w hich are responsible for toe
35 flexion. The tibial nerve also innervates the tibialis posterior muscle, w hich is responsible for inversion of the
36 foot. (Patients w ith injury to the tibial nerve often hold their lower extremity in a calcaneovalgocavus position-
37 the foot is dorsiflexed and everted .) After delivering its motor innervation, the tibial nerve terminates by dividing
38 into the medial and lateral plantar nerves to provide sensory innervation to the skin of the distal plantar surface
39 of the foot.
40

42
~imeRemaini~ ~ 0
I Tutor Feedback Suspend End Block
---------------------------------------------------------------------------------------------------------------------------------------------
1
2
3
4
5
6
7
8
9
10
11 Deep peroneal nerve
12
13 The tibial nerve is the large medial branch of the sciatic nerve that descends through the popliteal fossa
14 together w ith the popliteal vein and artery. Injury to this nerve can occur after deep penetrating trauma to the
15 popliteal fossa and can cause a number of different deficits due to its numerous functions. The tibial nerve
16 innervates the gastrocnemius, soleus and plantaris muscles, w hich are responsible for plantar flexion of the
17 foot. It also supplies the flexor digitorum longus and flexor hallucis longus, w hich are responsible for toe
18 flexion . The tibial nerve also innervates the tibialis posterior muscle, which is responsible for inversion of the
19 foot. (Patients w ith injury to the tibial nerve often hold their low er extremity in a calcaneovalgocavus position-
20 the foot is dorsiflexed and everted.) After delivering its motor innervation, the tibial nerve terminates by dividing
21 into the medial and lateral plantar nerves to provide sensory innervation to the skin of the distal plantar surface
22 of the foot.
23
24 (Choice A) The cutaneous branch of the femoral nerve innervates the majority of the skin of the anterior thigh .
25
26 (Choice B) Cutaneous branches of the saphenous nerve innervate the skin of the medial leg. The
27 saphenous nerve is a branch of the femoral nerve and courses to the medial leg together w ith the great
28 saphenous vein .
29
30 (Choice C) The medial aspect of the foot is innervated by the superficial peroneal nerve, a branch of the
31 common peroneal nerve, as w ell as by the saphenous nerve.
32
(Choice D) The superficial peroneal nerve innervates the dorsum of the foot, except for the skin between the
33
great toe and the second toe, w hich is innervated by the deep peroneal nerve.
34
35 Educational Objective:
36 The tibial nerve innervates the flexors of the low er leg, the extrinsic digital flexors of the toes, and the skin of
37 the sole of the foot.
38
39
Time Spent: 1 seconds Copyright USMLEW orld,LLC. Last updated: [1 2119/2012]
40

42
~imeRemaini~ ~ 0
I Tuto' Feedback Suspend End Block
---------------------------------------------------------------------------------------------------------------------------------------------
1
2
" Item: 42 of 46
Q.ld: 1633 [
111 f> Mark .<:J
Previous
[::>
Next
il
Lab Values
~
Notes
~
Calculator
3
4
5
6 A 34-year-old male presents to your office complaining of decreased hearing in both ears. He is a rock
7 musician and has been spending a lot of time in his studio recently. W here is the most likely location of ear
8 injury in this patient?
9
10
11 0 A. Tympanic membrane
12 0 B. Middle ear ossicles
13
14 0 C. Round w indow
15 0 D. Organ of Corti
16
17 0 E. Cupula
18
0 F. Auditory nerve
19
20
21
22
23
24
25
26
27
28
29
30
31
32
33
34
35
36
37
38
39
40
41 ~imeRemaini~ ~ 0
I Tuto' Feedback Suspend End Block
---------------------------------------------------------------------------------------------------------------------------------------------
1
2
" Item: 42 of 46
Q.ld: 1633 [
111 f> Mark .<:J
Previous
[::>
Next
il
Lab Values
~
Notes
~
Calculator
3
4 .; @ D. Organ of Corti [52%]
5 0 E. Cupula [3%]
6
7 0 F. Auditory nerve [5%]
8
9
Expla nation: User ld:
10
11 Transduction of mechanical auditory forces into nerve impulses occurs in the organ of Corti by the follow ing
12 steps: 1) Sound reaches the middle ear by vibrating the tympanic membrane. 2) This vibration is transferred
13 to the oval w indow by the ossicles. 3) Vibration of the oval w indow causes vibration of the basilar membrane,
14 w hich in turn causes bending of the hair cell cilia against the tectorial membrane. 4) Hair cell bending causes
15 oscillating hyperpolarization and depolarization of the auditory nerve, thus creating nerve impulses from
16 sound.
17
18 Noise-induced hearing loss results from trauma to the stereociliated hair cells of the organ of Corti. Prolonged
19 exposure to noises greater than 85 dB can cause distortion or fracture of the stereocilia due to shearing
20 forces against the tectorial membrane. High-frequency hearing is lost first, regardless of the frequency of the
21 sound causing the damage. Initial damage causes loss of hearing at frequencies of approximately 4000 Hz
22 (speech frequencies = 500-3000 Hz). The acoustic reflex dampens the effects of prolonged loud noise by
23 causing the stapedius and tensor tympani muscles to contract, lessening the responsiveness of the ossicles
24 to sound.
25
26 (Choice A) Perforation or rupture of the tympanic membrane can occur w ith infection, trauma, pressure
27 changes w hen the eustachian tube is blocked, or sudden very loud noises. Rupture of the tympanic
28 membrane causes conductive hearing loss.
29
30 (Choice B) Defects of the middle ear ossicles such as otosclerosis can cause conductive hearing loss.
31
32 (Choice C) The round w indow lies betw een the middle and inner ear and moves outward w hen the stapes
33 causes the oval w indow to move inw ard and vice versa .
34
(Choice E) The cupula lies at the apex of the cochlea and houses the cells that sense rotation .
35
36 (Choice F) The auditory nerve transmits sound impulses to the brainstem via the S'h cranial nerve. Defects in
37
this nerve can result from trauma, mass effect, demyelination, or local inflammatory processes, and can
38 cause nerve-based hearing loss.
39
40 e..J .-- .:-- -' 1"\t..:.--: -

~
41
F!!ck
---------------------------------------------------------------------------------------------------------------------------------------------
suW.nd EnQck
1
2
" Item: 42 of 46
Q.ld: 1633 [
111 f> Mark .<:J
Previous
[::>
Next
il
Lab Values
~
Notes
~
Calculator
3
4
5 Explanation: User ld:
6 Transduction of mechanical auditory forces into nerve impulses occurs in the organ of Corti by the follow ing
7 steps: 1) Sound reaches the middle ear by vibrating the tympanic membrane. 2) This vibration is transferred
8 to the oval w indow by the ossicles. 3) Vibration of the oval w indow causes vibration of the basilar membrane,
9 w hich in turn causes bending of the hair cell cilia against the tectorial membrane. 4) Hair cell bending causes
10
oscillating hyperpolarization and depolarization of the auditory nerve, thus creating nerve impulses from
11
sound.
12
13 Noise-induced hearing loss results from trauma to the stereociliated hair cells of the organ of Corti. Prolonged
14 exposure to noises greater than 85 dB can cause distortion or fracture of the stereocilia due to shearing
15 forces against the tectorial membrane. High-frequency hearing is lost first, regardless of the frequency of the
16 sound causing the damage. Initial damage causes loss of hearing at frequencies of approximately 4000 Hz
17 (speech frequencies = 500-3000 Hz). The acoustic reflex dampens the effects of prolonged loud noise by
18 causing the stapedius and tensor tympani muscles to contract, lessening the responsiveness of the ossicles
19 to sound.
20
21 (Choice A) Perforation or rupture of the tympanic membrane can occur w ith infection, trauma, pressure
22 changes w hen the eustachian tube is blocked, or sudden very loud noises. Rupture of the tympanic
23 membrane causes conductive hearing loss.
24
25 (Choice B) Defects of the middle ear ossicles such as otosclerosis can cause conductive hearing loss.
26
27 (Choice C) The round w indow lies betw een the middle and inner ear and moves outward w hen the stapes
28 causes the oval w indow to move inw ard and vice versa .
29
(Choice E) The cupula lies at the apex of the cochlea and houses the cells that sense rotation .
30
31
(Choice F) The auditory nerve transmits sound impulses to the brainstem via the S'h cranial nerve. Defects in
32
this nerve can result from trauma, mass effect, demyelination, or local inflammatory processes, and can
33
cause nerve-based hearing loss.
34
35 Educational Objective:
36 Prolonged exposure to loud noises causes hearing loss due to damage of the stereociliated hair cells of the
37 organ of Corti .
38
39
Time Spent: 1 seconds Copyright USMLEW orld,LLC. Last updated: [7/7/201 0]
40
41 ~imeRemaini~ ~ 0
I Tuto' Feedback Suspend End Block
---------------------------------------------------------------------------------------------------------------------------------------------
1
2
" Item: 43 of 46
Q.ld: 1634 [
111 f> Mark .<:J
Previous
[::>
Next
il
Lab Values
~
Notes
~
Calculator
3
4
5
6 A 19-year-old female presents to your office complaining of "right hand clumsiness." Physical examination
7 reveals decreased sensation over the fifth finger and a flattened hypothenar eminence. The affected nerve is
8 commonly injured at w hich of the follow ing locations?
9
10
11 0 A. Carpal tunnel
12 0 B. Hook of the hamate
13
14 0 C. Surgical neck of the humerus
15 0 D. Head of the radius
16
17 0 E. Coracobrachialis muscle
18
19
20
21
22
23
24
25
26
27
28
29
30
31
32
33
34
35
36
37
38
39
40
41
42
~imeRemaini~ ~ 0
I Tuto' Feedback Suspend End Block
---------------------------------------------------------------------------------------------------------------------------------------------
1
2
" Item: 43 of 46
Q.ld: 1634 [
111 f> Mark .<:J
Previous
[::>
Next
il
Lab Values
~
Notes
~
Calculator
3
4
.; @ B. Hook of the hamate [73%1
5
6 0 C. Surgical neck of the humerus [5%1
7 0 D. Head of the radius [8% 1
8
0 E. Coracobrachialis muscle [3%1
9
10
11 Explanation: User ld:
12
13 The ulnar nerve is a branch of the medial cord of the brachial plexus derived from the C8-T1 ventral rami . The
14 ulnar nerve provides sensory innervation to the fifth digit and the medial half of the fourth digit as w ell as to the
15 palmar and dorsal surfaces of the hand. The ulnar nerve also provides motor innervation to the flexor carpi
16 ulnaris and to the medial section of the flexor digitorum profundis in the forearm. In the hand, the ulnar nerve
17 innervates all of the palmar and dorsal interosseus muscles, the muscles of the hypothenar eminence, the
18 palmaris brevis muscle, the third and fourth lumbrical muscles, and the adductor pollicis muscle.
19
20 The ulnar nerve enters the forearm after passing behind the medial epicondyle of the humerus w here it is
21 covered by a small amount of overlying soft tissue. This region, sometimes referred to as the "funny bone," is
22 a common site of ulnar nerve injury. Ulnar nerve injury at this site classically causes a "claw hand" deformity.
23 In the w rist, the ulnar nerve passes between the hook of the hamate and the pisiform bone in a fibroosseous
24 tunnel know n as Guyon's canal. Ulnar nerve injury at Guyon's canal causes dysesthesia of the ulnar side of
25 the hand and w eakness of the intrinsic muscles of the hand.
26
27 (Choice A) In carpal tunnel syndrome, some factor reduces the size of the carpal tunnel causing median
28 nerve compression. Patients typically experience difficulty w ith fine motor control of the thumb.
29
(Choice C) Fracture of the surgical neck of the humerus may cause axillary nerve injury leading to paralysis
30
of the deltoid and teres minor muscles as w ell as loss of sensation of the lateral upper arm.
31
32 (Choice D) The deep branch of the radial nerve can be affected by radial head subluxation leading to
33 w eakness or paralysis of many of the muscles of the extensor compartment of the forearm .
34
35 (Choice E) The coracobrachialis muscle is an arm flexor that lies deep to the biceps brachii and overlies the
36 median nerve and brachial artery. It is innervated by the musculocutaneous nerve.
37
38 Educational Objective:
39 Ulnar nerve injury classically causes a "claw hand" deformity. The ulnar nerve can be injured either near the
40 mprli::.l pnir nnrlviP nf thP h11mpn 1c::: n r in r, vnn'c::: r :::m:::.l nP::~r thp hnnk nf thP h:::.m:::.tp :::.nrl nic;:;ifnrm hnnp in thP

~
41
42
---------------------------------------------------------------------------------------------------------------------------------------------
F!!ck suW.nd EnQck
1
2
3
4
5 0 E. Coracobrachialis muscle [3%]
6
7
Explanation: User ld:
8
9 The ulnar nerve is a branch of the medial cord of the brachial plexus derived from the C8-T1 ventral rami. The
10 ulnar nerve provides sensory innervation to the fifth digit and the medial half of the fourth digit as w ell as to the
11 palmar and dorsal surfaces of the hand. The ulnar nerve also provides motor innervation to the flexor carpi
12 ulnaris and to the medial section of the flexor digitorum profundis in the forearm . In the hand, the ulnar nerve
13 innervates all of the palmar and dorsal interosseus muscles, the muscles of the hypothenar eminence, the
14
palmaris brevis muscle, the third and fourth lumbrical muscles, and the adductor pollicis muscle.
15
16 The ulnar nerve enters the forearm after passing behind the medial epicondyle of the humerus w here it is
17 covered by a small amount of overlying soft tissue. This region, sometimes referred to as the "funny bone," is
18 a common site of ulnar nerve injury. Ulnar nerve injury at this site classically causes a "claw hand" deformity.
19 In the w rist, the ulnar nerve passes between the hook of the hamate and the pisiform bone in a fibroosseous
20 tunnel know n as Guyon's canal. Ulnar nerve injury at Guyon's canal causes dysesthesia of the ulnar side of
21 the hand and w eakness of the intrinsic muscles of the hand.
22
23 (Choice A) In carpal tunnel syndrome, some factor reduces the size of the carpal tunnel causing median
24 nerve compression. Patients typically experience difficulty w ith fine motor control of the thumb.
25
26 (Choice C) Fracture of the surgical neck of the humerus may cause axillary nerve injury leading to paralysis
27 of the deltoid and teres minor muscles as w ell as loss of sensation of the lateral upper arm.
28
29 (Choice D) The deep branch of the radial nerve can be affected by radial head subluxation leading to
30 w eakness or paralysis of many of the muscles of the ex1ensor compartment of the forearm.
31
(Choice E) The coracobrachialis muscle is an arm flexor that lies deep to the biceps brachii and overlies the
32
median nerve and brachial artery. It is innervated by the musculocutaneous nerve.
33
34 Educational Objective:
35 Ulnar nerve injury classically causes a "claw hand" deformity. The ulnar nerve can be injured either near the
36 medial epicondyle of the humerus or in Guyon's canal near the hook of the hamate and pisiform bone in the
37 w rist.
38
39
Time Spent: 1 seconds Copyright USMLEW orld,LLC. Last updated: [7/ 17/2012]
40
41
42
~imeRemaini~ ~ 0
I Tuto' Feedback Suspend End Block
---------------------------------------------------------------------------------------------------------------------------------------------
1
2
" Item: 44 of 46
Q.ld: 1813 [
111 f> Mark .<:J
Previous
[::>
Next
il
Lab Values
~
Notes
~
Calculator
3
4
5
6 A 23-year-old Caucasian female is brought to the ER unconscious after a motor vehicle accident. A linear
7 skull fracture in the area of junction of frontal, parietal, temporal, and sphenoid bones is seen on X-ray. A
8 branch of w hich of the follow ing arteries is most likely severed in this patient?
9
10
11 0 A. Occipital
12 0 B. Maxillary
13
14 0 C. Ophthalmic
15 0 D. Middle cerebral
16
17 0 E. Sphenopalatine
18
0 F. Facial
19
20
21
22
23
24
25
26
27
28
29
30
31
32
33
34
35
36
37
38
39
40
41
42
~imeRemaini~ ~ 0
I Tuto' Feedback Suspend End Block
---------------------------------------------------------------------------------------------------------------------------------------------
1
2
" Item: 44 of 46
Q.ld: 1813 [
111 f> Mark .<:J
Previous
[::>
Next
il
Lab Values
~
Notes
~
Calculator
3
4 v @ B. Maxillary [38%1
5
6 0 C. Ophthalmic [9%1
7 0 D. Middle cerebral [35%1
8 0 E. Sphenopalatine [6%1
9
10 0 F. Facial [7%1
11
12 Explanation: User ld:
13
14 The patient described in the question stem has suffered a fracture w here the frontal, parietal, temporal, and
15 sphenoid bones meet in the skull. This site is known as the pterion and is medically relevant due to the fact
16 that the bone is thin in this region and the middle meningeal artery courses w ithin the skull deep to this
17 point. As such, fractures to this region of the skull are at risk of damaging the middle meningeal artery and
18 causing an epidural hematoma. Epidural hematomas require prompt treatment because they are under
19 systemic arterial pressure and can expand rapidly leading to transtentorial herniation and palsy of the
20 oculomotor nerve.
21
22 The middle meningeal artery is a branch of the maxillary artery, w hich itself is a branch of the external carotid
23 artery.
24
25 (Choice A) The occipital artery arises opposite the facial artery from the external carotid, coursing posteriorly
26 instead of anteriorly. This vessel serves the posterior scalp and the sternocleidomastoids.
27
28 (Choice C) The ophthalmic artery is the first branch of the internal carotid artery, serving the eye and orbital
29 contents as w ell as the eyelids, forehead, nose, and nasal mucosa. The internal carotid artery has no
30 branches in the neck.
31
(Choice D) The middle cerebral artery is a branch of the internal carotid artery and supplies much of the
32
parietal and temporal regions of the brain . Injury to this artery or its branches can cause a subarachnoid or
33
intracerebral hemorrhage.
34
35 (Choice E) The sphenopalatine artery is a branch of the third part of the maxillary artery that supplies much of
36 the nasal mucosa. It anastomoses w ith branches of the ophthalmic and facial arteries w ithin the anterior part
37 of the nasal septum in a region known as Kiesselbach's plexus- a frequent site of nosebleeds.
38
39 (Choice F) The facial artery is a branch of the external carotid artery that courses over the mandible anterior
40 t " tho inC"'orl-il"'n nf tho I"''"''':IC'C"'Otar- 1"'1"'11 IC'I" lo t" C'l nnlu tho nr-:.1 ron inn -:.nrl nnC'o <:~C' uroll ':IC' th o h II" r -:.1 ron inn nf tho

~
41
42
---------------------------------------------------------------------------------------------------------------------------------------------
F!!ck suW.nd EnQck
1
2
" Item: 44 of 46
Q.ld: 1813 [
111 f> Mark .<:J
Previous
[::>
Next
il
Lab Values
~
Notes
~
Calculator
3
4 cxp1anat1on:
5
The patient described in the question stem has suffered a fracture w here the frontal, parietal, temporal, and
6
sphenoid bones meet in the skull. This site is know n as the pterion and is medically relevant due to the fact
7
that the bone is thin in this region and the middle meningeal artery courses w ithin the skull deep to this
8
point. As such, fractures to this region of the skull are at risk of damaging the middle meningeal artery and
9
causing an epidural hematoma. Epidural hematomas require prompt treatment because they are under
10
systemic arterial pressure and can expand rapidly leading to transtentorial herniation and palsy of the
11
oculomotor nerve.
12
13 The middle meningeal artery is a branch of the maxillary artery, w hich itself is a branch of the external carotid
14
artery.
15
16 (Choice A) The occipital artery arises opposite the facial artery from the external carotid, coursing posteriorly
17 instead of anteriorly. This vessel serves the posterior scalp and the sternocleidomastoids.
18
19 (Choice C) The ophthalmic artery is the first branch of the internal carotid artery, serving the eye and orbital
20 contents as w ell as the eyelids, forehead, nose, and nasal mucosa. The internal carotid artery has no
21 branches in the neck.
22
23 (Choice D) The middle cerebral artery is a branch of the internal carotid artery and supplies much of the
24 parietal and temporal regions of the brain. Injury to this artery or its branches can cause a subarachnoid or
25 intracerebral hemorrhage.
26
27 (Choice E) The sphenopalatine artery is a branch of the third part of the maxillary artery that supplies much of
28 the nasal mucosa. It anastomoses w ith branches of the ophthalmic and facial arteries w ithin the anterior part
29 of the nasal septum in a region know n as Kiesselbach's plexus - a frequent site of nosebleeds.
30
(Choice F) The facial artery is a branch of the external carotid artery that courses over the mandible anterior
31
to the insertion of the masseter muscle to supply the oral region and nose as w ell as the buccal region of the
32
cheeks.
33
34 Educational Objective:
35 The middle meningeal artery is a branch of the maxillary artery that enters the skull at the foramen spinosum
36 and courses intracranially deep to the pterion . Skull fractures at this site may cause laceration of this vessel,
37 leading to an epidural hematoma.
38
39
Time Spent: 1 seconds Copyright USMLEW orld,LLC. Last updated: [1 / 10/2013]
40
41
42
~imeRemaini~ ~ 0
I Tuto' Feedback Suspend End Block
---------------------------------------------------------------------------------------------------------------------------------------------
1
2
" Item: 45 of 46
Q.ld: 1732 [
111 f> Mark .<:J
Previous
[::>
Next
il
Lab Values
~
Notes
~
Calculator
3
4
5
6 A 53-year-old man presents to his physician's office w ith a dull ache in his right shoulder. He states that his
7 pain is w orse w ith movement and that it often interferes w ith his sleep. The patient cannot recall any
8 traumatic events prior to the start of the pain, but does report that he experienced increased discomfort after
9 helping his daughter move into her college dorm room one w eek ago. His past medical history includes
10 dyslipidemia and hypertension, for w hich he takes the appropriate medications. Physical examination reveals
11 localized tenderness just below the acromion. The physician asks the patient to abduct his arms to 90
12 degrees and flex them to 30 degrees w ith his thumbs pointing to the floor. She then applies dow nward force
13 to his arms. This maneuver elicits pain in the patient's right shoulder and reveals right-sided w eakness as
14 compared to the left. A tendon of w hich of the follow ing muscles is most likely inflamed in this patient?
15
16
17 0 A. Biceps brachii
18 0 B. Deltoid
19
20 0 C. Levator scapulae
21 0 D. Serratus anterior
22
23 0 E. Supraspinatus
24
25
26
27
28
29
30
31
32
33
34
35
36
37
38
39
40
41
42
~imeRemaini~ ~ 0
I Tuto' Feedback Suspend End Block
---------------------------------------------------------------------------------------------------------------------------------------------
1
2
" Item: 45 of 46
Q.ld: 1732 [
111 f> Mark .<:J
Previous
[::>
Next
il
Lab Values
~
Notes
~
Calculator
3

5
4 I ol @ E. Supraspinatus [62%]
6
7 Explanation: User ld:
8
Acromioclavicular joint
9
10
11
12
13
14
15
lAcromion

Supraspinatus tendon

16 Subdeltoid
17 bursa
18
19
20
21
Deltoid
22
23 muscle
24
25
26
27
28
29
30
31
32
33 U$N.t,.EWorld.UC ($) l-011
34
35
The rotator cuff is made up of the tendons of the following muscles: supraspinatus, infraspinatus, teres minor,
36
and subscapularis (SITS). The tendons of these rotator cuff muscles, along with the glenohumeral joint
37
ligaments and the tendon of the long head of the biceps brachii muscle, contribute to the stability of the
38
glenohumeral joint. The supraspinatus muscle is the main agonistic muscle responsible for the first 10 to 15
39
40 degrees of arm abduction . Above 30 degrees, the deltoid muscle becomes the main propagator of further
41
42
~imeRemaini~ ~ 0
I Tuto' Feedback Suspend End Block
---------------------------------------------------------------------------------------------------------------------------------------------
1
2
" Item: 45 of 46
Q.ld: 1732 [
111 f> Mark .<:J
Previous
[::>
Next
il
Lab Values
~
Notes
~
Calculator
3
4
5 The rotator cuff is made up of the tendons of the follow ing muscles: supraspinatus, infraspinatus, teres minor,
6 and subscapularis (SITS). The tendons of these rotator cuff muscles, along w ith the glenohumeral joint
7 ligaments and the tendon of the long head of the biceps brachii muscle, contribute to the stability of the
8 glenohumeral joint. The supraspinatus muscle is the main agonistic muscle responsible for the first 10 to 15
9 degrees of arm abduction . Above 30 degrees, the deltoid muscle becomes the main propagator of further
10 abduction, w hile the supraspinatus muscle stabilizes the glenohumeral joint.
11
12 Of the rotator cuff structures, the tendon of the supraspinatus muscle is most commonly affected in rotator
13 cuff syndrome. Due to its superior location, this tendon is vulnerable to chronic repeated trauma from
14 impingement between the head of the humerus and the acromioclavicular joint. Motions that typically cause
15 inflammation of this tendon are simultaneous abduction and flexion of the arm at the shoulder, but a traumatic
16 fall laterally on an outstretched hand can also cause injury. Inflammation is follow ed by fibrosis, w hich
17 w orsens the problem by increasing friction between the head of the humerus and the acromion, as w ell as
18 causing inflammation of the subacromial bursa . The physical exam maneuver described in the question stem
19 is know n as the "empty-can supraspinatus test," and is 90% specific for supraspinatus pathology.
20
21 (Choice A) The tendon of the long head of the biceps brachii muscle passes through the glenohumeral joint
22 to insert on the supraglenoid tubercle of the scapula . The short head of the tendon inserts directly onto the
23 anterior tip of the coracoid process of the scapula .
24
25 (Choice B) The deltoid originates on the clavicle, the acromion, and the spine of the scapula as the anterior,
26 lateral, and posterior parts of this muscle, respectively. Together, these parts of the deltoid insert by a
27 common tendon on the lateral surface of the humerus.
28
(Choice C) The tendons of the levator scapulae insert on the superomedial border of the scapula and on the
29
transverse processes of the C 1 through C4 vertebrae. The levator scapulae does not contribute to the
30
stability of the glenohumeral joint.
31
32 (Choice D) The serratus anterior muscle courses from the medial border of the scapula to insert onto the
33 lateral surfaces of the first eight ribs. Injury to the long thoracic nerve causes paralysis of this muscle and
34 "w inging" of the scapula .
35
36 Educational objective:
37 The supraspinatus muscle assists in abduction of the arm and stabilization of the glenohumeral joint. The
38 most commonly injured structure in rotator cuff syndrome is the tendon of the supraspinatus muscle. This
39 tendon is vulnerable to injury due to impingement between the acromion and the head of the humerus.
40
41
42
~imeRemaini~ ~ 0
I Tuto' Feedback Suspend End Block
---------------------------------------------------------------------------------------------------------------------------------------------
1
2
" Item: 45 of 46
Q.ld: 1732 [
111 f> Mark .<:J
Previous
[::>
Next
il
Lab Values
~
Notes
~
Calculator
3
4 Of the rotator cuff structures, the tendon of the supraspinatus muscle is most commonly affected in rotator
5 cuff syndrome. Due to its superior location, this tendon is vulnerable to chronic repeated trauma from
6 impingement between the head of the humerus and the acromioclavicular joint. Motions that typically cause
7 inflammation of this tendon are simultaneous abduction and flexion of the arm at the shoulder, but a traumatic
8 fall laterally on an outstretched hand can also cause injury. Inflammation is follow ed by fibrosis, which
9 w orsens the problem by increasing friction between the head of the humerus and the acromion, as w ell as
10 causing inflammation of the subacromial bursa . The physical exam maneuver described in the question stem
11 is known as the "empty-can supraspinatus test," and is 90% specific for supraspinatus pathology.
12
13 (Choice A) The tendon of the long head of the biceps brachii muscle passes through the glenohumeral joint
14 to insert on the supraglenoid tubercle of the scapula . The short head of the tendon inserts directly onto the
15 anterior tip of the coracoid process of the scapula .
16
17 (Choice B) The deltoid originates on the clavicle, the acromion, and the spine of the scapula as the anterior,
18 lateral, and posterior parts of this muscle, respectively. Together, these parts of the deltoid insert by a
19 common tendon on the lateral surface of the humerus.
20
(Choice C) The tendons of the levator scapulae insert on the superomedial border of the scapula and on the
21
transverse processes of the C 1 through C4 vertebrae. The levator scapulae does not contribute to the
22
stability of the glenohumeral joint.
23
24 (Choice D) The serratus anterior muscle courses from the medial border of the scapula to insert onto the
25 lateral surfaces of the first eight ribs. Injury to the long thoracic nerve causes paralysis of this muscle and
26 "winging" of the scapula .
27
28 Educational objective:
29 The supraspinatus muscle assists in abduction of the arm and stabilization of the glenohumeral joint. The
30 most commonly injured structure in rotator cuff syndrome is the tendon of the supraspinatus muscle. This
31 tendon is vulnerable to injury due to impingement between the acromion and the head of the humerus.
32
33 References:
34
35 1. Chronic shoulder pain: part I. Evaluation and diagnosis.
36 2. Rotator cuff disorders: recognition and management among patients with shoulder pain.
37
38
39
Time Spent: 1 seconds Copyright USMLEW orld,LLC. Last updated: [2123/2014]
40
41
42
~imeRemaini~ ~ 0
I Tuto' Feedback Suspend End Block
---------------------------------------------------------------------------------------------------------------------------------------------
1
2
" Item: 46 of 46
Q.ld: 1956 [
111 f> Mark .<:J
Previous
[::>
Next
il
Lab Values
~
Notes
~
Calculator
3
4
5
6 A 65-year-old Caucasian female is brought to the emergency department w ith severe left-sided hip pain after
7 falling in the bathroom. She is agitated and demands quick pain relief. A pelvic x-ray is show n below.
8
9
10
11
12
13
14
15
16
17
18
19
20
21
22
23
24
25
26
27
28
29 W hich of the follow ing arteries is most likely to be damaged in this patient?
30
31 0 A. Femoral
32
33 0 B. Medial circumflex
34 0 C. Lateral circumflex
35 0 D. Obturator
36
37 0 E. Inferior gluteal
38
39
40
41
42
~imeRemaini~ ~ 0
I Tuto' Feedback Suspend End Block
---------------------------------------------------------------------------------------------------------------------------------------------
1
2
" Item: 46 of 46
Q.ld: 1956 [
111 f> Mark .<:J
Previous
[::>
Next
il
Lab Values
~
Notes
~
Calculator
3
4 ol @ B. Medial circumflex [49%]
5
6 0 C. Lateral circumflex [19%]
7 0 D. Obturator [12%]
8 0 E. Inferior gluteal [6%]
9
10
11 Explanation: User ld:
12
13 This patient's x-ray show s a nondisplaced left subcapital femoral neck fracture. Note below how the femoral
14 neck is markedly shortened compared to the right side. The dashed lines demarcate the intertrochanteric
15 line, and the red dots label the fracture plane.
16
17
18
19
20
21
22
23
24
25
26
27
28
29
30
31
32
33
34
35
36 Femoral neck fractures are common in elderly patients w ith osteoporosis w ho have sustained a fall. The
37 femoral head and neck derive their blood supply from the superior and inferior gluteal arteries and the medial
38 and lateral femoral circumflex arteries, vessels that together form the trochanteric anastomosis. The medial
39 femoral circumflex artery makes the largest contribution to the blood supply of this region and is vulnerable to
40

~
41
42 F!!ck
---------------------------------------------------------------------------------------------------------------------------------------------
suW.nd EnQck
1
2
3
4
5
6
7
8
9
10
11
12
13 Femoral neck fractures are common in elderly patients w ith osteoporosis w ho have sustained a fall. The
14 femoral head and neck derive their blood supply from the superior and inferior gluteal arteries and the medial
15 and lateral femoral circumflex arteries, vessels that together form the trochanteric anastomosis. The medial
16 femoral circumflex artery makes the largest contribution to the blood supply of this region and is vulnerable to
17 damage from femoral neck fractures due to its close association w ith the posterior aspect of the femoral
18 neck. Injury or thrombosis of this vessel predisposes to avascular necrosis of the femoral head.
19
20 (Choice A) The femoral artery gives rise to the medial and lateral femoral circumflex arteries. Injury to the
21 femoral artery is unlikely here given its anatomical relationship to the fractured femoral head and because
22 low er extremity claudication and other signs/symptoms of seriously compromised limb perfusion (including
23 limb necrosis} w ould be expected w ith such an injury.
24
25 (Choice C) The lateral femoral circumflex artery courses anterior to the femoral neck and contributes to the
26 blood supply of the femoral head and neck.
27
28 (Choice D) The obturator artery gives rise to an artery that supplies the femoral head. This vessel is
29 especially important in children because it supplies blood to the region of the femoral head proximal to the
30 epiphyseal growth plate. This artery is of minimal clinical significance in adults.
31
(Choice E) The superior and inferior gluteal arteries contribute to the trochanteric anastomosis that supplies
32
the head and neck of the femur.
33
34 Educational Objective:
35 A femoral neck fracture can damage the blood supply to the femoral head and neck. This is most common
36 w ith displaced fractures. The medial femoral circumflex artery provides the majority of the blood supply to the
37 femoral head and neck; injury to this vessel can cause avascular necrosis of the femoral head.
38
39 Time Spent: 1 seconds Copyright USMLEW orld,LLC. Last updated: [7/7/201 OJ
40

~
41
42 F!!ck suW.nd EnQck
---------------------------------------------------------------------------------------------------------------------------------------------
2 Item: 1 of 45
Q. ld: 2126 [
111 f> Mark .<:J
Previous
[::>
Next
il
Lab Values
~
Notes
~
Calculator
3
4 A 55-year-old, right-handed man comes to the emergency department because of recent onset of severe,
5 throbbing, right-sided orbitofrontal headache and diplopia. His other medical problems include poorly
6 controlled hypertension and chronic tobacco use. Neurologic examination show s that he is aw ake, alert, and
7 oriented and follow s both simple and complex commands. Testing of the cranial nerves reveals intact visual
8 acuity bilaterally. Visual fields and optic fundi are normal. Examination show s anisocoria, w ith the right pupil
9 being dilated and nonreactive to both light and accommodation . He has evidence of both vertical and
10 horizontal binocular diplopia. The right eye is dow n and out w ith ipsilateral ptosis. The rest of the neurologic
11 examination is w ithin normal limits. CT angiography of the head reveals a large aneurysm in the posterior
12 fossa . A normal CT angiogram of the head is show n below .
13
14
15
16
17
18
19
12ol
~
22
23
24
25
26
27
28
29
30
31
32
33
34
35
36
37
38
39
40
41
42
3
4
5
6
7
8
9
10
11
12
13
14
15
16
17
18
19

[ill
~1
22
23
24
25
26
27
28
29 W hich of the following identifiers from the angiogram indicates the cerebral vessel most likely to have an
30 aneurysm causing this patient's neurologic deficits?
31
32
33 0 A. A
34 0 B. B
35 0 C. C
36
37 0 D. D
38 0 E. E
39 0 F. F
40
41
42
~imeRemaini~ ~ 0
I Tuto' Feedback Suspend End Block
---------------------------------------------------------------------------------------------------------------------------------------------
2
" Item: 1 of 45
Q. ld: 2126 [
111 f> Mark .<:J
Previous
[::>
Next
il
Lab Values
~
Notes
~
Calculator
3
4
.; @ B. B [48%]
5
6 0 C. C [22%]
7 0 D. D [14%]
8
9
0 E. E [7%]
10 0 F. F [4%]
11
12
13 Explanation: User ld:
14
This patient is presenting w ith right non- pupil-sparing third nerve palsy secondary to a compressive aneurysm
15 arising from the right posterior cerebral artery. It is important to remember that the third nerve courses
16 between the posterior cerebral and superior cerebellar arteries as it leaves the midbrain and is susceptible to
17 injury from an expanding aneurysm originating from these vessels. Chronic smoking and poorly controlled
18
hypertension are risk factors for developing intracranial aneurysms.
19

[ill The third cranial nerve (oculomotor) is a pure motor nerve carrying both the general visceral efferent (GVE)
~1 parasympathetic fibers and general somatic efferent (GSE) fibers. The GVE fibers course on the surface of
22 the oculomotor nerve and subserve the pupillary light and near-reflex pathways. The GSE fibers course w ithin
23 the interior of the oculomotor nerve and subserve the skeletal muscles of the orbit (e.g., superior rectus,
24 inferior rectus, medial rectus, inferior oblique, and levator palpebrae superioris ). The GVE fibers are more
25 susceptible to injury from a compression (e.g., aneurysm and tumor), w hile the GSE fibers are more
26 susceptible to injury from ischemia (e.g., small-vessel disease due to diabetes mellitus).
27
28 (Choice A) The basilar artery is formed by two vertebral arteries. An aneurysm arising from the basilar artery
29 typically does not lead to non- pupil-sparing third nerve palsy.
30
31 (Choice C) The posterior cerebral artery (PCA) arises from the basilar artery. An aneurysm arising from the
32 left PCA w ill cause non- pupil-sparing third nerve palsy on the left but not the right.
33
34 (Choice D) The superior cerebellar artery (SCA) arises from the basilar artery before it bifurcates into the two
35 posterior cerebral arteries. An aneurysm arising from the left SCA w ill cause non- pupil-sparing third nerve
36 palsy on the left but not the right.
37
(Choice E) Aneurysms rarely arise from the parapontine perforating artery.
38
39 (Choice F) The anterior inferior cerebellar artery (AICA) arises from the basilar artery. An aneurysm arising
40 , .. .
. , . .
41
42
~imeRemaini~ ~ 0
I Tuto' Feedback Suspend End Block
---------------------------------------------------------------------------------------------------------------------------------------------
2
" Item: 1 of 45
Q. ld: 2126 [
111 f> Mark .<:J
Previous
[::>
Next
il
Lab Values
~
Notes
~
Calculator
3
4
This patient is presenting w ith right non- pupil-sparing third nerve palsy secondary to a compressive aneurysm
5
arising from the right posterior cerebral artery. It is important to remember that the third nerve courses
6
between the posterior cerebral and superior cerebellar arteries as it leaves the midbrain and is susceptible to
7
injury from an expanding aneurysm originating from these vessels. Chronic smoking and poorly controlled
8
hypertension are risk factors for developing intracranial aneurysms.
9
10 The third cranial nerve (oculomotor} is a pure motor nerve carrying both the general visceral efferent (GVE}
11 parasympathetic fibers and general somatic efferent (GSE} fibers. The GVE fibers course on the surface of
12 the oculomotor nerve and subserve the pupillary light and near-reflex pathways. The GSE fibers course w ithin
13 the interior of the oculomotor nerve and subserve the skeletal muscles of the orbit (e.g., superior rectus,
14 inferior rectus, medial rectus, inferior oblique, and levator palpebrae superioris }. The GVE fibers are more
15 susceptible to injury from a compression (e.g., aneurysm and tumor}, w hile the GSE fibers are more
16 susceptible to injury from ischemia (e.g., small-vessel disease due to diabetes mellitus}.
17
18 (Choice A) The basilar artery is formed by two vertebral arteries. An aneurysm arising from the basilar artery
19 typically does not lead to non- pupil-sparing third nerve palsy.
[ill
~1 (Choice C) The posterior cerebral artery (PCA} arises from the basilar artery. An aneurysm arising from the
22 left PCA w ill cause non- pupil-sparing third nerve palsy on the left but not the right.
23
24 (Choice D) The superior cerebellar artery (SCA} arises from the basilar artery before it bifurcates into the two
25 posterior cerebral arteries. An aneurysm arising from the left SCA w ill cause non- pupil-sparing third nerve
26 palsy on the left but not the right.
27
28 (Choice E) Aneurysms rarely arise from the parapontine perforating artery.
29
(Choice F) The anterior inferior cerebellar artery (AICA} arises from the basilar artery. An aneurysm arising
30
from the AICA can compress the facial and vestibulocochlear nerves.
31
32 Educational objective:
33 The third cranial nerve (oculomotor} carrying general somatic efferent fiber and general visceral efferent
34 parasympathetic fiber exits the midbrain and courses between the posterior cerebral and superior cerebellar
35 arteries. An aneurysm arising from either artery can lead to a non- pupil-sparing third nerve palsy, w hich
36 clinically presents w ith unilateral headache, eye pain, diplopia, dilated nonreactive pupil, and ptosis w ith the
37 ipsilateral eye in a dow n and out position.
38
39
Time Spent: 8 seconds Copyright USMLEW orld,LLC. Last updated: [1 /7/2014]
40
41
42
~imeRemaini~ ~ 0
I Tuto' Feedback Suspend End Block
---------------------------------------------------------------------------------------------------------------------------------------------
1
" Item: 2 of 45
Q. ld: 513 [
111 f> Mark .<:J
Previous
[::>
Next
il
Lab Values
~
Notes
~
Calculator
3
4
5
6 A 26-year-old female presents to your office w ith a three-month history of amenorrhea . She also notes that
7 during this time her breasts have become engorged . She is taking a drug that helps her 'not to hear voices'
8 and acetaminophen for occasional headaches. You proceed w ith giving her a pregnancy test that returns w ith
9 a negative result. W hich of the follow ing dopaminergic pathw ay disturbances are most likely responsible for
10 this patient's symptoms?
11
12
13 0 A. Mesolimbic-mesocortical
14
15
0 B. Nigrostriatal
16 0 C. Tuberoinfundibular
17
18 0 D. Periventricular
19 0 E. lncertohypothalamic
[ill
~1
22
23
24
25
26
27
28
29
30
31
32
33
34
35
36
37
38
39
40
41
42
~imeRemaini~ ~ 0
I Tuto' Feedback Suspend End Block
---------------------------------------------------------------------------------------------------------------------------------------------
1
" Item: 2 of 45
Q. ld: 513 [
111 f> Mark .<:J
Previous
[::>
Next
il
Lab Values
~
Notes
~
Calculator
3
4 .; @ C. Tuberoinfundibular [56%]
5
0 D. Periventricular [4%]
6
7 0 E. lncertohypothalamic [14%]
8
9
Explanation: User ld:
10
11 The patient appears to be suffering from galactorrhea, a non-puerperal secretion of w atery or milky fluid that
12 contains neither pus nor blood. The tuberoinfundibular dopaminergic pathw ay connects the hypothalamus
13 and the pituitary gland and is responsible for dopamine-dependent prolactin tonic inhibition. The
14 neurotransmitter dopamine is secreted from these neural axons and inhibits prolactin secretion from the
15 anterior pituitary gland. Some antipsychotic drugs block dopamine in the tuberoinfundibular pathway, w hich
16 can cause an increase in blood prolactin levels, leading to hyperprolactinemia and abnormal lactation
17 (galactorrhea), disruptions to the menstrual cycle in w omen, and sexual dysfunction.
18
19 (Choice A) The mesolimbic-mesocortical pathway is primarily involved in regulating behavior. This area is
hyperactive in schizophrenia . Antipsychotic action of the neuroleptics is by primarily blocking the dopamine in
[ill
~1 the mesolimbic-mesocortical (frontal cortex) pathway.
22
23 (Choice B) In the nigrostriatal system the neurons are projected from the substancia nigra to the caudate
24 nucleus and putamen. This pathway primarily regulates coordination of voluntary movements. In this
25 pathway dopamine inhibits the release of acetylcholine. Degeneration of the substantia nigra thus causes
26 decreased dopamine and subsequent increased acetylcholine; this leads to movement disorders such as
27 Parkinsonism. Administering high potency antipsychotics can cause some dopamine antagonism in this
28 pathway resulting in drug-induced Parkinsonism.
29
30 (Choices D and E) The other 2 pathways are not really w ell defined and not important for purpose of board
31 examination .
32
33 Educational Objective:
34 Remember the 3 important Dopaminergic systems:
35 Pathway Function Diseases associated
36 Mesolimbic-mesocortical Regulates behavior Schizophrenia
37 Coordination of voluntary
Nigrostriatal Parkinsonism
38 movements
39 Tuberoinfundibular Controls prolactin secretion Hyperprolactinemia
40
41
42
~imeRemaini~ ~ 0
I Tuto' Feedback Suspend End Block
---------------------------------------------------------------------------------------------------------------------------------------------
1
" Item: 2 of 45
Q. ld: 513 [
111 f> Mark .<:J
Previous
[::>
Next
il
Lab Values
~
Notes
~
Calculator
3
4 0 D. Periventricular [4%]
5
0 E. lncertohypothalamic [14%]
6
7
8 Explanation: User ld:
9
10 The patient appears to be suffering from galactorrhea, a non-puerperal secretion of w atery or milky fluid that
11 contains neither pus nor blood. The tuberoinfundibular dopaminergic pathw ay connects the hypothalamus
12 and the pituitary gland and is responsible for dopamine-dependent prolactin tonic inhibition. The
13 neurotransmitter dopamine is secreted from these neural axons and inhibits prolactin secretion from the
14 anterior pituitary gland. Some antipsychotic drugs block dopamine in the tuberoinfundibular pathw ay, w hich
15 can cause an increase in blood prolactin levels, leading to hyperprolactinemia and abnormal lactation
16 (galactorrhea), disruptions to the menstrual cycle in w omen, and sexual dysfunction.
17
18 (Choice A) The mesolimbic-mesocortical pathw ay is primarily involved in regulating behavior. This area is
19 hyperactive in schizophrenia . Antipsychotic action of the neuroleptics is by primarily blocking the dopamine in
the mesolimbic-mesocortical (frontal cortex) pathw ay.
[ill
~1
22 (Choice B) In the nigrostriatal system the neurons are projected from the substancia nigra to the caudate
23 nucleus and putamen. This pathway primarily regulates coordination of voluntary movements. In this
24 pathway dopamine inhibits the release of acetylcholine. Degeneration of the substantia nigra thus causes
25 decreased dopamine and subsequent increased acetylcholine; this leads to movement disorders such as
26 Parkinsonism . Administering high potency antipsychotics can cause some dopamine antagonism in this
27 pathway resulting in drug-induced Parkinsonism.
28
(Choices D and E) The other 2 pathways are not really w ell defined and not important for purpose of board
29
examination .
30
31 Educational Objective:
32 Remember the 3 important Dopaminergic systems:
33 Pathway Function Diseases associated
34 Mesolimbic-mesocortical Regulates behavior Schizophrenia
35 Coordination of voluntary
36 Nigrostriatal Parkinsonism
movements
37 Tuberoinfundibular Controls prolactin secretion Hyperprolactinemia
38
39
Time Spent: 1 seconds Copyright USMLEW orld,LLC. Last updated: [2/16/2012]
40
41
42
~imeRemaini~ ~ 0
I Tuto' Feedback Suspend End Block
---------------------------------------------------------------------------------------------------------------------------------------------
1
2
" Item: 3 of 45
Q. ld: 2125 [
111 f> Mark .<:J
Previous
[::>
Next
il
Lab Values
~
Notes
~
Calculator
4 A 65-year-old man comes to the emergency department because of acute onset slurred speech . He also
5 complains of right-sided w eakness. He denies headache or loss of consciousness. His other medical
6 problems include hypertension and type 2 diabetes mellitus. Neurologic examination show s right-sided low er
7 facial droop and a Babinski response on the right. Motor strength is 3/5 on the right and 5/5 on the left. There
8 is also dysmetria and dysdiadochokinesia involving his right upper and low er extremities. Magnetic resonance
9 imaging of the brain reveals an acute infarct in the brainstem as show n in the image below .
10
11
12
13
14
15
16
17
18
19

[ill
~1
22
23
24
25
26
27
28
29
30
31
32
33
34
35
36
37
38
39
40
41
42
~imeRemaini~ ~ 0
I Tuto' Feedback Suspend End Block
---------------------------------------------------------------------------------------------------------------------------------------------
6
7
8
9
10
11
12
13
14
15
16
17
18
19

[ill
~1
22
23
24
25
26
27
28
29
30
31
The brainstem level affected by this patient's stroke gives rise to w hich of the follow ing cranial nerves?
32
33
34 0 A. Facial
35 0 B. Hypoglossal
36
37 0 C. Oculomotor
38 0 D. Trigeminal
39 0 E. Trochlear
40

~
41
42 F!!ck suW.nd EnQck
---------------------------------------------------------------------------------------------------------------------------------------------
1
2
" Item: 3 of 45
Q. ld: 2125 [
111 f> Mark .<:J
Previous
[::>
Next
il
Lab Values
~
Notes
~
Calculator
4
"' @ D. Trigeminal [1 8%]
5
6 0 E. Trochlear [6%]
7
8
9 Explanation: User ld:
10
11
12
13
14
15
16
17
18
19

[ill
~1
22
23
24
25
26
27
28
29
30
31
32
33
34
35
36
37
38
39
40

~
41
42 F!!ck suW.nd EnQck
---------------------------------------------------------------------------------------------------------------------------------------------
1
2
" Item: 3 of 45
Q. ld: 2125 [
111 f> Mark .<:J
Previous
[::>
Next
il
Lab Values
~
Notes
~
Calculator
4
5
6
7 Optic chiasm
8
9
10 Optic nerve (II) / Thalamus
11
12
13 Optic tract
14
15 Oculomotor
16 nerve (Ill)
17
18
19 Trochlear nerve (IV) ~ .
[ill
~1 Trigeminal nerve (V) ~ l
22
23 Abducens nerve (VI) -------
24
25 Facial nerve (VII) ~~
26 ____ Middle cerebellar
Vestibulococh lear
27 peduncle
nerve (VIII) --~
~ -
28
29 Glossopharyngeal ~
Olive
30 nerve(IX) ~
Hypoglossal nerve (XII) /;~
31
32
33
34 Vagus nerve (X)
35
36 Pyramidal decussation
Spinal accessory nerve (XI)
37
38 USMLEWOI'kt.LLC () 2011
39
40

~
41
42 F!!ck suW.nd
--------------------------------------------------------------------------------------------------------------------------------------
EnQck
Vagus nerve (X)
6
7 Pyramidal decussation
Spinal accessory nerve (XI)
8
9 USML(WoM.Ll( () 2011
10
11 This patient presents w ith an acute lacunar ischemic stroke (yellow arrow) of the medial pons at the level of
12 the middle cerebellar peduncle (brachium pontis). The trigeminal nerve arises from the lateral aspect of the
13 mid-pons at the level of the middle cerebellar peduncles, w hich are a key neuro-anatomic landmark for
14 locating the trigeminal nerve.
15
16 Infarcts involving the anterior pons (basis pontis) affect the corticospinal tract (contralateral hemiparesis and
17 Babinski's sign) and corticobulbar tract (contralateral lower facial palsy and dysarthria). Because the pontine
18 nuclei and pontocerebellar fibers are located in this region, infarction also causes contralateral dysmetria and
19 dysdiadochokinesia, resulting in a syndrome know n as ataxic hemiparesis. Although lesions in the
cerebellum result in ipsilateral deficits, lesions affecting the pontocerebellar fibers in the basis pontis result in
[ill
~1 contralateral deficits because these fibers enter the cerebellum through the contralateral cerebellar peduncle.
22
23 (Choice A) The facial nerve arises from the dorsolateral aspect of the caudal pons at the pontomedullary
24 junction, w hich is located at the level of the facial colliculus (below the middle cerebellar peduncles).
25
26 (Choice B) The hypoglossal nerve arises from the preolivary sulcus at the level of the rostral medulla.
27
28 (Choice C) The oculomotor nerve arises at the level of the superior colliculus and red nucleus, w hich are
29 located in the rostral mesencephalon (midbrain).
30
(Choice E) The trochlear nerve arises from behind the midbrain at the level of the inferior colliculus (below
31
the red nucleus). The trochlear nerve is the only cranial nerve to decussate before innervating its target (the
32
superior oblique muscle).
33
34 Educational objective:
35 The trigeminal nerve arises at the level of the middle cerebellar peduncle at the lateral aspect of the
36 mid-pons. Infarcts involving the anterior portion of the medial pons can produce dysarthria and contralateral
37 ataxic hemiparesis.
38
39 Time Spent: 4 seconds Copyright USMLEWorld,LLC. Last updated: [1 /29/201 4]
40

~
41
42 F!!ck suW.nd
---------------------------------------------------------------------------------------------------------------------------------------------
EnQck
1
2
" Item: 4 of 45
Q. ld: 1696 [
111 f> Mark .<:J
Previous
[::>
Next
il
Lab Values
~
Notes
~
Calculator
3

5
6 Show n below are the cerebral arteriography findings from a 45-year-old patient.
7
8
9
10
11
12
13
14
15
16
17
18
19

[ill
~1
22
23
24
25
26
27 Occlusion of the artery labeled 'X' w ould most likely result in an inability to:
28
29
30 0 A. Climb stairs
31 0 B. Grip
32 0 C. W histle
33
34 0 D. Sw allow food
35
36
37
38
39
40
41
42
~imeRemaini~ ~ 0
I Tuto' Feedback Suspend End Block
---------------------------------------------------------------------------------------------------------------------------------------------
1
2
" Item: 4 of 45
Q. ld: 1696 [
111 f> Mark .<:J
Previous
[::>
Next
il
Lab Values
~
Notes
~
Calculator
3

. 5 ~ @ A. Climb stairs [70%]


. 6 0 B. Grip [9%]
. 7
. 8
0 C. W histle [9%]
. 9 0 D. Swallow food [11 %]
10
11
Explanation: User ld:
12
13 Anterior communicat ing artery
14
15 Medial striate artery
16
(recurrent artery of Heubner)
17 Anterior cerebral artery
18
19

[ill
1~

22 Ophthalmic artery
23
24 Anterolateral central
Internal carotid artery (lenticulostriate) arteries
25
26
27 Middle cerebral
28 artery
29
30
31
32
33 Posterior communicating --~1( Anterior choroidal artery
34 artery
35
36 Posterior cerebral artery
37
38
39
40
41
42
1
2

6 Posterior communicating --~ Anterior choroidal artery


7 artery
8
9 Posterior cerebral artery
10
11
12
13
14
15 Basilar artery
16
17 Labyrinthine
18
Anterior inferior ---~ (internal acoustic) artery
19
cerebellar artery
[ill
~1 Vertebral artery
22 USMLEWorld, LLC 201 1
23
24
25 This cerebral angiogram image is projected axially, w ith the top of the image representing anterior regions and
26 the bottom representing posterior regions. The very large vessels seen bilaterally are the internal carotid
27 arteries. Each gives off a middle cerebral artery to supply more lateral regions and an anterior cerebral artery,
28 w hich extends medially. The above "X'' marks an anterior cerebral artery. Each anterior cerebral artery
29 supplies the medial region of the ipsilateral hemisphere, from the frontal pole to the parietooccipital sulcus.
30 Occlusion of the anterior cerebral artery would affect motor function of the contralateral leg and foot.
31
(Choices B, C and D) Motor control of the hand, head and neck would be affected by occlusion of the middle
32
cerebral artery.
33
34 Educational Objective:
35 The anterior cerebral arteries supply the medial portions of the two hemispheres. Occlusion causes
36 w eakness of the contralateral legs and sensory deficits of the contralateral legs, trunk and genitals. There
37 may also be behavior and mood changes due to injury of frontal lobe structures.
38
39
Time Spent: 2 seconds Copyright USMLEWorld,LLC. Last updated: [1 / 1/2014]
40
41
42
~imeRemaini~ ~ 0
I Tutor Feedback Suspend End Block
--------------------------------------------------------------------------------------------------------------------------------------
1
2
" Item: 5 of 45
Q. ld: 1741 [
111 f> Mark .<:J
Previous
[::>
Next
il
Lab Values
~
Notes
~
Calculator
3
4

6 A 34-year-old male begins treatment w ith combination chemotherapy for Hodgkin's lymphoma. Shortly
7 thereafter, he experiences severe vomiting and requires intravenous fluid supplementation. W hich of the
8 follow ing brain sites is responsible for his symptoms?
9
10
11
12
13
14
15
16
17
18
19

[ill
~1
22
23
24
25
26
27
28
29
30
31
32
33
34
35
36
37
38
39
40 () A A
41
42
~imeRemaini~ ~ 0
I Tuto' Feedback Suspend End Block
---------------------------------------------------------------------------------------------------------------------------------------------
7
8
9
10
11
12
13
14
15
16
17
18
19

[ill
~1
22
23
24
25
26
27
28
29
30
31
32
33
34 0 A. A
35 0 B. B
36
37 0 C. C
38 0 D. D
39 0 E. E
40
41
42
~imeRemaini~ ~ 0
I Tuto' Feedback Suspend End Block
----------------------------
9
10 0 A.A [19%]
11
12 0 B. B [1 8%]
13 0 C. C [11 %]
14 .; @ D. D [44%]
15
16 0 E. E [6%]
17
18
Explanation: User ld:
19
The vomiting that results from administration of systemic chemotherapy is believed to be triggered by the
[ill
~1 chemoreceptor trigger zone (CTZ), located on the dorsal surface of the medulla at the caudal end of the fourth
22 ventricle (Choice D) in a region known as the area postrema. The area postrema receives blood from
23 fenestrated vessels (absent blood-brain barrier), which allows it to sample chemicals circulating in the blood.
24
25 (Choice A} This is the thalamus, which helps to coordinate the sensory and motor systems.
26
27 (Choice B) This dorsal midbrain site is the location of the superior and inferior colliculi. The third ventricle is
28 located dorsal to these structures.
29
30 (Choice C) This is the ventral pons. Fibers of cranial nerves V-VIII, the corticospinal tract, the medial
31 lemniscus, and the lateral spinothalamic tract course through this region .
32
(Choice E) The cerebellum is marked as choice E.
33
34 Educational Objective:
35 Acute nausea following administration of systemic chemotherapy results from stimulation of the
36 chemoreceptor trigger zone (CTZ), which lies in the area postrema of the dorsal medulla near the fourth
37 ventricle.
38
39
Time Spent: 4 seconds Copyright USMLEWorld,LLC. Last updated: [4/4/2014]
40
41
42
~imeRemaini~ ~ 0
I Tuto' Feedback Suspend End Block
--------------------------------------------------------------------------------------------------------------------------------------
1
2
" Item: 6 of 45
Q. ld: 1690 [
111 f> Mark .<:J
Previous
[::>
Next
il
Lab Values
~
Notes
~
Calculator
3
4
5
A 23-year-old Caucasian male w ith involuntary movements undergoes an experimental radioisotope study to
localize metabolic abnormalities in his brain. The study show s abnormal metabolic activity in the caudate
7
nucleus. W hich of the follow ing on the gross brain section slide below corresponds to the location of the
8
abnormalities in this patient?
9
10
11
12
13
14
15
16
17
18
19

[ill
~1
22
23
24
25
26
27
28
29
30 Dlspl1yed with permission from Sprinaer Hnlthcue Ltd.
31 CCopyrlcht 1995, 2004 by Current Medicine
32
33
34 0 A. A
35 0 B. B
36
37 0 C. C
38 0 D. D
39 0 E. E
40
41
42
~imeRemaini~ ~ 0
I Tuto' Feedback Suspend End Block
---------------------------------------------------------------------------------------------------------------------------------------------
1
2
" Item: 6 of 45
Q. ld: 1690 [
111 f> Mark .<:J
Previous
[::>
Next
il
Lab Values
~
Notes
~
Calculator
3
4 ol @ A. A [71%]
5
0 B. B [5%]
7 0 C. C [3%]
8 0 D. D [12%]
9
10 0 E. E [8%]
11
12 Explanation: User ld:
13
14 Cotpus callosum

15 lateral ventrfde
16
17
18
lentiform nucleus
19

[ill
~1
1'\JtamenJ
Globus
22 palladius
23
24
25
26
27
28
29
30
31
32
33 Optic tract
34 Amygditlold nudeus

35
The patient described in the question stem is most likely suffering from Huntington's disease. This disease is
36
an autosomal dominant condition that results from an excessive number of CAG trinucleotide repeats as
37
compared to the unaffected population. Furthermore, families with this disease experience a process called
38
anticipation, which is a progressive expansion of the trinucleotide repeats in successive generations leading to
39
earlier expression of the disease in affected children of Huntington's patients. Imaging studies typically show
40
41
42
~imeRemaini~ ~ 0
I Tuto' Feedback Suspend End Block
------------------------------------------------------------------------------------------------------------------------------------------------------
1
2
3

Optic tract
Amygditlold nudeus
8
9 The patient described in the question stem is most likely suffering from Huntington's disease. This disease is
10 an autosomal dominant condition that results from an excessive number of CAG trinucleotide repeats as
11 compared to the unaffected population . Furthermore, families w ith this disease experience a process called
12 anticipation, w hich is a progressive expansion of the trinucleotide repeats in successive generations leading to
13 earlier expression of the disease in affected children of Huntington's patients. Imaging studies typically show
14 atrophy of the caudate, and metabolic imaging studies such as PET scanning show decreased metabolic
15 activity in the striatum, w hich is composed of the caudate and putamen.
16
17 The caudate nucleus (Choice A) is a curved or "C-shaped" structure in its complete form . The head of the
18 caudate can be identified in the inferolateral w alls of the anterior horns of the lateral ventricles. The head of
19 the caudate nucleus is separated from the globus pallidus and putamen by the internal capsule.
[ill
~1 (Choice B) This structure is the internal capsule. The posterior limb of the internal capsule separates the
22 globus pallidus and putamen from the thalamus and carries corticospinal motor and somatic sensory fibers
23 as w ell as visual and auditory fibers. The function of this limb is the most important to know for the USMLE.
24 The genu, or "knee", of the internal capsule lies between the anterior and posterior branches and carries
25 corticobulbar fibers. The anterior limb of the internal capsule separates the caudate nucleus from the globus
26 pallidus and putamen and carries a portion of the thalamocortical fibers.
27
28 (Choice C) This structure is the insular cortex (insula). The insula plays a role in the limbic system (emotion)
29 as w ell as in the coordination of some autonomic functions, particularly of the cardiac system.
30
31 (Choice D) This structure is the putamen.
32
(Choice E) This structure is the globus pallidus.
33
34 Educational Objective:
35 The head of the caudate lies in the inferolateral w all of the anterior horn of the lateral ventricle. It is separated
36 from the globus pallidus and putamen by the internal capsule. The caudate is atrophied in Huntington's
37 disease.
38
39 Time Spent: 2 seconds Copyright USMLEW orld,LLC. Last updated: [1 0/ 19/2013]
40
41
42
~imeRemaini~ ~ 0
I Tuto' Feedback Suspend End Block
---------------------------------------------------------------------------------------------------------------------------------------------
1
2
" Item: 7 of 45
Q. ld: 8859 [
111 f> Mark .<:J
Previous
[::>
Next
il
Lab Values
~
Notes
~
Calculator
3
4
5
6 A 32-year-old w oman describes five episodes of intractable vomiting over the last year. The episodes last
several hours and are associated w ith a sensation that the room is spinning or tilting . At these times, it is
8 difficult for her to w alk because she loses her balance. She cannot relate the timing of the episodes to any
9 particular inciting event. Physical examination reveals stability in the Romberg position and during tandem
10 w alk. Proprioception is intact. Dysfunction of w hich of the follow ing structures best explains this patient's
11 symptoms?
12
13
14 0 A. Posterior columns of the spinal cord
15 0 B. Vagal nerve
16
17 0 C. Optic tract
18 0 D. Inner ear
19
0 E. Cerebellum
[ill
~1
0 F. Frontal cortex
22
23
24
25
26
27
28
29
30
31
32
33
34
35
36
37
38
39
40
41
42
~imeRemaini~ ~ 0
I Tuto' Feedback Suspend End Block
---------------------------------------------------------------------------------------------------------------------------------------------
1
2
" Item: 7 of 45
Q. ld: 8859 [
111 f> Mark .<:J
Previous
[::>
Next
il
Lab Values
~
Notes
~
Calculator
3
4 u L:. U pttc tract [Z'7oJ
5 v @ D. Inner ear [84%]
6
0 E. Cerebellum [8%]
8 0 F. Frontal cortex [0%]
9
10
11 Explanation: User ld:
12 This patient is experiencing vertigo due to vestibular dysfunction. Vertigo is a sensation of motion w hen no
13 motion is present or an exaggerated sense of motion for a given bodily movement. It must be distinguished
14 from other sensations that are often described by patients using similar terminology, such as imbalance,
15 light-headedness, and syncope. Vertigo resulting from vestibular dysfunction tends to be of sudden onset,
16 interfere w ith w alking, and cause nausea and vomiting. Typical causes of vestibulopathy include Meniere's
17 disease, perilymphatic fistulas, benign positional vertigo, labyrinthitis, and acoustic neuromas.
18
19 (Choice A) Disease of the posterior column of the spinal cord can occur in syphilis (i.e., tabes dorsalis) and
vitamin 812 deficiency. Typical symptoms include ataxia, decreased proprioception and vibratory sense, and
[ill
~1 hyporeflexia. This patient's normal neurologic exam makes posterior column disease unlikely.
22
23 (Choice B) Vagal nerve damage can present as hoarseness, dysphagia, abnormal gastrointestinal motility,
24 or tachycardia . Vagal nerve dysfunction w ill not cause vertigo.
25
26 (Choice C) Optic tract dysfunction w ill result in visual problems. Vision deficits do not typically cause vertigo,
27 but may cause people to feel unsteady on their feet.
28
29 (Choice E) Cerebellar dysfunction presents as ataxia, imbalance, incoordination, and nystagmus. Patients
30 w ith cerebellar dysfunction w ill be unable to perform tandem w alking due to their imbalance.
31
(Choice F) The frontal cortex controls executive functions and response inhibition. Damage to the frontal
32
cortex results in "frontal release" symptoms, such as personality change, inability to organize or plan, and
33
disinhibition.
34
35 Educational Objective:
36 Vertigo is a sensation of excessive motion compared to physical reality. It is most commonly due to
37 dysfunction w ithin the vestibular system .
38
39
Time Spent: 1 seconds Copyright USMLEW orld,LLC. Last updated: [3/ 19/2014]
40
41
42
~imeRemaini~ ~ 0
I Tuto' Feedback Suspend End Block
---------------------------------------------------------------------------------------------------------------------------------------------
1
2
" Item: 8 of 45
Q. ld: 1534 [
111 f> Mark .<:J
Previous
[::>
Next
il
Lab Values
~
Notes
~
Calculator
3
4
5
6 A 22-year-old male presents to your office complaining of occasional headaches. On exam, you note several
7 pigmented spots on his trunk. You also discover a few rubbery cutaneous tumors on his neck. The cells
composing these skin tumors most likely originated from which of the following structures?
9
10
11 0 A. Surface ectoderm
12 0 B. Neuroectoderm
13
14 0 C. Neural crest
15 0 D. Mesoderm
16
17 0 E. Endoderm
18
0 F. Notochord
19

[ill
~1
22
23
24
25
26
27
28
29
30
31
32
33
34
35
36
37
38
39
40
41
42
~imeRemaini~ ~ 0
I Tuto' Feedback Suspend End Block
---------------------------------------------------------------------------------------------------------------------------------------------
1
2
" Item: 8 of 45
Q. ld: 1534 [
111 f> Mark .<:J
Previous
[::>
Next
il
Lab Values
~
Notes
~
Calculator
3
4
"' @ C. Neural crest [52%]
5
6 0 D. Mesoderm [4%]
7 0 E. Endoderm [2%]

9
0 F. Notochord [1 %]
10
11 Explanation: User ld:
12
13 This patient has pigmented lesions that are most likely a cafe-au-lait macules, and headaches that could be
14 due to a glioma- both of w hich are common presentations of Neurofibromatosis Type I.
15
16 The tumors of interest in the question stem are neurofibromas. Neurofibromas are tumors of Schw ann cells,
17 w hich are derived from the neural crest. On exam, cutaneous neurofibromas are skin-colored or pink nodules
18 w ith a rubbery texture that exhibit pathognomonic "buttonholing". The latter refers to the examiner's perception
19 that these tumors "buttonhole" dow n through the skin w ith gentle pressure.

[ill
~1 (Choice A) The surface ectoderm gives rise to the skin and its appendages, the nasal and oral epithelium, the
22 anal epithelium to the dentate line, and the lens and cornea .
23
24 (Choice B) The neuroectoderm gives rise to all neurons of the CNS, the neurohypophysis, the retina, the
25 pineal gland, preganglionic autonomic fibers, astrocytes, ependymal cells, and the choroid plexus.
26 (Choice D) The mesoderm is divided into paraxial, intermediate, and lateral sections and gives rise to
27
muscles, bones, blood, genital tissues, the dermis, and most organ systems. Notably, the mesoderm does
28 not give rise to any epithelial surfaces.
29
30 (Choice E) The endoderm gives rise to thyroid tissues (descendent from the base of the tongue) and the
31 epithelial linings of the Gl tract, lung, urethra, bladder, and outer and middle ear. The endoderm also gives rise
32 to the bulk of the liver, pancreas, and lungs, as these structures form from outpouchings of the primitive gut
33 tube.
34
35 (Choice F) The notochord almost completely regresses in humans. The only major vestige is the nucleus
36 pulposus in the intervertebral disks. Vertebrae form from the paraxial mesoderm.
37
38 Educational Objective:
39 Neurofibromatosis Type I (NF1, or von Recklinghausen's disease) is a common autosomal-dominant disorder
40 resultina from a defect in the NF-1 aene on chromosome 17. Cutaneous and subcutaneous neurofibromas

~
41
42 F!!ck
---------------------------------------------------------------------------------------------------------------------------------------------
suW.nd EnQck
1
2
" Item: 8 of 45
Q. ld: 1534 [
111 f> Mark .<:J
Previous
[::>
Next
il
Lab Values
~
Notes
~
Calculator
3
4 0 F. Notochord [1 %]
5
6
7 Explanation: User ld:

This patient has pigmented lesions that are most likely a cafe-au-lait macules, and headaches that could be
9
due to a glioma- both of w hich are common presentations of Neurofibromatosis Type I.
10
11 The tumors of interest in the question stem are neurofibromas. Neurofibromas are tumors of Schw ann cells,
12 w hich are derived from the neural crest. On exam, cutaneous neurofibromas are skin-colored or pink nodules
13 w ith a rubbery texture that exhibit pathognomonic "buttonholing". The latter refers to the examiner's perception
14 that these tumors "buttonhole" dow n through the skin w ith gentle pressure.
15
16 (Choice A) The surface ectoderm gives rise to the skin and its appendages, the nasal and oral epithelium, the
17 anal epithelium to the dentate line, and the lens and cornea .
18
19 (Choice B) The neuroectoderm gives rise to all neurons of the CNS, the neurohypophysis, the retina, the
pineal gland, preganglionic autonomic fibers, astrocytes, ependymal cells, and the choroid plexus.
[ill
~1
22 (Choice D) The mesoderm is divided into paraxial, intermediate, and lateral sections and gives rise to
23 muscles, bones, blood, genital tissues, the dermis, and most organ systems. Notably, the mesoderm does
24 not give rise to any epithelial surfaces.
25
26 (Choice E) The endoderm gives rise to thyroid tissues (descendent from the base of the tongue) and the
27 epithelial linings of the Gl tract, lung, urethra, bladder, and outer and middle ear. The endoderm also gives rise
28 to the bulk of the liver, pancreas, and lungs, as these structures form from outpouchings of the primitive gut
29 tube.
30
(Choice F) The notochord almost completely regresses in humans. The only major vestige is the nucleus
31
pulposus in the intervertebral disks. Vertebrae form from the paraxial mesoderm .
32
33 Educational Objective:
34 Neurofibromatosis Type I (NF1, or von Recklinghausen's disease) is a common autosomal-dominant disorder
35 resulting from a defect in the NF-1 gene on chromosome 17. Cutaneous and subcutaneous neurofibromas
36 are common in this condition and are tumors of Schw ann cells, w hich are embryologically derived from the
37 neural crest.
38
39
Time Spent: 2 seconds Copyright USMLEW orld,LLC. Last updated: [1 / 10/2013]
40
41
42
~imeRemaini~ ~ 0
I Tuto' Feedback Suspend End Block
---------------------------------------------------------------------------------------------------------------------------------------------
1
2
" Item: 9 of 45
Q. ld: 1694 [
111 f> Mark .<:J
Previous
[::>
Next
il
Lab Values
~
Notes
~
Calculator
3
4
5
6 A patient presents to the emergency room w ith w eak w rist extension . On physical examination, sensation to
7 the arm is intact. The affected nerve w as most likely injured at w hich of the follow ing locations?
8

10 0 A. Carpal tunnel
11 0 B. Hook of the hamate
12
13 0 C. Surgical neck of the humerus
14 0 D. Head of the radius
15
16 0 E. Coracobrachialis
17
18
19

[ill
~1
22
23
24
25
26
27
28
29
30
31
32
33
34
35
36
37
38
39
40
41
42
~imeRemaini~ ~ 0
I Tuto' Feedback Suspend End Block
---------------------------------------------------------------------------------------------------------------------------------------------
1
2
" Item: 9 of 45
Q. ld: 1694 [
111 f> Mark .<:J
Previous
[::>
Next
il
Lab Values
~
Notes
~
Calculator
3
4 .; @ D. Head of the radius {55%]
5 0 E. Coracobrachialis [8%]
6
7
8 Explanation: User ld:

10
11
12
13 Above the spiral groove, the radial
14 +- nerve gives off the posterior
15
16
.,"

i cutaneous nerve of the arm.

17 The posterior cutaneous nerve of the


18 forearm arises in the spiral groove.
19

[ill
~1
22
23
24
25
26
27
28
29
30
31
32
33
34
35 The deep branch of the radial nerve
. ,_ supplies the extensor compartment.
36
37
38
The superficial branch
39
40
41
42
~imeRemaini~ ~ 0
I Tuto' Feedback Suspend End Block
------------------------------------------------------------------------------------------------------------------------------------------------------
1
2
3
4
5
6
7
8 The deep branch of the radial nerve
+-II-. supplies the extensor compartment.
10
11
12 The superficial branch
supplies the skin on the
13
dorsum of the hand.
14
15
16 The radial nerve arises from the posterior cord of the brachial plexus. It innervates the muscles of the
17 posterior (extensor) compartment of the arm. After innervating the extensor compartment of the upper arm,
18 the radial nerve enters the forearm. At the lateral epicondyle of the humerus (where the humerus articulates
19 w ith the radius), the radial nerve divides into superficial and deep branches. The superficial branch of the
radial nerve is purely sensory; it provides somatic sensory innervation to the radial half of the dorsal hand. (It
[ill
~1 does not, how ever, provide sensory innervation to the dorsal portions of the distal first through third digits.
22 Sensory innervation of the palmar and dorsal surfaces of the distal 3 Y2 digits is provided by the lateral and
23 medial branches of the median nerve.)
24
25 Radial head subluxation ("nursemaid's elbow") is a common injury in children that can result from sudden
26 outward pulling on an extended and pronated arm (e.g. w hen an adult attempts to pull a child upw ard by the
27 arm). Radial head subluxation can damage the deep branch of the radial nerve, leading to w eakness or
28 paralysis of forearm extensor compartment muscles. 'Wrist drop" commonly results from radial nerve injury.
29 The deep branch of the radial nerve does not carry any somatic sensory innervation.
30
31 (Choice A) Carpal tunnel syndrome can result from any factor (e.g. inflammation) that reduces the size of the
32 carpal tunnel, compressing the median nerve. In addition to pain, patients classically experience difficulty w ith
33 fine motor control of the thumb.
34
(Choice B) In the w rist, the ulnar nerve passes between the hook of the hamate and the pisiform bone in a
35
fibroosseous tunnel know n as Guyon's canal. Ulnar nerve injury at this site causes dysesthesias of the ulnar
36
side of the hand and w eakness of the intrinsic muscles of the hand.
37
38 (Choice C) Fracture of the surgical neck of the humerus may cause axillary nerve injury leading to paralysis
39 of the deltoid and teres minor muscles as w ell as loss of sensation on the lateral upper arm.
40
41
42
~imeRemaini~ ~ 0
I Tuto' Feedback Suspend End Block
---------------------------------------------------------------------------------------------------------------------------------------------
1
2
" Item: 9 of 45
Q. ld: 1694 [
111 f> Mark .<:J
Previous
[::>
Next
il
Lab Values
~
Notes
~
Calculator
3
4 dorsum of lhe hand.
5
6 The radial nerve arises from the posterior cord of the brachial plexus. It innervates the muscles of the
7 posterior (extensor) compartment of the arm . After innervating the extensor compartment of the upper arm,
8 the radial nerve enters the forearm . At the lateral epicondyle of the humerus (where the humerus articulates
w ith the radius), the radial nerve divides into superficial and deep branches. The superficial branch of the
10 radial nerve is purely sensory; it provides somatic sensory innervation to the radial half of the dorsal hand. (It
11 does not, how ever, provide sensory innervation to the dorsal portions of the distal first through third digits.
12 Sensory innervation of the palmar and dorsal surfaces of the distal 3 Y2 digits is provided by the lateral and
13 medial branches of the median nerve.)
14
15 Radial head subluxation ('nursemaid's elbow") is a common injury in children that can result from sudden
16 outward pulling on an extended and pronated arm (e.g. w hen an adult attempts to pull a child upw ard by the
17 arm). Radial head subluxation can damage the deep branch of the radial nerve, leading to w eakness or
18 paralysis of forearm extensor compartment muscles. 'Wrist drop" commonly results from radial nerve injury.
19 The deep branch of the radial nerve does not carry any somatic sensory innervation.
[ill
~1 (Choice A) Carpal tunnel syndrome can result from any factor (e.g. inflammation) that reduces the size of the
22 carpal tunnel, compressing the median nerve. In addition to pain, patients classically experience difficulty w ith
23 fine motor control of the thumb.
24
25 (Choice B) In the w rist, the ulnar nerve passes between the hook of the hamate and the pisiform bone in a
26 fibroosseous tunnel know n as Guyon's canal. Ulnar nerve injury at this site causes dysesthesias of the ulnar
27 side of the hand and w eakness of the intrinsic muscles of the hand.
28
29 (Choice C) Fracture of the surgical neck of the humerus may cause axillary nerve injury leading to paralysis
30 of the deltoid and teres minor muscles as w ell as loss of sensation on the lateral upper arm.
31
(Choice E) The coracobrachialis muscle is an arm flexor that lies deep to the biceps brachii and overlies the
32
median nerve and brachial artery. It is innervated by the musculocutaneous nerve.
33
34 Educational Objective:
35 The deep branch of the radial nerve arises near the lateral epicondyle of the humerus, w here the humerus
36 articulates w ith the head of the radius. Radial head subluxation can damage the deep branch of the radial
37 nerve, causing w eakness of the forearm and hand extensors but no sensory deficits.
38
39
Time Spent: 1 seconds Copyright USMLEW orld,LLC. Last updated: [1 / 10/2013]
40
41
42
~ime Remainin g : o i : l ! ~ 0
I Tuto' Feedback Suspend End Block
---------------------------------------------------------------------------------------------------------------------------------------------
1
2
" Item: 10 of 45
Q.ld: 8522 [
lil f> Mark .<:J
Previous
[::>
Next
il
Lab Values
~
Notes
~
Calculator
3
4
5
6 A 67 -year-old woman w ith metastatic lung cancer complains of recent hoarseness and difficulty
7 sw allow ing. On examination, there is loss of the gag reflex on the left side and uvula deviation to the right
8 w hen the patient is prompted to say "ah ." There is also w eakness of her left sternocleidomastoid and
9 trapezius muscles. A lesion involving w hich of the following anatomical structures is most likely responsible
for the patient's symptoms?
11
12
13 0 A. Cribriform plate
14 0 B. Foramen ovale
15
16 0 C. Foramen rotundum
17 0 D. Hypoglossal canal
18
19 0 E. Internal acoustic meatus
[ill 0 F. Jugular foramen
~1
22
23
24
25
26
27
28
29
30
31
32
33
34
35
36
37
38
39
40
41
42
~imeRemaini~ ~ 0
I Tuto' Feedback Suspend End Block
---------------------------------------------------------------------------------------------------------------------------------------------
1
2
" Item: 10 of 45
Q.ld: 8522 [
lil f> Mark .<:J
Previous
[::>
Next
il
Lab Values
~
Notes
~
Calculator
3
4
5
I ~ @ F. Jugular foramen [74%]
6
7 Explanation: User ld:
8
9
Sku ll foramen Traversing structures
11
12
13 Anterior
Cribriform p late CN I olfactory bundles
14 cranial f ossa
15
16 CN II, opht halmic artery, central
17 Optic canal
18 retinal vein
19
CN Ill, IV, V1 , VI, ophthalmic vein,
[ill
~1
Superior orbital fissure
sympathetic fibers
22 Middle
23 cranial fossa
Foramen rotundum CN V2 (maxillary)
24
25
26 Foramen ovale CN V3 (mandibular)
27
28 Foramen spinosum Middle meningeal artery & vein
29
30
31 Int ernal acoustic meatus CNVII, VIII
32
33 CN IX, X, XI, jugular vein
Jugular foramen
34 Post erior
35 cran ial f ossa
36 Hypoglossal canal CN XII
37
38 Spinal root s of CN XI, brain stem,
39 Foramen magnum
40 vertebral arteries
41
42
~imeRemaini~ ~ 0
I Tuto' Feedback
--------------------------------------------------------------------------------------------------------------------------------------
Suspend End Block
1
2
" Item: 10 of 45
Q.ld: 8522 [
lil f> Mark .<:J
Previous
[::>
Next
il
Lab Values
~
Notes
~
Calculator
3
4
5
6 Skull foramen Traversing structures
7
8
9 Anterior
Cribriform plate CN I olfactory bundles
cranial f ossa
11
12 CN II, ophthalmic artery, central
13 Optic canal
14 retinal vein
15
16 CN Ill, IV, V1 , VI, ophthalmic vein,
Superior orbital fissure
17 sympathetic fibers
18 Middle
19 cranial fossa
Foramen rotundum CN V2 (maxillary)
[ill
~1
22 Foramen ovale CN V3 (mandibular)
23
24
Foramen spinosum Middle meningeal artery & vein
25
26
27 Int ernal acoustic meatus CNVII, VIII
28
29 CN IX, X, XI, jugular vein
Jugular foramen
30 Posterior
31 cranial fossa
32 Hypoglossal canal CN XII
33
34 Spinal root s of CN XI, brain stem,
35 Foramen magnum
36 vertebral arteries
37 >USM.LWorid, llC
38
39 This patient's symptoms are consistent with a lesion involving the jugular foramen, a large aperture located
40 . .. . . . . . , . . .. .. .
41
42
~imeRemaini~ ~ 0
I Tuto' Feedback Suspend End Block
--------------------------------------------------------------------------------------------------------------------------------------
1
2
" Item: 10 of 45
Q.ld: 8522 [
lil f> Mark .<:J
Previous
[::>
Next
il
Lab Values
~
Notes
~
Calculator
3
4 Foramen ovale CN V3 (mandibular)
5
6 Foramen spinosum Middle meningea l artery & vein
7
8
9 Internal acoustic meatus CN VII, VIII

11
Jugular foramen CN IX, X, XI, jugular vein
12 Posterior
13 cranial fossa
14 Hypoglossal canal CN XII
15
16 Spina l roots of CN XI, brain stem,
17 Foramen magnum
18 vertebral arteries
19 @USMLE.Wortd, llC

[ill
~1
This patient's symptoms are consistent with a lesion involving the jugular foramen, a large aperture located
22 in the base of the skull behind the carotid canal. It is formed by the petrous portion of the temporal bone and
23 the occipital bone. Passing through the jugular foramen are CN IX, X, and XI. Lesions of the jugular foramen
24
(from tumors, trauma, or infection) can result in jugular foramen (Vernet) syndrome, which is characterized by
25
CN IX, X, and XI dysfunction. Symptoms are related to the nerve affected:
26
27 Loss of taste from the posterior 1/3 of the tongue (CN IX}
28 Reduced parotid gland secretion (CN IX)
29 Loss of gag reflex (CN IX, X)
30 Dysphagia (CN IX, X)
31 Dysphonia/hoarseness (CN X)
32 Soft palate drop with deviation of the uvula tow ard the normal side (CN X)
33 Sternocleidomastoid and trapezius muscle paresis (CN XI)
34
35 Educational objective:
36 Lesions of the jugular foramen can result in jugular foramen (Vernet) syndrome, which is characterized by the
37 dysfunction of CN IX, X, and XI.
38
39
Time Spent: 1 seconds Copyright USMLEWorld,LLC. Last updated: [1 0/20/201 3]
40

~
41
42 F!!ck
--------------------------------------------------------------------------------------------------------------------------------------
suW.nd EnQck
1
2
" Item: 11 of 45
Q.ld: 8564 [
111 f> Mark .<:J
Previous
[::>
Next
il
Lab Values
~
Notes
~
Calculator
3
4
5
6 A 4-day-old premature infant in the neonatal intensive care unit develops a decreased level of consciousness
7 and hypotonia . She w as delivered vaginally at 30 w eeks of gestation and her birth w eight w as 1200 g (2 lb 10
8 oz). Physical examination reveals a lethargic infant w ith a w eak and high-pitched cry, prominent scalp veins,
9 and tense fontanels. Cranial ultrasound reveals blood in the lateral ventricles. W hich of the follow ing
10 structures is the most likely source of the bleeding?

12
13 0 A. Bridging cortical veins
14 0 B. Germinal matrix
15
16 0 C. Meningeal arteries
17 0 D. Sagittal sinus
18
19 0 E. Vessels of the circle of W illis

[ill
~1
22
23
24
25
26
27
28
29
30
31
32
33
34
35
36
37
38
39
40

~
41
42 F!!ck
---------------------------------------------------------------------------------------------------------------------------------------------
suW.nd EnQck
1
2
" Item: 11 of 45
Q.ld: 8564 [
111 f> Mark .<:J
Previous
[::>
Next
il
Lab Values
~
Notes
~
Calculator
3
4 ~ @ B. Germinal matrix [40%]
5
0 C. Meningeal arteries [8%]
6
7 0 D. Sagittal sinus [1 8%]
8 0 E. Vessels of the circle of Willis [1 6%]
9
10
Explanation: User ld:
12
13 Germinal matrix hemorrhage
14
15
16
17 Hemorrhage
18 extending into
19 lateral ventricle
[ill
~1 lateral
22 vent ricle
23
24
25
26
27
28
29
30
31
32
33
34
35
36
37
38
39
40

~
41
42 F!!ck suW.nd
--------------------------------------------------------------------------------------------------------------------------------------
EnQck
1
2
" Item: 11 of 45
Q.ld: 8564 [
111 f> Mark .<:J
Previous
[::>
Next
il
Lab Values
~
Notes
~
Calculator
3
4
<!>USMLEWO<ki.LLC
5
6 Intraventricular hemorrhage (IVH) is a common complication of prematurity that can lead to long-term
7 neurodevelopmental impairment. It occurs most frequently in infants born before 32 w eeks gestation and/or
8 w ith birth w eight < 1500 g, and almost always occurs w ithin the first 5 postnatal days. IVH in the new born can
9 be clinically silent or present w ith an altered level of consciousness, hypotonia, and decreased spontaneous
10 movements. Symptoms of catastrophic bleeding include a bulging anterior fontanelle, hypotension,
decerebrate posturing, tonic-clonic seizures, irregular respirations, and coma .
12
13 IVH in preterm infants usually originates from the germinal matrix, a highly cellular and vascularized layer in
14 the subventricular zone from w hich neurons and glial cells migrate out during brain development. The matrix
15 contains numerous thin-w alled vessels lacking the glial fibers that support other blood vessels throughout the
16 brain, w hich contributes to the risk of hemorrhage. It is especially vulnerable to hemodynamic instability as
17 premature infants can have impaired autoregulation of cerebral blood flow . Between 24-32 w eeks of
18 gestation, the germinal matrix becomes less prominent and its cellularity and vascularity decrease, reducing
19 the risk of IVH.
[ill
~1 (Choice A) Subdural hematoma can result from the rupture of cortical bridging veins. In infants, this can
22 occur from violent shaking (shaken baby syndrome).
23
24 (Choice C) Epidural hematoma frequently occurs due to head trauma . It is often associated w ith temporal
25 bone fracture and subsequent tearing of the middle meningeal artery.
26
27 (Choice D) The dural venous sinuses are venous channels found between layers of dura mater in the brain.
28 Certain conditions (eg, malignancy, hypercoagulable states, and trauma) can result in thrombosis w ithin the
29 dural sinuses.
30
(Choice E) Saccular (berry) aneurysms are the most common cause of subarachnoid hemorrhage. They
31
typically occur near the circle of W illis. Berry aneurysms are associated w ith Ehlers-Danlos syndrome and
32
autosomal dominant polycystic kidney disease.
33
34 Educational objective:
35 Neonatal intraventricular hemorrhage usually occurs in the fragile germinal matrix and increases in frequency
36 w ith decreasing age and birth w eight. It is a common complication of prematurity that can lead to long-term
37 neurodevelopmental impairment.
38
39
Time Spent: 1 seconds Copyright USMLEW orld,LLC. Last updated: [12116/2013]
40
41
42
~imeRemaini~ ~ 0
I Tuto' Feedback Suspend End Block
---------------------------------------------------------------------------------------------------------------------------------------------
1
2
" Item: 12 of 45
Q.ld: 1692 [
lil f> Mark .<:J
Previous
[::>
Next
il
Lab Values
~
Notes
~
Calculator
3
4
5
6 A 43-year-old Caucasian female presents to your office w ith back pain after dragging a heavy box. She
7 describes pain in the right posterior thigh and posterior calf shooting dow n to the foot. Physical examination
8 reveals symmetric knee reflexes, but the ankle jerk reflex is slightly diminished on the right. W hich of the
9 follow ing nerve roots is most likely affected in this patient?
10
11
O A.L2
13 0 B. L3
14
15 0 C.L4
16 0 D.L5
17
18 0 E. S1
19

[ill
~1
22
23
24
25
26
27
28
29
30
31
32
33
34
35
36
37
38
39
40
41
42
~imeRemaini~ ~ 0
I Tuto' Feedback Suspend End Block
---------------------------------------------------------------------------------------------------------------------------------------------
1
2
" Item: 12 of 45
Q.ld: 1692 [
lil f> Mark .<:J
Previous
[::>
Next
il
Lab Values
~
Notes
~
Calculator
3
4 0 D. L5 [15%]
5
.; @ E. S1 [67%]
6
7
8 Explanation: User ld:
9
10 The patient described in the question stem is suffering symptoms consistent w ith sciatic neuropathy, or
11 sciatica . Sciatic neuropathy can result from any irritation of the sciatic nerve or its roots. Common causes
include vertebral disc herniations, spinal canal stenosis, irritation by bone spurs, and irritation by the
13 piriformis. Classic symptoms of sciatica include w eakness of the posterior thigh muscles as w ell as
14 w eakness of all muscles below the knee. The sciatic nerve also provides sensory innervation to the posterior
15 thigh and calf as w ell as the plantar surface of the foot. Transection of the sciatic nerve w ould result in an
16 inability to flex the leg at the knee and paralysis of the muscles below the knee. The ankle reflex is diminished
17 in sciatic nerve injury, but the patellar reflex is unaffected as this reflex arc is carried by fibers of the femoral
18 nerve (L2 - L4).
19
The sciatic nerve is derived from the L4 through S3 nerve roots, so only choices C, D, and E are reasonable
[ill
~1 options. Sciatica most commonly involves L5 or S1. Of these, compression of the L5 root typically results in
22 posterior and lateral thigh and leg pain shooting to the inner foot. Compression of the S 1 root results in pain
23 strictly in the posterior thigh and leg shooting to the foot accompanied by w eakness of plantar flexion and loss
24 of the ankle jerk reflex.
25
26 (Choices A and B) L2 and L3 contribute to the lateral femoral cutaneous nerve as w ell as other nerves such
27 as the femoral, obturator, and genitofemoral. The lateral femoral cutaneous is a purely sensory nerve that
28 enters the thigh deep to the inguinal ligament and supplies most of the skin of the anterior thigh .
29
(Choice C) L4 contributes partially to the femoral, sciatic, obturator, superior gluteal, and quadratus femoris
30
nerves. This nerve provides no sensation to the posterior thigh or leg. Damage to L4 w ould result in
31
impairment of the knee jerk reflex.
32
33 Educational Objective:
34 Sciatica is a painful condition characterized by shooting pain dow n the posterior thigh and leg that typically
35 results from impingement of one of the spinal nerves as it leaves the vertebral column . Compression of the
36 S1 root results specifically in pain purely in the posterior thigh and leg as w ell as diminution of the ankle jerk
37 reflex.
38
39
Time Spent: 1 seconds Copyright USMLEW orld,LLC. Last updated: (1 /7/2014]
40
41
42
~imeRemaini~ ~ 0
I Tuto' Feedback Suspend End Block
---------------------------------------------------------------------------------------------------------------------------------------------
1
2
" Item: 13 of 45
Q.ld: 1691 [
lil f> Mark .<:J
Previous
[::>
Next
il
Lab Values
~
Notes
~
Calculator
3
4
5
6 An infant born to a 22-year-old female has difficulty feeding because of an underdeveloped mandible.
7 Development of which of the following structures is likely also impaired in this patient?
8
9
10 0 A. Styloid process
11 0 B. Lesser horn of the hyoid
12
0 C. Stapes
14 0 D. Malleus
15
16 0 E. Thyroid cartilage
17
18
19

[ill
~1
22
23
24
25
26
27
28
29
30
31
32
33
34
35
36
37
38
39
40
41
42
~imeRemaini~ ~ 0
I Tuto' Feedback Suspend End Block
---------------------------------------------------------------------------------------------------------------------------------------------
1
2
" Item: 13 of 45
Q.ld: 1691 [
lil f> Mark .<:J
Previous
[::>
Next
il
Lab Values
~
Notes
~
Calculator
3
4
5 0 A. Styloid process [1 5%]
6
0 B. Lesser horn of the hyoid [18%]
7
8 0 C. Stapes [1 3%]
9 ~ @ D. Malleus [44%]
10
0 E. Thyroid cartilage [1 0%]
11
12
Explanation: User ld:
14
15 The infant described in the question is suffering a defect in development of the first pharyngeal (branchial)
16 arch. There are six branchial arches in the developing embryo, and all but the fifth arch contribute structures
17 to the adult; the fifth arch regresses completely. Each pharyngeal arch is associated w ith a cranial nerve, and
18 know ledge of w hich nerve is paired w ith w hich arch can assist in remembering w hich adult structures are
19 derived from each arch . In addition, note that the muscular elements are formed by mesoderm and the bony
elements are formed by neural crest cells that migrate in.
[ill
~1
22 The first arch is associated w ith the trigeminal nerve (CN V). The neural crest cells of the first arch form the
23 bones associated w ith this structure, w hich include the maxilla, zygoma, mandible, vomer, palatine, incus,
24 and malleus (Choice D). Aberrant neural crest cell migration into the first arch can cause bony deformities
25 as described in the question stem . Mesodermal derivatives include all muscles of mastication, anterior belly
26 of the digastric, m ylohyoid, tensor tympani, and tensor veli palatini .
27
28 (Choices A, B and C) The styloid process, lesser horn of the hyoid, and the stapes are all neural crest
29 derivatives of the second pharyngeal arch . The second arch is associated w ith the facial nerve (CN VII) and
30 gives rise to the muscles of facial expression, the stylohyoid, the stapedius, and the posterior belly of the
31 digastric.
32
(Choice E) The fourth and sixth arches together make up the cartilaginous structures of the larynx.
33
34 Educational Objective:
35 The first arch can be poorly formed during embryonic development resulting in first arch syndrome.
36 Abnormalities include malformation of the mandible, maxilla, malleus, incus, zygoma, vomer, palate, and
37 temporal bone. The first arch is associated w ith the trigeminal nerve.
38
39
Time Spent: 1 seconds Copyright USMLEW orld,LLC. Last updated: [7/7/201 OJ
40
41
42
~imeRemaini~ ~ 0
I Tuto' Feedback Suspend End Block
---------------------------------------------------------------------------------------------------------------------------------------------
1
2
" Item: 14 of 45
Q.ld: 1933 [
lil f> Mark .<:J
Previous
[::>
Next
il
Lab Values
~
Notes
~
Calculator
3
4
5
6 A 43-year-old male complains of double vision w hen w alking dow n stairs. He has no other concerns. He eats
7 a balanced diet and takes one multivitamin daily. He jogs for 30 minutes every morning. His past medical
8 history is significant for type 2 diabetes mellitus, w hich is w ell controlled w ith glyburide. A lesion of w hich of
9 the follow ing structures is most likely responsible for this patient's complaints?
10
11
12 0 A. Optic nerve
13 0 B. Oculomotor nerve
15 0 C. Abducens nerve
16 0 D. Trochlear nerve
17
18 0 E. Medial longitudinal fasciculus
19

[ill
~1
22
23
24
25
26
27
28
29
30
31
32
33
34
35
36
37
38
39
40
41
42
~imeRemaini~ ~ 0
I Tuto' Feedback Suspend End Block
---------------------------------------------------------------------------------------------------------------------------------------------
1
2
" Item: 14 of 45
Q.ld: 1933 [
lil f> Mark .<:J
Previous
[::>
Next
il
Lab Values
~
Notes
~
Calculator
3
4 A 'l.:l-year-ota mate comp1a1ns or aou01e VISIOn w nen w atKtng aow n statrs. He nas no omer concerns. He eats
5 a balanced diet and takes one multivitamin daily. He jogs for 30 minutes every morning. His past medical
6 history is significant for type 2 diabetes mellitus, w hich is w ell controlled w ith glyburide. A lesion of w hich of
7 the follow ing structures is most likely responsible for this patient's complaints?
8
9 0 A. Optic nerve [7%]
10
11
0 B. Oculomotor nerve [20%]
12 0 C. Abducens nerve [9%]
13 v @ D. Trochlear nerve [55%]
0 E. Medial longitudinal fasciculus [9%]
15
16
17 Explanation: User ld:
18
19 Vertical diplopia is most noticeable w hen the affected eye looks tow ard the nose, as occurs w hen reading the
new spaper or w alking dow nstairs. One of the more common causes of vertical diplopia is a palsy of cranial
[ill
~1 nerve IV (trochlear nerve}. Most such palsies have traumatic or idiopathic etiologies.
22
23 (Choice A) The optic nerve (cranial nerve II} transmits visual information to the brain. Damage to this nerve
24 causes loss of vision.
25
26 (Choice B) The oculomotor nerve (cranial nerve Ill} innervates the superior rectus, medial rectus, inferior
27 rectus, and inferior oblique. These muscles collectively perform most eye movements. Palsies of this nerve
28 can cause vertical and horizontal diplopia, ptosis, and an enlarged and nonreactive pupil.
29
30 (Choice C) The abducens nerve (cranial nerve VI} innervates the lateral rectus, w hich is responsible for
31 abduction of the eye. Palsies of this nerve can cause horizontal diplopia and esotropia (inward deviation}.
32
(Choice E) A lesion of the medial longitudinal fasciculus is associated w ith internuclear ophthalmoplegia,
33
w hich presents w ith impaired horizontal eye movement and w eak adduction of the affected eye w ith
34
simultaneous abduction nystagmus of the contralateral eye.
35
36 Educational Objective:
37 Trochlear nerve palsy is characterized by vertical diplopia.
38
39
Time Spent: 1 seconds Copyright USMLEW orld,LLC. Last updated: [3/212014]
40
41
42
~imeRemaini~ ~ 0
I Tuto' Feedback Suspend End Block
---------------------------------------------------------------------------------------------------------------------------------------------
1
2
" Item: 15 of 45
Q.ld: 1687 [
lil f> Mark .<:J
Previous
[::>
Next
il
Lab Values
~
Notes
~
Calculator
3
4
5
A 45-year-old male presents with left arm clumsiness. Physical examination reveals motor w eakness and
6
increased muscle tone in the left arm. Specifically, with passive extension of the left arm there is initial
7
resistance then sudden release when maximum extension is reached. Pathology within which of the following
8
brain structures is most likely responsible?
9
10
11
12
13
14

16
17
18
19

[ill
~1
22
23
24
25
26
27
28
29
30 Dlspl1yed with permission from Sprinaer Hnlthcue Ltd.
31 CCopyrlcht 1995, 2004 by Current Medicine
32
33
34 0 A. A
35 0 B. B
36
37 0 C. C
38 0 D. D
39 0 E. E
40
41
42
~imeRemaini~ ~ 0
I Tuto' Feedback Suspend End Block
------------------------------------------------------------------------------------------------------------------------------------------------------
1
2
3
4
5
6 Dlspl1yed with permi ssion from Sprlncer Hnlthure Ltd.
7 CCopyrlcht 1995, 2004 by C.urrent Medicine
8
9
10 0 A.A [1 5%]
11
.; @ B. B [42%]
12
13 0 C. C [9%]
14 0 D. D [1 7%]
16 0 E. E [1 6%]
17
18
Explanation: User ld:
19
This patient has ''clasp knife spasticity," characterized by initial jerking resistance to passive extension
[ill
~1 follow ed by sudden release of resistance . This form of spasticity is typical of an upper motor neuron lesion.
22 Potential sites for upper motor neuron lesions include the corticospinal tracts of the spinal cord; the medulla,
23 pons and midbrain; the internal capsule; or the precentral gyrus (primary motor cortex). Of these, the internal
24 capsule is the only structure labeled above (Choice B). In general, upper motor neuron lesions cause spastic
25 paresis, hyperreflexia, and a positive Babinski sign. The clasp knife phenomenon is a form of spasticity that
26 results from a lack of upper motor neuron inhibitory control on the spinal muscle stretch reflex arc.
27
28 (Choice A) This is the caudate nucleus, the structure affected by Huntington's disease.
29
30 (Choice C) This is the insular cortex (insula), which plays a role in the limbic system (emotion).
31
32 (Choice D) This is the putamen .
33
(Choice E) This is the globus pallidus.
34
35 Educational Objective:
36 Upper motor neuron lesions cause spastic rigidity, hyperreflexia, and paresis. Corticospinal tract, internal
37 capsule (posterior limb), and primary motor cortex lesions can cause these symptoms.
38
39
Time Spent: 2 seconds Copyright USMLEWorld,LLC. Last updated: [11 / 10/2011]
40
41
42
~imeRemaini~ ~ 0
I Tuto' Feedback Suspend End Block
---------------------------------------------------------------------------------------------------------------------------------------------
1
2
" Item: 16 of 45
Q.ld: 1452 [
lil f> Mark .<:J
Previous
[::>
Next
il
Lab Values
~
Notes
~
Calculator
3
4
5
6 A 34-year-old male is unable to tolerate everyday sounds. Injury of w hich of the follow ing cranial nerves might
7 be responsible?
8
9
10 0 A. Hypoglossal
11
0 B. Facial
12
13 0 C. Accessory
14
15 0 D. Vagal
0 E. Glossopharyngeal
17
18
19

[ill
~1
22
23
24
25
26
27
28
29
30
31
32
33
34
35
36
37
38
39
40
41
42
~imeRemaini~ ~ 0
I Tuto' Feedback Suspend End Block
---------------------------------------------------------------------------------------------------------------------------------------------
1
2
" Item: 16 of 45
Q.ld: 1452 [
lil f> Mark .<:J
Previous
[::>
Next
il
Lab Values
~
Notes
~
Calculator
3
4
5
6 A 34-year-old male is unable to tolerate everyday sounds. Injury of w hich of the follow ing cranial nerves might
7 be responsible?
8
9
10
0 A. Hypoglossal [2%1
11 ~ @ B. Facial [78%1
12 0 C. Accessory [4%1
13
14
0 D. Vagal [8%1
15 0 E. Glossopharyngeal [7% 1

17
Explanation: User ld:
18
19 The middle ear cavity contains three auditory ossicles (malleus, incus and stapes) and two skeletal muscles
[ill
~1
(tensor tympani and stapedius) that participate in the transmission of sound from the tympanic membrane to
the inner ear. The stapedius muscle arises from the w all of the tympanic cavity and inserts on the neck of the
22 stapes. The body's smallest striated muscle, its function is to stabilize the stapes, the body's smallest bone.
23 The stapedius muscle is innervated by the stapedius nerve, a branch of facial nerve (CN VII). Paralysis of the
24 stapedius muscle allow s the stapes to oscillate more w idely, producing hyperacusis (increased sensitivity to
25 sound). Ipsilateral hyperacusis is a common finding in Bell's palsy (peripheral facial nerve paralysis). Other
26 findings in facial nerve paralysis include inability to close the eye or to smile on the affected side, as w ell as
27 ipsilateral increased salivation and loss of taste on the anterior 213 of the tongue.
28
29 Injury to the other .cranial nerves listed in the answ er choices does not lead to hyperacusis. Note also that the
30 tensor tympani muscle arises from the cartilaginous portion of the auditory tube and the adjoining part of the
31 sphenoid bone and inserts into the malleus. The tensor tympani draw s the tympanic membrane medially,
32 increasing its tension. This muscle is innervated by the mandibular branch of the trigeminal nerve (CN V3).
33
34 Educational Objective:
35 The stapedius muscle is innervated by the stapedius nerve, a branch of facial nerve (CN VII). Paralysis of the
36 stapedius muscle allow s w ider oscillation of the stapes, and leads to increased sensitivity to sound
37 (hyperacusis ).
38
39
Time Spent: 1 seconds Copyright USMLEW orld,LLC. Last updated: [11 / 10/20111
40
41
42
~imeRemaini~ ~ 0
I Tuto' Feedback Suspend End Block
---------------------------------------------------------------------------------------------------------------------------------------------
1
2
" Item: 17 of 45
Q.ld: 8594 [
111 f> Mark .<:J
Previous
[::>
Next
il
Lab Values
~
Notes
~
Calculator
3
4
5 A 57 -year-old man is brought to the emergency department follow ing a generalized tonic-clonic seizure. His
6 w ife reports that he has no history of seizures. How ever, she says that he has been complaining of
7 intermittent headaches, memory loss, and problems w ith his vision for the past 2 w eeks. Brain imaging
8 show s a solitary mass w ithin the right temporal lobe. W hich of the follow ing visual field defects is most likely
9 present in this patient?
10 Left Right
11
12
13 A.
14
15
16 0

18
19
B.
[ill
~1
22
23 0
24
25
26
27 C.
28
29
30 0
31
32
33
34 D.
35
36
37 0
38
39
40
41
42
~imeRemaini~ ~ 0
I Tuto' Feedback Suspend End Block
---------------------------------------------------------------------------------------------------------------------------------------------
1
2
" Item: 17 of 45
Q.ld: 8594 [
111 f> Mark .<:J
Previous
[::>
Next
il
Lab Values
~
Notes
~
Calculator
3
4
5 A.
6
7
8 0
9
10
11
12 B.
13
14
15 0
16

18
19 C.
[ill
~1
22 0
23
24
25
26 D.
27
28
29 0
30
31
32
33
E.
34
35
36
0
37
38
39
40
41
42
~imeRemaini~ ~ 0
I Tuto' Feedback Suspend End Block
---------------------------------------------------------------------------------------------------------------------------------------------
1
2
3
4
5
6
7
8
9
10 [60%]
11 E.
12
13
14
0
15
16

18 [7%]
19

[ill
~1
Explanation: User ld:
22 A. Monocular scotoma
23 Visual pathways from above
24
25
26
27
OQ
B. Right anopia
28
29
30
31
0
C. Bitemporal hemianopia
32
33
34 Optic
(a
35 D. Right nasal hemianopia
36

0 ~
37 Optic chiasma
38
39 Optic t ract
40
41
42
1
2
" Item: 17 of 45
Q.ld: 8594 [
111 f> Mark .<:J
Previous
[::>
Next
il
Lab Values
~
Notes
~
Calculator
3
4
A. Monocular scotoma
5 Visual pathways from above
6
7
8
9
OQ
B. Right anopia

0
10
11
12
13 C. Bitemporal hemianopia
14
15
16 Optic ne1rve--._
(a
D. Right nasal hemianopia
18
19

[ill
~1
Optic chiasma
0 ()
E. Left homonymous hemianopia
22
23
24
25
()
26 F. Left homonymous superior
27 quadrantanopla ("pie in the sky")
Lateral
28 geniculate
29 nucleus
30
31
G. Left homonymous inferior
32 quadrant anopia ("pie on the floor")
33
34
35 Optic radiation
36
37 H. Left homonymous hemianopia
H w ith macular sparing
38
39
40 ~~
~ F!!ck
41
42 suW.nd EnQ ck
------------------------------------------------------------------------------------------------------------------------------------------------------
1
2
3
4
5
6
7 Optic radiation
8
9 H. Left homonymous hemianopia
10 H w ith macular sparing
11
12
13
14
c) USMlEWorld. l lC
C)U
15 Damage to the visual pathw ay produces distinct types of visual field defects depending on the location of the
16 lesion . Visual perception begins w ith light from the nasal visual fields striking the temporal side of each retina
and light from the temporal visual fields striking the nasal side of each retina . Information from the retina is
18 then transmitted by the optic nerves to the optic chiasm. At the optic chiasm, optic nerve fibers from the nasal
19 half of each retina cross and project into the contralateral optic tract. In contrast, nerve fibers from the
temporal parts pass into the ipsilateral optic tract. The optic tract thus contains nerve fibers from the temporal
[ill
~1 part of the ipsilateral retina and the nasal part of the contralateral retina . Optic tract fibers project mainly to the
22 lateral geniculate nucleus (LGN), but also project to superior colliculus (reflex gaze), pretectal area (light
23 reflex), and the suprachiasmatic nucleus (circadian rhythms).
24
25 Axons from the LGN that project to the striate (primary visual) cortex are know n as the optic radiation (or
26 geniculocalcarine tract). The low er fibers of the optic radiation carry information from the low er retina (upper
27 contralateral visual field) and take a circuitous route anteriorly into the temporal lobe (Meyer's loop) before
28 reaching the lingual gyrus of the striate cortex. The upper fibers of the optic radiation carry information from
29 the upper retina (low er contralateral visual field) and pass more directly from the LGN through the parietal lobe
30 to reach the cuneus gyrus of the striate cortex.
31
Lesions in the temporal lobe can disrupt Meyer's loop and produce a contralateral superior quadrantanopia.
32
Temporal lobe lesions can also produce other neurologic manifestations, including aphasia (dominant
33
hemisphere lesions), memory deficits, seizures (complex partial and tonic-clonic), and hallucinations
34
(auditory, olfactory, and visual).
35
36 Educational objective:
37 Injury to Meyer's loop in the temporal lobe results in contralateral superior quadrantanopia.
38
39
Time Spent: 3 seconds Copyright USMLEW orld,LLC. Last updated: [213/2014]
40
41
42
~imeRemaini~ ~ 0
I Tuto' Feedback Suspend End Block
---------------------------------------------------------------------------------------------------------------------------------------------
1
2
3
4
5
6
7 Optic radiation
8
9 H. Left homonymous hemianopia
10 H w ith macular sparing
11
12
13
14
c) USMlEWorld. l lC
C)U
15 Damage to the visual pathw ay produces distinct types of visual field defects depending on the location of the
16 lesion . Visual perception begins w ith light from the nasal visual fields striking the temporal side of each retina
and light from the temporal visual fields striking the nasal side of each retina . Information from the retina is
18 then transmitted by the optic nerves to the optic chiasm. At the optic chiasm, optic nerve fibers from the nasal
19 half of each retina cross and project into the contralateral optic tract. In contrast, nerve fibers from the
temporal parts pass into the ipsilateral optic tract. The optic tract thus contains nerve fibers from the temporal
[ill
~1 part of the ipsilateral retina and the nasal part of the contralateral retina . Optic tract fibers project mainly to the
22 lateral geniculate nucleus (LGN), but also project to superior colliculus (reflex gaze), pretectal area (light
23 reflex), and the suprachiasmatic nucleus (circadian rhythms).
24
25 Axons from the LGN that project to the striate (primary visual) cortex are know n as the optic radiation (or
26 geniculocalcarine tract). The low er fibers of the optic radiation carry information from the low er retina (upper
27 contralateral visual field) and take a circuitous route anteriorly into the temporal lobe (Meyer's loop) before
28 reaching the lingual gyrus of the striate cortex. The upper fibers of the optic radiation carry information from
29 the upper retina (low er contralateral visual field) and pass more directly from the LGN through the parietal lobe
30 to reach the cuneus gyrus of the striate cortex.
31
Lesions in the temporal lobe can disrupt Meyer's loop and produce a contralateral superior quadrantanopia.
32
Temporal lobe lesions can also produce other neurologic manifestations, including aphasia (dominant
33
hemisphere lesions), memory deficits, seizures (complex partial and tonic-clonic), and hallucinations
34
(auditory, olfactory, and visual).
35
36 Educational objective:
37 Injury to Meyer's loop in the temporal lobe results in contralateral superior quadrantanopia.
38
39
Time Spent: 3 seconds Copyright USMLEW orld,LLC. Last updated: [213/2014]
40
41
42
~imeRemaini~ ~ 0
I Tuto' Feedback Suspend End Block
---------------------------------------------------------------------------------------------------------------------------------------------
1
2
" Item: 18 of 45
Q.ld: 1955 [
lil f> Mark .<:J
Previous
[::>
Next
il
Lab Values
~
Notes
~
Calculator
3
4
5
6 A 54-year-old man is brought to the emergency department by his w ife after he develops difficulty speaking .
7 W hen asked about the onset of his symptoms, he slow ly responds w ith "1... w eak ... morning..." and becomes
8 very frustrated. His w ife says that w hile filling out medical release forms, he w as able to grasp a pen w ithout
9 any problem but had trouble signing his name. On examination, the patient is able to state his first name w ith
10 difficulty and correctly points to different body parts on command . He also has mild w eakness involving his
11 right arm and leg. This patient's speech difficulties are most likely caused by a lesion affecting w hich of the
12 follow ing brain areas?
13
14
15
16
17 0 A. A
19 0 B. B

[ill 0 C. C
~1
22 0 D. D
23 0 E. E
24
25
0 F. F
26
27
28
29
30
31
32
33
34
35
36
37
38
39
40
41
42
~imeRemaini~ ~ 0
I Tuto' Feedback Suspend End Block
---------------------------------------------------------------------------------------------------------------------------------------------
1
2
" Item: 18 of 45
Q.ld: 1955 [
lil f> Mark .<:J
Previous
[::>
Next
il
Lab Values
~
Notes
~
Calculator
3
4
5 A 54-year-old man is brought to the emergency department by his w ife after he develops difficulty speaking .
6 W hen asked about the onset of his symptoms, he slow ly responds w ith "1... w eak ... morning..." and becomes
7 very frustrated. His w ife says that w hile filling out medical release forms, he w as able to grasp a pen w ithout
8 any problem but had trouble signing his name. On examination, the patient is able to state his first name w ith
9 difficulty and correctly points to different body parts on command . He also has mild w eakness involving his
10 right arm and leg. This patient's speech difficulties are most likely caused by a lesion affecting w hich of the
11 follow ing brain areas?
12
13
14
15
16
17

19

[ill
~1
22
23
24
25
26
27
28
29
30
31
32
33
34
35
36
37
38
USMLEWorld, LLC
39
40
41
42
~imeRemaini~ ~ 0
I Tuto' Feedback Suspend End Block
---------------------------------------------------------------------------------------------------------------------------------------------
1
2
" Item: 18 of 45
Q.ld: 1955 [
lil f> Mark .<:J
Previous
[::>
Next
il
Lab Values
~
Notes
~
Calculator
3
4
5
6
7
8
9
10
11
12
13
14
15
16
17

19

[ill
~1
22
23
24
25
26
27
28
29
30 USMLEWorld, LLC
31
32
33 0 A. A
34 0 B. B
35 0 C. C
36
37 0 D. D
38 0 E. E
39 0 F. F
40
41
42
~imeRemaini~ ~ 0
I Tuto' Feedback Suspend End Block
---------------------------------------------------------------------------------------------------------------------------------------------
1
2
" Item: 18 of 45
Q.ld: 1955 [
lil f> Mark .<:J
Previous
[::>
Next
il
Lab Values
~
Notes
~
Calculator
3
4 .; @ B. B [68%]
5 0 C. C [12%]
6
7 0 D. D [4%]
8 0 E.E [1 5%]
9 0 F. F [0%]
10
11
12 Explanation: User ld:
13
14
15 Common types of aphasia
16
17
3. Arcuate fasciculus
19

[ill
~1
22
23
24
25
26
27
28
29
30
31
32
33
34
35
36
37
38
39
40
41
42
1
2
" Item: 18 of 45
Q.ld: 1955 [
lil f> Mark .<:J
Previous
[::>
Next
il
Lab Values
~
Notes
~
Calculator
3
4
5 Common types of aphasia
6
7
8 3. Arcuate fasciculus
9
10
11
12
13
14
15
16
17

19

[ill
~1
22
23
24
25
26
27
28
29
30 1. Broca's area
31
32
33
34
35
36 2. Wernicke's area
37
38
39
40
41
42
1
2
" Item: 18 of 45
Q.ld: 1955 [
lil f> Mark .<:J
Previous
[::>
Next
il
Lab Values
~
Notes
~
Calculator
3
4
5
6
Ap hasia Spontaneous
7 Compreh ension Repetition Associated featu res
8 syndrome speech
9
10
11 Sparse& Relatively
1. Broca's Impaired Right hemiparesis
12 nonfluent preserved
13
14
15 Fluent &
Greatly Right superior visual
16 2 . Wernicke's voluminous but Impaired
17 d iminished field defect
lacks meaning
19
Fluent w ith Relatively
[ill
~1
3. Con d u ction
phonemic errors preserved
Very poor None
22
23
24 USMLEWorld, LLC
25
26 This patient can understand spoken language and follow commands (intact speech comprehension), making
27 Wernicke's aphasia unlikely. His speech is labored and lacking in small connecting words, and his frustration
28 indicates that he has insight into his speaking difficulties. These features are most consistent with Broca's
29 (motor, nonfluent) aphasia.
30
Patients with Broca's aphasia are able to communicate meaningfully, but their speech is slow and consists
31
primarily of nouns and verbs. Speech may be punctuated by pauses after each word as the patient attempts
32
to verbalize the next. Broca's aphasia classically results from damage to Broca's area of the brain, the region
33
responsible for all communicative motor planning (which is the reason these patients have difficulty writing
34
and signing). Broca's area is in the caudal part of the inferior frontal gyrus of the dominant (usually left)
35
hemisphere (Brodmann areas 44 and 45).
36
37 (Choice A) The frontal eye field is found near the caudal end of the middle frontal gyrus immediately
38 anterior to the precentral sulcus (Brodmann areas 6 and 8). Damage here would cause the eyes to deviate to
39 the ipsilateral side.
40
41
42
~imeRemaini~ ~ 0
I Tuto' Feedback Suspend End Block
--------------------------------------------------------------------------------------------------------------------------------------
1
2
" Item: 18 of 45
Q.ld: 1955 [
lil f> Mark .<:J
Previous
[::>
Next
il
Lab Values
~
Notes
~
Calculator
3
\ I I I V~V I ) I lVI IIIU'-'11~} Ut-'1 IU..,IU .
4
5 Patients with Broca's aphasia are able to communicate meaningfully, but their speech is slow and consists
6 primarily of nouns and verbs. Speech may be punctuated by pauses after each w ord as the patient attempts
7 to verbalize the next. Broca's aphasia classically results from damage to Broca's area of the brain, the region
8 responsible for all communicative motor planning (which is the reason these patients have difficulty writing
9 and signing). Broca's area is in the caudal part of the inferior frontal gyrus of the dominant (usually left)
10 hemisphere (Brodmann areas 44 and 45).
11
12 (Choice A) The frontal eye field is found near the caudal end of the middle frontal gyrus immediately
13 anterior to the precentral sulcus (Brodmann areas 6 and 8). Damage here w ould cause the eyes to deviate to
14 the ipsilateral side.
15
16 (Choice C) Lesions to the precentral gyrus (primary motor cortex) can cause dysarthria (eg, slurred or
17 explosive speech) due to paresis/paralysis of the skeletal muscles involved in movements of the mouth,
tongue, and larynx. Damage to Broca's area is often associated with right-sided hemiparesis and oral apraxia
19 due to extension of the lesion into the ipsilateral motor regions. How ever, it is the injury to Broca's area that
leads to the language deficit.
[ill
~1
22 (Choice D) This is the postcentral gyrus (primary somatosensory cortex). A lesion here w ould cause
23 sensation loss in the corresponding area of the contralateral body.
24
25 (Choice E) The caudal superior temporal gyrus (Brodmann area 22) is the location of Wernicke's area. A
26 lesion here w ould cause a sensory (receptive) aphasia. Wernicke's aphasia is also known as fluent aphasia
27 because speech flow s readily but is meaningless ("w ord salad"). Patients classically lack insight into their
28 problem.
29
(Choice F) A lesion to the visual cortex can cause cortical blindness or visual impairment that can make
30
reading and writing difficult. Spoken speech w ould be unaffected.
31
32 Educational objective:
33 Broca's (motor) aphasia is a nonfluent aphasia that results from damage to Broca's area in the inferior frontal
34 gyrus of the dominant hemisphere. Patients understand language but cannot properly formulate the motor
35 commands to form w ords or write. Speech is slow and fragmented, with short agrammatic phrases
36 (Broca-Broken speech). W ord-finding difficulty is often prominent. Patients have insight into their aphasia
37 and are often frustrated by it.
38
39
Time Spent: 5 seconds Copyright USMLEW orld,LLC. Last updated: [12111 /2013]
40
41
42
~imeRemaini~ ~ 0
I Tuto' Feedback Suspend End Block
---------------------------------------------------------------------------------------------------------------------------------------------
1
2
"
Item: 19 of 45
Q.ld: 1306 [
lil f> Mark .<:J
Previous
[::>
Next
il
Lab Values
~
Notes
~
Calculator
3
4
5
6 A 34-year-old female presents to your office w ith moderate hearing loss that she noticed several months ago.
7 She also complains of a noise in her right ear that is very disturbing. Physical examination show s right-sided
8 hearing loss, an asymmetric smile, and decreased corneal reflex in her right eye. This patient's condition
9 suggests an intracranial mass located in w hich of the follow ing locations?
10
11
12 0 A. Between the medulla and the cerebellar hemispheres
13
0 B. Above the diaphragma sellae
14
15 0 C. Over the lateral hemispheric fissure
16
0 D. Between the cerebellum and the lateral pons
17
18 0 E. In the parasagittal aspect of the brain convexity
0 F. Between the cerebellar peduncles

22
23
24
25
26
27
28
29
30
31
32
33
34
35
36
37
38
39
40
41
42
~imeRemaini~ ~ 0
I Tuto' Feedback Suspend End Block
---------------------------------------------------------------------------------------------------------------------------------------------
1
2
"
Item: 19 of 45
Q.ld: 1306 [
lil f> Mark .<:J
Previous
[::>
Next
il
Lab Values
~
Notes
~
Calculator
3
- ...... ....,., ..... , ~ .......... ~..... , .... ' ....... ' ................... '"" ................... l'-' ' "'J
4
5 ~ @ D. Between the cerebellum and the lateral pons [65%]
6 0 E. In the parasagittal aspect of the brain convexity [4%]
7
0 F. Between the cerebellar peduncles [6%]
8
9
10 Explanation: User ld:
11
12 Intracranial schw annomas are most commonly located at the cerebellopontine angle, w hich is between the
13 cerebellum and lateral pons. Schwannomas of this location arise from CN VIII and are called acoustic
14 neuromas. CN VII (facial nerve) and CN V (trigeminal nerve) are in proximity to CN VIII and may also be
15 affected by acoustic neuromas via compression . The patient in this vignette displays symptoms of CN VIII
16 involvement as w ell as CN V and CN VII.
17
18 1. The cochlear part of CN VIII mediates hearing. Its compression leads to sensorineural hearing loss
and tinnitus (ear ringing).
2. The vestibular part of CN VIII maintains balance. Its compression causes vertigo, dysequilibrium, and
nystagmus.
3. CN VII involvement leads to paralysis of the muscles of facial expression, loss of taste in the anterior
22
23 213 of the tongue, and hyperacusis (paralysis of the stapedius).
24 4. Compression of CN V causes loss of sensation around the mouth and nose, loss of corneal reflex,
and paralysis of the muscles of mastication.
25
26
27 Bilateral acoustic neuromas are associated w ith neurofibromatosis type 2.
28
29 (Choice B) Germ cell tumors arise in the pineal and suprasellar regions. They can cause aqueductal
30 stenosis (with obstructive hydrocephalus) and Parinaud syndrome (paralysis of vertical gaze).
31
32 (Choices C and E) Meningiomas are commonly found over the lateral hemispheric fissure and in the
33 parasagittal aspect of the brain convexity. On light microscopy, meningiomas have a w horled pattern of cell
34 growth- also, psammoma bodies are present. Clinically, meningiomas may manifest w ith headache, focal
35 deficit, or seizure.
36
Educational Objective:
37
Sensorineural hearing loss, tinnitus, paralysis of facial muscles, and loss of corneal reflex signify the
38
involvement of CN V, VII, and VIII. Simultaneous compression of these nerves is caused by tumor of the
39
cerebellopontine angle, w hich are most commonly acoustic neuromas.
40
41
42
~imeRemaini~ ~ 0
I Tuto' Feedback Suspend End Block
---------------------------------------------------------------------------------------------------------------------------------------------
1
2
"
Item: 19 of 45
Q.ld: 1306 [
lil f> Mark .<:J
Previous
[::>
Next
il
Lab Values
~
Notes
~
Calculator
3
4 0 E. In the parasagittal aspect of the brain convexity [4%]
5 0 F. Between the cerebellar peduncles [6%]
6
7
8 Explanation: User ld:
9
10 Intracranial schw annomas are most commonly located at the cerebellopontine angle, w hich is between the
11 cerebellum and lateral pons. Schwannomas of this location arise from CN VIII and are called acoustic
12 neuromas. CN VII (facial nerve) and CN V (trigeminal nerve) are in proximity to CN VIII and may also be
13 affected by acoustic neuromas via compression . The patient in this vignette displays symptoms of CN VIII
14 involvement as w ell as CN V and CN VII.
15
1. The cochlear part of CN VIII mediates hearing. Its compression leads to sensorineural hearing loss
16
and tinnitus (ear ringing).
17
2. The vestibular part of CN VIII maintains balance. Its compression causes vertigo, dysequilibrium, and
18
nystagmus.
3. CN VII involvement leads to paralysis of the muscles of facial expression, loss of taste in the anterior
213 of the tongue, and hyperacusis (paralysis of the stapedius).
4. Compression of CN V causes loss of sensation around the mouth and nose, loss of corneal reflex,
22
and paralysis of the muscles of mastication.
23
24
25 Bilateral acoustic neuromas are associated w ith neurofibromatosis type 2.
26
27 (Choice B) Germ cell tumors arise in the pineal and suprasellar regions. They can cause aqueductal
28 stenosis (with obstructive hydrocephalus) and Parinaud syndrome (paralysis of vertical gaze).
29
(Choices C and E) Meningiomas are commonly found over the lateral hemispheric fissure and in the
30
parasagittal aspect of the brain convexity. On light microscopy, meningiomas have a w horled pattern of cell
31
growth- also, psammoma bodies are present. Clinically, meningiomas may manifest w ith headache, focal
32
deficit, or seizure.
33
34 Educational Objective:
35 Sensorineural hearing loss, tinnitus, paralysis of facial muscles, and loss of corneal reflex signify the
36 involvement of CN V, VII, and VIII. Simultaneous compression of these nerves is caused by tumor of the
37 cerebellopontine angle, w hich are most commonly acoustic neuromas.
38
39
Time Spent: 1 seconds Copyright USMLEW orld,LLC. Last updated: [7/7/201 0]
40
41
42
~imeRemaini~ ~ 0
I Tuto' Feedback Suspend End Block
---------------------------------------------------------------------------------------------------------------------------------------------
1
2
" Item: 20 of 45
Q.ld : 2076 [
111 f> Mark .<:J
Previous
[::>
Next
il
Lab Values
~
Notes
~
Calculator
3
4
5
6 The following vignette applies to the next 2 items. The items in the set must be answered in sequential order.
7 Once you click Proceed to Next Item, you will not be able to add or change an answer.
8
9 A 62-year-old female comes to the ER complaining of difficulty w alking that started three hours ago. She also
10 says that she cannot feel her right side. Physical examination reveals loss of touch sensation affecting the
11 right upper and low er extremities and the right side of the face. Her muscle strength is 5/5 in all extremities.
12
13 Item 1 of 2
14 This patient has most likely suffered a stroke affecting w hich of the follow ing brain structures?
15
16 0 A. Frontal cortex
17
18 0 B. Internal capsule
19 0 C. Base of the pons
0 D. Caudate nucleus
22 0 E. Thalamus
23
24
25
26
27
28
29
30
31
32
33
34
35
36
37
38
39
40
41
42
~imeRemaini~ ~ 0
I Tuto' Feedback Suspend End Block
---------------------------------------------------------------------------------------------------------------------------------------------
1
2
" Item: 20 of 45
Q.ld: 2076 [
111 f> Mark .<:J
Previous
[::>
Next
il
Lab Values
~
Notes
~
Calculator
3
4
5
6 The follow ing vignette applies to the next 2 items. The items in the set must be answ ered in sequential order.
7 Once you click Proceed to Next Item, you w ill not be able to add or change an answ er.
8
9 A 62-year-old female comes to the ER complaining of difficulty w alking that started three hours ago. She also
10 says that she cannot feel her right side. Physical examination reveals loss of touch sensation affecting the
11 right upper and low er extremities and the right side of the face. Her muscle strength is 5/5 in all extremities.
12
13 Item 1 of 2
14 This patient has most likely suffered a stroke affecting w hich of the follow ing brain structures?
15
16 0 A. Frontal cortex [8%]
17 0 B. Internal capsule [29%]
18
19 0 C. Base of the pons [14%]
0 D. Caudate nucleus [4%]
., @ E. Thalamus [44%]
22
23
24 Explanation: User ld:
25
26 The patient described is experiencing the thalamic syndrome. In this syndrome, a vascular lesion of the
27 thalamus, such as an ischemic or hemorrhagic stroke, results in damage to the thalamic VPL and VPM
28 nuclei, causing complete contralateral sensory loss. There are also often severe proprioceptive defects that
29 lead to unsteady gait and falls. Patients w ith incomplete lesions or partial resolution of the thalamic lesion
30 may develop abnormal painful sensations on the affected side of the body.
31
32 (Choice A) Frontal cortex lesions cause deficits of higher brain function, altering patterns of spontaneity,
33 creativity, sexual interest and social behavior.
34
(Choice B) A lesion of the internal capsule would cause a motor deficit.
35
36 (Choice C) Ischemic events in the pons are frequently associated w ith coma, bilateral paralysis, decerebrate
37 rigidity, pinpoint pupils and often death.
38
39 (Choice D) Damage to the caudate nucleus is associated w ith transient hemiparesis as w ell as frontal lobe
40

~
41
42 F!!ck
---------------------------------------------------------------------------------------------------------------------------------------------
suW.nd EnQck
1
2
" Item: 21 of 45
Q.ld : 2077 [
111 f> Mark .<:J
Previous
[::>
Next
il
Lab Values
~
Notes
~
Calculator
3
4
5
6 Item 2 of 2
7
8 The patient w as diagnosed w ith a pure sensory stroke and received the appropriate treatment. Her symptoms
9 improved, and after a few w eeks of physical rehabilitation, she returned home to live w ith her daughter. Five
10 years later, the patient dies of a large m yocardial infarction. On autopsy, there are two 5-6 mm cavities in the
11 deep structures of her brain filled w ith clear fluid. W hich of the following best explains the autopsy findings?
12
13
14
0 A. Carotid atherosclerosis
15 0 B. Malignant cell infiltration
16
0 C. Mitral valve disease
17
18 0 D. Oligodendrocyte apoptosis
19
0 E. Small vessellipohyalinosis

22
23
24
25
26
27
28
29
30
31
32
33
34
35
36
37
38
39
40
41
42
~imeRemaini~ ~ 0
I Tuto' Feedback Suspend End Block
---------------------------------------------------------------------------------------------------------------------------------------------
1
2
" Item: 21 of 45
Q.ld : 2077 [
111 f> Mark .<:J
Previous
[::>
Next
il
Lab Values
~
Notes
~
Calculator
3
4 0 D. Oligodendrocyte apoptosis [36%]
5
6 " @ E. Small vessellipohyalinosis [36%]
7
8 Explanation: User ld:
9
10 Small, approximately 5mm cavities located w ithin the basal ganglia, posterior limb of the internal capsule,
11 pons, and cerebellum are know n as lacunar infarctions. These infarctions result from occlusion of the small
12 penetrating arteries that supply these structures, most commonly in the setting of chronic uncontrolled
13 hypertension or diabetes mellitus.
14
15 Lipohyalinosis and microatheromas are believed to be the primary causes of lacunar infarcts. Lipohyalinosis
16 is a destructive vessel lesion characterized by loss of normal arterial architecture, mural foam cells, and, in
17 acute cases, evidence of fibrinoid vessel w all necrosis. Microatheromas result from an accumulation of
18 lipid-laden macrophages w ithin the intimal layer of a vessel. In the case of lacunar infarcts, they cause
19 occlusion of a penetrating artery at or near its origin off the parent vessel.

(Choices A & C) Mitral valve disease and carotid atherosclerosis commonly cause cerebral infarction
22 secondary to embolism. They can also release microemboli, w hich are thought to cause lacunar infarcts
23 w hen no other pathological cause can be identified. How ever, lipohyalinosis and microatheroma are more
24 commonly identified as the cause of lacunar infarcts at the time of autopsy.
25
26 (Choice B) Malignant cell infiltration w ould tend to cause a space-occupying lesion rather than a small
27 cavity. The center of a large tumor may form a cavity due to insufficient perfusion w ith resultant necrosis, but
28 histopathologically, tumor w ould be identified around such a cavity. This patient's history is more consistent
29 w ith lacunar strokes.
30
(Choice D) Oligodendrocyte apoptosis occurs in multiple sclerosis, a demyelinating disease of the central
31
nervous system . The CNS lesions in multiple sclerosis do not form cavities; rather, the lesions are grossly
32
evident as pink patches in the w hite matter tracts.
33
34 Educational objective:
35 Lacunar infarctions are the result of small vessellipohyalinosis and atherosclerosis involving the penetrating
36 vessels supplying the deep brain structures. Uncontrolled hypertension and diabetes mellitus are risk factors
37 for this condition .
38
39
Time Spent: 2 seconds Copyright USMLEW orld,LLC. Last updated: [1 / 13/2014]
40
41
42
~imeRemaini~ ~ 0
I Tuto' Feedback Suspend End Block
---------------------------------------------------------------------------------------------------------------------------------------------
1
2
"
Item: 22 of 45
Q.ld: 1536 [
111 f> Mark .<:J
Previous
[::>
Next
il
Lab Values
~
Notes
~
Calculator
3
4
5
6 Muscle rigidity is observed in an experimental animal that has chemically-destroyed dopaminergic neurons of
7 the substantia nigra. The animal's rigidity fails to improve w ith continuous dopamine infusion. W hich of the
8 follow ing cell communications account for the lack of responsiveness to dopamine?
9
10
11 0 A. Desmosomes
12 0 B. Hemidesmosomes
13
14 0 C. Intermediate junctions
15 0 D. Fenestrae
16
17 0 E. Gap junctions
18
0 F. Tight junctions

23
24
25
26
27
28
29
30
31
32
33
34
35
36
37
38
39
40
41
42
~imeRemaini~ ~ 0
I Tuto' Feedback Suspend End Block
---------------------------------------------------------------------------------------------------------------------------------------------
1
2
"
Item: 22 of 45
Q.ld: 1536 [
111 f> Mark .<:J
Previous
[::>
Next
il
Lab Values
~
Notes
~
Calculator
3
4
5
Iv @ F. Tight junctions [57%]

6
7 Explanation: User ld:
8
9
10
11
12
13
14
15 Tight junctions
16
17
18

23
24
25
26
27
28
29
30
31
32
33
34
35
36
(~--------~~===-------~)
rype 4 collagen
37
38 The experimental animal is unresponsive to an intravenous dopamine infusion due to the fact that intravenous
39 dopamine is unable to cross the blood-brain barrier. Capillaries of the blood-brain barrier are not fenestrated,
40 so paracellular passage of fluid and dissolved material does not occur in the CNS. The primary mediators of
41
42
~imeRemaini~ ~ 0
I Tuto' Feedback Suspend End Block
---------------------------------------------------------------------------------------------------------------------------------------------
1
2
"
Item: 22 of 45
Q.ld: 1536 [
111 f> Mark .<:J
Previous
[::>
Next
il
Lab Values
~
Notes
~
Calculator
3
4
5

(~--------~~===-------~)
6
7 I ype 4 collagen
8
9 The experimental animal is unresponsive to an intravenous dopamine infusion due to the fact that intravenous
10 dopamine is unable to cross the blood-brain barrier. Capillaries of the blood-brain barrier are not fenestrated,
11
so paracellular passage of fluid and dissolved material does not occur in the CNS. The primary mediators of
12 the blood-brain barrier are tight junctions between the endothelial cells of CNS capillaries. Tight junctions,
13 also know n as zonula occludens, are formed via the interaction of specialized transmembrane proteins w ith
14
one another, such as occludens and claudens, on capillary endothelial cells. Due to tight junctions, the only
15 w ay that material can pass from the bloodstream to the brain is by transcellular movement, w hich is limited by
16 the size and lipophilicity of the molecule and by carrier-mediated transport.
17
18 (Choice A) Desmosomes are referred to as "spot w elds" and function as anchors holding adjoining cells
together. They are most notable in the skin w here they can be observed microscopically in the stratum
spinosum .

(Choice B) Hemidesmosomes are similar to desmosomes, but they serve to bind the basal layer of epithelial
23 cells to the basement membrane. Diseases of the hemidesmosome in the skin include bullous pemphigoid
24 and junctional epidermolysis bullosa .
25
26 (Choice C) Intermediate junctions occur on surface epithelial cells below tight junctions and assist in cell-cell
27 adhesion.
28
29 (Choice D) Fenestrae are small pores in endothelial cells that allow free exchange of intravascular fluid w ith
30 extravascular fluid in the tissue surrounding the capillary bed.
31
(Choice E) Gap junctions are channels between cells that allow the free exchange of solute and fluid. They
32
are typically formed by connexin proteins.
33
34 Educational Objective:
35 The tight junctions between endothelial cells in the capillary beds of the CNS form the blood-brain barrier.
36 Solutes and fluids cannot move freely across the capillary membrane in the CNS; material can only move
37 transcellularly by diffusing across the epithelial plasma membranes or by carrier-mediated transport.
38
39
Time Spent: 1 seconds Copyright USMLEW orld,LLC. Last updated: [7/7/201 0]
40
41
42
~imeRemaini~ ~ 0
I Tuto' Feedback Suspend End Block
---------------------------------------------------------------------------------------------------------------------------------------------
1
2
" Item: 23 of 45
Q.ld: 1815 [
111 f> Mark .<:J
Previous Next
[::> il
Lab Values
~
Notes
~
Calculator
3
4
5
6 The glossopharyngeal nerve is transected accidentally during a surgical procedure in a 45-year-old male.
7 W hich of the follow ing is most likely lost in this patient?
8
9
10 0 A. Taste sensation from the anterior two-thirds of the tongue
11 0 B. General sensation from the tonsillar lining
12
13 0 C. Salivary secretion from the submandibular gland
14
0 D. Protrusion of the tongue
15
16
17
18
19

[ill
1
22

24
25
26
27
28
29
30
31
32
33
34
35
36
37
38
39
40
41
42
~imeRemaini~ ~ 0
I Tuto' Feedback Suspend End Block
---------------------------------------------------------------------------------------------------------------------------------------------
1
2
" Item: 23 of 45
Q.ld: 1815 [
111 f> Mark .<:J
Previous
[::>
Next
il
Lab Values
~
Notes
~
Calculator
3
4
~ @ B. General sensation from the tonsillar lining [63%]
5
6 0 C. Salivary secretion from the submandibular gland (14%]
7 0 D. Protrusion of the tongue [12%]
8
9
10 Explanation: User ld:
11
12 The glossopharyngeal nerve (CN IX} originates in the medulla and exits the cranial cavity via the jugular
13 foramen . This nerve has numerous functions:
14
15 Somatic motor. stylopharyngeus muscle only
16 Parasympathetic: inferior salivatory nucleus-eN IX-otic ganglion- travels along auriculotemporal
17 nerve (CN V)-parotid gland secretion
18 General sensory. inner surface of tympanic membrane, Eustachian tube, posterior 1/3 of tongue,
19 tonsillar region, and upper pharynx (afferent portion of the gag reflex), carotid body and carotid sinus

[ill
1 Lesions of the glossopharyngeal nerve result in loss of the gag reflex (afferent limb), loss of sensation in the
22 upper pharynx, posterior tongue, tonsils and middle ear cavity, and loss of taste sensation on the posterior
one-third of the tongue.
24
25 (Choice A) Taste sensation from the anterior two-thirds of the tongue is mediated by the chorda tympani
26 branch of the facial nerve (CN VII).
27
28 (Choice C) Salivary secretion from the submandibular gland and the sublingual gland is mediated by
29 parasympathetic fibers originating in the superior salivatory nucleus carried on the facial nerve (CN VII) via the
30 chorda tympani nerve, the lingual nerve, and across the submandibular ganglion.
31
(Choice D) Protrusion of the tongue is mediated by motor efferent fibers carried by the hypoglossal nerve (CN
32
XII).
33
34 Educational Objective:
35 Lesions of the glossopharyngeal nerve result in loss of the gag reflex (afferent limb), loss of sensation in the
36 upper pharynx, posterior tongue, tonsils, and middle ear cavity, and loss of taste sensation on the posterior
37 one-third of the tongue.
38
39
Time Spent: 1 seconds Copyright USMLEW orld,LLC. Last updated: [7/7/201 OJ
40
41
42
~imeRemaini~ ~ 0
I Tuto' Feedback Suspend End Block
--------------------------------------------------------------------------------------------------------------------------------------
1
2
" Item: 24 of 45
Q.ld: 1689 [
111 f> Mark .<:J
Previous
[::>
Next
il
Lab Values
~
Notes
~
Calculator
3
4
5
6 A 54-year-old Caucasian male dies from a progressive neurological disorder. A brain section of the patient is
7 show n below .
8
9
10
11


12
13
14
15
16
17
18
19

[ill
1
22
23

25
26 Dlspl1yed with permi ssion from Sprlnaer Hnlthc.1re Ltd.
27 OCopyrl1ht 2002 by Current Medicine
28
29
30 W hich of the follow ing corresponds to the structure w ith cystic degeneration (black arrow)?
31
32 0 A. Globus pallidus
33
34 0 B. Putamen
35 0 C. Internal capsule
36 0 D. Caudate nucleus
37
38
0 E. Amygdala
39
40
41
42
~imeRemaini~ ~ 0
I Tuto' Feedback Suspend End Block
---------------------------------------------------------------------------------------------------------------------------------------------
1
2
" Item: 24 of 45
Q.ld: 1689 [
111 f> Mark .<:J
Previous
[::>
Next
il
Lab Values
~
Notes
~
Calculator
3
4 ol @ B. Putamen [51%]
5
6 0 C. Internal capsule [1 0%]
7 0 D. Caudate nucleus [16%]
8 0 E. Amygdala [3%]
9
10
11 Explanation: User ld:
12
13 Many progressive neurologic diseases may be associated w ith cystic degeneration of the putamen, but
14 Wilson's disease is the most likely of these to be emphasized on the USMLE.
15
16 The putamen (Choice B) can be identified on coronal sections (either in gross specimens as above or on
17 imaging studies) in a location immediately deep (medial) to the insula and lateral to the globus pallidus and
18 internal capsule.
19

[ill
1
22
23

25
II
26
27
28
29
30
31
32
33
34
35
I
36
37 Dlspl1yed with permission from Sprlncer Hnlthcue Ltd.

38 OCopyrl.cht 2002 by Current Medicine


39
40

~
41
42 F!!ck suW.nd
---------------------------------------------------------------------------------------------------------------------------------------------
EnQck
1
2
3
4
5
6
7
8
9
10
11
12
13
14
15
16
Dlspl1yed with permi ssion from Sprlnaer Hnlthc.1re Ltd.
17
OCopyrl1ht 2002 by Current Medicine
18
19

[ill
1
(Choice A) The globus pallidus is the orange-colored structure on the above image immediately medial to the
degenerated putamen . It is considered part of the basal ganglia along w ith the caudate and putamen.
22
23 (Choice C) The internal capsule is the w hite matter structure that separates the globus pallidus and putamen
from the caudate nucleus anteriorly and from the thalamus posteriorly on coronal sections.
25
26 (Choice D) In its full three dimensional structure, the caudate nucleus is a curved or "C-shaped" structure
27 that can be identified in the inferolateral w alls of the anterior horns of the lateral ventricles. How ever, only a
28 small cross-section is visible in the image above next to the lateral ventricles. This structure is affected in
29 Huntington's disease.
30
(Choice E) The amygdala is a group of neurons in the medial temporal lobe located adjacent to the
31
hippocampus that plays a role in the limbic system . The amygdala can be identified on the image above as
32
the circular grey matter structures in the medial temporal lobe immediately inferior to the globus pallidus and
33
putamen . The amygdala mediates fear and some sympathetic stimulation in humans.
34
35 Educational Objective:
36 W ilson's disease can cause cystic degeneration of the putamen as w ell as damage to other basal ganglia
37 structures. The putamen is located medial to the insula and lateral to the globus pallidus on coronal sections.
38
39
Time Spent: 2 seconds Copyright USMLEW orld,LLC. Last updated: [1 / 15/2014]
40
41
42
~imeRemaini~ ~ 0
I Tuto' Feedback Suspend End Block
---------------------------------------------------------------------------------------------------------------------------------------------
1
2
" Item: 25 of 45
Q.ld : 2127 [
111 f> Mark .<:J
Previous
[::>
Next
il
Lab Values
~
Notes
~
Calculator
3
4
A 70-year-old right-handed w oman comes to the emergency department w ith acute loss of vision on her left
5
side. She has no headache, slurred speech, difficulty sw allow ing, w eakness, or difficulty w alking . Her other
6
medical problems include hypertension, atrial fibrillation, and hyperlipidemia . Her blood pressure is 170/ 102
7
mm Hg and pulse is 90/min and irregular. On neurologic examination, she is alert and oriented to person,
8
place, and time. Her speech is fluent, and she follow s complex commands. Examination of her visual fields
9
reveals a left homonymous hemianopia w ith macular sparing. There is decreased sensation over the left side
10
of her body. The rest of the neurologic examination is w ithin normal limits. Brain magnetic resonance
11
imaging w ith diffusion protocol reveals an area of restricted diffusion, as show n below .
12
13
14
15
16
17
18
19

[ill
1
22
23
24

26
27
28
29
30
31
32
33
34
35
36
37
38
39
40
41
42
1
2
" Item: 25 of 45
Q.ld : 2127 [
111 f> Mark .<:J
Previous
[::>
Next
il
Lab Values
~
Notes
~
Calculator
3
U l 111::1 UUUY . I l it I t'~l U l li l t I I CUI UIUYI\.. t.X.CIIIIIIICIUUII I~ VVIU 1111 I IU I I I ICII Ill I Ill~ . 0 1 Clll l I I ICIYI Itlll...- I t~UIICIIII,.t:
4
5 imaging w ith diffusion protocol reveals an area of restricted diffusion, as show n below .
6
7
8
9
10
11
12
13
14
15
16
17
18
19

[ill
1
22
23
24

26
27
28
29
30
31
32
33
34
35
36
37
38 W hich of the follow ing cerebral arteries is most likely to be compromised in this patient?
39
40

~
41
42
--------------------------------------------------------------------------------------------------------------------------------------------------------------
F!!ck suW.nd EnQck
1
2
3
4
5
6
7
8
9
10
11
12
13
14
15
16
17
18
19

[ill
1
22
23
24

26
27
28
29
30 W hich of the follow ing cerebral arteries is most likely to be compromised in this patient?
31
32
33 0 A. Anterior cerebral artery
34 0 B. Anterior choroidal artery
35 0 C. Artery of Perc heron
36
37 0 D. Basilar artery
38 0 E. Middle cerebral artery
39 0 F. Posterior cerebral artery
40

~
41
42 F!!ck suW.nd EnQck
---------------------------------------------------------------------------------------------------------------------------------------------
1
2
3
4
5
6
7 Explanation: User ld:
8
9 Blood supply to the Cerebral Hemispheres
10
11
ACA
12 Head of caudate nucleus
13 MCA, superior d ivision
14
15 Internal capsule ACA, deep branches
16 (anterior Iimb)
17
18
19
Lentiform nucleus
[ill
22
23 inferior division
24 Globus
pallid us
26
27
28
29 Anterior
30 choroidal artery
31 Internal capsule
32 (posterior limb)
33
34
35 Thalamus
36
37
38
39
40
41
42
1
2
3
4
5 Occipital lobe
6
7 USMLWorid,llC 0 2011
PCA
8
9 This patient likely suffered an embolic stroke due to her underlying atrial fibrillation. Axial MRI of her brain
10 show s an area of restricted diffusion consistent w ith an acute infarct in the distribution of the right posterior
11 cerebral artery (PCA). (Axial images are interpreted as if view ed from the patient's feet, so the patient's right
12 side is show n on the left side of the image. Also note that diffusion-w eighted sequences have relatively low
13 resolution compared to T1 & T2 images, thus they often appear "blurry".) The PCA branches off the basilar
14 artery at the level of the pontomesencephalic junction and supplies cranial nerves Ill and IV and other
15 structures in the midbrain. It also supplies the thalamus, medial temporal lobe, splenium of the corpus
16 callosum, parahippocampal gyrus, fusiform gyrus, and occipital lobe.
17
18 The most common finding in PCA territory infarction is contralateral hemianopia, often w ith macular sparing
19 due to collateral circulation from the middle cerebral artery. Involvement of the lateral thalamus frequently

[ill
1
results in contralateral paresthesias and numbness affecting the face, trunk, and limbs. A variety of
cortical symptoms can also occur, including dyslexia, visual agnosia (ie, impaired visual recognition of
22 objects), and prosopagnosia (ie, inability to recognize faces).
23
24 (Choice A) The anterior cerebral artery distribution includes the inferior frontal, medial frontal, and parietal
lobes; anterior 4/5 of the corpus callosum; olfactory bulb and tract; anterior portions of the basal ganglia; and
26 internal capsule.
27
28 (Choice B) The anterior choroidal artery is the last branch off the internal carotid artery before it trifurcates. It
29 supplies the posterior limb of the internal capsule, optic tract, lateral geniculate body, choroid plexus, uncus,
30 hippocampus, and amygdala.
31
(Choice C) The artery of Perc heron branches off either the right or left PCA and supplies the bilateral thalami
32
and dorsal midbrain. It is considered to be a rare normal variant and can be seen in patients w ith bilateral
33
34 thalamic or dorsal midbrain strokes.
35 (Choice D) The basilar artery supplies a large portion of the brain, including most of the brainstem and
36 bilateral cerebellar hemispheres.
37
38 (Choice E) The middle cerebral artery distribution includes lateral convexity of the frontal, parietal, and
39 temporal lobes. It also supplies deep subcortical structures, such as the internal capsule and basal ganglia.
40 ........................ : ... :+1..: .... ...... : ............. ;..................... : ....... ...........+ ...... 1.-.+...........
'"""'"..... . , .... - - - ....... .................4- & ...... ,... .........'" ..........+ ......................................... . ...1 ....&: ... :+....

~
41
42
---------------------------------------------------------------------------------------------------------------------------------------------
F!!ck suW.nd EnQck
1
2
" Item: 25 of 45
Q.ld : 2127 [
111 f> Mark .<:J
Previous
[::>
Next
il
Lab Values
~
Notes
~
Calculator
3
''-'~VIUUVI I .... VI I 1pU1 '-''-" ~V I I U. I L. II IIU ~'-'"' ) U IU"' U 1'-' J V l~'-'1 I Upp'-'UI U""' I J / I I 1'-' J "'-Jr\ Ul Ul I .... I ''-'"' V II U ''-' UU..,IIUI
4
artery at the level of the pontomesencephalic junction and supplies cranial nerves Ill and IV and other
5
structures in the midbrain. It also supplies the thalamus, medial temporal lobe, splenium of the corpus
6
callosum, parahippocampal gyrus, fusiform gyrus, and occipital lobe.
7
8 The most common finding in PCA territory infarction is contralateral hemianopia, often w ith macular sparing
9 due to collateral circulation from the middle cerebral artery. Involvement of the lateral thalamus frequently
10 results in contralateral paresthesias and numbness affecting the face, trunk, and limbs. A variety of
11 cortical symptoms can also occur, including dyslexia, visual agnosia (ie, impaired visual recognition of
12 objects), and prosopagnosia (ie, inability to recognize faces).
13
14 (Choice A) The anterior cerebral artery distribution includes the inferior frontal, medial frontal, and parietal
15 lobes; anterior 4/5 of the corpus callosum; olfactory bulb and tract; anterior portions of the basal ganglia; and
16 internal capsule.
17
18 (Choice B) The anterior choroidal artery is the last branch off the internal carotid artery before it trifurcates. It
19 supplies the posterior limb of the internal capsule, optic tract, lateral geniculate body, choroid plexus, uncus,

[ill
1
hippocampus, and amygdala.

(Choice C) The artery of Perc heron branches off either the right or left PCA and supplies the bilateral thalami
22
23 and dorsal midbrain. It is considered to be a rare normal variant and can be seen in patients w ith bilateral
24 thalamic or dorsal midbrain strokes.

26 (Choice D) The basilar artery supplies a large portion of the brain, including most of the brainstem and
27 bilateral cerebellar hemispheres.
28
(Choice E) The middle cerebral artery distribution includes lateral convexity of the frontal, parietal, and
29
temporal lobes. It also supplies deep subcortical structures, such as the internal capsule and basal ganglia.
30
Ischemic stroke occurs most frequently w ithin this region, causing contralateral motor and sensory deficits
31
involving the upper and low er extremities and face.
32
33 Educational objective:
34 The posterior cerebral artery branches off the basilar artery and supplies cranial nerves Ill and IV and other
35 structures in the midbrain. It also supplies the thalamus, medial temporal lobe, splenium of the corpus
36 callosum, parahippocampal gyrus, fusiform gyrus, and occipital lobe. The most common finding w ith
37 posterior cerebral artery stroke is contralateral homonymous hemianopia, often w ith macular sparing.
38
39
Time Spent: 15 seconds Copyright USMLEWorld,LLC. Last updated: [4/2/2014]
40
41
42
~imeRemaini~ ~ 0
I Tuto' Feedback Suspend End Block
---------------------------------------------------------------------------------------------------------------------------------------------
1
2
" Item: 26 of 45
Q.ld: 8592 [
111 f> Mark .<:J
Previous
[::>
Next
il
Lab Values
~
Notes
~
Calculator
3
4
5
6 A 56-year-old woman is brought to the emergency department w ith sudden onset of headache and partial loss
7 of vision. She has a 3-year history of atrial fibrillation. Physical examination show s a visual field defect
8 involving the black areas illustrated below.
9
10 Left Right
11
12
13 Temporal Nasal Temporal
14
15
16
17
18 This patient most likely has occlusion of w hich of the follow ing arteries?
19

[ill
1
0 A. Anterior cerebral
0 B. Central retinal
22
23 0 C. Middle cerebral
24
0 D. Posterior cerebral
25
0 E. Posterior inferior cerebellar
27
28
29
30
31
32
33
34
35
36
37
38
39
40
41
42
~imeRemaini~ ~ 0
I Tuto' Feedback Suspend End Block
---------------------------------------------------------------------------------------------------------------------------------------------
1
2
" Item: 26 of 45
Q.ld: 8592 [
111 f> Mark .<:J
Previous
[::>
Next
il
Lab Values
~
Notes
~
Calculator
3
4 v @ D. Posterior cerebral [75%]
5
6 0 E. Posterior inferior cerebellar [4%]
7
8 Explanation: User ld:
9
10 A. Monocular scotoma
11 Vi sual pathways from above
12
13
14
15
OQ
B. Right anopia

16
17
18
19
0
C. Bitemporal hemianopia

[ill
1
22 Optic ne1rve-..._
(a
23 D. Right nasal hemianopia
24
25

27
Optic chiasma
0 ()
E. Left homonymous hemianopia
28
29
30
31
()
32 F. Left homonymous superior
33 quadrantanopla ("pie in the sky")
Lateral
34 geniculate
35 nucleus
36
37 G. Left homonymous inferior
38 quadrant anopia ("pie on t he floor")
39
40
41
42
1
2
" Item: 26 of 45
Q.ld: 8592 [
111 f> Mark .<:J
Previous
[::>
Next
il
Lab Values
~
Notes
~
Calculator
3
4 A. Monocular scotoma
5 Vi sual pathways from above
6
7
8
OQ
B. Right anopia
9
10
11
12
13
0
C. Bitemporal hemianopia
14
15
16 Optic ne1rve-..._
(a
17 D. Right nasal hemianopia
18
19

[ill
1
Optic chiasma
0 ()
E. Left homonymous hemianopia
22
23
24
25
()
F. Left homonymous superior
quadrantanopla ("pie in the sky")
27 Lateral
28 geniculate
29 nucleus
30
31 G. Left homonymous inferior
32 quadrant anopia ("pie on the floor")
33
34
Optic radiation
35
36 H. Left homonymous hemianopia
37 H w ith macular sparing
38
39
40
a) C)
~ F!!ck
41
42 suW.nd EnQ ck
------------------------------------------------------------------------------------------------------------------------------------------------------
1
2
" Item: 26 of 45
Q.ld: 8592 [
111 f> Mark .<:J
Previous
[::>
Next
il
Lab Values
~
Notes
~
Calculator
3
... -- - -- -- -; -- ---- -- --- -
4 H with macular sparing
5
6
7
8
~-~ USMLEWorld. LlC
C) ~
9 The described vision defect is consistent w ith an occlusion of the posterior cerebral artery (PCA), most likely
10 from an embolic event given this patient's history of atrial fibrillation. The PCA supplies the occipital lobe,
11 w hich contains the striate or primary visual cortex. The striate cortex of each hemisphere receives
12 information about the contralateral visual field from the ipsilateral lateral geniculate nucleus via the optic
13 radiation . Specifically, the cuneus gyrus of the striate cortex receives information from the upper retina (low er
14 visual field), w hile the lingual gyrus of the striate cortex receives information from the low er retina (upper visual
15 field).
16
17 Occlusion of the PCA leads to ischemic injury to the ipsilateral striate cortex, producing a contralateral
18 homonymous hemianopia w ith macular sparing. The macula is spared because collateral blood is supplied
19 by the middle cerebral artery to the occipital pole, w hich processes central visual information . Unilateral

[ill
1
infarcts involving only the cuneus gyrus or the lingual gyrus w ill produce a low er or upper quadrantanopia,
respectively.
22
23 (Choice A) Anterior cerebral artery occlusion manifests as contralateral w eakness that predominantly affects
24 the low er extremity.
25
(Choice B) Occlusion of the central retinal artery leads to sudden, painless, and complete loss of vision in
27 the affected eye.
28
29 (Choice C) Middle cerebral artery stroke is characterized by contralateral motor and sensory deficits (more
30 pronounced in the upper than low er limb) and homonymous hemianopia with macular involvement.
31
(Choice E) Occlusion of the posterior inferior cerebellar artery leads to lateral medullary (Wallenberg)
32
syndrome. It is characterized by contralateral loss of pain and temperature sensation; ipsilateral deficits of
33
34 CN V, VIII, IX, X, and XI; and Horner's syndrome.
35 Educational objective:
36 Lesions of the occipital cortex (eg, posterior cerebral artery occlusion) can produce contralateral
37 homonymous hemianopia w ith macular sparing.
38
39
Time Spent: 3 seconds Copyright USMLEW orld,LLC. Last updated: [213/2014]
40
41
42
~imeRemaini~ ~ 0
I Tuto' Feedback Suspend End Block
---------------------------------------------------------------------------------------------------------------------------------------------
1
2
" Item: 27 of 45
Q.ld: 1904 [
111 f> Mark .<:J
Previous
[::>
Next
il
Lab Values
~
Notes
~
Calculator
3
4
5
6 A 16-year-old male presents to your office w ith difficulty w alking. On physical examination, he has thoracic
7 kyphoscoliosis and multiple small scars on his hands. W hen asked about these, he reports that he
8 repeatedly accidentally burns his hands because he "cannot feel the heat." Neurologic examination reveals
9 1+ biceps reflexes and 3+ patellar reflexes bilaterally, as w ell as decreased muscle strength in the bilateral
10 upper and low er extremities. W hich of the follow ing is the most likely cause of this patient's problems?
11
12
13 0 A. Amyotrophic lateral sclerosis
14 0 B. Guillain-Barre syndrome
15
16 0 C. Syringomyelia
17 0 D. Normal pressure hydrocephalus
18
19 0 E. Multiple sclerosis

[ill
1
0 F. Medulloblastoma
22
23
24
25
26

28
29
30
31
32
33
34
35
36
37
38
39
40
41
42
~imeRemaini~ ~ 0
I Tuto' Feedback Suspend End Block
---------------------------------------------------------------------------------------------------------------------------------------------
1
2
" Item: 27 of 45
Q.ld: 1904 [
111 f> Mark .<:J
Previous
[::>
Next
il
Lab Values
~
Notes
~
Calculator
3
4 .; @ C. Syringomyelia [63%]
5 0 D. Normal pressure hydrocephalus [1%]
6
7 0 E. Multiple sclerosis [5%]
8 0 F. Medulloblastoma [3%]
9
10
11 Explanation: User ld:
12
13 This patient has chronic loss of upper extremity pain and temperature sensation, upper extremity w eakness
14 and hyporeflexia, low er extremity w eakness and hyperreflexia, and kyphoscoliosis. These findings are
15 consistent w ith a diagnosis of syringomyelia.
16
In syringomyelia, a central cystic dilation in the cervical spinal cord (a syrinx) slow ly enlarges,
17
characteristically causing damage to the ventral w hite commissure and anterior horns. The ventral w hite
18
commissure is the site of decussation of second-order lateral spinothalamic tract neurons, and the ventral
19
horns are the site of low er motor neuron cell bodies. The syrinx is most commonly situated at the C8-T1 cord
[ill
1
levels and may extend rostrally, caudally, and centrifugally. Further expansion w ithin the cervical cord in later
stages of the disease can produce lower extremity weakness and hyperreflexia (upper motor neuron defect)
22
by affecting the lateral corticospinal tract.
23
24 (Choice A) ALS is a progressive degenerative disease of upper and lower motor neurons only; it does not
25 cause sensory loss.
26
(Choice B) Guillain-Barre syndrome is a peripheral nervous system autoimmune demyelinating
28 polyneuropathy that typically causes lower motor neuron manifestations (e.g. areflexia, w eakness) in a
29 symmetric ascending pattern.
30
31 (Choice D) Normal pressure hydrocephalus is characterized by gait apraxia and urinary incontinence. It is
32 rare in patients younger than 60 years of age. It arises due to defective CSF resorption by the arachnoid
33 granulations.
34
35 (Choice E) Multiple sclerosis is an autoimmune CNS demyelinating disorder that presents w ith episodic
36 neurologic symptoms reflecting defects in random w hite matter tracts.
37
38 (Choice F) Medulloblastoma is the most common malignant brain tumor of children. It is a posterior fossa
39 tumor that causes symptoms related to obstructive hydrocephalus.
40
41
42
~imeRemaini~ ~ 0
I Tuto' Feedback Suspend End Block
---------------------------------------------------------------------------------------------------------------------------------------------
1
2
" Item: 27 of 45
Q.ld: 1904 [
111 f> Mark .<:J
Previous
[::>
Next
il
Lab Values
~
Notes
~
Calculator
3
4
5 Explanation: User ld:
6
This patient has chronic loss of upper extremity pain and temperature sensation, upper extremity w eakness
7
and hyporeflexia, low er extremity w eakness and hyperreflexia, and kyphoscoliosis. These findings are
8
consistent w ith a diagnosis of syringomyelia.
9
10 In syringomyelia, a central cystic dilation in the cervical spinal cord (a syrinx) slow ly enlarges,
11 characteristically causing damage to the ventral w hite commissure and anterior horns. The ventral w hite
12 commissure is the site of decussation of second-order lateral spinothalamic tract neurons, and the ventral
13 horns are the site of low er motor neuron cell bodies. The syrinx is most commonly situated at the C8-T1 cord
14 levels and may extend rostrally, caudally, and centrifugally. Further expansion w ithin the cervical cord in later
15 stages of the disease can produce low er extremity w eakness and hyperreflexia (upper motor neuron defect)
16 by affecting the lateral corticospinal tract.
17
18 (Choice A) ALS is a progressive degenerative disease of upper and low er motor neurons only; it does not
19 cause sensory loss.

[ill
1 (Choice B) Guillain-Barre syndrome is a peripheral nervous system autoimmune demyelinating
22 polyneuropathy that typically causes low er motor neuron manifestations (e.g. areflexia, w eakness) in a
23 symmetric ascending pattern.
24
25 (Choice D) Normal pressure hydrocephalus is characterized by gait apraxia and urinary incontinence. It is
26 rare in patients younger than 60 years of age. It arises due to defective CSF resorption by the arachnoid
granulations.
28
29 (Choice E) Multiple sclerosis is an autoimmune CNS demyelinating disorder that presents w ith episodic
30 neurologic symptoms reflecting defects in random w hite matter tracts.
31
(Choice F) Medulloblastoma is the most common malignant brain tumor of children. It is a posterior fossa
32
tumor that causes symptoms related to obstructive hydrocephalus.
33
34 Educational Objective:
35 The combination of fixed segmental loss of upper extremity pain and temperature sensation, upper extremity
36 low er motor neuron signs, and/or low er extremity upper motor neuron signs in the setting of scoliosis
37 suggests a diagnosis of syringomyelia .
38
39
Time Spent: 1 seconds Copyright USMLEW orld,LLC. Last updated: [7/7/201 0]
40
41
42
~imeRemaini~ ~ 0
I Tuto' Feedback Suspend End Block
---------------------------------------------------------------------------------------------------------------------------------------------
1
2
" Item: 28 of 45
Q.ld: 1693 [
111 f> Mark .<:J
Previous
[::>
Next
il
Lab Values
~
Notes
~
Calculator
3
4
5
6 A 53-year-old man comes to the emergency department because of severe low back pain. His back pain
7 started 2 w eeks ago w ithout history of trauma and has been keeping him aw ake at night. The pain has
8 acutely w orsened over the past 2 days and is not relieved w ith over-the-counter pain medications. He takes
9 no other medication. He has no know n drug allergies. He does not use tobacco, alcohol, or illicit drugs. His
10 vital signs are w ithin normal limits. Examination show s pain in the low back region w ith flexing of the back and
11 raising of the legs. Pinprick in the perianal area does not cause rapid contraction of the anal sphincter. W hich
12 of the follow ing nerve roots is most likely to be involved in this patient's condition?
13
14
15 0 A. T12
16 0 B. L2
17
18 0 C.L4
19 0 D.L5
[ill
1 0 E. S1
22 0 F. S4
23
24
25
26
27

29
30
31
32
33
34
35
36
37
38
39
40
41
42
~imeRemaini~ ~ 0
I Tuto' Feedback Suspend End Block
---------------------------------------------------------------------------------------------------------------------------------------------
1
2
" Item: 28 of 45
Q.ld: 1693 [
111 f> Mark .<:J
Previous
[::>
Next
il
Lab Values
~
Notes
~
Calculator
3
4 I .; @ F. S4 [51 %]
5
6
7 Explanation: User ld:
8
9 Dermatomes
10
11
12
13
14
15
16
17
18
19

[ill
1
22
23
24
25
26
27

29
30
31
32
33
34
35
36
37
38
39 VSMI..tWodd. uc (12011
.."
40
41
42
~imeRemaini~ ~ 0
I Tuto' Feedback Suspend End Block
------------------------------------------------------------------------------------------------------------------------------------------------------
1
2
" Item: 28 of 45
Q.ld: 1693 [
111 f> Mark .<:J
Previous
[::>
Next
il
Lab Values
~
Notes
~
Calculator
3
4
In an adult, the spinal cord terminates in a tapering fashion at the conus medullaris at approximately the L2
5
vertebral level. After this point, spinal nerves from the conus medullaris exit inferiorly through their respective
6
intervertebral foramina. This collection of spinal nerves (now considered peripheral nerves) is referred to as
7
the cauda equina (i.e., horse's tail). Conus medullaris syndrome refers to lesions at L2. It has symptoms of
8
flaccid paralysis of the bladder and rectum, impotence, and saddle (S3-S5 roots) anesthesia . There is usually
9
mild w eakness of the leg muscle if the lesion spares both the lumbar cord and the adjacent spinal and lumbar
10
nerve roots. Common causes include disk herniation, tumors, and spinal fractures.
11
12 In contrast, cauda equina syndrome typically results from a massive rupture of an intervertebral disk that is
13 capable of causing compression of two or more of the 18 spinal nerve roots of the cauda equina. How ever, it
14 can also occur due to any trauma or space-occupying lesion of the low er vertebral column. The cauda equina
15 nerve roots provide the sensory and motor innervation of most of the low er extremities, the pelvic floor, and
16 the sphincters.
17
18 Classic symptoms of cauda equina syndrome include low back pain radiating to one or both legs, saddle
19 anesthesia, loss of anocutaneous reflex (as in this patient), bow el and bladder dysfunction (S3-S5 roots), and

[ill
1
loss of ankle-jerk reflex w ith plantar flexion w eakness of the feet. Of the potentially involved spinal nerves, a
lesion involving S2 through S4 w ill cause the symptoms described in this patient, indicating impairment of the
22 pudendal nerve that innervates the perineum .
23
24 (Choice A) Compression at T12 results in impingement of the spinal cord w here the lumbar spinal nerves
25 originate.
26
27 (Choice B) L2 lesions cause anesthesia or paresthesias posteriorly in the gluteal region inferior to the iliac
crests and anteriorly on the thigh inferior to the femoral triangle and inguinal ligament.
29
30 (Choice C) The L3 and L4 spinal levels contribute to the femoral nerve and mediate the quadriceps (i.e.,
31 knee-jerk and patellar) reflex. Lesions in these levels cause loss of this reflex as w ell as paresthesias and
32 w eakness over the anterior thigh.
33
(Choice D) L5 lesions cause sensory loss or paresthesias over the anterior leg.
34
35 (Choice E) The S1 and S2 roots mediate the Achilles (i.e., ankle-jerk) reflex, w hich tests for the integrity of
36 the sciatic nerve. Lesions in these roots cause sensory loss or paresthesias over the posterior leg and thigh.
37
38 Educational objective:
39 Saddle anesthesia and loss of the anocutaneous reflex are symptoms of cauda equina syndrome, w hich is
40

~
41
42 F!!ck
---------------------------------------------------------------------------------------------------------------------------------------------
suW.nd EnQck
1
2
" Item: 28 of 45
Q.ld: 1693 [
111 f> Mark .<:J
Previous
[::>
Next
il
Lab Values
~
Notes
~
Calculator
3
4
5 In contrast, cauda equina syndrome typically results from a massive rupture of an intervertebral disk that is
6 capable of causing compression of two or more of the 18 spinal nerve roots of the cauda equina. How ever, it
7 can also occur due to any trauma or space-occupying lesion of the low er vertebral column. The cauda equina
8 nerve roots provide the sensory and motor innervation of most of the low er extremities, the pelvic floor, and
9 the sphincters.
10
Classic symptoms of cauda equina syndrome include low back pain radiating to one or both legs, saddle
11
anesthesia, loss of anocutaneous reflex (as in this patient}, bow el and bladder dysfunction (S3-S5 roots}, and
12
loss of ankle-jerk reflex w ith plantar flexion w eakness of the feet. Of the potentially involved spinal nerves, a
13
lesion involving S2 through S4 w ill cause the symptoms described in this patient, indicating impairment of the
14
pudendal nerve that innervates the perineum .
15
16 (Choice A) Compression at T12 results in impingement of the spinal cord w here the lumbar spinal nerves
17 originate.
18
19 (Choice B) L2 lesions cause anesthesia or paresthesias posteriorly in the gluteal region inferior to the iliac

[ill
1
crests and anteriorly on the thigh inferior to the femoral triangle and inguinal ligament.

22 (Choice C) The L3 and L4 spinal levels contribute to the femoral nerve and mediate the quadriceps (i.e.,
23 knee-jerk and patellar} reflex. Lesions in these levels cause loss of this reflex as w ell as paresthesias and
24 w eakness over the anterior thigh .
25
26 (Choice D) L5 lesions cause sensory loss or paresthesias over the anterior leg.
27
(Choice E) The S 1 and S2 roots mediate the Achilles (i.e., ankle-jerk} reflex, w hich tests for the integrity of
29 the sciatic nerve. Lesions in these roots cause sensory loss or paresthesias over the posterior leg and thigh.
30
Educational objective:
31
Saddle anesthesia and loss of the anocutaneous reflex are symptoms of cauda equina syndrome, w hich is
32
associated w ith damage to the S2 through S4 nerve roots.
33
34
35 References:
36 1. Cauda equma syndrome
37
38 I
39
Time Spent: 1 seconds Copyright USMLEW orld,LLC. Last updated: [1 /3/2014]
40
41
42
~imeRemaini~ ~ 0
I Tuto' Feedback Suspend End Block
---------------------------------------------------------------------------------------------------------------------------------------------
1
2
" Item: 29 of 45
Q.ld: 1537 [
111 f> Mark .<:J
Previous
[::>
Next
il
Lab Values
~
Notes
~
Calculator
3
4
A 53-year-old man w ho has not seen a physician in years presents to your office complaining of abdominal
5
distention. He states "Last month m y stomach started to sw ell up and it hasn't gotten any better". He has no
6
other medical problems. The patient admits to drinking 10-12 beers a day for the last 20 years. His
7
temperature is 36.7 C (98 F), blood pressure is 116/72 mm Hg, pulse is 78/min and respirations are 20/min.
8
On examination his abdomen is distended w ith engorged paraumbilical veins. There is also palmar erythema
9
and multiple spider angiomas are present. You decided to place him on a low -salt diet and start therapy w ith
10
furosemide and spironolactone, w ith subsequent improvement of his abdominal distention . Before beginning
11
this patient's treatment, w hich of the follow ing structures labeled on the image below is expected to have an
12
increased pressure?
13
14
15
16
17
18
19

[ill
1
22
23
24
25
26
27
28

30
31
32
33
34
35
36
37
38
39 0 A. A
40 (J RR
41
42
~imeRemaini~ ~ 0
I Tuto' Feedback Suspend End Block
---------------------------------------------------------------------------------------------------------------------------------------------
1
2
" Item: 29 of 45
Q.ld: 1537 [
111 f> Mark .<:J
Previous
[::>
Next
il
Lab Values
~
Notes
~
Calculator
3
4 temperature is 36.7 C (98 F), blood pressure is 116/72 mm Hg, pulse is 78/min and respirations are 20/min.
5 On examination his abdomen is distended w ith engorged paraumbilical veins. There is also palmar erythema
6 and multiple spider angiomas are present. You decided to place him on a low -salt diet and start therapy w ith
7 furosemide and spironolactone, w ith subsequent improvement of his abdominal distention . Before beginning
8 this patient's treatment, w hich of the follow ing structures labeled on the image below is expected to have an
9 increased pressure?
10
11
12
13
14
15
16
17
18
19

[ill
1
22
23
24
25
26
27
28

30
31
32
33
34
35 0 A. A
36
37 0 B. B
38 0 C. C
39 0 D. D
40
41
42
~imeRemaini~ ~ 0
I Tuto' Feedback Suspend End Block
---------------------------------------------------------------------------------------------------------------------------------------------
1
2
" Item: 29 of 45
Q.ld: 1537 [
111 f> Mark .<:J
Previous
[::>
Next
il
Lab Values
~
Notes
~
Calculator
3
4 ~ <!J C. C [71 %]
5 0 D. D [6%]
6
7
8 Explanation: User ld:
9
10
11
12
13
14
15
16
17
18
19

[ill
1
22
23
24
25
26
27
28

30
31
32
33
34 The patient described in the question stem is likely suffering from alcoholic cirrhosis. Alcoholic cirrhosis is a
35 form of micronodular cirrhosis associated w ith hepatocyte death follow ed by fine fibrosis of the liver. As
36 cirrhosis w orsens and the number of functioning hepatocytes decreases, the functional ability of the liver
37 diminishes. In advanced disease, portal blood has an increasingly difficult time passing through the liver
38 because the vasculature becomes compromised by the progressive fibrosis, causing portal hypertension . Of
39 the structures identified on the image above, only the portal and splenic veins are part of the portal venous
40 system . In this patient, hiqh pressure w ould be expected throuqhout the portal system , includinq the superior
41
42
~imeRemaini~ ~ 0
I Tuto' Feedback
---------------------------------------------------------------------------------------------------------------------------------------------
Suspend End Block
1
2
3
4
5
6
7 The patient described in the question stem is likely suffering from alcoholic cirrhosis. Alcoholic cirrhosis is a
8 form of micronodular cirrhosis associated w ith hepatocyte death follow ed by fine fibrosis of the liver. As
9 cirrhosis w orsens and the number of functioning hepatocytes decreases, the functional ability of the liver
10 diminishes. In advanced disease, portal blood has an increasingly difficult time passing through the liver
11 because the vasculature becomes compromised by the progressive fibrosis, causing portal hypertension . Of
12 the structures identified on the image above, only the portal and splenic veins are part of the portal venous
13 system. In this patient, high pressure w ould be expected throughout the portal system, including the superior
14 mesenteric, portal, and splenic veins. The effects of prolonged portal hypertension include varices at the four
15 sites of portocaval anastomoses (esophagus, rectum, umbilicus, and retroperitoneal}, as w ell as ascites.
16
17 The pathogenesis of ascites in patients w ith cirrhosis is complex. In addition to mechanical compromise of
18 portal vein flow by fibrotic tissue, vasoactive agents also play a role by causing dilatation of the splanchnic
19 arterial vasculature and further intrahepatic vasoconstriction . These processes result in increased portal vein

[ill
1
hydrostatic pressure leading to ascitic fluid formation, as w ell as decreased systemic perfusion pressure.
The kidney senses the decreased perfusion pressure (accentuated by renal vasoconstriction in hepatorenal
22 syndrome) and responds w ith avid retention of sodium and w ater, thus promoting further increase in ascitic
23 fluid formation. Treatment of ascites secondary to cirrhosis involves restriction of sodium intake combined
24 w ith diuretics. The most commonly prescribed initial therapy is a combination of furosemide and
25 spironolactone.
26
27 (Choice A) The descending abdominal aorta w ill have elevated pressures in patients w ith systemic
28 hypertension .

30 (Choice B) The inferior vena cava experiences increased pressure in cases of heart failure. This patient
31 does not present w ith the typical symptoms associated w ith heart failure, such as low er extremity edema and
32 shortness of breath .
33
(Choice D) The celiac trunk emerges from the aorta at this level (T12 / L1).
34
35 Educational Objective:
36 The portal vein can be identified on cross-sectional scans lying medial to (or just w ithin) the right lobe of the
37 liver and anterior to the inferior vena cava. The pressure in the portal system is elevated in liver cirrhosis.
38
39
Time Spent: 5 seconds Copyright USMLEW orld,LLC. Last updated: [219/2014]
40
41
42
~imeRemaini~ ~ 0
I Tuto' Feedback Suspend End Block
---------------------------------------------------------------------------------------------------------------------------------------------
1
2
" Item: 30 of 45
Q.ld: 842 [
111 f> Mark .<:J
Previous
[::>
Next
il
Lab Values
~
Notes
~
Calculator
3
4
5
6 A 54-year-old male undergoes evaluation for neurological symptoms. MRI scan show s bilateral infarcts in the
7 thalami . W hich of the follow ing sensory pathw ays reaches the cortex w ithout relay nuclei in the thalami?
8
9
10 0 A. Proprioception
11
12
0 B. Pain
13 0 C. Smell
14
15 0 D. Vision
16 0 E. Hearing
17
18 0 F. Taste
19

[ill
1
22
23
24
25
26
27
28
29

31
32
33
34
35
36
37
38
39
40
41
42
~imeRemaini~ ~ 0
I Tuto' Feedback Suspend End Block
---------------------------------------------------------------------------------------------------------------------------------------------
1
2
" Item: 30 of 45
Q.ld: 842
111 f> Mark .<:J
Previous
[::>
Next
il
Lab Values
~
Notes
~
Calculator
3
4 thalami . W hich of the follow ing sensory pathways reaches the cortex w ithout relay nuclei in the thalami?
5
6 0 A. Proprioception [7%]
7
8 0 B. Pain [6%]
9 .; @ C. Smell [69%]
10 0 D. Vision [1 0%]
11
12 0 E. Hearing [4%]
13 0 F. Taste [4%]
14
15
16 Explanation: User ld:
17
18 The thalamus is a part of diencephalon and has multiple functions. It translates information from all sensory
19 pathw ays other than olfaction and selectively distributes those impulses to appropriate parts of the cortex
("relay" function}. The follow ing thalamic nuclei receive input from sensory pathw ays:
[ill
1 1. Ventral posterolateral nucleus (VPL} receives input from the spinothalamic tract (pain and temperature
22 sensation} and medial lemniscus (position and proprioception}. It transmits impulses to primary
23 somatosensory cortex (Brodmann areas 3, 1 & 2}.
24 2. Ventral posteromedial nucleus (VPM} receives inputs from the trigeminal and gustatory pathways and
25 transmits them to the primary sensory cortex (Brodmann's areas 3, 1 & 2}.
26 3. Lateral geniculate body is a "relay" nucleus for the vision pathw ay. It receives impulses from the optic
27 nerve and transmits them via the optic radiations to the visual cortex (calcarine sulcus}.
28 4. Medial geniculate body is a part of the auditory pathw ay. It receives impulses from the superior olivary
29 nucleus and the inferior colliculus of the pons, and projects them to the auditory cortex of the temporal
lobe (Brodmann areas 41 & 42}.
31
32
The olfactory tract is the only sensory pathw ay w here input is not processed through the thalamus.
33
34 Educational Objective:
35 All sensory pathw ays except olfaction (smell} have relay nuclei in the thalamus. The VPL receives impulses
36 from spinothalamic and medial lemniscus pathw ays, the VPM from the trigeminal and gustatory pathw ays,
37 and the lateral and medial geniculate bodies are relay nuclei for the visual and auditory pathw ays, respectively.
38
39
Time Spent: 1 seconds Copyright USMLEWorld,LLC. Last updated: [7/7/201OJ
40
41
42
~imeRemaini~ ~ 0
I Tuto' Feedback Suspend End Block
---------------------------------------------------------------------------------------------------------------------------------------------
1
2
" Item: 31 of 45
Q.ld : 8635 [
lil f> Mark .<:J
Previous
[::>
Next
il
Lab Values
~
Notes
~
Calculator
3
4 A 31-year-old man is brought to the emergency department after a motorcycle accident. He w as found lying
5 in the middle of the road by first responders, w ho placed him in a rigid cervical collar and put him on a
6 backboard for spinal immobilization . The patient currently complains of severe pain in his neck and back. He
7 is alert and fully oriented. There are a number of lacerations and bruises on his body. Neurologic examination
8 suggests a spinal cord injury at the level show n in the illustration below .
9
10
11
12
13
14
15
16
17
18
19

[ill
1
22
23
24
25
26
27
28
29
30
W hich of the follow ing spinal cord levels is most likely to be injured?
32
33
34 0 A. C1
35 0 B. C7
36
37 0 C. T4
38 0 D. L5
39 0 E. S2
40
41
42
~imeRemaini~ ~ 0
I Tuto' Feedback Suspend End Block
---------------------------------------------------------------------------------------------------------------------------------------------
1
2
" Item: 31 of 45
Q.ld: 8635 [
lil f> Mark .<:J
Previous
[::>
Next
il
Lab Values
~
Notes
~
Calculator
3
4 v @ C. T4 [50%]
5
6 0 D. L5 [5%]
7 0 ES2 [1%]
8
9
10 Explanation: User ld:
11
12 Spinal cord secti ons through different levels
13
14
15
16
17
18
19

[ill
1 Dorsal
22
23 Dorsal columns
24
25
26
Cervical cord 11 Dorsal horn

27
28
;/ Lateral
29 - - column
30

I
Cervical

\
""- Lateral
enlargement
(0 -Tl) """ moto r nuclei
32 Ventral horn (limb muscles)
33 Medial motor
34 nuclei (axial muscles)
35
36
37 Dorsal columns
38
39
40
A'!
41
42
1
2
" Item: 31 of 45
Q.ld: 8635 [
lil f> Mark .<:J
Previous
[::>
Next
il
Lab Values
~
Notes
~
Calculator
3
4
5 Spinal cord sections through different levels
6
7
8
9
10
11
12
13
14 Dorsal
15
16 Dorsal columns
17
18 Cervical cord
I] Do rsal horn
19

[ill
1
7 Lateral
- column
22

I
23 Cervical

\
"'-. Lat eral
enlargement
24 ""' motor nuclei
(0-Tl)
25 Ventral horn (limb muscles)
26 Medial motor
27 nuclei (axial muscles)
28
29
30 Dorsal columns

32 /1
33 Thoracic cord
34 Lateral
35 _.;...----column
36

\
37

I
38
39
40

~
41
42 F!!ck suW.nd
------------------------------------------------------------------------------------------------------------------------------------------------------
EnQck
1
2
3
4 Dorsal columns
5
6
7
/1
Thoracic cord
8 Lateral
9 ~ column
10
11
12
13
14
15
16
/
Ventral horn Intermediolateral
cell column
17 (preganglionic sympathetics)
18
19

[ill
1
Dorsal nerve roots

22
23
Lumbar cord
1
24 - - - - Dorsal horn
25
26
27 Lumbosacral
28 enlargement ....._________Ventral horn
(Ll -$2)
29
30
"\
Ventral nerve roots
32
33
34
Sacral cord
35 Dorsal nerve roots
36
37
38
1
39
40
41
42
1
2
3
4
5 Dorsal nerve roots
6
7
Lumbar cord
1
8 - - - -Dorsal horn
9
10
11
Lumbosacral
12 enlargement -..________Ventral horn
13 (Ll -$2)
14
15 ~
16 Ventral nerve roots
17
18
19 Sacral cord
Dorsal nerve roots
[ill
1
1
22
23 > -Dorsal horn
24
25
26
27
28
29 ~Ventral hom
30
Ventral
USMLEWorld, LlC @ 2011
32
33 The spinal cord is a roughly round-to-oval structure containing a central "butterfly-like" area of grey matter
34 surrounded by w hite matter. The grey matter is divided into ventral (motor) horns and dorsal (sensory) horns.
35 Certain features of the cord vary w ith spinal level, including the amount of w hite matter (in comparison to grey
36 matter), the shape of the section (round vs oval), the size of the ventral horns, and the presence of lateral
37 horns. Moving proximally, the amount of w hite matter steadily increases and the sections become more ovoid
38 (particularly the upper thoracic and cervical regions). Further, the ventral horns of the low er cervical and
39 lumbosacral regions are more prominent as these areas innervate the muscles of the arms and legs,
40 ro.C'norti\ tOI \1

~
41
42 F!!ck suW.nd EnQck
---------------------------------------------------------------------------------------------------------------------------------------------
1
2
" Item: 31 of 45
Q.ld: 8635 [
lil f> Mark .<:J
Previous
[::>
Next
il
Lab Values
~
Notes
~
Calculator
3
4 The spinal cord is a roughly round-to-oval structure containing a central "butterfly-like" area of grey matter
5 surrounded by w hite matter. The grey matter is divided into ventral (motor) horns and dorsal (sensory) horns.
6 Certain features of the cord vary w ith spinal level, including the amount of w hite matter (in comparison to grey
7 matter), the shape of the section (round vs oval}, the size of the ventral horns, and the presence of lateral
8 horns. Moving proximally, the amount of w hite matter steadily increases and the sections become more ovoid
9 (particularly the upper thoracic and cervical regions). Further, the ventral horns of the low er cervical and
10 lumbosacral regions are more prominent as these areas innervate the muscles of the arms and legs,
11 respectively.
12
13 The transverse section presented in the question stem is representative of the T4 spinal level. A
14 characteristic that is unique to thoracic and early lumbar levels (T1-L2) is the presence of lateral horns, w hich
15 are made up of sympathetic preganglionic neurons (the intermediolateral nucleus). Additionally, the amount of
16 w hite matter in the mid-thoracic spinal cord appears larger in relation to the rather diminutive amount of grey
17 matter. Low er sections (lumbar and sacral} possess sparse w hite matter, more grey matter, and larger
18 ventral horns. Furthermore, the overall round shape of this section is consistent w ith sections taken at or
19 below the mid-thoracic region .

[ill
1 (Choice A) At the level of C1 , the shape of the transverse section is ovoid w ith a considerable amount of
w hite matter surrounding a dimunitive amount of grey matter. Both the gracile and cuneate fasciculi w ill also
22
23 be present at this level.
24
(Choice B) At the level of C7, the spinal cord appears oval-shaped . There is also a larger amount of grey
25
matter compared to the mid-thoracic region, particularly at the ventral horns.
26
27 (Choice D) At L5, the dorsal and ventral horns are large in comparison to the modest amount of w hite matter
28 present at this level. There are also no lateral horns.
29
30 (Choice E) In the sacral region, grey matter occupies most of the transverse section and the w hite matter
exists as a relatively thin mantle at the periphery of the cord.
32
33 Educational objective:
34 General features that aid in localizing a transverse spinal cord section:
35
More proximal levels have increasing amounts of w hite matter and more ovoid sections
36
Low er cervical and lumbosacral regions have large ventral horns
37
Thoracic and early lumbar sections (T1-L2) contain lateral grey matter horns
38
39
40

~
41
42 F!!ck
---------------------------------------------------------------------------------------------------------------------------------------------
suW.nd EnQck
1
2
" Item: 32 of 45
Q.ld: 1635 [
111 f> Mark .<:J
Previous
[::>
Next
il
Lab Values
~
Notes
~
Calculator
3
4
5
6 A patient presents to the emergency room complaining of w orsening headaches, confusion, and intermittent
7 vomiting. She is admitted, but later dies in the hospital. Examination of the brain at autopsy show s
8 enlargement of the lateral and third ventricles and a normal-sized fourth ventricle. W here is the most likely
9 site of obstruction in this patient?
10
11
12 0 A. Interventricular foramina of Monro
13
14
0 B. Cerebral aqueduct
15 0 C. Medial foramen of Magendie
16
17 0 D. Lateral foramina of Luschka
18 0 E. Arachnoid villi
19

[ill
1
22
23
24
25
26
27
28
29
30
31

33
34
35
36
37
38
39
40
41
42
~imeRemaini~ ~ 0
I Tuto' Feedback Suspend End Block
---------------------------------------------------------------------------------------------------------------------------------------------
1
2
" Item: 32 of 45
Q.ld: 1635 [
111 f> Mark .<:J
Previous
[::>
Next
il
Lab Values
~
Notes
~
Calculator
3
4 ., @ B. Cerebral aqueduct [80%]
5
6 0 C. Medial foramen of Magendie [4%]
7 0 D. Lateral foramina of Luschka [3%]
8 0 E. Arachnoid villi [1 %]
9
10
11 Explanation: User ld:
12
13 This patient had findings consistent w ith noncommunicating hydrocephalus resulting from obstruction of
14 cerebrospinal fluid (CSF} flow at the cerebral aqueduct.
15
16 CSF is formed by the choroid plexus in each of the four ventricles. The choroid plexus is composed of
17 ependymal cell-covered outgrowths of the pia mater blood vessels. The ependymal epithelium is a simple
18 ciliated columnar layer. Choroid plexus capillaries are fenestrated unlike the other CNS capillaries. The
19 ependymal cells form tight junctions w ith one another and selectively allow certain materials from the blood to
enter the CSF.
[ill
1
Once formed, CSF flow s from the lateral ventricles to the third ventricle via the interventricular foramina of
22
Monro. It then circulates to the fourth ventricle through the cerebral aqueduct (of Sylvius}. CSF then exits the
23
24 fourth ventricle and enters the subarachnoid space via one of three foramina, the lateral foramina of Luschka
or the medial foramen of Magendie. CSF is returned to the venous circulation via the arachnoid villi
25
(granulations}.
26
27 Other forms of hydrocephalus are briefly as follow s:
28
29 1. Communicating hydrocephalus: No blockage to CSF flow w ithin the brain or brainstem . The blockage
30 is in the subarachnoid space w here flow may be restricted by fibrosis follow ing inflammation.
31 2. Normal pressure hydrocephalus: CSF is not absorbed by the arachnoid villi.
3. Hydrocephalus ex vacuo: Increased CSF volume occurring in conditions of brain atrophy.
33 4 . Pseudotumor cerebri: A condition that classically occurs in overweight young females and is related to
34 decreased CSF outflow at the arachnoid villi.
35
36 Educational Objective:
37 CSF flow s from the third ventricle to the fourth ventricle via the cerebral aqueduct of Sylvius.
38
39
Time Spent: 1 seconds Copyright USMLEW orld,LLC. Last updated: [7/7/201 0]
40
41
42
~imeRemaini~ ~ 0
I Tuto' Feedback Suspend End Block
--------------------------------------------------------------------------------------------------------------------------------------
1
2
" Item: 33 of 45
Q.ld: 1742 [
111 f> Mark .<:J
Previous
[::>
Next
il
Lab Values
~
Notes
~
Calculator
3
4
5
6 A patient presents to your office w ith right leg numbness. W hen he w alks, you notice that he lifts his right foot
7 higher than he does his left, and that his right foot slaps to the ground w ith each step. On neurologic exam,
8 you also ascertain that he is unable to evert the right foot. W hich of the follow ing nerves has most likely been
9 injured?
10
11
12 0 A. Tibial
13 0 B. Common peroneal
14
15 0 C. Superficial peroneal
16 0 D. Sural
17
18 0 E. Obturator
19

[ill
1
22
23
24
25
26
27
28
29
30
31
32

34
35
36
37
38
39
40
41
42
~imeRemaini~ ~ 0
I Tuto' Feedback Suspend End Block
---------------------------------------------------------------------------------------------------------------------------------------------
1
2
" Item: 33 of 45
Q.ld: 1742 [
111 f> Mark .<:J
Previous
[::>
Next
il
Lab Values
~
Notes
~
Calculator
3
4 ~ @ B. Common peroneal [70%]
5
6 0 C. Superficial peroneal [12%]
7 0 D. Sural [2%]
8 0 E. Obturator [3%]
9
10
11 Explanation: User ld:
12
13 Sciatic nerve
14
15 ( ..............\
16
17
18
19 on peroneal nerve \\1! (
[ill
1
22
23
24
25
26
27 Tibial nerve Deep peroneal nerve
28
29
30
31 \
32

34
35
36
37
38
39 The common peroneal nerve is the most commonly injured nerve in the leg. It begins in the proximal popliteal
40 fossa, where the sciatic nerve divides into the tibial nerve and common peroneal nerve. From there, the
41
42
~imeRemaini~ ~ 0
I Tuto' Feedback Suspend End Block
---------------------------------------------------------------------------------------------------------------------------------------------
1
2
3
4
5
6
7
8
9
10
The common peroneal nerve is the most commonly injured nerve in the leg. It begins in the proximal popliteal
11
fossa, w here the sciatic nerve divides into the tibial nerve and common peroneal nerve. From there, the
12
common peroneal nerve courses laterally into the anterior compartment of the leg, passing superficially
13
around the head and neck of the fibula. Blunt trauma to the lateral leg and fractures of the neck of the fibula
14
can injure this nerve.
15
16 Patients w ith common peroneal nerve damage present w ith an equinovarus (plantarflexed and inverted)
17 posture of the affected foot due to paralysis of the peroneus longus and peroneus brevis muscles (mediate
18 foot eversion), paralysis of the tibialis anterior muscle (mediates dorsiflexion), and paralysis of the extrinsic
19 extensors of the toes. Injury to this nerve also causes loss of sensation to the anterolateral leg.
[ill
1 The classic finding on gait exam in patients w ith common peroneal nerve injury is "foot drop," w here the
22 affected leg is lifted high off of the ground w hile w alking due to an inability to dorsiflex the foot. The affected
23 foot w ill also classically slap to the ground w ith each step .
24
25 (Choice A) Tibial nerve injury causes dorsiflexion and eversion of the foot w ith sensory loss on the sole of the
26 foot.
27
28 (Choice C) The superficial peroneal nerve innervates the peroneus longus and peroneus brevis muscles in
29 the lateral compartment of the leg, w hich together evert the foot. It also provides sensory innervation to the
30 skin of the distal anterior leg and dorsal foot.
31
32 (Choice D) The sural nerve is a purely sensory nerve that supplies the posterolateral leg and lateral foot.

(Choice E) Obturator nerve injury classically causes spasms of the adductor muscles of the thigh.
34
35 Educational Objective:
36 Common peroneal nerve injury is common and typically results from trauma to the leg near the head of the
37 fibula. Signs include foot drop and a characteristic high-stepping gait.
38
39
Time Spent: 1 seconds Copyright USMLEW orld,LLC. Last updated: [7/7/201 0]
40
41
42
~imeRemaini~ ~ 0
I Tuto' Feedback Suspend End Block
---------------------------------------------------------------------------------------------------------------------------------------------
1
2
" Item: 34 of 45
Q.ld: 1814 [
111 f> Mark .<:J
Previous
[::>
Next
il
Lab Values
~
Notes
~
Calculator
3
4
5
6 A 34-year-old sw immer presents to your office w ith external ear infection . W hile inspecting the patient you
7 insert a speculum into the external auditory meatus in close contact w ith its posterior w all. The patient faints
8 during the procedure. Irritation of w hich of the follow ing nerves w as most likely induced by the procedure?
9
10
11 0 A. Vestibulocochlear
12 0 B. Vagal
13
14 0 C. Trigeminal
15 0 D. Facial
16
17 0 E. Accessory
18
19

[ill
1
22
23
24
25
26
27
28
29
30
31
32
33

35
36
37
38
39
40
41
42
~imeRemaini~ ~ 0
I Tuto' Feedback Suspend End Block
---------------------------------------------------------------------------------------------------------------------------------------------
1
2
" Item: 34 of 45
Q.ld: 1814 [
111 f> Mark .<:J
Previous
[::>
Next
il
Lab Values
~
Notes
~
Calculator
3
4
~ @ B. Vagal [58%]
5
6 0 C. Trigeminal [7%]
7 0 D. Facial [10%]
8
9 0 E. Accessory [1 %]
10
11 Explanation: User ld:
12
13 The patient described in the question has experienced a vasovagal syncope after stimulation of his posterior
14 external auditory canal by an otoscope speculum. In this form of syncope, parasympathetic outflow via the
15 vagus nerve (CN X) leads to decreased heart rate and blood pressure.
16
17 The posterior part of the external auditory canal is innervated by the small auricular branch of the vagus
18 nerve. Most of the remainder of the external auditory canal, including the external portion of the tympanic
19 membrane, is innervated by the mandibular division of the trigeminal nerve via its auriculotemporal branch.
The inner surface of the tympanic membrane is innervated by the glossopharyngeal nerve (CN IX} via its
[ill
1 tympanic branch .
22
23 (Choice A} The vestibulocochlear nerve (CN VIII) mediates hearing and vestibular proprioception . It does not
24 mediate any cutaneous sensation .
25
26 (Choice C) The mandibular branch of the trigeminal nerve (CN V3) does carry sensation to the majority of the
27 external auditory meatus, except for the posterior w all.
28
(Choice D) The facial nerve is represented in the middle ear by one of its branches, the chorda tympani
29
nerve. The chorda tympani nerve carries the afferent taste fibers from the anterior two-thirds of the tongue as
30
w ell as efferent parasympathetic preganglionic fibers that w ill ultimately innervate the submandibular and
31
sublingual salivary glands. CN VII has a small motor component to the stapedius and behind the ear but does
32
not carry cutaneous sensory information.
33
(Choice E) The accessory nerve (CN XI) provides voluntary motor innervation to the trapezius and the
35 sternocleidomastoid.
36
37 Educational Objective:
38 The vagus nerve provides some cutaneous sensation to the posterior external auditory canal via its small
39 auricular branch . Sensation to the rest of the canal is from CN V3. A vasovagal syncopal episode results
40 frnl"''"'' c-til"''""' l-:r.tinn nf t ho \ t <:aru IC' n ol"'\tO lo-:arlin,.. tl'\ ':1 rlorr-o-:aC'o in hlnnrl ni"OC'C'Ino -:anrl ho-:r.r+ r-':lto

~
41
42
---------------------------------------------------------------------------------------------------------------------------------------------
F!!ck suW.nd EnQck
1
2
" Item: 34 of 45
Q.ld: 1814 [
111 f> Mark .<:J
Previous
[::>
Next
il
Lab Values
~
Notes
~
Calculator
3
4
0 D. Facial [1 0%]
5
6 0 E. Accessory [1 %]
7
8
Explanation: User ld:
9
10 The patient described in the question has experienced a vasovagal syncope after stimulation of his posterior
11 external auditory canal by an otoscope speculum. In this form of syncope, parasympathetic outflow via the
12 vagus nerve (CN X) leads to decreased heart rate and blood pressure.
13
14 The posterior part of the external auditory canal is innervated by the small auricular branch of the vagus
15 nerve. Most of the remainder of the external auditory canal, including the external portion of the tympanic
16 membrane, is innervated by the mandibular division of the trigeminal nerve via its auriculotemporal branch.
17 The inner surface of the tympanic membrane is innervated by the glossopharyngeal nerve (CN IX) via its
18 tympanic branch .
19

[ill
1
(Choice A) The vestibulocochlear nerve (CN VIII) mediates hearing and vestibular proprioception . It does not
mediate any cutaneous sensation.
22
23 (Choice C) The mandibular branch of the trigeminal nerve (CN V3) does carry sensation to the majority of the
24 external auditory meatus, except for the posterior w all.
25
26 (Choice D) The facial nerve is represented in the middle ear by one of its branches, the chorda tympani
27 nerve. The chorda tympani nerve carries the afferent taste fibers from the anterior two-thirds of the tongue as
28 w ell as efferent parasympathetic preganglionic fibers that w ill ultimately innervate the submandibular and
29 sublingual salivary glands. CN VII has a small motor component to the stapedius and behind the ear but does
30 not carry cutaneous sensory information.
31
(Choice E) The accessory nerve (CN XI) provides voluntary motor innervation to the trapezius and the
32
sternocleidomastoid.
33
Educational Objective:
35 The vagus nerve provides some cutaneous sensation to the posterior external auditory canal via its small
36 auricular branch. Sensation to the rest of the canal is from CN V3. A vasovagal syncopal episode results
37 from stimulation of the vagus nerve, leading to a decrease in blood pressure and heart rate.
38
39
Time Spent: 1 seconds Copyright USMLEW orld,LLC. Last updated: [7/7/201 OJ
40
41
42
~imeRemaini~ ~ 0
I Tuto' Feedback Suspend End Block
---------------------------------------------------------------------------------------------------------------------------------------------
1
2
" Item: 35 of 45
Q.ld : 8595 [
111 f> Mark .<:J
Previous
[::>
Next
il
Lab Values
~
Notes
~
Calculator
3
4
5
6 A 62-year-old w oman comes to the physician w ith decreased vision. Over the last 6 w eeks, she has had
7 progressive difficulty w ith visualizing objects on her right side. She also has intermittent headaches that are
8 w orse in the morning. Her past medical history is significant for non-small cell lung carcinoma that w as
9 diagnosed 2 years ago and treated surgically. Physical examination reveals right homonymous hemianopia.
10 W hen light is show n in the left eye, both pupils constrict. How ever, w hen the light is immediately moved to the
11 right eye, her pupils appear to dilate. This patient's symptoms are most likely due to a left-sided lesion
12 involving w hich of the follow ing structures?
13
14
15 0 A. Frontal eye fields
16 0 B. Lateral geniculate nucleus
17
18 0 C. Meyer's loop
19 0 D. Optic radiation

[ill
1 0 E. Optic tract
22 0 F. Visual cortex
23
24
25
26
27
28
29
30
31
32
33
34

36
37
38
39
40
41
42
~imeRemaini~ ~ 0
I Tuto' Feedback Suspend End Block
---------------------------------------------------------------------------------------------------------------------------------------------
1
2
" Item: 35 of 45
Q.ld: 8595 [
111 f> Mark .<:J
Previous
[::>
Next
il
Lab Values
~
Notes
~
Calculator
3
4
" @ E. Optic tract[55%]
5
6 0 F. Visual cortex [4%]
7
8
Explanation: User ld:
9
10 Vi sual pathways from above
11
12
13
14
15
16
17
18
19 Left

[ill
1
22
23 Optic nerve
24
25
26 Optic chiasma
27
28
29
30
31
32
33
34

36 Lateral
37 geniculate
38 nucleus
39
40
41
42
1
2
" Item: 35 of 45
Q.ld: 8595 [
111 f> Mark .<:J
Previous
[::>
Next
il
Lab Values
~
Notes
~
Calculator
3
4
5
Visual pathways from above
6
7
8
9
10
11
12
13
Left
14
15
16
17
Optic nerve
18
19

[ill
1
22
23
24
25
26
27
28
29
30
31 Lateral
32 geniculate
33 nucleus
34

36
37
38
39
40
41
42
1
2
3
4
5
6
7
8 Optic radiation
9
10 C) USMLEWOI'kl. LlC
11
12 This patient likely has a lesion involving the left optic tract. The optic tract contains mixed optic nerve fibers
13 from the temporal part of the ipsilateral retina (nasal visual field) and fibers from the nasal part of the
14 contralateral retina (temporal visual field). Lesions of the optic tract subsequently result in a contralateral
15 homonymous hemianopia.
16
17 The afferent limb of the pupillary light reflex involves the retina, optic nerve, optic chiasm, and optic tract
18 fibers. Although the majority of optic tract fibers project to the lateral geniculate nucleus, some project onto
19 the pretectal nucleus in the midbrain. A relative afferent pupillary defect (Marcus Gunn pupil) often occurs w ith

[ill
1
lesions involving the optic nerve, such as optic neuritis in multiple sclerosis. How ever, optic tract lesions can
also cause a relative afferent pupillary defect in the contralateral eye, since the nasal portion of the retina
22 contributes more input to the pretectal nucleus than the temporal portion of the retina . This defect can be
23 observed during the swinging flashlight test since the patient's pupils constrict less (and therefore appear to
24 dilate) w hen a bright light is sw ung from the unaffected eye to the affected eye.
25
26 (Choice A) The frontal eye fields are regions located bilaterally in the prefrontal cortex. The right eye field
27 generates conjugate gaze movements to the left, and the left eye field generates conjugate gaze movements
28 to the right. W hen a frontal eye field is affected by an acute lesion, the influence of the contralateral eye field
29 predominates, causing the eyes to deviate tow ard the lesion.
30
31 (Choices B and D) Lesions involving the lateral geniculate nucleus or optic radiation can produce a
32 contralateral homonymous hemianopia. How ever, pupillary light reflexes w ould be normal.
33
34 (Choice C) A lesion in Meyer's loop w ould produce a contralateral upper quadrantanopia or "pie in the sky"
defect.
36
(Choice F) A lesion involving the visual cortex w ould produce a contralateral homonymous hemianopia w ith
37
macular sparing. Pupillary light reflexes w ould be intact.
38
39 Educational objective:
40 "
41
42
~imeRemaini~ ~ 0
I Tuto' Feedback Suspend
---------------------------------------------------------------------------------------------------------------------------------------------
End Block
1
2
" Item: 35 of 45
Q.ld : 8595 [
111 f> Mark .<:J
Previous
[::>
Next
il
Lab Values
~
Notes
~
Calculator
3
4 Optic radiation ......-
5
6 C) USMLEWOI'kl. LlC
7
8 This patient likely has a lesion involving the left optic tract. The optic tract contains mixed optic nerve fibers
9 from the temporal part of the ipsilateral retina (nasal visual field) and fibers from the nasal part of the
10 contralateral retina (temporal visual field). Lesions of the optic tract subsequently result in a contralateral
11 homonymous hemianopia.
12
13 The afferent limb of the pupillary light reflex involves the retina, optic nerve, optic chiasm, and optic tract
14 fibers. Although the majority of optic tract fibers project to the lateral geniculate nucleus, some project onto
15 the pretectal nucleus in the midbrain. A relative afferent pupillary defect (Marcus Gunn pupil) often occurs with
16 lesions involving the optic nerve, such as optic neuritis in multiple sclerosis. However, optic tract lesions can
17 also cause a relative afferent pupillary defect in the contralateral eye, since the nasal portion of the retina
18 contributes more input to the pretectal nucleus than the temporal portion of the retina . This defect can be
19 observed during the swinging flashlight test since the patient's pupils constrict less (and therefore appear to
[ill
1
dilate) when a bright light is swung from the unaffected eye to the affected eye.

22 (Choice A) The frontal eye fields are regions located bilaterally in the prefrontal cortex. The right eye field
23 generates conjugate gaze movements to the left, and the left eye field generates conjugate gaze movements
24 to the right. When a frontal eye field is affected by an acute lesion, the influence of the contralateral eye field
25 predominates, causing the eyes to deviate toward the lesion.
26
27 (Choices B and D) Lesions involving the lateral geniculate nucleus or optic radiation can produce a
28 contralateral homonymous hemianopia. However, pupillary light reflexes would be normal.
29
30 (Choice C) A lesion in Meyer's loop would produce a contralateral upper quadrantanopia or "pie in the sky"
31 defect.
32
(Choice F) A lesion involving the visual cortex would produce a contralateral homonymous hemianopia with
33
macular sparing. Pupillary light reflexes would be intact.
34
Educational objective:
36 A lesion in the optic tract can produce contralateral homonymous hemianopia and a relative afferent pupillary
37 defect (Marcus Gunn pupil) in the contralateral eye.
38
39
Time Spent: 1 seconds Copyright USMLEWorld,LLC. Last updated: [213/2014]
40
41
42
~imeRemaini~ ~ 0
I Tuto' Feedback Suspend End Block
---------------------------------------------------------------------------------------------------------------------------------------------
1
2
" Item: 36 of 45
Q.ld: 1846 [
111 f> Mark .<:J
Previous
[::>
Next
il
Lab Values
~
Notes
~
Calculator
3
4
5
6 A highly agitated 54-year-old man is brought to the emergency department by his family because he is unable
7 to effectively communicate. He speaks clearly and w ith conviction but his sentences are incomprehensible.
8 He does not appear to understand the doctor's questions, does not follow oral or w ritten instructions, and
9 cannot repeat simple phrases. Branch occlusion of w hich of the following arteries is most likely responsible
10 for this patient's condition?
11
12
13 0 A. Anterior cerebral artery
14 0 B. Anterior inferior cerebellar artery
15
16 0 C. Middle cerebral artery
17 0 D. Posterior cerebral artery
18
19 0 E. Posterior inferior cerebellar artery

[ill
1
22
23
24
25
26
27
28
29
30
31
32
33
34
35

37
38
39
40
41
42
~imeRemaini~ ~ 0
I Tuto' Feedback Suspend End Block
---------------------------------------------------------------------------------------------------------------------------------------------
1
2
" Item: 36 of 45
Q.ld: 1846 [
111 f> Mark .<:J
Previous
[::>
Next
il
Lab Values
~
Notes
~
Calculator
3
4 ., @ C. Middle cerebral artery [90%]
5
6 0 D. Posterior cerebral artery [5%]
7 0 E. Posterior inferior cerebellar artery [0%]
8
9
10 Explanation: User ld:
11
This patient has fluent aphasia, a form of receptive aphasia characterized by speech that is fluent, w ell
12
articulated, and melodic - but meaningless. Individuals stricken w ith fluent aphasia cannot understand verbal
13
14 or w ritten language. Repetition is usually affected to the same degree as the impairment in comprehension .
Typically, patients w ill have no aw areness of their cognitive deficit. Fluent aphasia is often produced by a
15
lesion in Wernicke's area, w hich is located in the auditory association cortex w ithin the posterior portion of the
16
superior temporal gyrus in the dominant (usually left) temporal lobe.
17
18
The regions of the cerebral cortex supplied by the middle cerebral artery (MCA) are approximated by the red
19
oval in the image below . The inferior terminal MCA branches supply W ernicke's area.
[ill
1
22 Wernicke's aphasia
23
24
25 Middle cerebral artery territory
26
27
28
29
30 Broca's area
31 Wern icke's area
32
33
34
35

37
38
39
40
41
42
1
2
" Item: 36 of 45
Q.ld: 1846 [
111 f> Mark .<:J
Previous
[::>
Next
il
Lab Values
~
Notes
~
Calculator
3
4
5 Wernicke's aphasia
6
7
8 M iddle cerebral artery territory
9
10
11
12
13 Broca's area
14 Wern icke's area
15
16
17
18
19

[ill
1
22
23
24
25
26
27
28
29
30
31
32
33
34
35

37
38
39
40
41
42
1
2
3
4
5
6
7
8
9
10
11
12
13
14
15
16
17
18
19

[ill
1
22
23
24
25
26
USMLEWorld UC
27
28
(Choices A, B, 0, and E) The anterior cerebral artery, anterior inferior cerebellar artery, posterior cerebral
29
artery, and posterior inferior cerebellar artery do not typically service W ernicke's area . Consequently,
30
occlusions of these arteries w ould not be associated w ith W ernicke's aphasia .
31
32 Educational objective:
33 A lesion in W ernicke's area can cause receptive aphasia, w hich is characterized by w ell-articulated,
34 nonsensical speech paired with a lack of language comprehension (Wernicke-Word salad). W ernicke's area
35 is located in the auditory association cortex within the posterior portion of the superior temporal gyrus in the
dominant ternporallobe. The rniddle cerebral artery supplies Broca's area (superior division) and W ernicke's
37 area (inferior division).
38
39
Tirne Spent: 1 seconds Copyright USMLEW orld,LLC. Last updated: [11 /20/2013]
40
41
42
~imeRemaini~ ~ 0
I Tuto' Feedback Suspend End Block
---------------------------------------------------------------------------------------------------------------------------------------------
1
2
" Item: 37 of 45
Q.ld : 2129 [
111 f> Mark .<:J
Previous
[::>
Next
il
Lab Values
~
Notes
~
Calculator
3
4 A 55-year-old, right-handed man is brought to the emergency department after police find him unresponsive
5 on the street. In triage, he appears disheveled and homeless, w ith a strong odor of alcohol coming from his
6 mouth. The patient is started on intravenous fluids w ith dextrose and admitted to the hospital for alcohol
7 intoxication. Over the next several days, he slow ly recovers. How ever, neurologic examination show s
8 substantial difficulty w ith short-term memory, nystagmus, ophthalmoplegia, and ataxia .
9
10
11
12
13
14
15
16
17
18
19

[ill
1
22
23
24
25
26
27
28
29
30
31
32
33
34
35 W hich of the follow ing identifiers from the brain image indicates the neural structure most likely to be
36 damaged in this patient?

38 0 A. A
39
40
0 B. B
41
42
~imeRemaini~ ~ 0
I Tuto' Feedback Suspend End Block
---------------------------------------------------------------------------------------------------------------------------------------------
1
2
" Item: 37 of 45
Q.ld : 2129 [
111 f> Mark .<:J
Previous
[::>
Next
il
Lab Values
~
Notes
~
Calculator
3
4
5
6
7
8
9
10
11
12
13
14
15
16
17
18
19

[ill
1
22
23
24
25
26
27
28
29 W hich of the following identifiers from the brain image indicates the neural structure most likely to be
30 damaged in this patient?
31
32
33 0 A. A
34 0 B. B
35 0 C. C
36
0 D. D
38 0 E. E
39 0 F. F
40
41
42
~imeRemaini~ ~ 0
I Tuto' Feedback Suspend End Block
---------------------------------------------------------------------------------------------------------------------------------------------
1
2
" Item: 37 of 45
Q.ld : 2129 [
111 f> Mark .<:J
Previous
[::>
Next
il
Lab Values
~
Notes
~
Calculator
3
4 v @ B. B [59%]
5
6 0 C. C [5%]
7 0 D. D [11 %]
8 0 E. E [1 6%]
9
10 0 F. F [3%]
11
12 Explanation: User ld:
13
14 Corpus callosum Choroid plexus
15
16 Tha lamus
17 Septum pellucidum
18
19
Parieto-occipital
[ill
1 Fornix sulcus
22
23
24 Pineal gland
25
26 Calcarine sulcus
27 (visual area)
28
29
30 Hypothalamus
31
32 Mammillary body
33
34
Cerebellum
35
36
Medulla
38 oblongata
39 USMI.EW01Id., Ll( (0 2011
40
41
42
~imeRemaini~ ~ 0
I Tuto' Feedback Suspend End Block
---------------------------------------------------------------------------------------------------------------------------------------------
1
2
" Item: 37 of 45
Q.ld : 2129 [
111 f> Mark .<:J
Previous
[::>
Next
il
Lab Values
~
Notes
~
Calculator
3
4
5 Chronic alcohol users are malnourished and have low levels of thiamine at baseline. Intravenous dextrose
6 administration w ithout prior thiamine supplementation in these patients results in an acute drop of thiamine
7 level, leading to W ernicke encephalopathy. The clinical presentation of W ernicke encephalopathy includes
8 ataxia, nystagmus, ophthalmoplegia, and anterograde amnesia . The chronic effects of thiamine deficiency
9 lead to Korsakoff psychosis, w hich is characterized by anterograde and retrograde amnesia, apathy, lack of
10 insight, and confabulation.
11
12 Thiamine is a key coenzyme for pyruvate dehydrogenase, w hich is involved in glucose metabolism. Thiamine
13 deficiency results in the brain's inability to properly metabolize glucose and turn it into energy. The structure in
14 the brain that most frequently undergoes necrosis in the setting of thiamine deficiency is the mammillary
15 body. The mammillary body is part of the Papez circuit, w hich is a neural pathway of the limbic system that is
16 involved in the cortical control of emotion and memory.
17
(Choice A) The fornix is a bundle of axons originating from the hippocampal subiculum that projects to the
18
mammillary body. Because the fornix is part of the Papez circuit, damage to this structure can lead to a
19
W ernicke encephalopathy. How ever, the most frequently affected structure in patients w ith W ernicke
[ill
1
encephalopathy is the mammillary body.
22 (Choice C) The basis pontis is not frequently affected in W ernicke encephalopathy.
23
24 (Choice D) The dorsal thalamus is an egg-shaped structure in the diencephalon that is responsible for
25 relaying information from the spinal cord and other subcortical structures to the cortex. The anterior nucleus
26 of the thalamus, w hich is part of the Papez circuit, can be affected in patients w ith thiamine deficiency and
27 lead to W ernicke encephalopathy but is not the most frequently affected structure.
28
29 (Choice F) The splenium of corpus callosum is not typically affected in W ernicke encephalopathy. Lesions
30 (demyelinating or ischemic) in this structure w ill clinically result in alexia w ithout agraphia.
31
32 (Choice E) The inferior colliculus is part of the auditory pathw ay and not typically affected in W ernicke
33 encephalopathy.
34
35 Educational objective:
36 Thiamine (i.e., vitamin 81 ) deficiency acutely leads to W ernicke encephalopathy and chronically leads to
Korsakoff psychosis. The neural structure most frequently affected in patients w ith W ernicke encephalopathy
38 is the mammillary body, w hich is part of the Papez circuit. Alcoholic or malnourished patients should receive
39 intravenous thiamine supplementation before intravenous dextrose administration because giving dextrose
40 w ithout prior thiamine can precipitate a W ernicke encephalopathy.
41
42
~imeRemaini~ ~ 0
I Tuto' Feedback Suspend End Block
---------------------------------------------------------------------------------------------------------------------------------------------
1
2
" Item: 37 of 45
Q.ld : 2129 [
111 f> Mark .<:J
Previous
[::>
Next
il
Lab Values
~
Notes
~
Calculator
3
I I IIUI 1111 ''-'I~ U '''-'J I..V'-'11-L.J I I 1'-' l V I t-'J' UVU~'-' U'-'1 I J U I V~'-'1 IU~'-' ) VI' I 11"-1 I I~ II I V V I V'-'U II I ~IU.._.V..,.._, I I 1'-'U.UVII"'I I 1 . I I IIUI IIIII'-'
4
deficiency results in the brain's inability to properly metabolize glucose and turn it into energy. The structure in
5
the brain that most frequently undergoes necrosis in the setting of thiamine deficiency is the mammillary
6
body. The mammillary body is part of the Papez circuit, which is a neural pathway of the limbic system that is
7
involved in the cortical control of emotion and memory.
8
9 (Choice A) The fornix is a bundle of axons originating from the hippocampal subiculum that projects to the
10 mammillary body. Because the fornix is part of the Papez circuit, damage to this structure can lead to a
11 W ernicke encephalopathy. How ever, the most frequently affected structure in patients with Wernicke
12 encephalopathy is the mammillary body.
13
14 (Choice C) The basis pontis is not frequently affected in Wernicke encephalopathy.
15
16 (Choice D) The dorsal thalamus is an egg-shaped structure in the diencephalon that is responsible for
17 relaying information from the spinal cord and other subcortical structures to the cortex. The anterior nucleus
18 of the thalamus, which is part of the Papez circuit, can be affected in patients with thiamine deficiency and
19 lead to Wernicke encephalopathy but is not the most frequently affected structure.

[ill
1 (Choice F) The splenium of corpus callosum is not typically affected in Wernicke encephalopathy. Lesions
(demyelinating or ischemic) in this structure will clinically result in alexia without agraphia .
22
23
24 (Choice E) The inferior colliculus is part of the auditory pathw ay and not typically affected in Wernicke
25 encephalopathy.
26
Educational objective:
27
Thiamine (i.e., vitamin 81 ) deficiency acutely leads to Wernicke encephalopathy and chronically leads to
28
Korsakoff psychosis. The neural structure most frequently affected in patients with Wernicke encephalopathy
29
is the mammillary body, which is part of the Papez circuit. Alcoholic or malnourished patients should receive
30
intravenous thiamine supplementation before intravenous dex1rose administration because giving dex1rose
31
without prior thiamine can precipitate a W ernicke encephalopathy.
32
33
34 References:
35 1. Wernicke encephalopathy: MR findings at clinical presentation in twenty-six alcoholic and
36 nonalcoholic patients.

38
39
Time Spent: 4 seconds Copyright USMLEW orld,LLC. Last updated: [1 0/ 11/2013]
40
41
42
~imeRemaini~ ~ 0
I Tuto' Feedback Suspend End Block
---------------------------------------------------------------------------------------------------------------------------------------------
1
2
" Item: 38 of 45
Q.ld: 2024 [
111 f> Mark .<:J
Previous
[::>
Next
il
Lab Values
~
Notes
~
Calculator
3
4
5
6 A patient recovering from a traumatic injury to the face now finds himself unable to chew his food . On
7 physical examination, his jaw deviates to the right side. The nerve that has been injured in this patient exits
8 the skull through w hich of the following foramina?
9
10
11 0 A. Foramen rotundum
12
13
0 B. Foramen ovale
14 0 C. Foramen spinosum
15
16 0 D. Foramen lacerum
17 0 E. Jugular foramen
18
19

[ill
1
22
23
24
25
26
27
28
29
30
31
32
33
34
35
36
37

39
40

~
41
42 F!!ck
---------------------------------------------------------------------------------------------------------------------------------------------
suW.nd EnQck
1
2
" Item: 38 of 45
Q.ld: 2024 [
111 f> Mark .<:J
Previous
[::>
Next
il
Lab Values
~
Notes
~
Calculator
3
4
5 0 A. Foramen rotundum [1 6%]
6
.; @ B. Foramen ovale [61%]
7
8 0 C. Foramen spinosum [12%]
9 0 D. Foramen lacerum [4%]
10
11 0 E. Jugular foramen [7%]
12
13 Explanation: User ld:
14
15 This patient has symptoms consistent w ith injury of the third division of the trigeminal nerve (CN V3,
16 mandibular nerve), w hich provides sensory innervation to the face and motor innervation to the muscles of
17 mastication. The muscles of mastication include the three muscles that close the jaw (masseter, medial
18 pterygoid, and temporalis muscles), and the single muscle that opens the jaw (lateral pterygoid muscle).
19 Thus, a patient w ho is unable to chew food or hold the jaw closed due to w eakness Oaw deviation) has likely

[ill
1
suffered an injury to this nerve. The third branch of the trigeminal nerve exits the skull through the foramen
ovale, w hich also contains the lesser petrosal nerve, the accessory meningeal artery, and the emissary veins .
22
23 (Choice A) The second division of the trigeminal nerve (CN V2, maxillary nerve) exits the skull via the foramen
24 rotundum .
25
26 (Choice C) The foramen spinosum contains the middle meningeal artery and middle meningeal vein and the
27 meningeal (recurrent) branch of CN V3.
28
29 (Choice D) The foramen lacerum contains the meningeal branch of the ascending pharyngeal artery and the
30 artery and nerve of the pterygoid canal. This foramen is otherwise occluded by cartilage.
31
(Choice E) The jugular foramen contains cranial nerves IX, X and XI, the inferior petrosal and sigmoid
32
sinuses, and the posterior meningeal artery.
33
34 Educational Objective:
35 The third branch of the trigeminal nerve (CN V3) exits the skull through the foramen ovale and innervates the
36 muscles of mastication, including the masseter, the medial and lateral pterygoids, and the temporalis
37 muscles.

39
Time Spent: 1 seconds Copyright USMLEW orld,LLC. Last updated: [7/7/201 OJ
40

~
41
42 F!!ck
---------------------------------------------------------------------------------------------------------------------------------------------
suW.nd EnQck
1
2
" Item: 39 of 45
Q.ld: 1743 [
111 f> Mark .<:J
Previous
[::>
Next
il
Lab Values
~
Notes
~
Calculator
3
4
5
6 A 53-year-old male complains of a "strange feeling" in his right hand. On physical examination, there is loss of
7 sensation over the 51" digit. W hich of the follow ing nerves has most likely been injured?
8
9
10 0 A. Ulnar
11 0 B. Median
12
13 0 C. Radial
14
0 D. Musculocutaneous
15
16 0 E. Axillary
17
18
19

[ill
1
22
23
24
25
26
27
28
29
30
31
32
33
34
35
36
37
38

40

~
41
42 F!!ck
---------------------------------------------------------------------------------------------------------------------------------------------
suW.nd EnQck
1
2
" Item: 39 of 45
Q.ld: 1743 [
111 f> Mark .<:J
Previous
[::>
Next
il
Lab Values
~
Notes
~
Calculator
3
4
.; @ A. Ulnar [91 %]
5
6 0 B. Median [4%]
7 0 C. Radial [3%]
8
9
0 D. Musculocutaneous [1 %]
10 0 E. Axillary [0%]
11
12
13 Explanation: User ld:
14
Loss of sensation in the 5th digit can only be due to injury of the ulnar nerve. The ulnar nerve innervates the
15
flexor carpi ulnaris and the medial portion of the flexor digitorum profundus as well as most of the intrinsic
16
muscles of the hand. Injury to the ulnar nerve results in a "claw hand" deformity as well as anesthesia over
17
the medial 1Y2 digits.
18
19 The ulnar nerve is a branch of the medial cord of the brachial plexus (CB-T1) that courses posteromedially in
[ill
1
the upper arm and enters the forearm after passing posterior to the medial epicondyle of the humerus. This
site, the "funny bone," is a common site of ulnar nerve injury. The ulnar nerve is also often injured where it
22 passes between the hook of the hamate and the pisiform bone in the wrist within Guyon's canal.
23
24 (Choice B) Median nerve injury classically results from carpal tunnel syndrome or fractures of the
25 supracondylar humerus. Patients lose sensory innervation to the majority of the palmar and dorsal 1st
26 through 3rd digits. Motor deficits include loss of extrinsic finger flexion, loss of thumb movement and loss of
27 pronation. There may be flattening of the thenar eminence due to denervation atrophy, producing an 'ape
28 hand' deformity.
29
30 (Choice C) The radial nerve innervates the extensor compartment of the forearm. Injury causes wrist drop.
31 Patients also lose sensation to the posterior arm, posterior forearm, and a small portion of the dorsal thumb.
32
33 (Choice D) Musculocutaneous nerve injury causes loss of sensation over the lateral forearm and paralysis of
34 the forearm flexors at the elbow.
35
36 (Choice E) Axillary nerve injury can result from fracture of the surgical neck of the humerus or other shoulder
37 trauma . Patients lose sensation over the lateral upper arm.
38
Educational Objective:
Ulnar nerve injury causes sensation loss to the medial 1Y2 digits of the hand.
40

~
41
42 F!!ck
---------------------------------------------------------------------------------------------------------------------------------------------
suW.nd EnQck
1
2
" Item: 39 of 45
Q.ld: 1743 [
111 f> Mark .<:J
Previous
[::>
Next
il
Lab Values
~
Notes
~
Calculator
3
4 u ts. Meat an [4 "l'oJ
5 0 C. Radial [3%]
6
0 D. Musculocutaneous [1 %]
7
8 0 E. Axillary [0%]
9
10
11 Explanation: User ld:
12 Loss of sensation in the 5th digit can only be due to injury of the ulnar nerve. The ulnar nerve innervates the
13 flexor carpi ulnaris and the medial portion of the flexor digitorum profundus as w ell as most of the intrinsic
14 muscles of the hand. Injury to the ulnar nerve results in a "claw hand" deformity as w ell as anesthesia over
15 the medial 1Y. digits.
16
17 The ulnar nerve is a branch of the medial cord of the brachial plexus (C8-T1) that courses posteromedially in
18 the upper arm and enters the forearm after passing posterior to the medial epicondyle of the humerus. This
19 site, the "funny bone," is a common site of ulnar nerve injury. The ulnar nerve is also often injured where it

[ill
1
passes between the hook of the hamate and the pisiform bone in the w rist w ithin Guyon's canal.

22 (Choice B) Median nerve injury classically results from carpal tunnel syndrome or fractures of the
23 supracondylar humerus. Patients lose sensory innervation to the majority of the palmar and dorsal 1st
24 through 3rd digits. Motor deficits include loss of extrinsic finger flexion, loss of thumb movement and loss of
25 pronation. There may be flattening of the thenar eminence due to denervation atrophy, producing an 'ape
26 hand' deformity.
27
28 (Choice C) The radial nerve innervates the extensor compartment of the forearm. Injury causes wrist drop.
29 Patients also lose sensation to the posterior arm, posterior forearm, and a small portion of the dorsal thumb.
30
31 (Choice D) Musculocutaneous nerve injury causes loss of sensation over the lateral forearm and paralysis of
32 the forearm flexors at the elbow.
33
(Choice E) Axillary nerve injury can result from fracture of the surgical neck of the humerus or other shoulder
34
trauma . Patients lose sensation over the lateral upper arm .
35
36 Educational Objective:
37 Ulnar nerve injury causes sensation loss to the medial 1Y. digits of the hand.
38
Time Spent: 1 seconds Copyright USMLEWorld,LLC. Last updated: [1 / 10/2013]
40

~
41
42 F!!ck
---------------------------------------------------------------------------------------------------------------------------------------------
suW.nd EnQck
1
2
" Item: 40 of 45
Q.ld: 1453 [
111 f> Mark .<:J
Previous
[::>
Next
il
Lab Values
~
Notes
~
Calculator
3
4
5
6 A 35-year-old male comes to your office complaining of a painful tongue sore. He also complains that over
7 the last 2 w eeks he has had a fever and been experiencing m yalgias and arthralgias. He has no other
8 medical problems. The patient w orks as a driver for a local delivery service. He is homosexual and admits to
9 having unprotected sex w ith a stranger approximately 1 month ago. Physical examination reveals a rash over
10 his trunk and cervical lymphadenopathy. An ulcer is located on the median sulcus of the tongue 1 em anterior
11 to the foramen cecum . You decide to draw blood to test for an HIV viral load and prescribe an oral analgesic
12 gel to relieve his ulcer related pain . The pain sensation from his ulcer is most likely carried by w hich of the
13 follow ing nerves?
14
15
16 0 A. Chorda tympani
17 0 B. Glossopharyngeal nerve
18
19 0 C. Mandibular division of the trigeminal nerve

[ill
1
0
0
D. Maxillary division of the trigeminal nerve
E. Vagus nerve
22
23
24
25
26
27
28
29
30
31
32
33
34
35
36
37
38
39

~
41
42 F!!ck
---------------------------------------------------------------------------------------------------------------------------------------------
suW.nd EnQck
1
2
" Item: 40 of 45
Q.ld: 1453 [
111 f> Mark .<:J
Previous
[::>
Next
il
Lab Values
~
Notes
~
Calculator
3
4 .; @ C. Mandibular division of the trigeminal nerve [50%]
5
0 D. Maxillary division of the trigeminal nerve [9%]
6
7 0 E. Vagus nerve [3%]
8
9
Explanation: User ld:
10
11 Innervation of the tongue is complex, as there are motor, general sensory and gustatory (taste) components.
12
13 1. Motor innervation of the tongue is provided by the hypoglossal nerve (CN XII), w ith the exception of
14 the palatoglossus muscle, w hich is innervated by the vagus nerve (CN X).
15 2. General sensory innervation of the tongue (including touch, pain, pressure, and temperature
16 sensation) is provided by:
17
o Anterior 2/3 of the tongue: mandibular branch of trigeminal nerve (CN V3}
18
19 o Posterior 1/3 of the tongue: glossopharyngeal nerve (CN IX)
o Posterior area of the tongue root: vagus nerve (CN X)
[ill
1 3. Gustatory innervation (taste buds) is as follow s:
22
23 o Anterior 2/3 of the tongue: chorda tympani branch of facial nerve (CN VII)
24 o Posterior 1/3 of the tongue: glossopharyngeal nerve (CN IX)
25 o Posterior area of the tongue root and taste buds of the larynx and upper esophagus: vagus
26 nerve (CN X)
27
28
29 The terminal sulcus delineates the anterior 213 of the tongue from the posterior 1/3. The foramen cecum is
30 located along the terminal sulcus at the midline. Thus, an ulcer 1 em distal to the foramen cecum w ould be
31 located on the anterior 2/3 of the tongue. Pain from this region (as w ell as sensations of touch, pressure, and
32 temperature) are transmitted by the mandibular branch of the trigeminal nerve.
33 Vagal Nerve
34
35 Glossopharyngeal Glossopharyngeal
36 N erve Nerve
37
38
39

41
42
1
2
" Item: 40 of 45
Q.ld: 1453 [
111 f> Mark .<:J
Previous
[::>
Next
il
Lab Values
~
Notes
~
Calculator
3
4
5 Explanation: User ld:
6
Innervation of the tongue is complex, as there are motor, general sensory and gustatory (taste) components.
7
8 1. Motor innervation of the tongue is provided by the hypoglossal nerve (CN XII), w ith the exception of
9 the palatoglossus muscle, w hich is innervated by the vagus nerve (CN X).
10 2. General sensory innervation of the tongue (including touch, pain, pressure, and temperature
11 sensation) is provided by:
12
13 o Anterior 213 of the tongue: mandibular branch of trigeminal nerve (CN V3 )
14 o Posterior 1/3 of the tongue: glossopharyngeal nerve (CN IX)
15 o Posterior area of the tongue root: vagus nerve (CN X)
16 3. Gustatory innervation (taste buds) is as follow s:
17
18 o Anterior 2/3 of the tongue: chorda tympani branch of facial nerve (CN VII)
19 o Posterior 1/3 of the tongue: glossopharyngeal nerve (CN IX)
o Posterior area of the tongue root and taste buds of the larynx and upper esophagus: vagus
[ill
1 nerve (CN X)
22
23
24 The terminal sulcus delineates the anterior 2/3 of the tongue from the posterior 1/3. The foramen cecum is
25 located along the terminal sulcus at the midline. Thus, an ulcer 1 em distal to the foramen cecum would be
26 located on the anterior 213 of the tongue. Pain from this region (as w ell as sensations of touch, pressure, and
27 temperature) are transmitted by the mandibular branch of the trigeminal nerve.
28 Vagal Nerve
29
30 Glossopharyngeal Glossopharyngeal
31 Nerve Nerve
32
33
34
35
oo 0 Oo
0 0
36
37
38 Ulcer
Chorda tympani and
39
trigeminal nerve
41
42
1
2
" Item: 40 of 45
Q.ld: 1453 [
111 f> Mark .<:J
Previous
[::>
Next
il
Lab Values
~
Notes
~
Calculator
3
4
5 The terminal sulcus delineates the anterior 213 of the tongue from the posterior 1/3. The foramen cecum is
6 located along the terminal sulcus at the midline. Thus, an ulcer 1 em distal to the foramen cecum w ould be
7 located on the anterior 213 of the tongue. Pain from this region (as w ell as sensations of touch, pressure, and
8 temperature) are transmitted by the mandibular branch of the trigeminal nerve.
9 Vagal Nerve
10
11 Glossopharyngeal Glossopharyngeal
12 N erve Nerve
13
14
15
16
17
18
19 Ulcer
C horda tympani and
[ill
1
trigeminal nerve
22
23
24
25
26
27 (Choice A) The chorda tympani branch of the facial nerve (CN VII) transmits gustatory sensation from
28 anterior 2/3 of the tongue but not painful stimuli.
29
30 (Choice B) The glossopharyngeal nerve transmits taste, pain, temperature and touch stimuli from the
31 posterior 1/3 of the tongue.
32
33 (Choice D) The maxillary division of the trigeminal nerve does not participate in tongue innervation.
34
35 (Choice E) The vagus nerve innervates the far posterior area of the tongue root, transmitting both gustatory
36 and general sensory stimuli.
37
38 Educational Objective:
39 General sensation from the anterior 213 of the tongue is carried by the mandibular division of the trigeminal
nerve. Gustatory innervation of anterior 2/3 of the tongue is provided by chorda tympani branch of the facial
41
42
~imeRemaini~ ~ 0
I Tuto' Feedback Suspend End Block
---------------------------------------------------------------------------------------------------------------------------------------------
1
2
" Item: 40 of 45
Q.ld: 1453 [
111 f> Mark .<:J
Previous
[::>
Next
il
Lab Values
~
Notes
~
Calculator
3
I V'-'OlCU VI I UIC OlllCIIVI ../ oJ V I liiC l U IIl:::fUC. I OIIIIIVIII Ull~ I~IVII \0~ YY CII 0~ ~CII~OUVII~ VI lVU\.,.1 1) tJIC~~UIC) OIIU
4
temperature) are transmitted by the mandibular branch of the trigeminal nerve.
5
6 Vagal Nerve
7
8 Glossopharyngeal Glossopharyngeal
9 N erve Nerve
10
11
12
13
14
15
Ulcer
16 C horda tympani and
17 trigeminal nerve
18
19

[ill
1
22
23
24 (Choice A) The chorda tympani branch of the facial nerve (CN VII) transmits gustatory sensation from
25 anterior 213 of the tongue but not painful stimuli.
26
27 (Choice B) The glossopharyngeal nerve transmits taste, pain, temperature and touch stimuli from the
28 posterior 1/3 of the tongue.
29
30 (Choice D) The maxillary division of the trigeminal nerve does not participate in tongue innervation.
31
(Choice E) The vagus nerve innervates the far posterior area of the tongue root, transmitting both gustatory
32
and general sensory stimuli.
33
34 Educational Objective:
35 General sensation from the anterior 213 of the tongue is carried by the mandibular division of the trigeminal
36 nerve. Gustatory innervation of anterior 213 of the tongue is provided by chorda tympani branch of the facial
37 nerve.
38
39
Time Spent: 1 seconds Copyright USMLEW orld,LLC. Last updated: [219/2014]
41
42
~imeRemaini~ ~ 0
I Tuto' Feedback Suspend End Block
---------------------------------------------------------------------------------------------------------------------------------------------
1
2
" Item: 41 of 45
Q.ld: 8701 [
111 f> Mark .<:J
Previous
[::>
Next
il
Lab Values
~
Notes
~
Calculator
3
4
5
6 A 59-year-old man comes to the physician because of diplopia and a drooping right eyelid . His symptoms
7 began 1 day ago and have since w orsened. Physical examination show s a dilated right pupil that is
8 nonreactive to both light and accommodation . There is vertical and horizontal diplopia. W hen the patient is
9 asked to stare straight ahead, the right eye is directed inferiorly and laterally w ith respect to his left eye.
10 Magnetic resonance imaging of the brain show s an aneurysm involving the right posterior communicating
11 artery. W hich of the follow ing muscles is most likely to remain functionally intact in this patient?
12
13
14 0 A. Inferior oblique
15 0 B. Inferior rectus
16
17 0 C. Lateral rectus
18 0 D. Levator palpebrae
19
0 E. Medial rectus
[ill
1 0 F. Superior rectus
22
23
24
25
26
27
28
29
30
31
32
33
34
35
36
37
38
39
40

42
~imeRemaini~ ~ 0
I Tuto' Feedback Suspend End Block
---------------------------------------------------------------------------------------------------------------------------------------------
1
2
" Item: 41 of 45
Q.ld: 8701 [
111 f> Mark .<:J
Previous
[::>
Next
il
Lab Values
~
Notes
~
Calculator
3
4
5
6 A 59-year-old man comes to the physician because of diplopia and a drooping right eyelid . His symptoms
7 began 1 day ago and have since w orsened. Physical examination show s a dilated right pupil that is
8 nonreactive to both light and accommodation . There is vertical and horizontal diplopia. W hen the patient is
9 asked to stare straight ahead, the right eye is directed inferiorly and laterally w ith respect to his left eye.
10 Magnetic resonance imaging of the brain show s an aneurysm involving the right posterior communicating
11 artery. W hich of the follow ing muscles is most likely to remain functionally intact in this patient?
12
13
14 0 A. Inferior oblique [8%1
15 0 B. Inferior rectus [2%1
16 " @ C. Lateral rectus [82%1
17
18 0 D. Levator palpebrae [3%1
19 0 E. Medial rectus [2% 1

[ill
1
0 F. Superior rectus [3% 1

22
23 Explanation: User ld:
24
25
26
27
28 This patient presents w ith a right third nerve (CN Ill) palsy secondary to a compressive aneurysm . Lesions
29 involving CN Ill cause ptosis (drooping of the upper eyelid) and impair adduction, depression, and elevation of
30 the eye. As a result, diagonal (eg, horizontal and vertical) diplopia is frequently present and the eye rests in a
31 "dow n and out" position . Pupillary constriction and accommodation can also be affected as CN Ill carries
32 parasympathetic fibers to the ciliary muscle and the iris sphincter.
33
34 Educational objective:
35 Lesions involving CN Ill cause ptosis, a dow nward and laterally deviated eye, impaired pupillary constriction
36 and accommodation, and diagonal diplopia. The most dreaded cause of CN Ill palsy is an enlarging
37 intracranial aneurysm.
38
39 Time Spent: 1 seconds Copyright USMLEW orld,LLC. Last updated: [3/9/20141
40

42
~imeRemaini~ ~ 0
I Tuto' Feedback Suspend End Block
---------------------------------------------------------------------------------------------------------------------------------------------
1
2
" Item: 41 of 45
Q.ld: 8701 [
111 f> Mark .<:J
Previous
[::>
Next
il
Lab Values
~
Notes
~
Calculator
3
4
., @ C. Lateral rectus [82%1
5
6 0 D. Levator palpebrae [3%1
7 0 E. Medial rectus [2%1
8
0 F. Superior rectus [3%1
9
10
11 Explanation: User ld:
12
13
14 Eye muscle(s) Oculomotor
Nerve Lesion findings
15 in nervated functions
16
17
18
. Eye deviated downward & laterally
19
Superior rectus, medial Adduction,
. Diagonal diplopia
[ill CN Ill
rect us, inferior rectus,
inferior oblique & levator
depression,
elevation &
. Dilation of pupil & loss of
1
accommodation
22
23
palpebrae superioris external rotation
. Ptosis
24
25
26 CNIV Superior oblique
Internal rotation,
depression &
. Eye deviated upward
27
abduction
. Vertical & torsional diplopia
28
29
30 Eye deviated medially
CNVI Abduction
31 Lateral rectus
. Horizontal diplopia
32
33 0 USMl.EWo<ld, llC
34
35 This patient presents w ith a right third nerve (CN Ill) palsy secondary to a compressive aneurysm . Lesions
36 involving CN Ill cause ptosis (drooping of the upper eyelid) and impair adduction, depression, and elevation of
37 the eye. As a result, diagonal (eg, horizontal and vertical) diplopia is frequently present and the eye rests in a
38 "dow n and out" position . Pupillary constriction and accommodation can also be affected as CN Ill carries
39 parasympathetic fibers to the ciliary muscle and the iris sphincter.
40
~imeRemaini~ ~ 0
I Tuto' Feedback Suspend End Block
--------------------------------------------------------------------------------------------------------------------------------------
1
2
" Item: 41 of 45
Q.ld: 8701 [
111 f> Mark .<:J
Previous
[::>
Next
il
Lab Values
~
Notes
~
Calculator
3
4
5 Explanation: User ld:
6
7
8 Eye musde(s) Ocu lomotor
Nerve lesion findi ngs
9 innervated functions
10
11 . Eye deviated downward & laterally
12 Superior rectus, medial Adduction,
. Diagonal diplopia
13
14 CN Ill
rect us, inferior rectus,
inferior oblique & levator
depression,
elevation &
. Dilation of pupil & loss of
15 accommodation
16
17
palpebrae superioris external rotation
. Ptosis
18
19 Internal rotation,
. Eye deviated upward
[ill
1
CN IV Superior oblique depression &
abduction
. Vertical & torsional diplopia
22
23
24
. Eye deviated medially
25
CNVI Lateral rectus Abduction
. Horizontal diplopia
26
<!) USMLEI....,kl.LLC
27
28
This patient presents with a right third nerve (CN Ill) palsy secondary to a compressive aneurysm. l esions
29
involving CN Ill cause ptosis (drooping of the upper eyelid) and impair adduction, depression, and elevation of
30
the eye. As a result, diagonal (eg, horizontal and vertical) diplopia is frequently present and the eye rests in a
31
"down and out" position. Pupillary constriction and accommodation can also be affected as CN Ill carries
32
parasympathetic fibers to the ciliary muscle and the iris sphincter.
33
34 Educational objective:
35 l esions involving CN Ill cause ptosis, a downward and laterally deviated eye, impaired pupillary constriction
36 and accommodation, and diagonal diplopia. The most dreaded cause of CN Ill palsy is an enlarging
37 intracranial aneurysm.
38
39
Time Spent: 1 seconds Copyright USMLEWorld,llC. Last updated: [3/9/2014]
40
~imeRemaini~ ~ 0
I Tuto' Feedback Suspend End Block
--------------------------------------------------------------------------------------------------------------------------------------
1
2
" Item: 42 of 45
Q.ld: 1150 [
111 f> Mark .<:J
Previous
[::>
Next
il
Lab Values
~
Notes
~
Calculator
3
4
A 43-year-old female undergoes brain MRI for a long history of complex neurological complaints. The findings
5
from the imaging study are show n below.
6
7
8
9
10
11
12
13
14
15
16
17
18
19

[ill
1
22
23
24
25
26
27
28
29
30
31
32
33
34
35
36 You suspect that this patient initially suffered from w hich of the follow ing?
37
38
39 0 A. Tinnitus and unilateral deafness
40 () R HP.<'trl<'lr.hP.!': <'tnrl <'!no!':mi<'!

~
41
F!!ck suW.nd
---------------------------------------------------------------------------------------------------------------------------------------------
EnQck
1
2
3
4
5
6
7
8
9
10
11
12
13
14
15
16
17
18
19

[ill
1
22
23
24
25
26
27
28
29
30
31
You suspect that this patient initially suffered from w hich of the following?
32
33
34 0 A. Tinnitus and unilateral deafness
35 0 B. Headaches and anosmia
36
37 0 C. Progressive leg w eakness and spasticity
38 0 D. Unilateral ptosis and gaze palsy
39 0 E. Bitemporal visual field defects
40
41 ~imeRemaini~ ~ 0
I Tuto' Feedback Suspend End Block
---------------------------------------------------------------------------------------------------------------------------------------------
1
2
" Item: 42 of 45
Q.ld: 1150 [
111 f> Mark .<:J
Previous
[::>
Next
il
Lab Values
~
Notes
~
Calculator
3
4 0 B. Headaches and anosmia [8%1
5 " @ C. Progressive leg weakness and spasticity [83%1
6
7 0 D. Unilateral ptosis and gaze palsy [4%1
8 0 E. Bitemporal visual field defects [2%1
9
10
11 Explanation: User ld:
12
The image above show s a tumor adjacent to the falx cerebri and compressing the medial surface of the
13
hemisphere. This is a typical meningioma location. Compression of the parasagittal cortex causes
14
contralateral spastic paralysis of the leg. Focal seizures may also occur.
15
16 Meningiomas are the second-most common brain tumors in adults (the most common is glioblastoma
17 multiforme). These benign and slow -grow ing tumors arise from arachnoid villi. On light microscopy,
18 meningiomas are composed of concentrically-arranged meningothelial cells. Psammoma bodies (concentric
19 laminar calcifications) are characteristic.
[ill
1 Meningiomas are commonly located in the parasagittal region; they can also be found adjacent to the lateral
22 convexity of the hemisphere, in the region of the sphenoid w ing and olfactory groove.
23
24 (Choice A) Tinnitus and unilateral hearing loss are the common presenting features of a cerebellopontine
25 angle tumor, such as an acoustic neuroma.
26
27 (Choice B) Meningiomas located in the olfactory groove present w ith headache and anosmia.
28
29 (Choice D) Unilateral ptosis and gaze palsy ('dow n and out" gaze) are features of CN Ill paralysis. CN Ill can
30 suffer damage from compression (by berry aneurysm or uncal herniation) or via nerve ischemia (diabetes
31 mellitus).
32
(Choice E) Bitemporal hemianopia occurs due to compression of the central part of the optic chiasm.
33
Pituitary adenoma and craniopharyngioma are common causes.
34
35 Educational Objective:
36 Meningiomas are located adjacent to the cerebral surface. Parasagittal meningiomas cause contralateral
37 spastic paresis of the leg due to compression of the leg-foot motor area .
38
39
Time Spent: 7 seconds Copyright USMLEW orld,LLC. Last updated: [11 /10/20111
40
41 ~imeRemaini~ ~ 0
I Tuto' Feedback Suspend End Block
---------------------------------------------------------------------------------------------------------------------------------------------
1
2
" Item: 43 of 45
Q.ld: 1854 [
111 f> Mark .<:J
Previous
[::>
Next
il
Lab Values
~
Notes
~
Calculator
3
4
5
6 A 6-month-old female has a head circumference that is in the 971h percentile for her age. CT scan of the head
7 reveals significant dilation of the lateral ventricles. W hich of the follow ing is the most likely long-term
8 complication of this patient's condition?
9
10
11 0 A. Infection
12
13 0 B. Tumor
14 0 C. Muscle hypertonicity
15
16 0 D. Lower extremity hyporeflexia
17 0 E. Subarachnoid hemorrhage
18
19

[ill
1
22
23
24
25
26
27
28
29
30
31
32
33
34
35
36
37
38
39
40
41
42
~imeRemaini~ ~ 0
I Tuto' Feedback Suspend End Block
---------------------------------------------------------------------------------------------------------------------------------------------
1
2
" Item: 43 of 45
Q.ld: 1854 [
111 f> Mark .<:J
Previous
[::>
Next
il
Lab Values
~
Notes
~
Calculator
3
4
5
6 A 6-month-old female has a head circumference that is in the 971h percentile for her age. CT scan of the head
7 reveals significant dilation of the lateral ventricles. W hich of the follow ing is the most likely long-term
8 complication of this patient's condition?
9
10
11 0 A. Infection [9%]
12 0 B. Tumor [5%]
13
"' @ C. Muscle hypertonicity [53%]
14
15 0 D. Low er extremity hyporeflexia [22%]
16 0 E. Subarachnoid hemorrhage [10%]
17
18
19 Explanation: User ld:

[ill
1 Hydrocephalus is any increase in cerebrospinal fluid (CSF) volume. In infancy and early childhood, the cause
is usually impaired CSF outflow due to congenital abnormalities like Type II Arnold-Chiari malformations,
22
23 hereditary aqueductal stenosis, and prenatal infections (e.g. toxoplasmosis). Hydrocephalus in early infancy
24 typically presents w ith macrocephaly (defined as head circumference greater than two standard deviations
25 above the mean for gender and age), irritability, poor feeding, muscle hypertonicity and hyperreflexia .
26 Hypertonicity and hyperreflexia result from upper motor neuron damage caused by stretching of the
27 periventricular pyramidal tracts.
28
(Choices A, B and E) Infection, tumor, and subarachnoid hemorrhage are potential causes of acquired
29
hydrocephalus but do not result from hydrocephalus.
30
31 (Choice D) Low er extremity hyporeflexia w ould result from damage to the low er motor neurons of the low er
32 extremities, as can occur in poliomyelitis or W erdnig-Hoffman disease.
33
34 Educational Objective:
35 Hydrocephalus in infants presents w ith irritability, poor feeding, increased head circumference and enlarged
36 ventricles on CT. Long-term sequelae of hydrocephalus include low er extremity spasticity due to stretching of
37 the periventricular pyramidal tracts, visual disturbances, and learning disabilities.
38
39 Time Spent: 2 seconds Copyright USMLEW orld,LLC. Last updated: [1 / 10/2013]
40
41
42
~imeRemaini~ ~ 0
I Tuto' Feedback Suspend End Block
---------------------------------------------------------------------------------------------------------------------------------------------
1
2
" Item: 44 of 45
Q.ld : 2020 [
111 f> Mark .<:J
Previous
[::>
Next
il
Lab Values
~
Notes
~
Calculator
3
4
5
6 A 5-year-old male demonstrates persistent food-seeking behavior. He is obese, and his mother reports that
7 he sometimes exhibits aggressive and bizarre behaviors. These clinical findings may be explained by a lesion
8 in w hich of the follow ing hypothalamic nuclei?
9
10
11 0 A. Lateral
12 0 B. Suprachiasmatic
13
14 0 C. Supraoptic
15 0 D. Anterior
16
17 0 E. Ventromedial
18
19

[ill
1
22
23
24
25
26
27
28
29
30
31
32
33
34
35
36
37
38
39
40
41
42
~imeRemaini~ ~ 0
I Tuto' Feedback Suspend End Block
---------------------------------------------------------------------------------------------------------------------------------------------
1
2
" Item: 44 of 45
Q.ld : 2020 [
111 f> Mark .<:J
Previous
[::>
Next
il
Lab Values
~
Notes
~
Calculator
3
4 0 A. Lateral [17%]
5
6 0 B. Suprachiasmatic [7%]
7 0 C. Supraoptic [5%]
8 0 D. Anterior [7%]
9
10 v @ E. Ventromedial [64%]
11
12 Explanation: User ld:
13
14 The body's satiety center resides w ithin the ventromedial nucleus of the hypothalamus. The neurons here
15 detect blood glucose elevations and signal satiety in response. Bilateral lesions of the ventromedial nuclei
16 cause hyperphagia and obesity. Episodes of rage and/or savage behavior are also sometimes noted. The
17 most common cause of such lesions is tumor invasion, for example by a craniopharyngioma . Lesions of the
18 ventromedial nuclei are the cause of obesity in only exceedingly rare cases.
19

[ill
1
(Choice A) The lateral nuclei of the hypothalamus signal hunger. Bilateral lesions w ould cause one to lose
the desire to eat. In adults, starvation ensues, in infants, failure to thrive.
22
23 (Choice B) The suprachiasmatic nuclei are located immediately above the optic chiasm and receive visual
24 input from the retina via the optic tract. The suprachiasmatic nuclei help to regulate circadian rhythms by
25 relaying light information to other hypothalamic nuclei and to the pineal gland.
26
27 (Choice C) The supraoptic and paraventricular nuclei produce vasopressin (ADH) and oxytocin, w hich are
28 carried dow n axons to the posterior pituitary (neurohypophysis), w hich releases these hormones into the
29 circulation .
30
(Choice D) W hen the body temperature rises, the anterior hypothalamic nuclei coordinate cooling by
31
stimulating the parasympathetic nervous system to produce vasodilation and sweating. Destruction of these
32
nuclei can result in uncontrolled hyperthermia and death. The posterior hypothalamic nuclei regulate heat
33
conservation and heat production w hen the body is cold.
34
35 Educational Objective:
36 The ventromedial hypothalamic nuclei contain the satiety center and regulate food intake. Lesions of the
37 ventromedial nuclei result in obesity secondary to hyperphagia as w ell as aggressive, savage behavior.
38
39
Time Spent: 1 seconds Copyright USMLEW orld,LLC. Last updated: [7/7/201 OJ
40
41
42
~imeRemaini~ ~ 0
I Tuto' Feedback Suspend End Block
---------------------------------------------------------------------------------------------------------------------------------------------
1
2
" Item: 45 of 45
Q.ld: 1735 [
111 f> Mark .<:J
Previous
[::>
Next
il
Lab Values
~
Notes
~
Calculator
3
4
5
6 A 23-year-old Caucasian w oman suffers from an acute attack of optic neuritis. She has a history of
7 relapsing-remitting multiple sclerosis. An MRI of her brain show s an area of demyelinization involving her left
8 optic nerve. W hich of the follow ing is most likely to be found on physical examination?
9
10 Light in the right eye Light in the left eye
11 Right pupil Left pupil Right pupil Left pupil
12
13
14
15
16 A. Constrict Constrict No change No change
17
0
18
19 Constrict No change No change No change
0 B.
[ill
1
Constrict Constrict No change
22 Constrict
0 C.
23
24
25 Constrict No change Constrict No change
0 D.
26
27
28 Constrict Constrict No change Constrict
0 E.
29
30
31
32
33
34
35
36
37
38
39
40
41
42
~imeRemaini~ ~ 0
I Tuto' Feedback Suspend End Block
---------------------------------------------------------------------------------------------------------------------------------------------
1
2
" Item: 45 of 45
Q.ld: 1735 [
111 f> Mark .<:J
Previous
[::>
Next
il
Lab Values
~
Notes
~
Calculator
3
4
5 .; @ A. Constrict Constrict No change No change
[70%]
"
6
7
Constrict No change No change No change
8 0 B. (1 0%]
9
10 Constrict Constrict No change
Constrict
11 0 C. (11 %]
12
13 Constrict No change Constrict No change
14 0 D. [7%]
15
16 Constrict Constrict No change Constrict
17 0 E. [2%]
18
19

[ill
1
Explanation: User ld:

22 Pupillary light reflex


23
24
25
26
27
28
29
30
31
32
33
34
35
36
37
38
39
40
41
42
1
2
" Item: 45 of 45
Q.ld: 1735 [
111 f> Mark .<:J
Previous
[::>
Next
il
Lab Values
~
Notes
~
Calculator
3
4
5 Pupillary light reflex
6
7
8
9
10
11
12
13
14
15
16
17
18
19

[ill
1
22
23
24
25
26
27
28
29
30
31
32
33
34
Edinger-Westpha l nucleus
35
36
37 Pretectal nucleus
(I) USMLE\Vorld. LLC
38
39
40 Thic:: n::aticnt ic::: cvnQ.ricnr inn ::a \ tcrv r n rnrYin n ::~ri Hcrc::: c cffcrt n f rn1 1ltiniQ. c:::r lcrn c::ic:: rlc rn v clini?::atin n nf the n ntir

~
41
42
--------------------------------------------------------------------------------------------------------------------------------------------------------------
F!!ck suW.nd EnQck
1
2
3
4
5
6
7
8
9
10
11 Edinger-Westphal nucleus
12
13
Pretectal nucleus
14
@ USMLEWorld, lLC
15
16
17 This patient is experiencing a very common adverse effect of multiple sclerosis: demyelinization of the optic
18 nerve (CN II}. This condition causes impairment of the light reflex pathway, since the afferent (sensory} limb
19 is carried by the optic nerve. As light enters the eye, the retina generates a signal that is transmitted along the
optic nerve bilaterally to the pretectal nucleus, located in the superior colliculus. From there, fibers project to
[ill
1 the ipsilateral and contralateral Edinger-W estphal nuclei, w ith the decussating fibers traversing w ithin the
posterior commissure.
22
23
24 The efferent (motor} fibers of the reflex pathw ay originate from parasympathetic preganglionic neurons in the
25 Edinger-W estphal nucleus. These travel w ithin the oculomotor nerve to the ciliary ganglion . There they
26 synapse on parasympathetic pos.ganglionic neurons, w hich in turn project fibers that innervate the sphincter
27 of the iris.
28
As w ith many patients suffering from multiple sclerosis, this patient's injury is isolated to only 1 optic nerve.
29
Thus, light entering the affected eye (in this case, the left} w ill not be transmitted to the midbrain, and neither
30
pupil w ill respond . How ever, light in the contralateral eye w ill be transmitted normally to the midbrain, and the
31
response w ill be transmitted along the unaffected oculomotor nerves to constrict the pupils of both eyes.
32
33 Educational objective:
34 The afferent limb of the light reflex pathw ay is the optic nerve; the efferent limb is the parasympathetic fibers of
35 the oculomotor nerve. W hen an optic nerve is damaged, light in that eye w ill cause neither pupil to constrict
36 (the nerve can't sense the light}. How ever, light in the contralateral eye w ill cause both pupils to constrict
37 (because the motor pathw ays are intact}.
38
39
Time Spent: 2 seconds Copyright USMLEW orld,LLC. Last updated: [1 / 14/2014]
40
41
42
~imeRemaini~ ~ 0
I Tuto' Feedback Suspend End Block
---------------------------------------------------------------------------------------------------------------------------------------------
2 Item: 1 of 12
Q. ld: 1100 [
lil f> Mark .<:J
Previous
[::>
Next
il
Lab Values
~
Notes
~
Calculator
3
4
5
A 21-year-old football player is brought to the emergency room complaining of back pain . An X-ray show s a
6
fractured 121" rib on the left side. W hich of the follow ing structures is most likely to be lacerated by the broken
7
rib?
8
9 0 A. Left kidney
10
11 0 B. Spleen
12
0 C. Liver
0 D. Visceral pleura
0 E. Pancreas

~imeRemaini~ ~ 0
I Tuto' Feedback Suspend End Block
---------------------------------------------------------------------------------------------------------------------------------------------
2 Item: 1 of 12
Q. ld: 1100 [
lil f> Mark .<:J
Previous
[::>
Next
il
Lab Values
~
Notes
~
Calculator
3
4 fractured 12'" rib on the left side. Which of the following structures is most likely to be lacerated by the broken
5 rib?
6 .; @ A. Left kidney [50%]
7
8 0 B. Spleen [38%]
9 0 C. Liver [2%]
10 0 D. Visceral pleura [6%]
11
12 0 E. Pancreas [3%]

Explanation: User ld:

The 121h rib overlies the parietal pleura medially and the kidney laterally. Recall that the 11 th and 121h ribs are
"floating" ribs, meaning that they are not bound to the anterior rib cage by cartilage as are the more superior
"false" ribs. For this reason, the distal tip of the left 121h rib can be displaced into the retroperitoneum when
fractured, lacerating the left kidney.

(Choice B) The spleen lies in the posterior superior portion of the left abdominal cavity. The left 91h, 1O'h and
11 th ribs overlie the spleen.

(Choice C) The liver occupies much of the right upper quadrant of the abdominal cavity. The 81h, gth, 1O'h and
11 th ribs overlie the liver's posterior surface on the right. The positions of both the liver and spleen shift slightly
with diaphragmatic movement during inhalation and exhalation.

(Choice D) The visceral pleura envelop the lungs. At rest, the inferior margin of the left lung lies at the level of
the 1O'h rib in the midscapular line. During maximal inhalation, the lung may descend to the level of the most
medial portion of the 121h rib, but fractures of the 1st through 61h ribs have the greatest chance of damaging
the visceral pleura.

(Choice E) The pancreas is a partially retroperitoneal organ that overlies the body of the second lumbar
vertebra. Classically, crushing abdominal trauma is the injury that causes pancreatic damage.

Educational Objective:
The left kidney lies immediately deep to the tip of the 121h rib on the left.

Time Spent: 4 seconds Copyright USMLEWorld,LLC. Last updated: [11 /10/2011]

~imeRemaini~ ~ 0
I Tuto' Feedback Suspend End Block
--------------------------------------------------------------------------------------------------------------------------------------
-
1

3
4
5
6
Item: 2 of 12
Q. ld: 654 [
lil f> Mark .<:J
Previous
[::>
Next
il
Lab Values
~
Notes
~
Calculator

A 23-year-old male w ho has been previously healthy is brought to the ER by his friends after a street fight.
7 They tell you that they w ere "walking dow n the street minding their ow n business w hen a guy jumped out from
8 behind a dumpster and stabbed him in the chest". They state that they disarmed the man after his single
9 attack and that he ran off into an alley. The patient is conscious but in distress. Physical examination reveals
10 a stab w ound at the fifth intercostal space along the left midclavicular line. W hich of the follow ing is most
11 likely punctured in this patient?
12

0 A. Azygous vein
0 B. Inferior vena cava
0 C. Left atrium
0 D. Left lung
0 E. Right ventricle

~imeRemaini~ ~ 0
I Tuto' Feedback Suspend End Block
---------------------------------------------------------------------------------------------------------------------------------------------
-
1

3
4
5
6
Item: 2 of 12
Q. ld: 654 [

~ @ D. Left lung [50%]


0 E. Right ventricle [39%]
lil f> Mark .<:J
Previous
[::>
Next
il
Lab Values
~
Notes
~
Calculator

7
8 Explanation: User ld:
9
10
11
12

USMLEWod<t lLC () 2011

Most of the volume of the thoracic cavity is occupied by the lungs. The apex of each lung extends into the
nPr k :::mnrn vim:::~tPi v thrPP tn fn11r r PntimPtPrc::: :::~hmtP thP firc;:;t rih ThP h 1nn h::~c;:;pc::: :::.rp in rlirprt r nnt::~r t w ith thP

~imeRemaini~ ~ 0
I Tuto' Feedback Suspend End Block
------------------------------------------------------------------------------------------------------------------------------------------------------
-
1

3
4
5
6
Item: 2 of 12
Q. ld: 654 [

USMLEWO<Id. LLC <1> 2011


lil f> Mark .<:J
Previous
[::>
Next
il
Lab Values
~
Notes
~
Calculator

7
Most of the volume of the thoracic cavity is occupied by the lungs. The apex of each lung extends into the
8
neck approximately three to four centimeters above the first rib. The lung bases are in direct contact w ith the
9
diaphragm, w hich separates the right lung from the right lobe of the liver, and the left lung from the left lobe of
10
the liver, the stomach, and the spleen . The mediastinal surface of each lung has a cardiac impression that
11
accommodates the heart.
12
The heart is located behind the sternum and its anterior surface is partially covered by the lungs. The apex is
formed by the left ventricle and is covered by the left lung. It lies behind the fifth left intercostal space at the left
midclavicular line. The sternocostal (anterior) surface of the heart is formed chiefly by the right ventricle. The
diaphragmatic (inferior) surface is formed by left and right ventricles, and is in contact w ith the central tendon
of the diaphragm. The posterior surface of the heart is formed mainly by the left atrium .

Penetrating injury involving the fifth intercostal space at the left midclavicular line w ould most likely injure the
left lung. Penetration of the left lung at this location could lead to injury of the apex of the heart (left ventricle)
as w ell, if the v.ound ~-~ere deep enough. All other heart chambers lie medial to the left midclavicular line and
w ould not be affected.

(Choice A) The azygos vein lies in the posterior mediastinum immediately to the right of the midline. It drains
blood from the posterior intercostal veins into the superior vena cava.

(Choice B) The inferior vena cava is located in the mediastinum, lies to the right of midline, and drains into
the right atrium .

(Choice C) The left atrium is located posterior to the left ventricle and forms the base of the heart.

(Choice E) The right ventricle forms the anterior (sternal) surface of the heart and the majority of its inferior
border. It lies medially to the left midclavicular line.

Educational Objective:
The left ventricle forms the apex of the heart and reaches as far as the fifth intercostal space at the left
midclavicular line. All other chambers of the heart lie medial to the left midclavicular line. The lungs overlap
much of the anterior surface of the heart.

Time Spent: 2 seconds Copyright USMLEW orld,LLC. Last updated: [11/212013]

~imeRemaini~ ~ 0
I Tuto' Feedback Suspend End Block
---------------------------------------------------------------------------------------------------------------------------------------------
1
il
-
2

4
5
6
7
Item: 3 of 12
Q. ld: 2021 [
lil f> Mark .<:J
Previous
[::>
Next

A 23-year-old man comes to the physician complaining of right-sided testicular sw elling . He first noticed the
Lab Values
~
Notes
~
Calculator

sw elling 1 w eek ago w hile getting ready for w ork. He denies any testicular pain or history of
8 trauma . How ever, he has noticed a heavy, pressing sensation involving his scrotum and low er
9 abdomen. Physical examination reveals asymmetric swelling of the right testis, and subsequent
10 ultrasonography show s a solid testicular mass. If malignant, this patient's tumor is most likely to spread to
11 w hich of the follow ing lymph node groups?
12

0 A. Superficial inguinal
0 B. Deep inguinal
0 C. External iliac
0 D. Common iliac
0 E. Inferior mesenteric
0 F. Para-aortic

~imeRemaini~ ~ 0
I Tuto' Feedback Suspend End Block
---------------------------------------------------------------------------------------------------------------------------------------------
Item: 3 of 12
Q. ld: 2021 [
lil f> Mark .<:J
Previous
[::>
Next
il
Lab Values
~
Notes
~
Calculator
4 I .; @ F. Para-aortic [75%]
5
6
7 Explanation: User ld:
8
9 Lymph vessels & nodes of male genitalia
10
11
12

Para-aortic nodes
----:>----:,~;___ __ (receive lymph
drainage from testis)

Deep inguinal nodes


..,....---;-...;!--- - (receive lymph drainage
from glans penis &
superficial nodes)

Superficial inguinal nodes


~---,.-,--+- (receive lymph drainage
Item: 3 of 12
Q. ld: 2021 [
lil f> Mark .<:J
Previous
[::>
Next
il
Lab Values
~
Notes
~
Calculator
4
5
Lymph vessels & nodes of male genitalia
6
7
8
9
10
11 Para-aortic nodes
12 -:::=------':~--- (receive lymph
drainage from testis)

Deep inguinal nodes


~---;--..;!----- (receive lymph drainage
from glans penis &
superficial nodes)

Superficial inguinal nodes


->-----:'--:--~- (receive lymph d rainage
from scrotum)
1

-
2

4
5
6
7
Deep inguinal nodes
~---;--..;!----- (receive lymph drain age
from glans penis &
8
superficial nodes)
9
10
11 Super ficial inguinal nodes
12 ->------:'--:--~- (receive lymph d rainage
from scrotum)

@ USMLEWolld, LLC

In general, the lymph drainage from a particular organ follow s the path of the arterial supply to that site. During
fetal development, the testes originate w ithin the retroperitoneum and establish their arterial supply from the
abdominal aorta . The testes subsequently descend through the inguinal canals into the scrotum, taking w ith
them their arterial, venous, and lymphatic supplies. Thus, lymph from the testes drains through lymph
channels directly back to the para-aortic (retroperitoneal) lymph nodes.

(Choice A) The superficial inguinal lymph nodes are located on the anterior thigh inferior to the inguinal
ligament. These nodes drain nearly all cutaneous structures inferior to the umbilicus, including the external
genitalia and the anus up to the pectinate line.

(Choice B) The deep inguinal nodes reside under the fascia lata on the medial side of the femoral vein . They
receive afferents from the superficial inguinal nodes and deep lymphatic trunks along the femoral vessels.
The lymphatics from the glans penis and clitoris also drain directly to these nodes.

(Choice C) The external iliac nodes drain the superficial and deep inguinal nodes and the deep lymphatics of
the abdominal w all below the umbilicus.

~ F!!ck suW.nd
---------------------------------------------------------------------------------------------------------------------------------------------
EnQck
1

-
2

4
5
6
7
8
9 @ USMLEWorld, UC
10
11 In general, the lymph drainage from a particular organ follow s the path of the arterial supply to that site. During
12 fetal development, the testes originate w ithin the retroperitoneum and establish their arterial supply from the
abdominal aorta . The testes subsequently descend through the inguinal canals into the scrotum, taking w ith
them their arterial, venous, and lymphatic supplies. Thus, lymph from the testes drains through lymph
channels directly back to the para-aortic (retroperitoneal) lymph nodes.

(Choice A) The superficial inguinal lymph nodes are located on the anterior thigh inferior to the inguinal
ligament. These nodes drain nearly all cutaneous structures inferior to the umbilicus, including the external
genitalia and the anus up to the pectinate line.

(Choice B) The deep inguinal nodes reside under the fascia lata on the medial side of the femoral vein . They
receive afferents from the superficial inguinal nodes and deep lymphatic trunks along the femoral vessels.
The lymphatics from the glans penis and clitoris also drain directly to these nodes.

(Choice C) The external iliac nodes drain the superficial and deep inguinal nodes and the deep lymphatics of
the abdominal w all below the umbilicus.

(Choice D) The common iliac nodes are located alongside the common iliac artery and drain the internal and
external iliac nodes.

(Choice E) The inferior mesenteric nodes drain the structures supplied w ith arterial blood by branches of the
inferior mesenteric artery (eg, the left colic, sigmoid, and superior rectal arteries). Thus, these nodes drain
the descending and sigmoid colon as w ell as the upper part of the rectum . Their efferents drain to pre-aortic
nodes.

Educational objective:
lymph from the testes drains through lymph channels directly back to the para-aortic lymph nodes. In
contrast, lymph from the scrotum drains to the superficial inguinal lymph nodes.

Time Spent: 1 seconds Copyright USMlEW orld,llC. l ast updated: [2124/2014]

~ F!!ck suW.nd
---------------------------------------------------------------------------------------------------------------------------------------------
EnQck
1
2 Item: 4 of 12 lil f> Mark .<:J [::> il ~ ~

-
Q. ld: 1745 [ Previous Next Lab Values Notes Calculator
3

5
6 A 63-year-old male w ith a history of stroke and resultant severe oropharyngeal dysphagia develops a right
7 low er lobe pneumonia after an episode of vomiting. This type of aspiration pneumonia commonly affects the
8 right low er lung lobe because:
9
10
11
A. The right main bronchus is longer than the left main bronchus
12

0 B. The right main bronchus is straighter than the left main bronchus
0 C. The right main bronchus is narrow er than the left main bronchus
0 D. The right low er lobe receives a richer blood supply than the left low er lobe
0 E. Vascular resistance is higher in the right low er lobe than the right upper lobe

~ F!!ck suW.nd
---------------------------------------------------------------------------------------------------------------------------------------------
EnQck
1
2 Item: 4 of 12 lil f> Mark .<:J [::> il ~ ~

-
Q. ld: 1745 [ Previous Next Lab Values Notes Calculator
3

5
6 A 63-year-old male w ith a history of stroke and resultant severe oropharyngeal dysphagia develops a right
7 low er lobe pneumonia after an episode of vomiting. This type of aspiration pneumonia commonly affects the
8 right low er lung lobe because:
9
10
11
0 A. The right main bronchus is longer than the left main bronchus [2%]
12 .; @ B. The right main bronchus is straighter than the left main bronchus [94%]
0 C. The right main bronchus is narrow er than the left main bronchus [1 %]
0 D. The right low er lobe receives a richer blood supply than the left low er lobe (1 %]
0 E. Vascular resistance is higher in the right low er lobe than the right upper lobe [1 %]

Explanation: User ld:

The right main bronchus is more prone to foreign body aspiration than the left main bronchus because the
right main bronchus has a larger diameter, is shorter, and is oriented more vertically than the left main
bronchus. This fact can be remembered using the mnemonic, "Sw allow a bite, goes dow n the right."

Educational objective:
Aspirated or inhaled particles are most likely to become lodged in the right main bronchus or its branches
because this bronchus is shorter, w ider and more vertically oriented than the left main bronchus.

Time Spent: 2 seconds Copyright USMLEW orld,LLC. Last updated: [1 / 1/2014]

~imeRemaini~ ~ 0
I Tuto' Feedback Suspend End Block
---------------------------------------------------------------------------------------------------------------------------------------------
1
2 Item: 5 of 12
Q. ld: 1535 [
lil f> Mark .<:J
Previous
[::>
Next
il
Lab Values
~
Notes
~
Calculator
3

-
4

6
7
8
9
A 26-year-old male presents to the ER w ith a sharp pain in his neck and shoulder. He has some breathing
difficulty secondary to pain w ith inspiration. His past medical history is not significant except for a mild
respiratory disease a few days ago. His blood pressure is 120/70 mmHg and his heart rate is 100/min. The
pain experienced by this patient is most likely carried by w hich of the follow ing nerves?
10
11
12 0 A. Phrenic
0 B. Accessory
0 C. Vagus
0 D. Superior epigastric
0 E. Long thoracic

~imeRemaini~ ~ 0
I Tuto' Feedback Suspend End Block
---------------------------------------------------------------------------------------------------------------------------------------------
1
2 Item: 5 of 12
Q. ld: 1535 [
lil f> Mark .<:J
Previous
[::>
Next
il
Lab Values
~
Notes
~
Calculator
3

-
4

6
7
8
9
v @ A. Phrenic [72%]
0
0
0
B. Accessory [13%]
C. Vagus [9%]
D. Superior epigastric [1 %]

10 0 E. Long thoracic (5%]


11
12
Explanation: User ld:

Pleuritic chest pain can result from any condition that causes inflammation of the pleura . The pleura is divided
into segments, as follow s:
1. Visceral pleura: The visceral pleura, or pulmonary pleura, covers all surfaces of the lungs, including
the surfaces w ithin the pulmonary fissures.
2. Parietal pleura: This represents the remainder of the pleura that is not in contact w ith the lungs and
can be subdivided as follow s:

Costal pleura: Covers the thoracic w all including the ribs, sternum, intercostal spaces, costal
cartilages, and the sides of the thoracic vertebrae.
Mediastinal pleura: Covers the mediastinum
Diaphragmatic pleura: Covers the surface of the diaphragm located w ithin the thoracic cavity
Cervical pleura: Extends w ith the apices of the lung into the neck.

The parietal pleura is innervated by somatic sensory (sensory afferent} nerves, w hich allow the sensation of
sharp and localized pain. The phrenic nerve, w hich is derived from the C3-C5 nerve roots, delivers motor
innervation to the diaphragm and additionally carries pain fibers from the diaphragmatic and mediastinal
pleura . Irritation of the pleura in either of these areas w ill cause a sharp pain w orsened by inspiration that w ill
be "referred" to the C3-C5 distribution, w hich lies at the base of the neck and over the shoulder. Sensory
innervation of the remainder of the parietal pleura is accomplished by intercostal nerves.

(Choice B) The spinal accessory nerve is the 11 th cranial nerve and provides motor innervation to the
sternocleidomastoid and trapezius muscles.

(Choice C) The vagus nerve is the 1O'h cranial nerve and is the major source of parasympathetic innervation
to the viscera of the chest w all and the foregut.

~imeRemaini~ ~ 0
I Tuto' Feedback Suspend End Block
---------------------------------------------------------------------------------------------------------------------------------------------
1
2 Item: 5 of 12
Q. ld: 1535 [
lil f> Mark .<:J
Previous
[::>
Next
il
Lab Values
~
Notes
~
Calculator
3

-
4

6
7
8
9
into segments, as follow s:
1. Visceral pleura: The visceral pleura, or pulmonary pleura, covers all surfaces of the lungs, including
the surfaces w ithin the pulmonary fissures.
2. Parietal pleura: This represents the remainder of the pleura that is not in contact w ith the lungs and
can be subdivided as follow s:
10 Costal pleura: Covers the thoracic w all including the ribs, sternum, intercostal spaces, costal
11 cartilages, and the sides of the thoracic vertebrae.
12 Mediastinal pleura: Covers the mediastinum
Diaphragmatic pleura: Covers the surface of the diaphragm located w ithin the thoracic cavity
Cervical pleura: Extends w ith the apices of the lung into the neck.

The parietal pleura is innervated by somatic sensory (sensory afferent) nerves, w hich allow the sensation of
sharp and localized pain. The phrenic nerve, w hich is derived from the C3-C5 nerve roots, delivers motor
innervation to the diaphragm and additionally carries pain fibers from the diaphragmatic and mediastinal
pleura . Irritation of the pleura in either of these areas w ill cause a sharp pain w orsened by inspiration that w ill
be "referred" to the C3-C5 distribution, w hich lies at the base of the neck and over the shoulder. Sensory
innervation of the remainder of the parietal pleura is accomplished by intercostal nerves.

(Choice B) The spinal accessory nerve is the 11 '" cranial nerve and provides motor innervation to the
sternocleidomastoid and trapezius muscles.

(Choice C) The vagus nerve is the 101" cranial nerve and is the major source of parasympathetic innervation
to the viscera of the chest w all and the foregut.

(Choice D) There is no superior epigastric nerve. The superior epigastric artery and vein course over the
superior half of the abdominal w all and anastomose w ith the internal thoracics and the inferior epigastrics.

(Choice E) The long thoracic nerve innervates the serratus anterior. Damage to this nerve causes w inged
scapula .

Educational Objective:
Irritation of the mediastinal or diaphragmatic parietal pleura w ill cause sharp pain, w orse on inspiration, in the
C3-C5 distribution. Pain sensation from these areas is carried by the phrenic nerve.

Time Spent: 1 seconds Copyright USMLEW orld,LLC. Last updated: [7/7/201 0]

~imeRemaini~ ~ 0
I Tuto' Feedback Suspend End Block
---------------------------------------------------------------------------------------------------------------------------------------------
1
2 Item: 6 of 12
Q. ld: 1739 [
lil f> Mark .<:J
Previous
[::>
Next
il
Lab Values
~
Notes
~
Calculator
3
4

-
5

7
8
9
10
A 23-year-old female is examined by an obstetrician in the delivery room . Starting intravaginally, he injects
lidocaine solution near the tip of the ischial spine. W hich of the follow ing nerves is most likely blocked by the
injection?

11 O A. Genitofemoral
12 0 B. Iliohypogastric
o c. Lateral femoral cutaneous
o o. Obturator
O E. Pudendal
0 F. Inferior gluteal

~imeRemaini~ ~ 0
I Tuto' Feedback Suspend End Block
---------------------------------------------------------------------------------------------------------------------------------------------
1
2 Item: 6 of 12
Q. ld: 1739 [
lil f> Mark .<:J
Previous
[::>
Next
il
Lab Values
~
Notes
~
Calculator
3
4

-
~ @ E. Pudendal [87%]
5
0 F. Inferior gluteal [2%]
7
8
9 Explanation: User ld:
10
A pudendal nerve block is one method of providing anesthesia during childbirth, and this is the block that is
11
described in this scenario. Sometimes this method is used if a w oman has progressed too far in labor to
12
receive epidural anesthesia. The pudendal nerve is derived from the S2 - S4 nerve roots and provides
sensory innervation to the perineum and genitals (of both sexes) as w ell as motor innervation to the sphincter
urethrae and the external anal sphincter.

W hen administering a pudendal nerve block, the physician generally palpates intravaginally for the
ischial spines and attempts to administer the anesthetic agent in that location .

(Choice A} The genitofemoral nerve originates at L1 and L2 and courses on the anterior surface of the psoas
muscle. It splits into the genital and femoral branches, innervating the scrotum / labia majora and cutaneously
innervating the femoral triangle, respectively.

(Choice B) The iliohypogastric nerve is derived from T12 / L1 and courses in that dermatome to innervate the
skin overlying the iliac crests.

(Choice C) The lateral femoral cutaneous nerve is derived from L2 and L3. It courses deep to the inguinal
ligament to innervate the skin on the anterolateral thigh.

(Choice D) The obturator nerve is derived from L3 and L4 and serves to provide motor innervation to the
medial thigh (adductors).

(Choice F) The inferior gluteal nerve is derived from L5 - S2 roots and provides motor innervation to the
gluteus maximus.

Educational Objective:
A pudendal nerve block can be performed by injecting anesthetic intravaginally in the region of the ischial
spine. Blocking the pudendal nerve provides anesthesia to the majority of the perineum; additional blockade of
the genitofemoral and ilioinguinal nerves would provide complete perineal and genital anesthesia.

Time Spent: 1 seconds Copyright USMLEWorld,LLC. Last updated: [2/22/2012]

~imeRemaini~ ~ 0
I Tuto' Feedback Suspend End Block
---------------------------------------------------------------------------------------------------------------------------------------------
1
2 Item: 7 of 12
Q. ld: 1695 [
lil f> Mark .<:J
Previous
[::>
Next
il
Lab Values
~
Notes
~
Calculator
3
4
5

-
6

8
9
10
11
A 22-year-old male is brought to the emergency room after being involved in a fight in a nearby bar. His blood
pressure is 100/70 mmHg and his heart rate is 100/min. Physical examination reveals a penetrating
right-sided neck injury just above the clavicle and lateral to the manubrium sterni. W hich of the follow ing
structures w as most likely injured?

12 0 A. Inferior thyroid artery


0 B. Ansa cervicalis
0 C. Lung pleura
0 D. Accessory nerve
0 E. Carotid body

~imeRemaini~ ~ 0
I Tuto' Feedback Suspend End Block
---------------------------------------------------------------------------------------------------------------------------------------------
1
2 Item: 7 of 12
Q. ld: 1695 [
lil f> Mark .<:J
Previous
[::>
Next
il
Lab Values
~
Notes
~
Calculator
3
4 v @ C. Lung pleura [53%]
5

-
6 0 D. Accessory nerve [9%]
0 E. Carotid body [ 15%]
8
9
10 Explanation: User ld:
11
12

The patient likely has a tension pneumothorax, a condition w here air enters the pleural cavity during inspiration
but is not expelled during exhalation. An increasing volume of air accumulates w ithin the pleural space,
ultimately causing deviation of the lungs and mediastinum to the opposite side of the chest. The increased
pressure w ithin the chest cavity causes decreased systemic venous return to the heart leading to decreased
cardiac output. Signs and symptoms of tension pneumothorax include tachycardia, hypotension, tachypnea,
hypoxia, and absence of breath sounds and hyperresonance to percussion on the affected side. A chest
X-ray w ill confirm the diagnosis; treatment is emergent needle thoracostomy or chest tube.

In patients w ith neck injuries, it is important to remember that the lung apices and cervical pleura extend above
thP r 1::::.\Jir IP ;::mrl firc;:;t rih thrn1 1nh thp c;:;11nP.rinr thnr:::.r ir ::~nP.rtllrP intn thp npr k Ac::. thp :::.hmtP r:::~rli nnr:::mh

~
---------------------------------------------------------------------------------------------------------------------------------------------
F!!ck suW.nd EnQck
1
2
3
4
5

-
6

8
9
10
11
The patient likely has a tension pneumothorax, a condition w here air enters the pleural cavity during inspiration
but is not expelled during exhalation. An increasing volume of air accumulates w ithin the pleural space,
ultimately causing deviation of the lungs and mediastinum to the opposite side of the chest. The increased
pressure w ithin the chest cavity causes decreased systemic venous return to the heart leading to decreased
12 cardiac output. Signs and symptoms of tension pneumothorax include tachycardia, hypotension, tachypnea,
hypoxia, and absence of breath sounds and hyperresonance to percussion on the affected side. A chest
X-ray w ill confirm the diagnosis; treatment is emergent needle thoracostomy or chest tube.

In patients w ith neck injuries, it is important to remember that the lung apices and cervical pleura extend above
the clavicle and first rib through the superior thoracic aperture into the neck. As the above radiograph
illustrates, stab w ounds immediately above the clavicle and lateral to the manubrium can puncture the pleura
and cause pneumothorax, tension pneumothorax or hemothorax.

(Choice A) The inferior thyroid artery arises from the thyrocervical trunk, w hich itself arises from the
subclavian artery. The inferior thyroid artery courses posterior to the carotid artery and jugular vein and
supplies the inferior pole of the thyroid gland.

(Choice B) The ansa cervicalis arises from the C1 , C2 and C3 nerve roots and innervates the sternohyoid,
sternothyroid and omohyoid muscles of the anterior neck. Penetrating trauma to the neck superior to the
cricoid cartilage might injure this nerve.

(Choice D) The accessory nerve (CN XI) innervates the sternocleidomastoid and trapezius muscles. This
nerve may be injured during surgery involving the posterior triangle of the neck (a region bounded by the
sternocleidomastoid muscle, trapezius muscle, and clavicle).

(Choice E) The carotid body, w hich contains 0 2 , C0 2 and W chemoreceptors, lies at the bifurcation of the
common carotid artery oust inferior to the hyoid bone).

Educational Objective:
The lung apices extend above the level of the clavicle and first rib through the superior thoracic aperture.
Penetrating injury in this area may lead to pneumothorax.

Time Spent: 1 seconds Copyright USMLEW orld,LLC. Last updated: [7/7/201 0]

~imeRemaini~ ~ 0
I Tuto' Feedback Suspend End Block
---------------------------------------------------------------------------------------------------------------------------------------------
1
2 Item: 8 of 12
Q. ld: 1746 [
lil f> Mark .<:J
Previous
[::>
Next
il
Lab Values
~
Notes
~
Calculator
3
4 A 56-year-old male smoker is being evaluated for right shoulder pain . You suspect a malignancy in the
5 location marked by a star below .
6

-
7

9
10
11
12

W hich of the follow ing additional findings is likely to also be present in this patient as a result of local tumor
extension?

0 A. Bitemporal hemianopsia
0 B. Unilateral deafness
0 C. Ptosis
0 D. Horizontal nystagmus
0 E. Anosmia

~imeRemaini~ ~ 0
I Tuto' Feedback Suspend End Block
---------------------------------------------------------------------------------------------------------------------------------------------
1
2 Item: 8 of 12
Q. ld: 1746 [
lil f> Mark .<:J
Previous
[::>
Next
il
Lab Values
~
Notes
~
Calculator
3
4
5 0 A. Bitemporal hemianopsia [2%]
6

-
7
0 B. Unilateral deafness [2%]
" @ C. Ptosis [90%]
9 0 D. Horizontal nystagmus [2%]
10
11
0 E. Anosmia [3%]
12
Explanation: User ld:

The star on the chest x-ray above marks the pulmonary apex. Apical lung tumors, or Pancoast tumors
(named for the radiologist w ho first described them), can cause a variety of symptoms due to invasion of
nearby structures. Most commonly, Pancoast tumors cause shoulder pain due to invasion of the parietal
pleura, vertebrae, and superior ribs. Invasion of the brachial plexus can cause w eakness and paresthesias of
the arm, particularly at sites innervated by the inferior trunk. Superior vena cava (SVC) compression by tumor
can cause SVC syndrome, and recurrent laryngeal nerve involvement can cause hoarseness. Horner
syndrome, the triad of ipsilateral miosis, ptosis and anhidrosis, occurs in up to 50% of patients w ith Pancoast
tumors, and occurs due to tumor invasion of the paravertebral sympathetic chain.

(Choice A) Bitemporal hemianopsia classically results from masses w ithin the hypophysial fossa, such as
pituitary tumors. Prolactinomas are the most common pituitary tumors.

(Choice B) Unilateral deafness can result from many insults including cerumen impaction, damage to the
conductive system, or damage to cranial nerve VIII.

(Choice D) Horizontal nystagmus can result from disease of the peripheral vestibular system or severe
damage to one of the cerebral hemispheres.

(Choice E) Anosmia (loss of the sense of smell) is associated w ith a variety of potential causes including
Kallman syndrome, toxin exposure, malignancy invading the olfactory region of the CNS or nasopharynx, etc.

Educational Objective:
Apical lung tumors are called Pancoast tumors. Pancoast tumors can cause Horner syndrome, SVC
syndrome, arm w eakness, arm paresthesias, and hoarseness.

Time Spent: 3 seconds Copyright USMLEW orld,LLC. Last updated: [7/7/201 OJ

~imeRemaini~ ~ 0
I Tuto' Feedback Suspend End Block
---------------------------------------------------------------------------------------------------------------------------------------------
1
2 Item: 9 of 12
Q. ld: 844 [
lil f> Mark .<:J
Previous
[::>
Next
il
Lab Values
~
Notes
~
Calculator
3
4
5
6 A medical resident performs a diagnostic procedure on 32-year-old male. He inserts a needle along the upper
7 border of the 1onrib at the right midaxillary line. W hich of the follow ing is most likely to be punctured as a

-
8

10
11
12
result of the intervention?

0 A. Right lower lung lobe


0 B. Liver
0 C. Intercostal artery
0 D. Intercostal nerve
0 E. Hepatic veins

~imeRemaini~ ~ 0
I Tuto' Feedback Suspend End Block
---------------------------------------------------------------------------------------------------------------------------------------------
1
2 Item: 9 of 12
Q. ld: 844 [
lil f> Mark .<:J
Previous
[::>
Next
il
Lab Values
~
Notes
~
Calculator
3
4 v ~ ts. uver [ol '1oJ
5 0 C. Intercostal artery [7%]
6
0 D. Intercostal nerve [1 3%]
7

-
8

10
11
12
0 E. Hepatic veins [2%]

Explanation: User ld:

To avoid complications w hile performing a thoracentesis, it is necessary to remember the location of the
lungs, pleura, and other organs of the chest and upper abdomen . The portion of pleura that covers the
surface of the lung is called the visceral pleura . The parietal pleura lines the inner surface of the chest w all
and the diaphragm, and is innervated by somatic sensory nerves. The diaphragm is bound to the inferior
margin of the thorax, and the portion of parietal pleura that covers the diaphragm extends to the follow ing
levels
Lower border of
Midclavicular line Midaxillary line Paravertebral line
the pleura
Upper border of 1o~
Right 7" rib 12"' rib
rib
Low er border of 1o~
Left 7"' rib 12"' rib
rib I
The low er border of the lung is usually located tw o intercostal spaces above the respective pleural border.
Thoracentesis, therefore, should be performed between 5"' and 7"' ribs along the midclavicular line, the 7"' and
9~ ribs along the midaxillary line, and 9" and 11" ribs along the paravertebral line. If the needle is inserted
higher, there is a risk of lung injury. Insertion of the needle below the 9"' rib at the middle axillary line on the
right may cause liver injury (Choice B).

(Choice A) The low er border of the right lung is located two intercostal spaces above the pleural border. The
lung is not likely to be injured by insertion of a needle into the 1o~ intercostal space at the midaxillary line as
described.

(Choices C and D) The intercostal vein, artery, and nerve lie in the subcostal groove on the lower border of
the rib. Thoracentesis should be performed along the upper border of the rib to prevent injury to these
structures.

~ F!!ck
---------------------------------------------------------------------------------------------------------------------------------------------
suW.nd EnQck
1
2 Item: 9 of 12
Q. ld: 844 [
lil f> Mark .<:J
Previous
[::>
Next
il
Lab Values
~
Notes
~
Calculator
3
I U CI VUIU '-'U I I I tJIIl.CIUUII~ VVIIIIt: IJCI I UII I IIIIY C1 U IUICH... tlllt~l~ , Ill~ IU:l.t:~~cu y lU l ti i i CI I IUtl li l t IUl.CIUUI I Ul li l t
4
5
lungs, pleura, and other organs of the chest and upper abdomen. The portion of pleura that covers the
6
surface of the lung is called the visceral pleura . The parietal pleura lines the inner surface of the chest w all
7
and the diaphragm, and is innervated by somatic sensory nerves. The diaphragm is bound to the inferior

-
8
margin of the thorax, and the portion of parietal pleura that covers the diaphragm extends to the follow ing
levels
10 Lower border of
Midclavicular line Midaxillary line Paravertebral line
11 the pleura
12
Upper border of 1o~
Right 7'" rib 12"' rib
rib
Low er border of 1o~
Left 7'" rib 12"' rib
rib

The low er border of the lung is usually located two intercostal spaces above the respective pleural border.
Thoracentesis, therefore, should be performed between 5'" and 7"' ribs along the midclavicular line, the 7'" and
911 ribs along the midaxillary line, and 9~ and 11"' ribs along the paravertebral line. If the needle is inserted
higher, there is a risk of lung injury. Insertion of the needle below the 9"' rib at the middle axillary line on the
right may cause liver injury (Choice B).

(Choice A) The low er border of the right lung is located two intercostal spaces above the pleural border. The
lung is not likely to be injured by insertion of a needle into the 1o~ intercostal space at the midaxillary line as
described.

(Choices C and D) The intercostal vein, artery, and nerve lie in the subcostal groove on the low er border of
the rib. Thoracentesis should be performed along the upper border of the rib to prevent injury to these
structures.

(Choice E) Hepatic veins are not likely to be injured during thoracocentesis.

Educational Objective:
Thoracocentesis should be performed above the 7"' rib in midclavicular line, the 9"' rib along midaxillary line
and the 11'" rib along posterior scapular line. Insertion of a needle low er than these points increases the risk
of penetrating abdominal structures, and insertion of the needle on the inferior margin of the rib risks striking
the subcostal neurovascular bundle.

Time Spent: 1 seconds Copyright USMLEWorld,LLC. Last updated: [1 / 16/2013]

~imeRemaini~ ~ 0
I Tuto' Feedback Suspend End Block
---------------------------------------------------------------------------------------------------------------------------------------------
1
2 Item: 10 of 12
Q.ld: 1883 [
lil f> Mark .<:J
Previous
[::>
Next
il
Lab Values
~
Notes
~
Calculator
3
4
5
6 A 32-year-old female complains of periodic dyspnea, dizziness and sharp chest pain. Her chest X-ray is
7 show n below .
8

11
12

The arrow points to w hich structure?

0 A. Superior vena cava


0 B. Coronary sinus
0 C. Right ventricle
0 D. Inferior vena cava
0 E. Right atrium
0 F. Pulmonary artery

~imeRemaini~ ~ 0
I Tuto' Feedback Suspend End Block
---------------------------------------------------------------------------------------------------------------------------------------------
1
2 Item: 10 of 12
Q.ld: 1883 [
lil f> Mark .<:J
Previous
[::>
Next
il
Lab Values
~
Notes
~
Calculator
3
4 v @ E. Right atrium [69%]
5
6
0 F. Pulmonary artery [2%]
7
8

-
Explanation: User ld:
9
The arrow on the chest X-ray above points to the right atrium. On postero-anterior (PA) chest X-ray
11 projections, the right atrium composes most of the right side of the cardiac silhouette. The right atrium
12 receives venous blood from the superior and inferior vena cavae, which compose the superior and inferior
portions of the right cardiac silhouette on this type of film .

Normal Chest Radiograph Anatomy:

.,/'Aortic Knob
~onary Artery
./left Alrium
Righi A~ Left Ventricle
......... tl'
vena cava
......... ~ "'ft" """'""

(Choice A) The superior vena cava is formed behind the first costal cartilage by the confluence of the right
and left brachiocephalic veins. On PA chest X-ray, it is the flattened opacity parallel to the vertebral column
that terminates inferiorly at the right atrium.

(Choice B) The coronary sinus receives most of the venous drainage from the cardiac circulation. It courses

~ F!!ck
---------------------------------------------------------------------------------------------------------------------------------------------
suW.nd EnQck
1
2
3
4
5
6
7
8

-
Right AI~
9
.........
11
Inferior Vana Cava
12
~

(Choice A) The superior vena cava is formed behind the first costal cartilage by the confluence of the right
and left brachiocephalic veins. On PA chest X-ray, it is the flattened opacity parallel to the vertebral column
that terminates inferiorly at the right atrium.

(Choice B) The coronary sinus receives most of the venous drainage from the cardiac circulation . It courses
posteriorly around the heart in the coronary sulcus to empty directly into the right atrium. It is difficult to
visualize on standard chest X-rays.

(Choice C) The right ventricle forms the anterior w all of the heart and is best seen on lateral chest X-rays.

(Choice D) The inferior vena cava receives venous blood from the low er extremities and abdomen and
empties into the right atrium in the thorax. It composes the most inferior edge of the right border of the cardiac
silhouette.

(Choice F) The pulmonary artery can be seen on an PA chest X-ray on the left side of the cardiomediastinal
silhouette just below the aortic arch .

Educational Objective:
The right atrium receives venous blood from both the superior vena cava and the inferior vena cava. The right
atrium makes up the majority of the right border of the heart on PA chest films. The right ventricle forms the
anterior w all of the heart and is best seen on lateral chest X-rays. The SVC and IVC compose the superior
and inferior borders of the cardiac silhouette on the right side.

Time Spent: 2 seconds Copyright USMLEW orld,LLC. Last updated: [7/7/201 0]

~imeRemaini~ ~ 0
I Tutor Feedback Suspend End Block
---------------------------------------------------------------------------------------------------------------------------------------------
1
2 Item: 11 of 12
Q.ld: 8703 [
111 f> Mark .<:J
Previous
[::>
Next
il
Lab Values
~
Notes
~
Calculator
3
4
5
6 A 23-year-old man comes to the emergency department complaining that he has food stuck in his throat. His
7 symptoms started 2 hours ago after eating fish at a local seafood restaurant. He has tried coughing and
8 sw allow ing multiple times in an attempt to clear the food, but has so far been unsuccessful. The patient
9

-
denies any difficulty w ith breathing. He does not appear to be in any distress on physical examination.
10 Laryngoscopy reveals a fish bone lodged in the left piriform recess. During an attempt to retrieve the fish
bone, a nerve is injured deep to the mucosa overlying the recess. Which of the follow ing is most likely to be
12 impaired in this patient?

0 A. Cough reflex
0 B. Gag reflex
0 C. Mastication
0 D. Salivation
0 E. Taste sensation

~imeRemaini~ ~ 0
I Tuto' Feedback Suspend End Block
---------------------------------------------------------------------------------------------------------------------------------------------
1
2 Item: 11 of 12
Q.ld: 8703 [
111 f> Mark .<:J
Previous
[::>
Next
il
Lab Values
~
Notes
~
Calculator
3
4
5 ., @ A. Cough reflex [35%]
6 0 B. Gag reflex (55%]
7
0 C. Mastication [2%]
8
9 0 D. Salivation [3%]
10 0 E. Taste sensation [5%]

12
Explanation: User ld:

Posterior view of pharynx

Palatine tonsils --~

Aryepiglottic fold

Epiglottis
Internal branch of
superior laryngeal

Mucosa overlying
internal laryngeal _ _, Thyroid cartilage
nerve
1
2
3
4
5
6
nerve
7
8 Mucosa overlying
9 Thyroid cartilage

-
internal laryngeal
10
nerve
12

Piriform recess

@USMLEWOIId,llC

The piriform recesses are small cavities that lie on either side of the laryngeal orifice. They are bounded
medially by the aryepiglottic folds and laterally by the thyroid cartilage and thyrohyoid membrane. During
normal sw allow ing, food is diverted by the epiglottis laterally through the piriform recesses into the esophagus
w ithout endangering the airway.

A thin layer of mucosa overlying the piriform recess is all that protects the superficially coursing internal
laryngeal nerve, a branch of the superior laryngeal nerve (CN X). Unlike the recurrent and external laryngeal
nerves that carry motor fibers to the muscles involved in vocal cord function, the internal laryngeal nerve
contains only sensory and autonomic fibers. It mediates the afferent limb of the cough reflex by carrying
sensation from the mucosa superior to the vocal cords. Foreign bodies, such as chicken or fish bones, can
become lodged in the piriform recess. The internal laryngeal nerve can be damaged by the sharpness of
these objects or attempts to retrieve them .

(Choice B) The afferent limb of the gag reflex is mediated predominantly by the qlossopharvnqeal nerve

~imeRemaini~ ~ 0
I Tuto' Feedback Suspend End Block
---------------------------------------------------------------------------------------------------------------------------------------------
1
2
3
4
5
6
7
8 @ USMLEWOrld, LLC
9

10

12
The piriform recesses are small cavities that lie on either side of the laryngeal orifice. They are bounded
medially by the aryepiglottic folds and laterally by the thyroid cartilage and thyrohyoid membrane. During
normal sw allow ing, food is diverted by the epiglottis laterally through the piriform recesses into the esophagus
w ithout endangering the airway.

A thin layer of mucosa overlying the piriform recess is all that protects the superficially coursing internal
laryngeal nerve, a branch of the superior laryngeal nerve (CN X). Unlike the recurrent and external laryngeal
nerves that carry motor fibers to the muscles involved in vocal cord function, the internal laryngeal nerve
contains only sensory and autonomic fibers. It mediates the afferent limb of the cough reflex by carrying
sensation from the mucosa superior to the vocal cords. Foreign bodies, such as chicken or fish bones, can
become lodged in the piriform recess. The internal laryngeal nerve can be damaged by the sharpness of
these objects or attempts to retrieve them .

(Choice B) The afferent limb of the gag reflex is mediated predominantly by the glossopharyngeal nerve
(CN IX}, w hile the efferent limb is carried by the vagus nerve (CN X). The internal laryngeal nerve does not
carry motor fibers and mediates sensation mainly from larynx and epiglottis (stimulation of these areas
induces coughing, not gagging).

(Choice D) Salivation is mediated in part by parasympathetic fibers originating from the glossopharyngeal
nerve. These fibers synapse on the otic ganglion, and postganglionic fibers travel via the auriculotemporal
nerve to reach the parotid gland.

(Choice E) Taste from the base of the tongue is mediated by the glossopharyngeal nerve (CN IX); taste from
the anterior two-thirds of the tongue is mediated by the facial nerve (CN VII).

Educational objective:
The internal laryngeal nerve mediates the afferent limb of the cough reflex above the vocal cords. Foreign
bodies (eg, chicken or fish bones) can become lodged in the piriform recess and may cause damage to the
nerve, impairing the cough reflex.

Time Spent: 1 seconds Copyright USMLEWorld,LLC. Last updated: [1 0/ 15/2013]

~imeRemaini~ ~ 0
I Tutor Feedback Suspend End Block
---------------------------------------------------------------------------------------------------------------------------------------------
1
2 Item: 12 of 12
Q.ld: 2 [
lil f> Mark .<:J
Previous
[::>
Next
il
Lab Values
~
Notes
~
Calculator
3
4 A patient is being evaluated for recurrent urinary tract infections. Her abdominal CT scan is show n below .
5
6
7
8
9
10

-
11

W hich of the follow ing most likely prevented the proper ascent of the anomalous organ seen on the CT scan?

0 A. Superior mesenteric artery


0 B. Vitellointestinal duct
0 C. Persistent urachus
0 D. Inferior vena cava
0 E. Inferior mesenteric artery

~ F!!ck suW.nd
---------------------------------------------------------------------------------------------------------------------------------------------
EnQck
1
2 Item: 12 of 12
Q.ld: 2 [
lil f> Mark .<:J
Previous
[::>
Next
il
Lab Values
~
Notes
~
Calculator
3
4 I .; @ E. Inferior mesenteric artery [75%]
5
6
7 Explanation: User ld:
8 Horseshoe kidney
9
10
11

- - - - - - Inferior
mesenteric a.
1
2 Item: 12 of 12
Q.ld: 2 [
lil f> Mark .<:J
Previous
[::>
Next
il
Lab Values
~
Notes
~
Calculator
3
4 Horseshoe kidney
5
6
7
8
9
10

-
11

----~Inferior
mesenteric a.

USMLEWorld, LLC 2010

~ F!!ck suW.nd EnQck


---------------------------------------------------------------------------------------------------------------------------------------------
1
2 Item: 12 of 12
Q.ld: 2 [
lil f> Mark .<:J
Previous
[::>
Next
il
Lab Values
~
Notes
~
Calculator
3
4 USMLEWorld, LLC 2010
5
6
The abdominal CT scan show s the kidneys joined at their low er poles. This is the most common variant of
7
horseshoe kidney, occurring in 90% of cases. In the other 10%, fusion occurs at the upper poles. Horseshoe
8
kidney is found in approximately 1 out of every 500 autopsies and may lead to urinary tract obstruction and the
9
development of hydronephrosis.
10

-
11 During embryogenesis, kidney development proceeds through the pronephros, mesonephros, and
metanephros stages. The fetal metanephros is initially located in the sacral region. In adults the mature
kidneys are located at vertebral levels T12-L3 (the kidney is slightly low er on the right compared to the left
side). The relative ascent of the kidneys from the sacral region to their normal anatomic position results from
the disproportionately rapid growth of the caudal part of the embryo.

W hen fusion of the low er or upper poles of the kidney occurs, the central part of the new ly formed horseshoe
kidney lies across the midline anterior to the great vessels. This centrally located isthmus is trapped by the
inferior mesenteric artery during the ascent of the kidney.

(Choice A) The superior mesenteric artery is located above the inferior mesenteric artery. It leaves the aorta
at the level of L 1 and does not serve as an obstacle for the ascent of a horseshoe kidney.

(Choice B) W hen the vitelline duct persists, a connection is formed between the intestinal lumen and the
outside of the body at the umbilicus. Discharge of meconium from the umbilicus w ill be seen in this condition.

(Choice C) A direct connection between the bladder lumen and the outside of the body at the umbilicus is
called a persistent urachus or urachal fistula . This condition w ould likely have been identified earlier in the
patient's life.

(Choice D) The inferior vena cava lies posterior to the isthmus of a horseshoe kidney and w ould not obstruct
its ascent.

Educational objective:
In horseshoe kidney, both kidneys are fused together at the poles in early embryonic life. The isthmus of
horseshoe kidney usually lies anterior to the aorta and inferior vena cava and posterior to the inferior
mesenteric artery. This centrally located isthmus becomes trapped behind the inferior mesenteric artery
during the relative ascent of the kidney.

Time Spent: 4 seconds Copyright USMLEW orld,LLC. Last updated: [1 0/23/2013]

~ F!!ck
---------------------------------------------------------------------------------------------------------------------------------------------
suW.nd EnQck

Das könnte Ihnen auch gefallen